Surgery Flashcards

1
Q

Pathophysiology of acute pancreatitis

A

Pancreatic enzymes released and activated-> multi stage process

Oedema + fluid shift + vomiting —> hypovolaemic shock

Enzymes—-> autodigestion

Vessel autodigestion—> retroperitoneal haemorrhage

Inflammation—–> pancreatic necrosis

How well did you know this?
1
Not at all
2
3
4
5
Perfectly
2
Q

Implications of pancreatic necrosis

A

Super-added infection in 50% of patients with necrosis

How well did you know this?
1
Not at all
2
3
4
5
Perfectly
3
Q

Epidemiology of acute pancreatitis

A

1% of surgical admissions

4th and 5th decades

10% mortality

How well did you know this?
1
Not at all
2
3
4
5
Perfectly
4
Q

Aetiology of pancreatitis

A

Idiopathic (?microstones)

Gallstones

Ethanol

Trauma

Steroids

Mumps (+ other infections e.g. Coxsackie B)

Autoimmune: PAN

Scorpion (Trinidadian)

Hyperlipidaemia, Hypercalcaemia, Hypothermia

ERCP

Drugs: thiazides, azathioprine

How well did you know this?
1
Not at all
2
3
4
5
Perfectly
5
Q

Severe epigastric pain radiating to the back

May be relieved by sitting forward

Vomiting

A

?Acute pancreatitis

How well did you know this?
1
Not at all
2
3
4
5
Perfectly
6
Q

Raised HR, Raised RR

Fever

Hypovolaemia—> shock

Epigastric tenderness

Jaundice

Ileus (absent bowel sounds)

Ecchymoses

A

?Acute pancreatitis

How well did you know this?
1
Not at all
2
3
4
5
Perfectly
7
Q

Grey turner’s

A

Flank ecchymoses

How well did you know this?
1
Not at all
2
3
4
5
Perfectly
8
Q

Cullen’s

A

Periumbilical ecchymosis (tracks up falciform)

How well did you know this?
1
Not at all
2
3
4
5
Perfectly
9
Q
A

Grey Turner’s sign

Flank ecchymosis

Acute pancreatitis

How well did you know this?
1
Not at all
2
3
4
5
Perfectly
10
Q
A

Cullens sign

Peri-umbilical ecchymosis

Acute pancreatitis

How well did you know this?
1
Not at all
2
3
4
5
Perfectly
11
Q

Ddx for acute pancreatitis

A

Perforated duodenal ulcer

Mesenteric infarction

MI

How well did you know this?
1
Not at all
2
3
4
5
Perfectly
12
Q

Difference between Glasgow and Ranson criteria

A

Glasgow criteria valid for EtOH and gallstones
whereas Ranson only applicable to Etoh and can only be fully applied after 48 hours

How well did you know this?
1
Not at all
2
3
4
5
Perfectly
13
Q

Components of modified glasgow score

PANCREAS

A

PaO2 <8kPA

Age >55

Neutrophils >15 x 10^9

Ca <2mM

Renal function, U >16mM

Enzymes: LDH >600iu/L, AST >200 iu/L

Albumin <32 g/L

Sugar >10mM

How well did you know this?
1
Not at all
2
3
4
5
Perfectly
14
Q

Modified Glasgow criteria cut offs

A

1= mild

2= moderate

3= severe

How well did you know this?
1
Not at all
2
3
4
5
Perfectly
15
Q

Ix in acute pancreatitis and what would be seen

Bloods

A

Bloods:

FBC- raised WCC

Raised amylase (>1000/3x ULN) and raised lipase

U+Es: dehydration and renal failure

LFTs: cholestatic picture, raised AST, raised LDH

Ca: reduced

Glucose: raised

CRP: monitor progress, >150 after 48 hours= severe

ABG: reduecd O2 suggests ARDS

How well did you know this?
1
Not at all
2
3
4
5
Perfectly
16
Q

Ix in acute pancreatitis and what would be seen

Urine

A

Glucose

Raised conjugated bilirubin

Reduced urobiliongen

How well did you know this?
1
Not at all
2
3
4
5
Perfectly
17
Q

Ix in acute pancreatitis and what would be seen

Imaging

A

CXR: ARDS, exclude perforated DU

AXR: sentinel loop, pancreatic calcification

USS: gallstones and dilated ducts, inflammation

Contrast CT: Balthazar severity score

How well did you know this?
1
Not at all
2
3
4
5
Perfectly
18
Q

Cut offs for amylase in acute pancreatitis

A

>1000/ 3xULN

How well did you know this?
1
Not at all
2
3
4
5
Perfectly
19
Q

Difference between lipase and amylase

A

Lipase is more sensitive and speciic

How well did you know this?
1
Not at all
2
3
4
5
Perfectly
20
Q

CRP >150 after 48hrs in acute pancreatitis

A

Severe

How well did you know this?
1
Not at all
2
3
4
5
Perfectly
21
Q

What is used to grade severity of pancreatitis on CT?

A

Balthazar severity score

How well did you know this?
1
Not at all
2
3
4
5
Perfectly
22
Q

Complications of acute pancreatitis

Early: systemic

A

Respiratory: ARDS, pleural effusion

Shock: hypovolaemic or septic

Renal failure

DIC

Metabolic: hypocalcaemia, raised glucose, metabolic acidosis

How well did you know this?
1
Not at all
2
3
4
5
Perfectly
23
Q

Complications of acute pancreatitis

Late (>1w)

A

Pancreatic necrosis

Pancreatic infection

Pancreatic abscess: may form in pseudocyst or in pancreas, may require open or percutaenous drainage

Bleeding: e.g. from splenic artery, may require embolisation

Thrombosis: splenic artery, GDA or colic branches of SMA, may subsequently lead to bowel necrosis. Portal vein, may subsequently lead to portal HTN

Fistula formation: pancreato-cutaneous due to skin breakdown

How well did you know this?
1
Not at all
2
3
4
5
Perfectly
24
Q

Def: pancreatic pseudocyst

A

Collection of pancreatic fluid in the lesser sac, surrounded by granulation tissue

Occurs in 20% especially in EtOHic pancreatitis

How well did you know this?
1
Not at all
2
3
4
5
Perfectly
25
Q

4-w after acute attack

Persisting abdominal pain

Epigastric mass-> early satiety

A

?Pancreatic pseudocyst

How well did you know this?
1
Not at all
2
3
4
5
Perfectly
26
Q

Complications of pancreatic pseudocyst

A

Infection-> abscess

Obstruction of duodenum or common bile duct

How well did you know this?
1
Not at all
2
3
4
5
Perfectly
27
Q

Ix in pancreatic pseudocyst

A

Persistently raised amylase +/- deranged LFTs

USS/CT

How well did you know this?
1
Not at all
2
3
4
5
Perfectly
28
Q

Management of pancreatic pseudocyst

A

<6cm: spontaneous resolution

>6cm:

Endoscopic cyst-gastrostomy

Percutaenous drainage under US/CT

How well did you know this?
1
Not at all
2
3
4
5
Perfectly
29
Q

Causes of chronic pancreatitis

AGITS

A

Alcohol: 70%

Genetic: CF, HH

Immune: lymphoplasmacytic sclerosing pancreatitis (raised IgG4)

TGs raised

Structural: obstruction by tumour, pancreas divisum

How well did you know this?
1
Not at all
2
3
4
5
Perfectly
30
Q
A

Sentinal loop

Focal dilated proximal jejunal loop in the LUQ

Acute pancreatitis

How well did you know this?
1
Not at all
2
3
4
5
Perfectly
31
Q
A

Pancreatic calcification

How well did you know this?
1
Not at all
2
3
4
5
Perfectly
32
Q

Epigastric pain: bores through to back

Relieved by sitting back or hot water bottle: erythema ab igne

Exacerbated by fatty food or EtOH

Steatorrhoea and weight loss

(Polyuria and polydipsia)

(Epigastric mass)

A

Chronic pancreatitis

(DM)

(pancreatic pseudocyst)

How well did you know this?
1
Not at all
2
3
4
5
Perfectly
33
Q

Complications of chronic pancreatitis

A

Pseudocyst

DM

Pancreatic cancer

Pancreatic swelling-> biliary obstruction

Splenic vein thrombosis

How well did you know this?
1
Not at all
2
3
4
5
Perfectly
34
Q

Ix in chronic pancreatitis

A

Raised glucose

Reduced faecal elastase= indicative of reduced exocrine function

LFTs

USS: pseudocyst

AXR: speckled pancreatic clacifications

CT: pancreatic calcifications

How well did you know this?
1
Not at all
2
3
4
5
Perfectly
35
Q

Serum amylase in chronic pancreatitis

A

serum amylase level is not routinely raised, is not diagnostic in chronic pancreatitis, and testing should therefore not be performed.

How well did you know this?
1
Not at all
2
3
4
5
Perfectly
36
Q

Px of chronic pancreatitis

A

1/3rd die within 10y

How well did you know this?
1
Not at all
2
3
4
5
Perfectly
37
Q

Conservative management of chronic pancreatitis

A

No EToH or smoking

Reduced fat and increased carb diet

How well did you know this?
1
Not at all
2
3
4
5
Perfectly
38
Q

Medical management of chronic pancreatitis

A

Analgesia: NSAIDS/paracetamol first line. ?weak opiate (codeine phospohate) if not effective. May need additional pain relief in primary care e.g. coeliac plexus block.

Enzyme supplementation e.g. Creon

Cobalamin/thiamine supplementation

ADEK vitamins

DM Rx

(Octreotide: somatostatin analgoe that inhibits pancreatic enzyme secretions)

How well did you know this?
1
Not at all
2
3
4
5
Perfectly
39
Q

Screening in chronic pancreatitis

A

DM

Osteoporosis

How well did you know this?
1
Not at all
2
3
4
5
Perfectly
40
Q

Indications for Surgical Mx of chronic pancreatitis

A

Unremitting pain

Weight loss

Duct blockage

How well did you know this?
1
Not at all
2
3
4
5
Perfectly
41
Q

Surgical options of management of chronic pancreatitis

A

Distal pancreatectomy: Whipple’s

Pancreaticojejunostomy: drainage

Endoscopic stenting

How well did you know this?
1
Not at all
2
3
4
5
Perfectly
42
Q

Monitoring in management of acute pancreatitis

A

Manage at appropriate level e.g. ITU if severe

Constant reassessment:

Hourly TPR, UO

Daily FBC, U+Es, Ca, glucose, amylase

How well did you know this?
1
Not at all
2
3
4
5
Perfectly
43
Q

Medical management of acute pancreatitis

A

ABC approach

Aggressive fluid resus: keep UO >30ml/h

Catheter+/- CVP

Pancreatic rest:

NBM

NGT if vomiting

TPN may be required

Analagesia:

pethidine or buprenorphine ± intravenous (IV) benzodiazepines. Morphine is relatively contra-indicated because of possible spastic effect on the sphincter of Oddi.

Antibiotics:

Not routinely given, use if suspected infection or before ERCP

Penems often used, but use to treat specific infections

Mx complications

How well did you know this?
1
Not at all
2
3
4
5
Perfectly
44
Q

Mx of the complications of acute pancreatitis

A

ARDS: O2 therapy or ventilation

Raied glucose: insulin sliding scale

Ca

EToH withdrawal: chlordiazepoxide

How well did you know this?
1
Not at all
2
3
4
5
Perfectly
45
Q

ERCP use in acute pancreatitis

A

Diagnostic

Can be used if pancreatitis with dilated ducts secondary to gallstones. ERCP and sphincterectomy -> reduces complications

How well did you know this?
1
Not at all
2
3
4
5
Perfectly
46
Q

Indications for surgical management of acute pancreatitis

A

Infected pancreatic necrosis

Pseudocyst or abscess

Unsure re dx

How well did you know this?
1
Not at all
2
3
4
5
Perfectly
47
Q

Operations used in treatment of acute pancreatitis

A

Laparotomy + necrosectomy (pancreatic debridement)

Laparotomy + peritoneal lavage

Laparostomy: abdomen left open with sterile packs in ITU

How well did you know this?
1
Not at all
2
3
4
5
Perfectly
48
Q

Epidemiology of gallstones

A

~8% of the population >40 years

Incidence increasing

Slight increased incidence in females

90% of gallstones remain asymptomatic

How well did you know this?
1
Not at all
2
3
4
5
Perfectly
49
Q

5Fs of gallstones

A

Fair

Fat

Female

Forty

Fertile

How well did you know this?
1
Not at all
2
3
4
5
Perfectly
50
Q

What is the general composition of gallstones

A

Phopsholipids: lecithin

Bile pigments (broken down Hb)

Cholesterol

How well did you know this?
1
Not at all
2
3
4
5
Perfectly
51
Q

What are the relative proportions of the different types of gall stones?

A

Mixed stones: 75%

Cholesterol stones: 20%

Pigment stones: 5%

How well did you know this?
1
Not at all
2
3
4
5
Perfectly
52
Q

What is the aetiology of gallstones

A

Lithogenic bile: Admirand’s triangle

Biliary sepsis

GB hypomotility-> stasis: pregnancy, OCP, TPN, fasting

How well did you know this?
1
Not at all
2
3
4
5
Perfectly
53
Q

What is admirand’s triangle

A

A delicate balacnce exists between the levels of bile acids, phospholipids and cholesterol

When this balance is disrupted, especially when there is supersaturation with cholesterol, there is predisoposition to the formation of lithogenic bile and the conseuqent development of cholesterol-type gallstones.

This is because when cholesterol supersaturates it tends to crystallise and in the presence of enucleating factors can be a nidus for stone formation

How well did you know this?
1
Not at all
2
3
4
5
Perfectly
54
Q

Large often solitary gallbladder stone

A

Cholsterol

How well did you know this?
1
Not at all
2
3
4
5
Perfectly
55
Q

Formation of cholesterol gallstones

A

According to Admirand’s triangle:

decreased bile salts

decreased lecithin

increased cholesterol

How well did you know this?
1
Not at all
2
3
4
5
Perfectly
56
Q

What are the risk factors for the development of cholesterol stones?

A

Female

OCP/pregnancy

Increasing age

High fat diet + obesity

Racial e.g. American Indian tribes

Loss of terminal ileum (reduction in bile salt reabsorption)

How well did you know this?
1
Not at all
2
3
4
5
Perfectly
57
Q

What is the composition of pigment stones in the gall bladder

A

Calcium bilirubinate

How well did you know this?
1
Not at all
2
3
4
5
Perfectly
58
Q

With what are pigment GB stones associated?

A

Haemolysis

How well did you know this?
1
Not at all
2
3
4
5
Perfectly
59
Q

Small, black gritty, fragile GB stones

A

Pigment stones

How well did you know this?
1
Not at all
2
3
4
5
Perfectly
60
Q

Often multiple GB stones with cholesterol as the major component

A

Mixed stones

How well did you know this?
1
Not at all
2
3
4
5
Perfectly
61
Q

What are the complications of gallstones

A

In the gallbladder:

Biliary colic

Acute cholecystitis +/- empyema

Chronic cholecystitis

Mucocele

Carcinoma

Mirizzi’s syndrome

In the CBD:

Obstructive jaundice

Pancreatitis

Cholangitis

In the gut:

Gallstone ileus

How well did you know this?
1
Not at all
2
3
4
5
Perfectly
62
Q

Mirizzi’s syndrome

A

Mirizzi’s syndrome is a rare complication in which a gallstone becomes impacted in the cystic duct or neck of the gallbladder causing compression of the common bile duct (CBD) or common hepatic duct, resulting in obstruction and jaundice. The obstructive jaundice can be caused by direct extrinsic compression by the stone or from fibrosis caused by chronic cholecystitis (inflammation). A cholecystocholedochal fistula can occur

How well did you know this?
1
Not at all
2
3
4
5
Perfectly
63
Q

Pathogenesis of biliary colic

A

Gallbladder spasm against a stone impacted in the neck of the gallbladder: Hartmann’s pouch

Less commonly, the stone may be in the CBD

How well did you know this?
1
Not at all
2
3
4
5
Perfectly
64
Q

What is Hartmann’s pouch?

A

a spheroid or conical pouch at the junction of the neck of the gallbladder and the cystic duct.

How well did you know this?
1
Not at all
2
3
4
5
Perfectly
65
Q

RUQ pain radiating to back (scapular region)

Associated with sweating, pallor, N+V

Attacks may be precipitated by fatty food and last <6h

O/E tenderness in right hypochondrium

(jaundice)

A

?Biliary colic

(if stone is in the CBD)

How well did you know this?
1
Not at all
2
3
4
5
Perfectly
66
Q

DDx for biliary colic

A

Cholecystitis/other gallstone disease

Pancreatitis

Bowel perforation

How well did you know this?
1
Not at all
2
3
4
5
Perfectly
67
Q

Ix in biliary colic

Urine

A

Same work up as cholecystitis

Urine:

Bilirubin, urobilinogen, Hb

How well did you know this?
1
Not at all
2
3
4
5
Perfectly
68
Q

Ix in biliary colic

Bloods

A

Bloods: FBC, U+E, amylase, LFTs, G+S, clotting CRP

How well did you know this?
1
Not at all
2
3
4
5
Perfectly
69
Q

Ix in biliary colic

Imaging

A

AXR: 10% of gallstones are radio-opaque

Erect CXR: ?perforation

USS:

stones: acoustic shadow

dilated ducts >6mm

Inflamed GB: wall oedema

How well did you know this?
1
Not at all
2
3
4
5
Perfectly
70
Q

Indications for MRCP in biliary colic

A

bile duct dilated and or liver function tests abnormal and USS has not detected CBD stones

How well did you know this?
1
Not at all
2
3
4
5
Perfectly
71
Q

What is the indication for endoscopic USS in biliary colic

A

If MRCP does not allow diagnosis to be made

How well did you know this?
1
Not at all
2
3
4
5
Perfectly
72
Q

Mx of biliary colic

A

Conservative:

Rehydrate and NBM

Opioid analgesia: morphine 5-10mg/2h max

High recurrence rate therefore surgical management is favoured

NB asymptomatic gallbladder stones do not need treatment

Surgical:

Laparoscopic cholecystectomy:

Urgent (within 1w of diagnosis in those with acute cholecystitis)

Elective at 6-12w

Percutaenous choleystotomy

How well did you know this?
1
Not at all
2
3
4
5
Perfectly
73
Q

Indications for percutaenous cholecystotomy

A

Management of gallbladder empyema when:

surgery is contraindicated at presentation and conservative management is unsuccessful.

Reconsider lap chole for people whho have had percutaenous cholecystotomy once they are well enough for surgery

How well did you know this?
1
Not at all
2
3
4
5
Perfectly
74
Q

Mx of CBD stones

A

Offer bile duct clearance and lap chole to people with symptomatic or asymptomatic CBD stones

Clear the bile duct:

surgically at the time of lap chole

or with ERCP before or at time of surgery

If the bile duct cannot be cleared with ERCP, use bilirary stenting to achieve drainage as a definitive measure until definitive clearance

How well did you know this?
1
Not at all
2
3
4
5
Perfectly
75
Q

Pathogenesis of acute cholecystitis

A

Stone or sludge impaction in Hartmann’s pouch leading to chemical and or bacterial inflammation

5% are acalculous: sepsis, burns, DM

How well did you know this?
1
Not at all
2
3
4
5
Perfectly
76
Q

Sequelae of acute cholecystitis

A

Resolution +/- recurrence

Gangrene and rarely perf

Chronic cholecystitis

Empyema

How well did you know this?
1
Not at all
2
3
4
5
Perfectly
77
Q

Servere RUQ pain

Continuous radiating to right scapula and epigastrium

Fever

Vomiting

A

?Acute cholecystitis

How well did you know this?
1
Not at all
2
3
4
5
Perfectly
78
Q

Local peritonism in RUQ, tachycardia with shallow breathing

+/- jaundice

Murphy’s sign

Phlegmon

Boas’s sign

A

?Acute cholecystitis

How well did you know this?
1
Not at all
2
3
4
5
Perfectly
79
Q

What is Murphy’s sign

A

2 fingers over the GB and ask patient to breath in

Pain and breath catch, must be -ve on the left

How well did you know this?
1
Not at all
2
3
4
5
Perfectly
80
Q

What is phlegmon

A

May be palpable in acute cholecystitis

Mass of adherent omentum and bowel

How well did you know this?
1
Not at all
2
3
4
5
Perfectly
81
Q

What is Boas’ sign

A

Hyperaesthesia below the right scapula

How well did you know this?
1
Not at all
2
3
4
5
Perfectly
82
Q

Ix in acute cholecystitis

Urine

A

Bilirubin, urobilinogen

How well did you know this?
1
Not at all
2
3
4
5
Perfectly
83
Q

Ix in acute cholecystitis

Bloods

A

FBC: raised WCC

U+E: dehydration from vomiting

Amylase, LFTs, G+S, clotting, CRP

How well did you know this?
1
Not at all
2
3
4
5
Perfectly
84
Q

Ix in acute cholecystitis

Imaging

A

AXR: gallstone, porcelain GB

Erect CXR: perforation

USS:

stones, acoustic shadow

Dilated ducts >6mm

Inflamed GB: wall oedema

MRCP if dilated ducts seen on USS

How well did you know this?
1
Not at all
2
3
4
5
Perfectly
85
Q
A

Porcelain gallbladder is an uncommon manifestation of chronic cholecystitis, characterized by intramural calcification of the gallbladder wall [1]. The term “porcelain gallbladder” is used to describe the bluish discoloration and brittle consistency of the gallbladder wall seen in this condition [

How well did you know this?
1
Not at all
2
3
4
5
Perfectly
86
Q

Mx of acute cholecystitis

A

Admit

Conservative:

NBM

Fluid resuscitation

Analgesia: paracetamol, diclofenac, codeine, naproxen, IM pethidine

Abx:

cef and met

80-90% settle over 24-48h

Deterioration may be suggestive of perforation/empyema

Surgical:

May be elective surgery at 6-12w (once inflammation has reduced)

Or if <72h can perform lap chole in acute phase

How well did you know this?
1
Not at all
2
3
4
5
Perfectly
87
Q

High fever

RUQ mass

A

Think ?GB empyema

Percutaenous drainage via cholecystotomy may be indicated

How well did you know this?
1
Not at all
2
3
4
5
Perfectly
88
Q

Vague upper abdominal discomfort

Distension, bloating

Nausea

Flatulence, burping

Symptoms exacerbated by fatty foods

A

Flatulent dyspepsia

?Chronic cholecystitis

How well did you know this?
1
Not at all
2
3
4
5
Perfectly
89
Q

Ddx in chronic cholecystitis

A

PUD

IBS

Hiatus hernia

Chronic pancreatitis

How well did you know this?
1
Not at all
2
3
4
5
Perfectly
90
Q

Ix in chronic cholecystitis

A

AXR: porcelain GB

US: stones, fibrotic, shrunken GB

MRCP

How well did you know this?
1
Not at all
2
3
4
5
Perfectly
91
Q

Mx of chronic cholecystitis

A

Medical: bile salts (not very effective)

Surgical:

Elective cholecystectomy

ERCP first if USS shows dilated ducts and stones

How well did you know this?
1
Not at all
2
3
4
5
Perfectly
92
Q

What are the features of gallstone mucocele

A

Neck of the GB blocked by stone but contents remain sterile

Can be very large-> palpable mass

May become infected–> empyema

How well did you know this?
1
Not at all
2
3
4
5
Perfectly
93
Q

Features of GB carcinoma

A

Rare

Associated with gallstones and gallbladder polyps

May see porcelain GB

Incidental Ca found in 0.5-1% of lap choles

How well did you know this?
1
Not at all
2
3
4
5
Perfectly
94
Q

Features of Gallstone ileus

A

Large stone >2.5cm erodes from GB into duodenum through a cholecysto-duodenal fistula 2o to inflammation

May impact in the distal ileum leading to obstruction

Rigler’s triad

How well did you know this?
1
Not at all
2
3
4
5
Perfectly
95
Q

What is Rigler’s triad

A

Pneumobilia

SBO

Gallstone in RLQ

How well did you know this?
1
Not at all
2
3
4
5
Perfectly
96
Q
A

Rigler triad consists of three findings seen in gallstone ileus:

pneumobilia

small bowel obstruction

gallstone, usually in the right iliac fossa

Rigler triad should not be confused with Rigler sign or the Hoffman-Rigler sign.

How well did you know this?
1
Not at all
2
3
4
5
Perfectly
97
Q

What is Bouveret’s syndrome?

A

Gastric outlet obstruction due to gallstone

How well did you know this?
1
Not at all
2
3
4
5
Perfectly
98
Q

Causes of obstructive jaundice

Rule of 30%s…

A

30% stones

30% Ca head of the pancreas

30% other

How well did you know this?
1
Not at all
2
3
4
5
Perfectly
99
Q

What are other causes of obstructive jaundice

A

LNs @ porta hepatis: TB, Ca

Inflammatory: PBC, PSC

Drugs: OCP, sulfonylureas, fluclox

Neoplastic: cholangiocarcinoma

Mirizzi’s syndrome

How well did you know this?
1
Not at all
2
3
4
5
Perfectly
100
Q

Jaundice

Dark urine, pale stools

Itch

A

Obstructive jaundice

Itch due to bile salts

How well did you know this?
1
Not at all
2
3
4
5
Perfectly
101
Q

At what [serum] is obstructive jaundice noticeable and where is it seen first

A

Noticable at ~50mM

Seen at tongue frenulum first

How well did you know this?
1
Not at all
2
3
4
5
Perfectly
102
Q

Ix in obstructive jaundice

Urine

A

Dark

Raised bilirubin

Reduced urobilinogen

How well did you know this?
1
Not at all
2
3
4
5
Perfectly
103
Q

Ix in obstructive jaundice

Bloods

A

FBC: raised WCC in cholangitis

U+Es: hepatorenal syndrome

LFTs: raised conjugated bilirubin, ++ ALP, +AST/ALT

Clotting: reduced VitK–> Raised INR

G+S: may need ERCP

Immune: AMA, ANCA, ANA

How well did you know this?
1
Not at all
2
3
4
5
Perfectly
104
Q

Ix in obstructive jaundice

Imaging

A

AXR: may visualise stone

Pneumobilia suggests gas forming infection

USS:

DIlated ducts >6mm

Stones (95% accurtate)

Tumour

MRCP or ERCP

PTC: percutaneous transhepatic colangiography

How well did you know this?
1
Not at all
2
3
4
5
Perfectly
105
Q

What is PCT

A

Percutaneous transhepatic cholangiography (PTHC or PTC) or percutaneous hepatic cholangiogram is a radiologic technique used to visualize the anatomy of the biliary tract. A contrast medium is injected into a bile duct in the liver, after which X-rays are taken.

How well did you know this?
1
Not at all
2
3
4
5
Perfectly
106
Q
A

Pneumobilia

May be suggestive of a gas forming infection

How well did you know this?
1
Not at all
2
3
4
5
Perfectly
107
Q

Mx of stone causing obstructive jaundice

A

Conservative:

Monitor LFTs, stone passage may lead to resolution

Vits ADEK

Analgesia

Cholestyramine

Interventional:

If no resolution, worsening LFTs or cholangitis

ERCP with sphincterectomy and stone extraction

Surgical:

Open/lap stone removal with T tube placement

T tube cholangiogram 8d later to confirm stone removal

Delayed cholecystectomy to prevent recurrence

How well did you know this?
1
Not at all
2
3
4
5
Perfectly
108
Q

MOA cholestyramine

A

Cholestyramine is a bile acid sequestrant. It works by helping the body remove bile acids, which can lower cholesterol levels in the blood. The medicine is also used to relieve itching that’s caused by a bile duct blockage

How well did you know this?
1
Not at all
2
3
4
5
Perfectly
109
Q

Features of ascending cholangitis

A

Charcot’s triad

Reynolds pentad

How well did you know this?
1
Not at all
2
3
4
5
Perfectly
110
Q

What is Charcot’s triad

A

Fever/rigors

RUQ pain

Jaundice

How well did you know this?
1
Not at all
2
3
4
5
Perfectly
111
Q

What is Reynolds pentad

A

Charcot’s tirad

Shock

Confusion

How well did you know this?
1
Not at all
2
3
4
5
Perfectly
112
Q

Mx of ascending cholangitis

A

Cef and met

1st ERCP

2nd: open or lap stone removal with T tube drain

How well did you know this?
1
Not at all
2
3
4
5
Perfectly
113
Q

Risk factors for pancreatic carcinoma

SINED

A

Smoking

Inflammation: chronic pancreatitis

Nutrition: high fat diet

EtOH

DM

How well did you know this?
1
Not at all
2
3
4
5
Perfectly
114
Q

Pathology of pancreatic carcinoma

A

90% are ductal adenocarcinomas

Present late, metastasise early

Direct extension to local structures, lymphatics, haematogenously to liver and lungs

How well did you know this?
1
Not at all
2
3
4
5
Perfectly
115
Q

Location of pancreatic adenocarcinoma

A

60% in head

25% in body

15% in tail

How well did you know this?
1
Not at all
2
3
4
5
Perfectly
116
Q

(Male) >60y

Painless obstructive jaundice picture

or epigastric pain radiating to back and relieved by sitting forward

Anorexia, weight loss and malabsorption

Acute pancreatitis

Sudden onset DM

A

?Pancreatic carcinoma

How well did you know this?
1
Not at all
2
3
4
5
Perfectly
117
Q

Palpable gallbladder

Jaundice

Epigastric mass

(Trousseau’s sign)

Splenomegaly

Ascites

A

Pancreatic adenocarcinoma

(Thrombophlebitis migrans)

Splenomegaly due to PV thrombosis leading to portal HTN

How well did you know this?
1
Not at all
2
3
4
5
Perfectly
118
Q

What is Courvossier’s law

A

In the presence of painless obstructive jaundice, a palpable gall bladder is unlikely to be due to stones

How well did you know this?
1
Not at all
2
3
4
5
Perfectly
119
Q

Why does Coruvossier’s sign occur

A

Cause unlikely to be gallstones, gallstones form over an extended period of time resulting in a shrunken, fibrotic gall bladder that does not easily distend, this is less likely to be palpable on examination

In contrast, the gallbladder is more enlarged in pathologies that cause obstruction of the biliary tree over longer periods of time e.g. pancreatic malgiancny leading to passive distension from back pressure

How well did you know this?
1
Not at all
2
3
4
5
Perfectly
120
Q

Ix in pancreatic carcinoma

Bloods

A

Cholestatic LFTs

Ca 19-9

Raised Ca

How well did you know this?
1
Not at all
2
3
4
5
Perfectly
121
Q

Ix in pancreatic carcinoma

Imaging

A

USS: pancreatic mass, dilated ducts, hepatic mets, guide biopsy

EUS: better than CT/MRI for staging

CXR: mets

Laparoscopy: mets, staging

How well did you know this?
1
Not at all
2
3
4
5
Perfectly
122
Q

ERCP in pancreatic caricnoma

A

Shows anatomy

Allows stenting

Biopsy of peri-ampullary lesions

How well did you know this?
1
Not at all
2
3
4
5
Perfectly
123
Q

5y survival of pancreatic carcinoma

A

<2%

Mean survival <6m

How well did you know this?
1
Not at all
2
3
4
5
Perfectly
124
Q

Mx of pancreatic carcinoma

A

Sx:

fit, nomets, tumour <3cm (<10% patients)

Whipple’s pancreaticoduodenectomy

Distal pancreatectomy

Post-op chemo delays progression

Palliation:

Endoscopic/percutaneous stenting of CBD

Paliative bypass surgery: cholecystojejunosostomy, gastrojejunostomy

Pain relief: may need coeliac plexus block

How well did you know this?
1
Not at all
2
3
4
5
Perfectly
125
Q

Epidemiology of pancreatic endocrine neoplasia

A

30-60 y/o

15% associated with MEN1

How well did you know this?
1
Not at all
2
3
4
5
Perfectly
126
Q

With which MEN are pancreatic endocrine neoplasias associated?

A

MEN1

How well did you know this?
1
Not at all
2
3
4
5
Perfectly
127
Q

Fasting/exercise induced hypoglycaemia

Confusion, stuipor< LOC

Raised insulin, raised c-peptide, reduced glucose

A

Insulinoma

How well did you know this?
1
Not at all
2
3
4
5
Perfectly
128
Q

Hypergastrinaaemia–> hyperchlorhydia—> PUD and chronic diarrhoea due to inactivation of pancreatic enzymes

A

Gastrinoma (Zollinger-Ellison)

How well did you know this?
1
Not at all
2
3
4
5
Perfectly
129
Q

Raised glucagon

Necrolytic migratory erythema

A

Glucagonoma

How well did you know this?
1
Not at all
2
3
4
5
Perfectly
130
Q
A

Necrolytic migratory erythema

Characteristic rash of glucagonoma

How well did you know this?
1
Not at all
2
3
4
5
Perfectly
131
Q

Watery diarrhoea

Hypokalaemia

Achlorrhydia

Acidosis

A

VIPoma/ Verner-Morrison/ WDHA syndrome

How well did you know this?
1
Not at all
2
3
4
5
Perfectly
132
Q

MOA somatostatin

A

Inhibits glucagon and insulin release

Inhibits pancreatic enzyme secretion

How well did you know this?
1
Not at all
2
3
4
5
Perfectly
133
Q

DM

Steatorrhoea

Gallstones

Usually very malignant tumour with poor prognosis

A

Somatostatinoma

How well did you know this?
1
Not at all
2
3
4
5
Perfectly
134
Q

Favoured sites are stomach and duodenum followed by jejunum and ileum

Usually located in submucosa

Maybe visualised as a sessile mass

May cause pain from localised inflammation or more rarely, mucosal bleeding

2% of tumours arise in this tissue

A

Ectopic pancreas

How well did you know this?
1
Not at all
2
3
4
5
Perfectly
135
Q

Failuire of fusion of dorsal and ventral buds—> bulk of pancreas drains through smaller accessory duct.

Usually asymptomatic

May present with chronic pancreatitis

A

Pancreas divisum

How well did you know this?
1
Not at all
2
3
4
5
Perfectly
136
Q

Fusion of dorsal and ventral buds around duodenum

May present with infantile duodenal obstruction

A

Annular pancreas

How well did you know this?
1
Not at all
2
3
4
5
Perfectly
137
Q
A

Pancreas divisum

How well did you know this?
1
Not at all
2
3
4
5
Perfectly
138
Q
A

Annular pancreas

How well did you know this?
1
Not at all
2
3
4
5
Perfectly
139
Q

What is a Klatskin tumour

A

Typically occuring cholangiocarcinoma at confluence of right and left hepatic ducts

How well did you know this?
1
Not at all
2
3
4
5
Perfectly
140
Q

Features of cholangiocarcinoma

A

Rare bile duct tumour

Adenocarcinoma

[Klatskin tumour[

How well did you know this?
1
Not at all
2
3
4
5
Perfectly
141
Q
A

Klatskin tumour

How well did you know this?
1
Not at all
2
3
4
5
Perfectly
142
Q

Risk factors for cholangiocarcinoma

A

PSC

UC

Choledocholithiasis

Hep B/C

Choledochal cysts

Lynch 2

Flukes

How well did you know this?
1
Not at all
2
3
4
5
Perfectly
143
Q

Progressive painless obstructive jaundice

GB not palpable

Steatorrhoea

Weight loss

A

?Cholangiocarcinoma

How well did you know this?
1
Not at all
2
3
4
5
Perfectly
144
Q

Ix in cholangiocarcinoma

A

Cholestatic LFTs

CA19-9

How well did you know this?
1
Not at all
2
3
4
5
Perfectly
145
Q

Mx of cholangiocarcinoma

A

Poor Px

Palliative stenting

How well did you know this?
1
Not at all
2
3
4
5
Perfectly
146
Q

Pathophysiology of Hydatid cyst

A

Zoonotic infection by Echinococcus granulosus

Occurs in sheep-rearing communities

Parasite penetrates the portal system and infects the liver—> calcified cyst

How well did you know this?
1
Not at all
2
3
4
5
Perfectly
147
Q

Mostly asymptomatic

Pressure effects: non-specific pain, abdominal fullness, obstructive jaundice

Rupture: biliary colic, jaundice, urticaria, anaphylaxis, 2o infection

A

?Hydatid cyst

How well did you know this?
1
Not at all
2
3
4
5
Perfectly
148
Q

Ix in hydatid cyst

A

Eosinophilia

CT

How well did you know this?
1
Not at all
2
3
4
5
Perfectly
149
Q

Rx in hydatid cyst

A

Rx: albendazole

Sx: cystectomy for large cysts

How well did you know this?
1
Not at all
2
3
4
5
Perfectly
150
Q

What are the aims of pre-operative assessment and planning

A

Informed consent

Assess risk vs benefits

Optimise fitness of patient

Check anaesthesia/analgesia type with anaesthetist

How well did you know this?
1
Not at all
2
3
4
5
Perfectly
151
Q

OP CHECS

A

Operative fitness: cardiorespiratory comorbidities

Pills

Consent

History: MI, asthma, HTN, jaundice, complications of anaesthesia, DVT

Ease of intubation: neck arthritis, dentures, loose teeth

Clexane: DVT prophylaxis

Site: correct and marked

How well did you know this?
1
Not at all
2
3
4
5
Perfectly
152
Q

Anticoagulants pre-op

A

Balance risk of haemorrhage with risk of thrombosis

Avoid epidural, spinal and regional blocks

How well did you know this?
1
Not at all
2
3
4
5
Perfectly
153
Q

AED pre-op

A

Continue as usual

Post-op give IV or via NGT if unable to tolerate orally

How well did you know this?
1
Not at all
2
3
4
5
Perfectly
154
Q

OCP/HRT pre-op

A

Stop 4w before major/leg sx

Restart 2w post-op if mobile

How well did you know this?
1
Not at all
2
3
4
5
Perfectly
155
Q

Beta-blockers pre-op

A

Continue as normal

How well did you know this?
1
Not at all
2
3
4
5
Perfectly
156
Q

What are the routine pre-op bloods?

A

FBC

U+Es

G+S

Clotting

Glucose

How well did you know this?
1
Not at all
2
3
4
5
Perfectly
157
Q

What specific bloods are used pre-op

A

LFTs: liver disease, EtOH, jaundice

TFT: thyroid disease

Electrophoresis if from risk background

How well did you know this?
1
Not at all
2
3
4
5
Perfectly
158
Q

Cross-match for gastrectomy

A

4u

How well did you know this?
1
Not at all
2
3
4
5
Perfectly
159
Q

X-match for AAA

A

6u

How well did you know this?
1
Not at all
2
3
4
5
Perfectly
160
Q

How is cardiorespiratory function assessed pre-operatively

A

CXR: cardiorespiratory disease/symptoms, >65 y/o

Echo: poor LV function,?murmurs

ECG: HTN, Hx of cardiac disease, >55y

Cardiopulmonary exercise testing

PFT: known pulmonary disease or obesity

How well did you know this?
1
Not at all
2
3
4
5
Perfectly
161
Q

Why may you do an C spine XR pre-operatively

A

Flexion and extension views, in RA/AS

How well did you know this?
1
Not at all
2
3
4
5
Perfectly
162
Q

What are the different ASA grades

A

1: Normally healthy
2: Mild systemic disease
3: Severe systemic disease that limits activity
4: Systemic disease which is a constant threat to life
5: Moribund, not expected to survive 24h even with operation

How well did you know this?
1
Not at all
2
3
4
5
Perfectly
163
Q

NBM pre-op

A

>2h for clear fluids

>6h for solids

How well did you know this?
1
Not at all
2
3
4
5
Perfectly
164
Q

Bowel prep indications

A

May be needed in left-sided operations

Not usually needed in right-sided procedures

Necessity is controversial

How well did you know this?
1
Not at all
2
3
4
5
Perfectly
165
Q

What are the risks of bowel prep

A

Liquid bowel contents spilling during Sx

Electrolyte disturbance

Dehydration

Increased rate of post-op anastamotic leak

How well did you know this?
1
Not at all
2
3
4
5
Perfectly
166
Q

Indications for prophylactic Abx

A

Gi Sx

Joint replacement

How well did you know this?
1
Not at all
2
3
4
5
Perfectly
167
Q

Elective GI Sx post-op infection rate

A

20%

How well did you know this?
1
Not at all
2
3
4
5
Perfectly
168
Q

When are prophylactic Abx given preoperatively

A

15-60 mins before Sx

How well did you know this?
1
Not at all
2
3
4
5
Perfectly
169
Q

What are some bowel prep options

A

Picloax: picosulfate and Mg citrate

Klean-prep: macrogol

How well did you know this?
1
Not at all
2
3
4
5
Perfectly
170
Q

Abx prophlyaxis for

Biliary sx

A

Cef 1.5g and met 500mg IV

How well did you know this?
1
Not at all
2
3
4
5
Perfectly
171
Q

Abx prophlyaxis for

CR or appendicetomy

A

Cef and Met TDS

How well did you know this?
1
Not at all
2
3
4
5
Perfectly
172
Q

Abx prophlyaxis for

Vascular Sx

A

Co-amoxiclav 1.2g IV TDS

How well did you know this?
1
Not at all
2
3
4
5
Perfectly
173
Q

DVT prophylaxis in surgery

A

Low risk: early mobilisation

Medium risk: early mobilisation + TEDs + 20mg enoxaparin

High: early mobilisation + TEDS + 40mg enoxaparin and intermittent compression boots perioperatievely

How well did you know this?
1
Not at all
2
3
4
5
Perfectly
174
Q

Risk of DM in surgery

A

Increased risk of post-operative complications

How well did you know this?
1
Not at all
2
3
4
5
Perfectly
175
Q

How does DM increase the risk of post-operative complications?

A

Sx leads to the release of stress hormones which antagonise insulin

Patients are NBM

Increased risk of infection

Coexistant IHD and PVD

How well did you know this?
1
Not at all
2
3
4
5
Perfectly
176
Q

What should be done in DM patients pre-operatively?

A

Urine dipstick: proteinuria

Venous glucose

U+Es: K

How well did you know this?
1
Not at all
2
3
4
5
Perfectly
177
Q

What is the practical management of patients with IDDM when having surgery

A

Put patient first on list and inform surgeon and anaesthetist

Some centres prefer to use GKI infusions (mixture of glucose, insulin and potassium)

Sliding scale is not always necessary for minor ops, if in doubt, liase with diabetes specialist nurse

How well did you know this?
1
Not at all
2
3
4
5
Perfectly
178
Q

Management of insuilin pre and peri-operatively

A

Stop long acting insulin the night before

Omit AM insulin if sx if in the morning

Start sliding scale

How well did you know this?
1
Not at all
2
3
4
5
Perfectly
179
Q

Outline sliding scale

A

5% Dextroes with 20mmol KCl 125ml/hr

Infusion pump with c50u actrapid

Check CPG hourly and adjust insulin rate

How well did you know this?
1
Not at all
2
3
4
5
Perfectly
180
Q

What is target glucose when on a sliding scale

A

7-11mM

How well did you know this?
1
Not at all
2
3
4
5
Perfectly
181
Q

Post-op Mx of sliding scale

A

Continue sliding scale until tolerating food

Switch to SC regimen around meal

How well did you know this?
1
Not at all
2
3
4
5
Perfectly
182
Q

Mx of NIDDM and surgery

A

If glucose control poor (fasting glucose >10mM) treat as IDDM

Omit oral hypoglycaemics on the AM of Sx

Eating post-op: resume oral hypoglycaemics with meal

Not eating post-op: check fasting glucose on AM of surgery, start insulin sliding scale. Consult specialist team about restrting PO Rx

How well did you know this?
1
Not at all
2
3
4
5
Perfectly
183
Q

Mx of diet controlled DM in surgery

A

Usually no problem

Patient may be briefly insulin-dependant post-op

Monitor CPGs

How well did you know this?
1
Not at all
2
3
4
5
Perfectly
184
Q

Risks of steroids and surgery

A

Poor wound healing

Infection

Adrenal crisis

How well did you know this?
1
Not at all
2
3
4
5
Perfectly
185
Q

Mx of LT steroids and surgery

A

Need to increase steroids to cope with stress

Consider cover if high-dose steroids have been received within the past year

How well did you know this?
1
Not at all
2
3
4
5
Perfectly
186
Q

Steroid Rx in major surgery

A

Hydrocortisone 50-100mg IV with pre-med then 6-8 hourly for 3d

Usual pre-operative steroids +25 mg hydrocortisone @ induction +100 mg day-1 for 48 – 72h

How well did you know this?
1
Not at all
2
3
4
5
Perfectly
187
Q

Steroid Rx in minor surgery

A

As for major but only for 24h

How well did you know this?
1
Not at all
2
3
4
5
Perfectly
188
Q

Jaundice and surgery

A

Best to avoid operating

Use ERCP instead

How well did you know this?
1
Not at all
2
3
4
5
Perfectly
189
Q

Risks of jaundice and surgery

A

Patients with obstructive jaundice have increased risk of post-op renal failure, need to maintain a good UO

Coagulopathy

Increased infection risk—> cholangitis

How well did you know this?
1
Not at all
2
3
4
5
Perfectly
190
Q

Pre-Op Mx of jaundice

A

Avoid morphine in pre-med due to possible spastic effect on the sphincter of Oddi

Check clotting and consider pre-op vit K

Give 1L NS pre-op unless CCF-> moderate diuresis

Catheterise to monitor UO

Abx prophlyaxis

How well did you know this?
1
Not at all
2
3
4
5
Perfectly
191
Q

Intra-op Mx of jaundice

A

Hourly UO monitoring

NS titrated to output

How well did you know this?
1
Not at all
2
3
4
5
Perfectly
192
Q

Post-op mx of jaundice

A

Intensive monitoring of fluid status

Consider CVP and frusemide if poor output despite NS

How well did you know this?
1
Not at all
2
3
4
5
Perfectly
193
Q

Anticoagulated patients and surgery

A

Risk of haemorrhage vs risk of thrombosis

Consultant surgeon, anaesthetist and haemotologist

Very minor surgery can be undertaken without stopping warfarin if INR <3.5

Avoid spinal, epidural and regional blocks

In general continue aspirin/clopidogrel unless risk of bleeding is high, in which case stop 7d before surgery

How well did you know this?
1
Not at all
2
3
4
5
Perfectly
194
Q

Mx of anticoagulated with low-thromboembolic risk e.g. AF

A

Stop warfarin 5d pre-op, need INR <1.5

Restart next day

How well did you know this?
1
Not at all
2
3
4
5
Perfectly
195
Q

Mx of anticoagulated with high thromboembolic risk e.g. valves, recurrent VTE

A

Need bridging with LMWH

Stop warfaring 5d pre-op and start LMWH

Stop LMWH 12-18h pre-op

Restart LMWH 6h post-op

Restart warfarin next day

Stop LMWH when INR >2

How well did you know this?
1
Not at all
2
3
4
5
Perfectly
196
Q

Mx of anticoagulated patients requiring emergency surgery

A

Discontinue warfarin

Vit K 5mg slow IV

Request FFP or PCC (prothrombin complext concentrate) to cover surgery

How well did you know this?
1
Not at all
2
3
4
5
Perfectly
197
Q

Risks of COPD and surgery

A

Basal atelectasis

Aspiration

Chest infection

How well did you know this?
1
Not at all
2
3
4
5
Perfectly
198
Q

Pre-op Mx of smoking/COPD

A

CXR

PFTs

Physio

Quit smoking at least 4w prior to surgery

How well did you know this?
1
Not at all
2
3
4
5
Perfectly
199
Q

What are the aims of anaesthesia

A

Hypnosis, analgesia, muscle relaxation

How well did you know this?
1
Not at all
2
3
4
5
Perfectly
200
Q

Principles of anaesthesia

A

Induction

Muscle relaxation

Airway control

Maintenance

End of anaesthesia

How well did you know this?
1
Not at all
2
3
4
5
Perfectly
201
Q

Induction agent for anaesthesia

A

IV propofol

How well did you know this?
1
Not at all
2
3
4
5
Perfectly
202
Q

Muscle relaxation for anaesthesia

A

Depolarising: suxamethonium

Non-depolarising: vecuronium , atracurium

How well did you know this?
1
Not at all
2
3
4
5
Perfectly
203
Q

Airway control for anaesthesia

A

ET tube

LMA

How well did you know this?
1
Not at all
2
3
4
5
Perfectly
204
Q

Maintenance for anaesthesia

A

Usually volatile agent added to NO2/O2 mix e.g. halothan, enflurane

How well did you know this?
1
Not at all
2
3
4
5
Perfectly
205
Q

End of anaesthesia

A

Change inspired gas to 100% O2

Reverse paralysis: neostigmine and atropine (prevents muscarinic side effects)

How well did you know this?
1
Not at all
2
3
4
5
Perfectly
206
Q

Pre-medications for anaesthesia

7As

A

Anxiolytics and Amnesia: temazepam

Analgesics: opioids, NSAIDs, paracetamol

Anti-emetics: ondansteron 4mg/ metoclopramide 10mg

Antacids: lansoprazole

Anti-sialogue: glycopyrolate (reduces secretion)

Antibiotics

How well did you know this?
1
Not at all
2
3
4
5
Perfectly
207
Q

Features of regional anaesthesia

A

May be used for minor procedures or if unsuitable for GA

Nerve or spinal blocks

Use long-acting agents e.g. bupivacaine

How well did you know this?
1
Not at all
2
3
4
5
Perfectly
208
Q

What are the contraindications to nerve/spinal blocks

A

Local infection

Clotting abnormality

How well did you know this?
1
Not at all
2
3
4
5
Perfectly
209
Q

Complications of

Propofol induction

A

Cardiorespiratory depression

How well did you know this?
1
Not at all
2
3
4
5
Perfectly
210
Q

Complications of

Intubation

A

Oro-pharyngeal injury with laryngoscope

Oesophageal intubation

How well did you know this?
1
Not at all
2
3
4
5
Perfectly
211
Q

Complications of

Loss of pain sensation

A

Urinary retention

Pressure necrosis

Nerve palsies

How well did you know this?
1
Not at all
2
3
4
5
Perfectly
212
Q

Complications of

Loss of muscle power in anaesthesia

A

Corneal abrasion

No cough—> atelectasis + pneumonia

How well did you know this?
1
Not at all
2
3
4
5
Perfectly
213
Q

Complications of

Malignant hyperpyrexia

A

Rare complication precipitated by halothane or suxamethonium

AD inheritance

Rapid rise in temperature and masseter spasm

Rx: dantrolene and cooling

How well did you know this?
1
Not at all
2
3
4
5
Perfectly
214
Q

Complications of

Anaphylaxis in anaesthetics

A

Rare

Possible triggers:

antibiotics

colloid

NM blockers

How well did you know this?
1
Not at all
2
3
4
5
Perfectly
215
Q

What is the necessity of analgesia in surgery

A

Pain-> autonomic acitvation–> arteriolar constriction-> reduced wound perfusion–> impaired wound healing

Pain-> decreased mobilisation-> increased VTE and decreased function

Pain-> decreased respiratory excursion and decreased cough–> atelectasis and pneumonia

Humanitarian considerations

How well did you know this?
1
Not at all
2
3
4
5
Perfectly
216
Q

General guidance for anaesthesia

A

Give regular doses at fixed intervals

Consider best route: PO if possible

PCA (patient-controlled) should be considered: morphine, fentanyl

Follow stepwise approach

Liaise with acute pain service

How well did you know this?
1
Not at all
2
3
4
5
Perfectly
217
Q

Pre-op analgesia

A

Epidural e.g. bupivacaine

How well did you know this?
1
Not at all
2
3
4
5
Perfectly
218
Q

End-op analgesia

A

Infiltrate wound edge with LA

Infiltrate major regional nerves with LA

How well did you know this?
1
Not at all
2
3
4
5
Perfectly
219
Q

Features of spinal/epidural anaesthesia

A

Decreased SE as drugs more localised

First line for major bowel resection

Caution: respiratory depression, neurogenic shock (reduced BP)

How well did you know this?
1
Not at all
2
3
4
5
Perfectly
220
Q

What is the WHO pain ladder

A

Non-opiod +/- adjuvants

Weak opiod + non-opioid +/- adjuvants

Strong opiod + non-opioid +/- adjuvants

How well did you know this?
1
Not at all
2
3
4
5
Perfectly
221
Q

Non-opioid pain medications

A

Paracetamol

NSAIDs: ibuprofen, diclofenac

How well did you know this?
1
Not at all
2
3
4
5
Perfectly
222
Q

Weak opioid pain medications

A

Codeine

Dihydrocodeine

Tramadol

How well did you know this?
1
Not at all
2
3
4
5
Perfectly
223
Q

Strong opioid analgesia

A

Morphine

Oxycodone

Fentanyl

How well did you know this?
1
Not at all
2
3
4
5
Perfectly
224
Q

What is ERAS

A

Enhanced recovery after surgery

Commonly employed in colorectal and orthopaedic surgery

How well did you know this?
1
Not at all
2
3
4
5
Perfectly
225
Q

Aims of EPAR

A

Optimisie pre-op preparation for surgery

Avoid iatrogenic problems e.g. ileus

Minimise adverse physiological/immunological responses to surgery: (raised cortisol, reduced insulin) (hypercoagulbility) (immunosuppression)

Increase speed of recover and return to function

Recognise abnormal recovery and allow early intervention

How well did you know this?
1
Not at all
2
3
4
5
Perfectly
226
Q

Pre-op approach EPAR

A

Optimisation:

Aggressive physiological optimisation e.g. Hydration, BP, anaemia, DM, co-morbidities

Smoking cessation >4w before surgery

Admission on day of surgery, avoidance of prolonged fast

Carb loading prior to surgery

Fully informed patient encouraged to participate in recovery

How well did you know this?
1
Not at all
2
3
4
5
Perfectly
227
Q

Intra-op approach EPAR

A

Short-acting anaesthetic agents

Epidural use

Minimally invasive technqiues

Avoid drains and NGTs where possible

How well did you know this?
1
Not at all
2
3
4
5
Perfectly
228
Q

Post-op approach to EPAR

A

Aggressive Rx of pain and nausea

Early mobilisation and PT

Early resumption of oral intake

Early discontinuation of IV fluids

Remove drains and urinary catheters ASAP

How well did you know this?
1
Not at all
2
3
4
5
Perfectly
229
Q

How can surgical complications be characterised

A

Immediate (<24h)

Early (1d-1m)

Late (>1m)

How well did you know this?
1
Not at all
2
3
4
5
Perfectly
230
Q

What are the immediate surgical complications

A

<24h

Intubation leading to oropharyngeal trauma

Surgical trauma to local structures

Primary or reactive haemorrhage

How well did you know this?
1
Not at all
2
3
4
5
Perfectly
231
Q

What are the early surgical complications

A

1d-1m

Secondary haemorrhage

VTE

Urinary retention

Atelectasis and pneumonia

Wound infection and dehiscence

Antibitoic associated colitis

How well did you know this?
1
Not at all
2
3
4
5
Perfectly
232
Q

What are the late surgical complications

A

>1m

Scarring

Neuropathy

Failure or recurrence

How well did you know this?
1
Not at all
2
3
4
5
Perfectly
233
Q

How can post-op haemorrhage be classified?

A

Primary

Reactive

Secondary

How well did you know this?
1
Not at all
2
3
4
5
Perfectly
234
Q

What is primary haemorrhage

A

Continuous bleeding starting during surgery

How well did you know this?
1
Not at all
2
3
4
5
Perfectly
235
Q

What is reactive bleeding

A

Bleeding at the end of surgery or early post-op

2o to increased CO and BP

How well did you know this?
1
Not at all
2
3
4
5
Perfectly
236
Q

What is secondary bleeding

A

Bleeding >24h post op

Usually due to infection

How well did you know this?
1
Not at all
2
3
4
5
Perfectly
237
Q

What are the causes of post-op urinary retention?

A

Drugs: opioids, epidural/spinal, anti-AChM

Pain: sympathetic activation leading to sphincter contraction

Psychogenic: hospital environment

How well did you know this?
1
Not at all
2
3
4
5
Perfectly
238
Q

What are the risk factors for post-op urinary retention?

A

Male

Increasing age

Neuropathy e.g. DM, EtOH

BPH

Surgery type: hernia and anorectal

How well did you know this?
1
Not at all
2
3
4
5
Perfectly
239
Q

Mx of post-op urinary retention

A

Conservative:

Privacy, ambulation, void to running taps or in hot bath, analgesia

Catheterise +/- gent 2.5mg/kg IV stat

TWOC: if failed may be sent home with silicone catheter and urology output f/up

How well did you know this?
1
Not at all
2
3
4
5
Perfectly
240
Q

Features of pulmonary atelectasis

A

Occurs after nearly every GA

Mucus plugging + absorption of distal air—> collapse

How well did you know this?
1
Not at all
2
3
4
5
Perfectly
241
Q

What are the causes of pulmonary atelectasis

A

Pre-op smoking

Anaesthetic: increases mucus production, reduced mucociliary clearance

Pain inhibitng respiratory excursion and cough

How well did you know this?
1
Not at all
2
3
4
5
Perfectly
242
Q

First 48 hours post-sx

Mild pyrexia

Dyspnoea

Dull bases with reduced air entry

A

?Pulmonary atelectasis

How well did you know this?
1
Not at all
2
3
4
5
Perfectly
243
Q

Mx of pulmonary atelectasis

A

Analgesia to aid coughing

Chest PT

How well did you know this?
1
Not at all
2
3
4
5
Perfectly
244
Q

Features of wound infection

A

5-7d post-op

S. aureus and coliforms

How well did you know this?
1
Not at all
2
3
4
5
Perfectly
245
Q

How can operations be classified

A

Clean

Clean/cont

Contaminated

Dirty

How well did you know this?
1
Not at all
2
3
4
5
Perfectly
246
Q

What is a clean operation

A

Incision of uninfected skin without opening viscus

How well did you know this?
1
Not at all
2
3
4
5
Perfectly
247
Q

What is a clean/cont operation

A

Intraoperative beach of viscus (not colon)

How well did you know this?
1
Not at all
2
3
4
5
Perfectly
248
Q

What is a contaminated operation?

A

Breach of viscus and spillage or opening of colon

How well did you know this?
1
Not at all
2
3
4
5
Perfectly
249
Q

What is a dirty operation

A

Site already contaminated- faeces, urine, trauma

How well did you know this?
1
Not at all
2
3
4
5
Perfectly
250
Q

How can the risk factors for post-op wound infection be classified?

A

Pre-operative

Operative

Post-operative

How well did you know this?
1
Not at all
2
3
4
5
Perfectly
251
Q

What are the pre-op risk factors for wound infection

A

Increasing age

Comorbidities e.g. DM

Pre-existing infection e.g. appendix perf

Patient colonisation e.g. MRSA

How well did you know this?
1
Not at all
2
3
4
5
Perfectly
252
Q

What are the operative risk factors for wound infection?

A

Op classification and wound infection risk

Duration

Technical: pre-op Abx, asepsis

How well did you know this?
1
Not at all
2
3
4
5
Perfectly
253
Q

What are the post-operative risk factors for wound infection

A

Contamination to wound from staff

How well did you know this?
1
Not at all
2
3
4
5
Perfectly
254
Q

Mx of wound infection

A

Regular wound dressing

Abx

Abscess drainage

How well did you know this?
1
Not at all
2
3
4
5
Perfectly
255
Q

Occurs 10d post-op

Preceded by serosanguinous discharge from wound

A

Wound dehiscence

How well did you know this?
1
Not at all
2
3
4
5
Perfectly
256
Q

How can the risk factors for wound dehiscence be classified?

A

Pre-op

Op

Post-op

How well did you know this?
1
Not at all
2
3
4
5
Perfectly
257
Q

What are the pre-op risk factors for wound dehiscence?

A

Increasing age

Smoking

Obesity, malnutrition, cachexia

Comorbidities e.g. DM, uraemia, chronic cough, Cancer

Drugs: steroids, chemo, RTx

How well did you know this?
1
Not at all
2
3
4
5
Perfectly
258
Q

What are the operative risk factors for wound dehiscence?

A

Length and orientation of incision

Closure technique: follow Jenkin’s rule

Suture material

How well did you know this?
1
Not at all
2
3
4
5
Perfectly
259
Q

What is Jenkin’s rule?

A

It is a rule for closure of the abdominal wound. It states that for a continuous suture, the length of suture used should be at least four times the length of the wound with sutures 1cm apart and with 1cm bites of the wound edge

(More recent research has shown that the optimal ratio may in fact be 6:1)

How well did you know this?
1
Not at all
2
3
4
5
Perfectly
260
Q

What are the post-operative risk factors for wound dehiscence?

A

Increased IAP e.g. prolonged ileus leading to distension

Infection

Haematoma/seroma formation

How well did you know this?
1
Not at all
2
3
4
5
Perfectly
261
Q

Mx of wound dehiscence

A

Replace abdo contents and cover with sterile soaked gauze

IV Abx: cef and met

Opioid analgesia

Call senior and arrange theatre

Repair:

wash bowel, debride wound edge, close with deep non-absorbable sutures e.g. nylon

May require VAC dressing or grafting

How well did you know this?
1
Not at all
2
3
4
5
Perfectly
262
Q

What are the Cxs of cholecystectomy

A

Conversion to open: 5%

CBD injury: 0.3%

Bile leak

Retained stones needing ERCP

Fat intolerance/ loose stools

How well did you know this?
1
Not at all
2
3
4
5
Perfectly
263
Q

What are the early complications of inguinal hernia repair?

A

Haematoma/seroma formation: 10%

Intra-abdominal injury (laparoscopic)

Infection

Urinary retention

How well did you know this?
1
Not at all
2
3
4
5
Perfectly
264
Q

What are the late complications of inguinal hernia repair?

A

Recurrence (<2%)

Ischaemic orchitis (0.5%)

Chronic groin pain/paraesthesia (5%)

How well did you know this?
1
Not at all
2
3
4
5
Perfectly
265
Q

What are the complications of appendicectomy

A

Abscess formation

Fallopian tube trauma

Right hemicolectomy e.g. for carcinoid, caecal necrosis

How well did you know this?
1
Not at all
2
3
4
5
Perfectly
266
Q

What are the early complications of colonic surgery?

A

Ileus

AAC (acute acalculous cholecystitis)

Anastomotic leak

Enterocutaneous fistulae

Abdominal or pelvic abscess

How well did you know this?
1
Not at all
2
3
4
5
Perfectly
267
Q

What are the late complications of colonic surgery?

A

Adhesions leading to obstruction

Incisional hernia

How well did you know this?
1
Not at all
2
3
4
5
Perfectly
268
Q

What are the causes of post-op ileus?

A

Bowel handling

Anaesthesia

Electrolyte imbalance

How well did you know this?
1
Not at all
2
3
4
5
Perfectly
269
Q

Post surgery:

Distension

Constipation +/- vomiting

Absent bowel sounds

A

?post-op ileus

How well did you know this?
1
Not at all
2
3
4
5
Perfectly
270
Q

Mx of post-op ileus

A

IV fluids and NGT

TPN if prolonged

How well did you know this?
1
Not at all
2
3
4
5
Perfectly
271
Q

What are the complications of anorectal surgery?

A

Anal incontinence

Stenosis

Anal fissure

How well did you know this?
1
Not at all
2
3
4
5
Perfectly
272
Q

What are the complications of small bowel surgery?

A

Short gut syndrome (<250cm)

How well did you know this?
1
Not at all
2
3
4
5
Perfectly
273
Q

Def: short gut syndrome

A

Malabsorption disorder caused by the surgical removal of a large portion of the small intestine

How well did you know this?
1
Not at all
2
3
4
5
Perfectly
274
Q

Abdominal pain

Diarrhea and steatorrhea (oily or sticky stool, which can be malodorous)

Fluid depletion

Weight loss and malnutrition

Fatigue

A

?Small gut syndrome

May also have complications caused by malabsorption in vitamin absoprtion e.g. anaemia, hyperkeratosis, easy brusing, muscle spasms, poor blood clotting and bone pain

How well did you know this?
1
Not at all
2
3
4
5
Perfectly
275
Q

What are the complications of splenectomy

A

Gastric dilatation 2o to gastric ileus: prevent with NGT

Thrombocytosis-> VTE

Infection: encapsulated organisms

How well did you know this?
1
Not at all
2
3
4
5
Perfectly
276
Q

What are the complications of arterial surgery

A

Thrombosis and embolisation

Anastomotic leak

Graft infection

How well did you know this?
1
Not at all
2
3
4
5
Perfectly
277
Q

What are the complications of aortic surgery?

A

Gut ischaemia

Renal failure

Aorto-enteric fistula

Anterior spinal syndrome

Emboli: distal ischaemia–> trash foot

How well did you know this?
1
Not at all
2
3
4
5
Perfectly
278
Q

Complications of breast surgery

A

Arm lymphoedema

Skin necrosis

Seroma

How well did you know this?
1
Not at all
2
3
4
5
Perfectly
279
Q

What are the complications of urological surgery?

A

Sepsis

Uroma: extravasation or urine

How well did you know this?
1
Not at all
2
3
4
5
Perfectly
280
Q

What are the complications of prostatectomy?

A

Urinary incontinence

Erectile dysfunction

Retrograde ejaculation

Prostatitis

How well did you know this?
1
Not at all
2
3
4
5
Perfectly
281
Q

What are the complications of thyroidectomy?

A

Wound haematoma–> tracheal obstruction

Recurrent laryngeal nerve trauma-> hoarse voice

  • transient in 1.5%
  • permanent in 0.5%

Damage to R nerve more common due to it being more medial

Hypoparathyroidism—> hypocalcaemia

Thyroid storm

Hypothyroidism

How well did you know this?
1
Not at all
2
3
4
5
Perfectly
282
Q

What are the complications of tracheostomy?

A

Stenosis

Mediastinitis

Surgical emphysema

How well did you know this?
1
Not at all
2
3
4
5
Perfectly
283
Q

What are the complications of # repair?

A

Mal/non-union

Osteomyelitis

AVN

Compartment syndrome

How well did you know this?
1
Not at all
2
3
4
5
Perfectly
284
Q

What are the complications of hip replacement?

A

Deep infection

VTE

Dislocation

Nerve injury: sciatic, SGN

Leg length discrepancy

How well did you know this?
1
Not at all
2
3
4
5
Perfectly
285
Q

What are the complications of cardiothoracic surgery

A

Pneumo/haemothorax

Infection: mediastinitis, empyema

How well did you know this?
1
Not at all
2
3
4
5
Perfectly
286
Q

How can post-op pyrexia be classified?

A

Early 0-5d

Delayed >5d

How well did you know this?
1
Not at all
2
3
4
5
Perfectly
287
Q

What are the early causes of post-op pyrexia

A

0-5d post-op

Blood transfusion

Physiological: SIRS from trauma, 0-1d

Pulmonary atelectasis: 24-48hr

Infection: UTI, superficial thrombophlebitis, cellulitis

Drug reaction

How well did you know this?
1
Not at all
2
3
4
5
Perfectly
288
Q

What are the delayed causes of post-op pyreixa

A

>5d post-op

Pneumonia

VTE: 5-10d

Wound infection: 5-7d

Anastomotic leak: 7d

Collection: 5-20d

How well did you know this?
1
Not at all
2
3
4
5
Perfectly
289
Q

Examination work up in post-op pyrexia

A

Obs, notes and drug chart

Wound

Abdo + DRE

Legs

Chest

Lines

Urine

Stool

How well did you know this?
1
Not at all
2
3
4
5
Perfectly
290
Q

What are the Ix in post-op pyrexia?

A

Urine: dip, MC+S

Bloods: FBC, CRP, cultures +/- LFTs

Cultures: wound swabs, CVP tip for culture

CXR

How well did you know this?
1
Not at all
2
3
4
5
Perfectly
291
Q

Cause of post-op pneumonia

A

Anaesthesia-> atelectasis

Pain-> reduced cough

Sx-> immunosuppression

How well did you know this?
1
Not at all
2
3
4
5
Perfectly
292
Q

Rx of post-op pneumonia

A

Chest physio: encourage coughing

Good analgesia

Abx

How well did you know this?
1
Not at all
2
3
4
5
Perfectly
293
Q

Malaise

Swinging fever, rigors

Localised peritonitis

Shoulder tip pain (if subphrenic)

Post-op

A

?Collection

How well did you know this?
1
Not at all
2
3
4
5
Perfectly
294
Q

What are the locations of collections?

A

Pelvic: 4-10d post-op

Subphrenic: present 7-21d post-op

Paracolic gutters

Lesser sac

Hepatorenal recess (Morrison’s space)

Small bowel (interloop spaces)

How well did you know this?
1
Not at all
2
3
4
5
Perfectly
295
Q

What is Morrison’s space

A

Hepatorenal recess

How well did you know this?
1
Not at all
2
3
4
5
Perfectly
296
Q

Ix in collection

A

FBC, CRP, cultures

USS, CT

Diagnostic lap

How well did you know this?
1
Not at all
2
3
4
5
Perfectly
297
Q

Rx of post-op collection

A

Abx

Drainage/washout

How well did you know this?
1
Not at all
2
3
4
5
Perfectly
298
Q

Def: cellulitis

A

Acute infection of subcutaneous connective tissue

How well did you know this?
1
Not at all
2
3
4
5
Perfectly
299
Q

Cause of cellulitis:

A

Beta haemolytic strep and staph aureus

How well did you know this?
1
Not at all
2
3
4
5
Perfectly
300
Q

Pain, swelling, erythema and warmth

Systemic upset

+/- lymphadenopathy

A

Cellulitis

How well did you know this?
1
Not at all
2
3
4
5
Perfectly
301
Q

Rx of cellulitis

A

Benpen IV

Pen V and fluclox PO

How well did you know this?
1
Not at all
2
3
4
5
Perfectly
302
Q

Def: diverticulum

A

Out-pouching of tubular structure

How well did you know this?
1
Not at all
2
3
4
5
Perfectly
303
Q

True diverticulum

A

Composed of complete wall e.g. Meckel’s

How well did you know this?
1
Not at all
2
3
4
5
Perfectly
304
Q

Def: false diverticulum

A

Composed of mucosa only (pharyngeal, colonic)

How well did you know this?
1
Not at all
2
3
4
5
Perfectly
305
Q

Diverticular disease

A

Symptomatic diverticulosis

How well did you know this?
1
Not at all
2
3
4
5
Perfectly
306
Q

Def: diverticulitis

A

Inflammation of diverticula

How well did you know this?
1
Not at all
2
3
4
5
Perfectly
307
Q

Epidemiology of diverticular disease

A

30% have diverticulosis by 60 years

F>M

How well did you know this?
1
Not at all
2
3
4
5
Perfectly
308
Q

Pathophysiology of diverticular disease

A

Associated with increased intraluminal pressure: low fibre diet has no osmotic effect to keep stool wet

Mucosa herniates through muscularis propria at points of weakness i.e. where perforating arteries enter

Most commonly located in the sigmoid colon

How well did you know this?
1
Not at all
2
3
4
5
Perfectly
309
Q

What is Saint’s triad

Unifying factor?

A

Hiatus hernia

Cholelithiasis

Diverticular disease

?Obesity

How well did you know this?
1
Not at all
2
3
4
5
Perfectly
310
Q

Symptoms of diverticular disease

A

Altered bowel habit +/- left sided colic, relieved by defecation

Nausea

Flatulance

How well did you know this?
1
Not at all
2
3
4
5
Perfectly
311
Q

Altered bowel habit +/- left sided colic, relieved by defecation

Nausea

Flatulance

A

?Diverticular disease

How well did you know this?
1
Not at all
2
3
4
5
Perfectly
312
Q

Rx for diverticular disease

A

High fibre diet

Mebeverine may help

Elective resection for chronic pain

How well did you know this?
1
Not at all
2
3
4
5
Perfectly
313
Q

Pathophysiology of diverticulitis

A

Inspissated faeces-> obstruction of diverticulum

Think in elderly patient with previous history of constipation

How well did you know this?
1
Not at all
2
3
4
5
Perfectly
314
Q

Presentation of diverticulitis

A

Abdominal pain and tenderness

Typically LIF

Localised peritonitis

Pyrexia

How well did you know this?
1
Not at all
2
3
4
5
Perfectly
315
Q

Abdominal pain and tenderness

Typically LIF

Localised peritonitis

Pyrexia

A

Diverticulitis

How well did you know this?
1
Not at all
2
3
4
5
Perfectly
316
Q

Ix in diverticulitis: bloods

A

FBC

Raised WCC

Raised CRP/ESR

Amylase

G+S/x-match

How well did you know this?
1
Not at all
2
3
4
5
Perfectly
317
Q

Ix in diverticulitis: imaging

A

Erect CXR: ?perforation

AXR: fluid level/air in bowel wall

Contrast CT

Gastrograffin enema

How well did you know this?
1
Not at all
2
3
4
5
Perfectly
318
Q

Ix in diverticulitis: endoscopy

A

Flexi sig

Colonscopy (not used in acute attack)

How well did you know this?
1
Not at all
2
3
4
5
Perfectly
319
Q

What can be used to grade diverticulitis perforation

A

Hinchey grading

How well did you know this?
1
Not at all
2
3
4
5
Perfectly
320
Q

Hinchey 1

A

Localised para-colonic abscess- surgery rarely needed

How well did you know this?
1
Not at all
2
3
4
5
Perfectly
321
Q

Hinchey 2

A

Large abscess extending into pelvis

May resolve without surgery

How well did you know this?
1
Not at all
2
3
4
5
Perfectly
322
Q

Hinchey 3

A

Generalised purulent peritonitis

Surgery needed

How well did you know this?
1
Not at all
2
3
4
5
Perfectly
323
Q

Hinchey 4

A

Generalised faecal peritonitis

Surgery needed

How well did you know this?
1
Not at all
2
3
4
5
Perfectly
324
Q

Mx of mild acute diverticulitis

A

Can be treated at home with bowel rest (fluids only)

and abx- some conflicting evidence about use of abx in uncomplicated diverticular disease.. (augmentin +/- met)

How well did you know this?
1
Not at all
2
3
4
5
Perfectly
325
Q

Indications for admission acute diverticulitis

A

Unwell

Fluids can’t be tolerated

Pain can’t be controlled

How well did you know this?
1
Not at all
2
3
4
5
Perfectly
326
Q

Medical Mx of acute diverticulitis

A

NBM

IV fluids

Analgesia: paracetamol

Antibiotics: cef and met

Most cases settle

How well did you know this?
1
Not at all
2
3
4
5
Perfectly
327
Q

NSAIDs and opioid analgesics in diverticulitis

A

Have been identified as risk factors for perforation

How well did you know this?
1
Not at all
2
3
4
5
Perfectly
328
Q

Indications for surgical management of diverticulitis

A

Perforation

Large haemorrhage

Stricture leading to obstruction

How well did you know this?
1
Not at all
2
3
4
5
Perfectly
329
Q

Surgical procedure used in diverticulitis

A

Harmann’s to resect diseased bowel

How well did you know this?
1
Not at all
2
3
4
5
Perfectly
330
Q

Complications of diverticulitis

A

Perforation

Haemorrhage

Abscess

Fistulae

Strictures

How well did you know this?
1
Not at all
2
3
4
5
Perfectly
331
Q

Sudden onset pain

Generalised peritonitis and shock

+/- preceding diverticulitis

A

?Perforation

How well did you know this?
1
Not at all
2
3
4
5
Perfectly
332
Q

Mx of perforation in diverticulitis

A

CXR: free air under the diaphragm

Rx: Hartmann’s

How well did you know this?
1
Not at all
2
3
4
5
Perfectly
333
Q

Suddeen painless, bright red PR bleed

Following diverticulitis

A

?Haemorrhage

How well did you know this?
1
Not at all
2
3
4
5
Perfectly
334
Q

Ix in ?haemorrhage in diverticulitis

A

Mesenteric angiography or colonoscopy

How well did you know this?
1
Not at all
2
3
4
5
Perfectly
335
Q

Mx of haemorrhage in diverticulitis

A

Usually stops spontaneously

May need transfusion

Colonoscopy +/- diathermy/adrenaline

Embolisation

Resection

How well did you know this?
1
Not at all
2
3
4
5
Perfectly
336
Q

Walled off perforation

Swinging fever

Localising signs e.g. boggy rectal mass

Leukocytosis

Hx of diverticulitis

A

?Abscess

How well did you know this?
1
Not at all
2
3
4
5
Perfectly
337
Q

Mx of diverticulitis abscess

A

Abx + CT/US guided drainage

How well did you know this?
1
Not at all
2
3
4
5
Perfectly
338
Q

Types of of fistulae following diverticulitis

A

Enterocolic

Colovaginal

Colovesicular

How well did you know this?
1
Not at all
2
3
4
5
Perfectly
339
Q

Pneumaturia + intractable UTIs

A

?Colovesciular fistula

How well did you know this?
1
Not at all
2
3
4
5
Perfectly
340
Q

What may occur after diverticulitis

A

Colon may heal with fibrous strictures

How well did you know this?
1
Not at all
2
3
4
5
Perfectly
341
Q

Mx of strictures post diverticulitis

A

Resection (usually with primary anastomosis)

Stenting

How well did you know this?
1
Not at all
2
3
4
5
Perfectly
342
Q

How can bowel obstruction be classified

A

Simple

Closed loop

Strangulated

How well did you know this?
1
Not at all
2
3
4
5
Perfectly
343
Q

Simple bowel obstruction

A

1 obstructing point + no vascular compromise

May be partial or complete

How well did you know this?
1
Not at all
2
3
4
5
Perfectly
344
Q

Closed-loop bowel obstruction

A

Bowel obstructed at two points

Left CRC with competent ileocaecal valve. Volvulus

Gross distension can lead to perforation

How well did you know this?
1
Not at all
2
3
4
5
Perfectly
345
Q

Strangulated bowel obstruction

A

Compromised blood supply

Localised, constant pain + peritonism

Fever and raised WCC

How well did you know this?
1
Not at all
2
3
4
5
Perfectly
346
Q

What are the commonest causes of SBO

A

Adhesions: 60%

Hernia

How well did you know this?
1
Not at all
2
3
4
5
Perfectly
347
Q

What are the commonest causes of LBO

A

Colorectal neoplasia (60%)

Diverticular stricture (20%)

Volvulus 5%

How well did you know this?
1
Not at all
2
3
4
5
Perfectly
348
Q

How can bowel obstruction be classified

A

Non-mechanical

Mechanical

How well did you know this?
1
Not at all
2
3
4
5
Perfectly
349
Q

What is non-mechanical bowel obstruction?

A

Paralytic ileus

How well did you know this?
1
Not at all
2
3
4
5
Perfectly
350
Q

Causes of paralytic ileus

A

Post-op

Peritonitis

Pancreatitis or any localised inflammation

Posions/drugs: anti-AChM

Pseudo-obstruction

Metabolic: hypokalaemia, hyponatraemia, hypomagnesia, uraemia

Mesenteric ischaemia

How well did you know this?
1
Not at all
2
3
4
5
Perfectly
351
Q

Intestinal pseudo-obstruction

A

Intestinal pseudo-obstruction is a clinical syndrome caused by severe impairment in the ability of the intestines to push food through. It is characterized by the signs and symptoms of intestinal obstruction without any lesion in the intestinal lumen.[1] Clinical features can include abdominal pain, nausea, severe distension, vomiting, dysphagia, diarrhea and constipation, depending upon the part of the gastrointestinal tract involved.[2] The condition can begin at any age and it can be a primary condition (idiopathic or inherited) or caused by another disease (secondary).[3]

It can be chronic[4] or acute.[5]

How well did you know this?
1
Not at all
2
3
4
5
Perfectly
352
Q

How can mechanical bowel obstruction be classified?

A

Intraluminal

Intramural

Extramural

How well did you know this?
1
Not at all
2
3
4
5
Perfectly
353
Q

What are the intraluminal causes of bowel obstruction

A

Impacted matter: faeces, worms, bezoars

Intussuception

Gallstones

How well did you know this?
1
Not at all
2
3
4
5
Perfectly
354
Q

What are the intramural causes of mechanical obstruction?

A

Benign stricutre: IBD, Sx, ischaemic colitis, diverticulitis, RTx

Neoplasia

Congenital atresia

How well did you know this?
1
Not at all
2
3
4
5
Perfectly
355
Q

What are the extramural causes of bowel obstruction

A

Hernia

Adhesions

Volvulus (sigmoid, caecal, gastric)

Extrinsic compression: pseudocyst, abscess, haematoma, tumour e.g. ovarian, congenital bands e.g. Ladds

How well did you know this?
1
Not at all
2
3
4
5
Perfectly
356
Q

Ladd’s bands

A

Ladd’s bands, sometimes called bands of Ladd, are fibrous stalks of peritoneal tissue that attach the cecum to the abdominal wall and create an obstruction of the duodenum. This condition is found in malrotation of the intestine.

How well did you know this?
1
Not at all
2
3
4
5
Perfectly
357
Q

Presentation of bowel obstruction

A

Abdominal pain:

Colicky, central but level depends on gut region, constant/localised pain suggest strangulation or impending perforation

Distension:

with lower bowel involvement

Vomiting:

early in high, late or absent in low

Absolute constipation

How well did you know this?
1
Not at all
2
3
4
5
Perfectly
358
Q

Abdominal pain:

Colicky, central but level depends on gut region, constant/localised pain suggest strangulation or impending perforation

Distension:

with lower bowel involvement

Vomiting:

early in high, late or absent in low

Absolute constipation

A

?Bowel obstruction

How well did you know this?
1
Not at all
2
3
4
5
Perfectly
359
Q

Absolute constipation

A

Of flatus and faeces

How well did you know this?
1
Not at all
2
3
4
5
Perfectly
360
Q

Examination findings in bowel obstruction

A

Tachycardia: hypovolaemia, strangulation
Dehydration, hypovolaemia

Fever: suggests inflammatory disease or strangulation

Surgical scars

Hernias

Mass: neoplastic or inflammatory

Bowel sounds: increased in mechanical obstruction, reduced in ileus

PR: empty rectum, rectal mass, hard impacted stool, blood from higher pathology

How well did you know this?
1
Not at all
2
3
4
5
Perfectly
361
Q

Bowel obstruction, increased bowel sounds

A

Mechanical obstruction

How well did you know this?
1
Not at all
2
3
4
5
Perfectly
362
Q

Bowel obstruction, reduced bowel sounds

A

Ileus

How well did you know this?
1
Not at all
2
3
4
5
Perfectly
363
Q

Ix in bowel obstruction: bloods

A

FBC: raised WCC

U+Es: dehydration, electrolyte abnormalities

Amylase: ++ in strangulation/perforation

VBG: raised lactate in strangulation

G+S, clotting: may need Sx

How well did you know this?
1
Not at all
2
3
4
5
Perfectly
364
Q

Ix in bowel obstruction: imaging

A

Erect CXR

AXR +/- erect film for fluid levels

CT: can show transition point

How well did you know this?
1
Not at all
2
3
4
5
Perfectly
365
Q

Ix in bowel obstruction: gastrograffin studies

A

Look for mechanical obstruction: no free flow

Follow through or enema

Follow through may relive mild mechanical obstruction, usually adhesional

How well did you know this?
1
Not at all
2
3
4
5
Perfectly
366
Q

Ix in bowel obstruction: colonoscopy

A

Can be used in some cases

Risk of perforation

May be used to therapeutically stent

How well did you know this?
1
Not at all
2
3
4
5
Perfectly
367
Q

AXR findings in ileus

A

Both small and large bowel may be visible

No clear transition point

How well did you know this?
1
Not at all
2
3
4
5
Perfectly
368
Q

SBO AXR findings

A

Diameter >3

Central

Valvulae coniventes: completely across

LB gas absent

Many loops

Many, short fluid levels

How well did you know this?
1
Not at all
2
3
4
5
Perfectly
369
Q
A
How well did you know this?
1
Not at all
2
3
4
5
Perfectly
370
Q

LBO AXR findings

A

>6cm (caecum >9cm)

Peripheral loccation

Haustra- partially across

LB gas present in rectum

Few loops

Few, long fluid levels

How well did you know this?
1
Not at all
2
3
4
5
Perfectly
371
Q
A
How well did you know this?
1
Not at all
2
3
4
5
Perfectly
372
Q
A
How well did you know this?
1
Not at all
2
3
4
5
Perfectly
373
Q

Def: Meckel’s diverticulum

A

Ileal remnant of vitellointestinal duct

Joins yoke sac to midgut lumen

How well did you know this?
1
Not at all
2
3
4
5
Perfectly
374
Q

Features of Meckel’s diverticulum

A

A true diverticulum

2 inches long

2ft from ileocaecal vavle on antimesenteric border

2% of population

2% symptomatic

Contain ectopic gastric or pancreatic tissue

How well did you know this?
1
Not at all
2
3
4
5
Perfectly
375
Q

Symptomatic Meckel’s presentation

A

Rectal bleeding from gastric mucosa

Diverticulitis mimicking appendicitis

Intussuception

Volvulus

Malignant change: adenocarcinoma

Raspberry tumour: musoca protruding at umbilicius: vitello-intestinal fistula

Littre’s hernia

How well did you know this?
1
Not at all
2
3
4
5
Perfectly
376
Q

Rectal bleeding from gastric mucosa

Diverticulitis mimicking appendicitis

Intussuception

Volvulus

Malignant change: adenocarcinoma

Raspberry tumour: musoca protruding at umbilicius: vitello-intestinal fistula

Littre’s hernia

A

Meckels diverticulum

How well did you know this?
1
Not at all
2
3
4
5
Perfectly
377
Q

Littre hernia

A

Littre hernia is a hernia containing a Meckel’s diverticulum. Also known as a persistent omphalomesenteric duct hernia. It is most frequently encountered in the inguinal region.

How well did you know this?
1
Not at all
2
3
4
5
Perfectly
378
Q

Dx of Meckel’s

A

Tc pertechnecate scan +ve in 70% (detects gastric mucosa)

How well did you know this?
1
Not at all
2
3
4
5
Perfectly
379
Q

Rx Meckels

A

Surgical resection

How well did you know this?
1
Not at all
2
3
4
5
Perfectly
380
Q

Def: intussuception

A

Portion of the intestine (the intussuception) is invaginated into its own lumen (the intussuscipiens)

How well did you know this?
1
Not at all
2
3
4
5
Perfectly
381
Q

Causes of intussuception

A

Hypertrophied Peyer’s patch

Meckel’s

HSP

Petuz-Jeghers

Lymphoma

How well did you know this?
1
Not at all
2
3
4
5
Perfectly
382
Q

Presentation of intussuception

A

6-12mo

Colicky abdo pain: episodic incosolable crying, drawing up legs +/- bilious vomiting

Redcurrent jelly stools

Sausage shaped abdominal mass

How well did you know this?
1
Not at all
2
3
4
5
Perfectly
383
Q

6-12mo

Colicky abdo pain: episodic incosolable crying, drawing up legs +/- bilious vomiting

Redcurrent jelly stools

Sausage shaped abdominal mass

A

Intussuception

How well did you know this?
1
Not at all
2
3
4
5
Perfectly
384
Q

NB intussuception in adults

A

Rarely occurs

If it does, consider neoplasm as a lead point

How well did you know this?
1
Not at all
2
3
4
5
Perfectly
385
Q

Mx of intussusception

A

Resuscitate and X-match, NGT

US + reduction by air enema

Sx if irreducible

How well did you know this?
1
Not at all
2
3
4
5
Perfectly
386
Q

Def: mesenteric adenitits

A

URTI/viral infeciotn-> enlargement of mesenteric LNs= pain tenderness and fever

How well did you know this?
1
Not at all
2
3
4
5
Perfectly
387
Q

Differentiating features of mesenteric adenitis

A

Post URTI

Headache + photophobia

Higher temperature

Tenderness is more generalised

Lymphocytosis

How well did you know this?
1
Not at all
2
3
4
5
Perfectly
388
Q

Gross anatomy: External ear

A

Auricle

External auditory meatus

How well did you know this?
1
Not at all
2
3
4
5
Perfectly
389
Q

Gross antomy: middle ear

A

Tympanic:

malleus, incus and stapes

How well did you know this?
1
Not at all
2
3
4
5
Perfectly
390
Q

Gross anatomy: inner ear

A

Semicircular canals, vestibule, cochlea

How well did you know this?
1
Not at all
2
3
4
5
Perfectly
391
Q
A
How well did you know this?
1
Not at all
2
3
4
5
Perfectly
392
Q

Purpose of audiometry

A

Quantify loss and determine its nature

How well did you know this?
1
Not at all
2
3
4
5
Perfectly
393
Q

Features of pure tone audiometry

A

Headphones deliver tones at different frequencies and strengths

Patient indicates when sound appears and disappears

Mastoid vibrator- bone conduction threshold

Threshold at different frequences are plotted to give an audiogram

How well did you know this?
1
Not at all
2
3
4
5
Perfectly
394
Q

Purpose of tympanometry

A

Measures stiffness of the ear drum

Evaluates middle ear function

How well did you know this?
1
Not at all
2
3
4
5
Perfectly
395
Q

Flat tympanogram=

A

Mid ear fluid or perforation

How well did you know this?
1
Not at all
2
3
4
5
Perfectly
396
Q

Shifted tympanogram=

A

+/- mid ear pressure

How well did you know this?
1
Not at all
2
3
4
5
Perfectly
397
Q

Features of evoked response audiometry

A

Auditory stimulus with measurement of elicited brain response by surface electrode

Used for neonatal screening if otoacoustic emission test negative

How well did you know this?
1
Not at all
2
3
4
5
Perfectly
398
Q

Presentation of otitis externa

A

Watery discharge

Itch

Pain and tragal tenderness

How well did you know this?
1
Not at all
2
3
4
5
Perfectly
399
Q

Watery discharge

Itch

Pain and tragal tenderness

A

?Otitis externa

How well did you know this?
1
Not at all
2
3
4
5
Perfectly
400
Q

Causes of otitis externa

A

Moisture e.g. swimming

Trauma e.g. fingernails

Absence of wax

Hearing aid

How well did you know this?
1
Not at all
2
3
4
5
Perfectly
401
Q

Organisms causing otitis externa

A

Pseudomonas

Staph aureus

How well did you know this?
1
Not at all
2
3
4
5
Perfectly
402
Q
A

OE

How well did you know this?
1
Not at all
2
3
4
5
Perfectly
403
Q
A

OE

How well did you know this?
1
Not at all
2
3
4
5
Perfectly
404
Q

Mx of otitis externa

A

Conservative: remove precipitating factors

Medical:

Analgesia e.g. paracetamol or ibuprofen

Treat inflammation:

Topical acetic acid

For more severe cases consider topical antibitoic +/- corticosteroid

Surgical:

?Cleaning of ear if required

How well did you know this?
1
Not at all
2
3
4
5
Perfectly
405
Q

Rx in non-infected eczematous OE

A

Betamethasone

How well did you know this?
1
Not at all
2
3
4
5
Perfectly
406
Q

Combination therapies for OE

A

Betamethasone with neomycin

Hydrocortisone with gentamicin

How well did you know this?
1
Not at all
2
3
4
5
Perfectly
407
Q

Rx for fungal OE

A

Consider clotrimazole

How well did you know this?
1
Not at all
2
3
4
5
Perfectly
408
Q

Consideration for amingoglycoside treatment of OE

A

If perf can lead to ototoxicity

How well did you know this?
1
Not at all
2
3
4
5
Perfectly
409
Q

Severe otalgia which is worse at night

Copious otorrhoea

Granulation tissue in the canal

(90% in DM)

A

Malignant otitis externa

How well did you know this?
1
Not at all
2
3
4
5
Perfectly
410
Q

Implications of malignant OE

A

Life-threatening infection which can lead to skull osteomyletitis

Look for tenderness of mastoid process

How well did you know this?
1
Not at all
2
3
4
5
Perfectly
411
Q

Rx of malignant OE

A

Admit for IV abx

+/- surgical debridement

How well did you know this?
1
Not at all
2
3
4
5
Perfectly
412
Q
A

Malignant OE

How well did you know this?
1
Not at all
2
3
4
5
Perfectly
413
Q

Def: bullous myringitis

A

Painful haemorrhagic blisters on deep meatal skin and tympanic membrane

Associated with influenza infection

How well did you know this?
1
Not at all
2
3
4
5
Perfectly
414
Q

With what is bullous myringitis associated

A

Influenza infection

How well did you know this?
1
Not at all
2
3
4
5
Perfectly
415
Q
A

Bullous myringitis

How well did you know this?
1
Not at all
2
3
4
5
Perfectly
416
Q

What are the symptoms of TMJ dysfunction

A

Earache (referred pain from auriculo temporal nerve)

Facial pain

Joint-clicking/popping

Teeth-grinding (bruxism)

Stress (associated with depression)

How well did you know this?
1
Not at all
2
3
4
5
Perfectly
417
Q

Joint tenderness exacerbated by lateral movements of open jaw

A

?TMJ dysfunction

How well did you know this?
1
Not at all
2
3
4
5
Perfectly
418
Q

Ix of TMJ dysfunction

A

MRI

How well did you know this?
1
Not at all
2
3
4
5
Perfectly
419
Q

Mx of TMJ dysfunction

A

NSAIDs

stabilising orthodontic occlusal prostheses

How well did you know this?
1
Not at all
2
3
4
5
Perfectly
420
Q

How can OM be classified?

A

Acute

OME

Chronic

Chronic suppurative OM

How well did you know this?
1
Not at all
2
3
4
5
Perfectly
421
Q

Acute OM=

A

Acute phase OM

How well did you know this?
1
Not at all
2
3
4
5
Perfectly
422
Q

OME=

A

Effusion after symptom regression

How well did you know this?
1
Not at all
2
3
4
5
Perfectly
423
Q

Chronic OM=

A

Effusion >3mo if bilateral or >6mo if unilateral

How well did you know this?
1
Not at all
2
3
4
5
Perfectly
424
Q

Def: chronic suppurative OM

A

Ear discharge with hearing loss and evidence of central drum perforation

How well did you know this?
1
Not at all
2
3
4
5
Perfectly
425
Q

Presentation of acute OM

A

Usually children post viral UTI

Rapid onset ear pain, tugging

Irrritability, anorexia, vomiting

Purulent discharge if perforation

How well did you know this?
1
Not at all
2
3
4
5
Perfectly
426
Q

Usually children post viral UTI

Rapid onset ear pain, tugging

Irrritability, anorexia, vomiting

Purulent discharge if perforation

A

Acute OM

How well did you know this?
1
Not at all
2
3
4
5
Perfectly
427
Q

Bulging red TM

Fever

A

Acute OM

How well did you know this?
1
Not at all
2
3
4
5
Perfectly
428
Q

Rx in acute OM

A

Paracetamol

Amoxicillin: may used delayed presecription

How well did you know this?
1
Not at all
2
3
4
5
Perfectly
429
Q

How can the cx of OM be classified

A

Intratemporal

Intracranial

Systemic

How well did you know this?
1
Not at all
2
3
4
5
Perfectly
430
Q

Intratemporal complications of OM

A

OME

Perforation of TM

Mastoidis

Facial N. palsy

How well did you know this?
1
Not at all
2
3
4
5
Perfectly
431
Q

Intracranial Cxs of OM

A

Meningitis/encephalitis

Brain abscess

Sub/epidural abscess

How well did you know this?
1
Not at all
2
3
4
5
Perfectly
432
Q

Systemic cxs of OM

A

Bacteraemia

Septic arthritis

IE

How well did you know this?
1
Not at all
2
3
4
5
Perfectly
433
Q
A

Acute OM

How well did you know this?
1
Not at all
2
3
4
5
Perfectly
434
Q

Features of delayed antibiotic prescribing strategies

A

Should be started if symptoms not improving within 4d of onset of symptoms or signficiant worsening of symptoms

Safety net

How well did you know this?
1
Not at all
2
3
4
5
Perfectly
435
Q

Inattention at school

Poor speech development

Hearing impairment

A

?OME

How well did you know this?
1
Not at all
2
3
4
5
Perfectly
436
Q
A

OME

Retracted, dull TM

Fluid level

How well did you know this?
1
Not at all
2
3
4
5
Perfectly
437
Q

Audiometry in OME

A

Flat tympanogram

How well did you know this?
1
Not at all
2
3
4
5
Perfectly
438
Q

Mx of OME

A

Usually resolves spontaenously

Consider grommets if persistent hearing loss

SE: infections and tympanosclerosis

How well did you know this?
1
Not at all
2
3
4
5
Perfectly
439
Q

Painless discharge with hearing loss

A

Chronic suppurative OM

How well did you know this?
1
Not at all
2
3
4
5
Perfectly
440
Q
A

Chronic suppurative OM

How well did you know this?
1
Not at all
2
3
4
5
Perfectly
441
Q

Rx chronic suppurative OM

A

Aural toilet

Abx/steroid ear drops

How well did you know this?
1
Not at all
2
3
4
5
Perfectly
442
Q

Complications of chronic suppurative OM

A

Cholesteatoma

How well did you know this?
1
Not at all
2
3
4
5
Perfectly
443
Q

Def: mastoiditis

A

Middle-ear inflammation-> destruction of mastoid air cells and abscess formation

How well did you know this?
1
Not at all
2
3
4
5
Perfectly
444
Q

Fever

Mastoid tenderness

Protruding auricle

A

?Mastoiditis

How well did you know this?
1
Not at all
2
3
4
5
Perfectly
445
Q

Ix in mastoiditis

A

CT

How well did you know this?
1
Not at all
2
3
4
5
Perfectly
446
Q

Mx in mastoiditis

A

IV Abx

Myringotomy and mastoidectomy

How well did you know this?
1
Not at all
2
3
4
5
Perfectly
447
Q

Def: cholesteatoma

A

Locally destructive expansion of stratified squamous epithelium within the middle ear

How well did you know this?
1
Not at all
2
3
4
5
Perfectly
448
Q

How can cholesteatoma be classified?

A

Congenital

Acquired: secondary to attic perforation in chronic suppurative OM

How well did you know this?
1
Not at all
2
3
4
5
Perfectly
449
Q

Foul smelling white discharge

Headache/pain

CN involvement: vertigo, deafness, facial paralysis

A

?Cholesteatoma

How well did you know this?
1
Not at all
2
3
4
5
Perfectly
450
Q
A

Cholesteatoma

Pearly white with surrounding inflammation

How well did you know this?
1
Not at all
2
3
4
5
Perfectly
451
Q

Cx of cholesteatoma

A

Deafness (ossicle destruction)

Meningitis

Cerebral abscess

How well did you know this?
1
Not at all
2
3
4
5
Perfectly
452
Q

Mx of cholesteatoma

A

Sx

How well did you know this?
1
Not at all
2
3
4
5
Perfectly
453
Q

Def: tinnitus

A

Sensation of sound without external sound stimulation

How well did you know this?
1
Not at all
2
3
4
5
Perfectly
454
Q

How can causes of tinnitus be classified?

A

Specific

General

Drugs

How well did you know this?
1
Not at all
2
3
4
5
Perfectly
455
Q

Specific causes of tinnitus

A

Meniere’s

Acoustic neuroma

Otosclerosis

Noise-induced

Head injury

Hearing loss e.g. presbyacusis

How well did you know this?
1
Not at all
2
3
4
5
Perfectly
456
Q

Presbyacusis

A

Loss of hearing that gradually occurs in most individuals as they grow older

How well did you know this?
1
Not at all
2
3
4
5
Perfectly
457
Q

General causes of tinnitus

A

Raised BP

Anaemia

How well did you know this?
1
Not at all
2
3
4
5
Perfectly
458
Q

Drugs causing tinnitus

A

Aspirin

Aminoglycosides

Loop diuretics

ETOH

How well did you know this?
1
Not at all
2
3
4
5
Perfectly
459
Q

Components of tinnitus history

A

Character: constant or pulsatile

Unilateral (acoustic neuroma)

FHx: otoscleoris

Alleviating/exacerbating factors e.g. worse at night

Associations:

Vertigo: Meniere’s, acoustic neuroma

Deafness: Meniere’s, acoustic neuroma

Causes e.g. head injury, noise, drugs, Fhx

How well did you know this?
1
Not at all
2
3
4
5
Perfectly
460
Q

Def: otosclerosis

A

Hereditary disorder causing progressive deagness due to bone overgrowth in the inner ear

How well did you know this?
1
Not at all
2
3
4
5
Perfectly
461
Q

Examination in tinnitus

A

Otoscopy

Tuning fork tests

Pulse and BP

How well did you know this?
1
Not at all
2
3
4
5
Perfectly
462
Q

Ix in tinnitus

A

Audiometry and tympanogram

MRI if unilateral to exclude acoustic neuroma

How well did you know this?
1
Not at all
2
3
4
5
Perfectly
463
Q

Mx of tinnitus

A

Treat underlying causes

Psych support: tinnitus retraining therapy

Hypnotics at night may help

How well did you know this?
1
Not at all
2
3
4
5
Perfectly
464
Q

Def: vertigo

A

The illusion of movement

How well did you know this?
1
Not at all
2
3
4
5
Perfectly
465
Q

How can the causes of vertigo be classified

A

Peripheral/vestibular

Central

Drugs

How well did you know this?
1
Not at all
2
3
4
5
Perfectly
466
Q

Peripheral causes of vertigo

A

Meniere’s

BPPV

Labyrinthitis

How well did you know this?
1
Not at all
2
3
4
5
Perfectly
467
Q

Central causes of vertigo

A

Acoustic neuroma

MS

Vertebrobasilar insufficiency

Stroke

Head injury

Inner ear syphillis

How well did you know this?
1
Not at all
2
3
4
5
Perfectly
468
Q

Drugs causing vertigo

A

Gentamicin

Loop diuretics

Metronidazole

Co-trimoxazole

How well did you know this?
1
Not at all
2
3
4
5
Perfectly
469
Q

Components of vertigo history

A

Is it true vertigo or light-headedness: which way are things moving: spinning/whirling when not moving (vertigo), sense of imbalance or staggering when walking (disequilibirium), light headedness (presyncope), dizziness caused by hyperventilation

Timespan

Associated symptoms: n/v, hearing loss, tinnitus, nystagmus

How well did you know this?
1
Not at all
2
3
4
5
Perfectly
470
Q

Nystagmus

A

Condition of involuntary eye movement

can be physiological or pathological

How well did you know this?
1
Not at all
2
3
4
5
Perfectly
471
Q

Examinations and tests in vertigo

A

Hearing

Cranial nerve

Cerebellar and gait

Rombergs +ve= vestibular or proprioception

Hallpike manouvre

Audiometry, calorimetry, LP, MRI

How well did you know this?
1
Not at all
2
3
4
5
Perfectly
472
Q

Meniere’s triad

A

Vertigo

Tinnitus

Hearing loss

How well did you know this?
1
Not at all
2
3
4
5
Perfectly
473
Q

Pathology of Meniere’s disease

A

Dilatation of endolymph spaces of membranous labyrinth (endolymphatic oedema)

How well did you know this?
1
Not at all
2
3
4
5
Perfectly
474
Q

Attacks occuring in clusters lasting up to 12h

Progressive SNHL

Vertigo and n+v

Tinnitus

Aural fullness (pressure in the ears)

A

?Meniere’s

How well did you know this?
1
Not at all
2
3
4
5
Perfectly
475
Q

Audiometry showing low frequency SNHL which fluctuates

A

Meniere’s

How well did you know this?
1
Not at all
2
3
4
5
Perfectly
476
Q

Mx: Meniere’s disease

A

Medical:

Symptomatic reliefe: prochlorperazine (if severe) or betahistine or cyclizine

Surgical:

Gentamicin instillation via grommets

Saccus decompression

How well did you know this?
1
Not at all
2
3
4
5
Perfectly
477
Q

Follows febrile illness (e.g. URTI)

Sudden vomiting

Severe vertigo exacerbated by head movemenet

A

Vestibular neuronitis/viral labyrinthistis

How well did you know this?
1
Not at all
2
3
4
5
Perfectly
478
Q

Mx of viral labyrinthitis

A

Cyclizine

Improvement in days

How well did you know this?
1
Not at all
2
3
4
5
Perfectly
479
Q

Pathology of BPV

A

Displacement of otoliths in semicircular canals

Common after head injury

How well did you know this?
1
Not at all
2
3
4
5
Perfectly
480
Q

Sudden rotational vertigo for <30s
Provoked by head turning

Nystagmus

A

BPV

How well did you know this?
1
Not at all
2
3
4
5
Perfectly
481
Q

Features of classic BPPV

A

Geotropic nystagmus with the problem ear down

Predominantly rotatory fast phase toward undermost ear

Latency (a few seconds)

Limited duration (< 20 s)

Reversal upon return to upright position

Response decline upon repetitive provocation

How well did you know this?
1
Not at all
2
3
4
5
Perfectly
482
Q

How can nystagmus be classified

A

Begins as a slow pursuit movement followed by a fast rapid, resetting phase

Named by direction of the fast phase

Right or left beating (horizontal nystagmus)

Up-beating or down-beating (vertical)
Or direction changing

If the movements are not horizontal or vertical then the nystagmus is rotational (clockwise or counter-clockwsie)

Can also have visual evoked nystagamus (VER)- implies central lesion

How well did you know this?
1
Not at all
2
3
4
5
Perfectly
483
Q

Causes of BPV

A

Idiopathic

Head injury

Otosclerosis

Post-viral

How well did you know this?
1
Not at all
2
3
4
5
Perfectly
484
Q

Dx of BPV

A

Hallpike manoeuvre-> upbeat torsional nystagmus

How well did you know this?
1
Not at all
2
3
4
5
Perfectly
485
Q

Rx of BPV

A

Self-limiting

Epley manoeuvre

Betahistine

How well did you know this?
1
Not at all
2
3
4
5
Perfectly
486
Q

Difference between primary and secondary otalgia

A

Primary there is usually abnormality on examination

Secondary is normal looking ear

How well did you know this?
1
Not at all
2
3
4
5
Perfectly
487
Q

Causes of primary otalgia

A

OE/OM

FB

Barotrauma

Rarely:

OME

Ramsay hunt

Perichorditis

Cellulitis

Relapsing polychondritis etc.

How well did you know this?
1
Not at all
2
3
4
5
Perfectly
488
Q

Causes of secondary otalgia

A

Can be classified on basis of nerve territory

e.g.

Trrigeminal TMJ problems

Facial: CPA lesions etc

Glossopharyngeal: tumours in PNS/pharynx

Vagus: tumours in pharynx/larynx, GORD

Spinal nerves: arthritis/tumours

How well did you know this?
1
Not at all
2
3
4
5
Perfectly
489
Q

Borders of the anterior triangle of the neck

A

Mandible

Midline

Sternocleidomastoid

How well did you know this?
1
Not at all
2
3
4
5
Perfectly
490
Q

Borders of the posterior triangle of the neck

A

Sternocleidomastoid

Trapezius

Clavicle

How well did you know this?
1
Not at all
2
3
4
5
Perfectly
491
Q

Quinsy=

A

Peritonsillar abscess

How well did you know this?
1
Not at all
2
3
4
5
Perfectly
492
Q

Definition of a definitive airway

A

In the trachea

Cuffed below the vocal cords

Attached to oxygen

Secured

How well did you know this?
1
Not at all
2
3
4
5
Perfectly
493
Q

What are the indications for intubation

A

A: protection and patency

B: respiratory failure, increase FRC, decreased WOB, secretion management, to facilitate bronchoscopy

C: minimise oxygen consumption and optimise O2 delivery

D: unresponsive to pain, prevent brain injury

E: temperature control

How well did you know this?
1
Not at all
2
3
4
5
Perfectly
494
Q

Causes of stridor

A

Children: croup, inhaled FB, tracheitis, abscess (retropharyngeal, peritonsillar), anaphylaxis, epiglottits, laryngomalacia, VC dysfunction, subglottic stenosis, laryngeal web, laryngeal tumours, tracheomalacia, choanal atresia, tracheal stenosis

Adults: anaphylaxis, laryngitis, epiglottits/supraglottitis, FB, abscess, laryngospasm, tumour

How well did you know this?
1
Not at all
2
3
4
5
Perfectly
495
Q

Sound of stridor

A

Insipiratory

NB: stridor can be biphasic if obstruction is at the level of the glottis

How well did you know this?
1
Not at all
2
3
4
5
Perfectly
496
Q

Causative organisms in epiglottits

A

Strep

Staph

HiB

Pseudomonas

Moraxella catarrhalis

TB

How well did you know this?
1
Not at all
2
3
4
5
Perfectly
497
Q

Rapid onset

Unwell

Odynophagia

Drooling

Fever

Anterior tenderness over hyoid bone

Lymphadenopathy

Tripod sign

Progressing rapibldy to SOB, resp distress, airway obstruction, stridor

A

?Epiglottis/supraglottits

How well did you know this?
1
Not at all
2
3
4
5
Perfectly
498
Q

Ix in epiglottits

A

Airway prep

Lateral neck XR

Bloods +/- cultures

How well did you know this?
1
Not at all
2
3
4
5
Perfectly
499
Q

Mx of epiglottits

A

IV antibiotics: usually 3rd gen cephalosporin

Steroids

Intubation +/- cricothyroidotomy c trachy if airway obstructed

How well did you know this?
1
Not at all
2
3
4
5
Perfectly
500
Q

Mx of stridor

A

ABCDE

Appropriate area e.g. A&E resus

Adrenaline (1:1000) nebulised

Steroids: budenoside nebs + IV dex

O2

Intubation if needed

Cricothyroidotomy either needle or surgical

How well did you know this?
1
Not at all
2
3
4
5
Perfectly
501
Q

Site of location of cricothyroidotomy?

A

Through cricothyroid membrane inbetween thyroid cartilage and cricoid

Can be done with large bore cannulae or surgically

How well did you know this?
1
Not at all
2
3
4
5
Perfectly
502
Q

Mx of epistaxis

A
  1. External compresion: 90% anterior, 10% posterior, lean forward and distal part of nose, spit anything that enters mouth
  2. Packing:

Anterior: merocel (nasal tampon)/ Rapid rhino

48hrs

After removal cauterise and give naseptin cream (NB contain peanut oil)

Posterior packing: urinary catheter, insert until tip seen at back of mouth, inflate balloon slowly, don’t let go of catheter

  1. Cautery using silver nitrate
  2. Theatre

Surgical ligation of the sphenopalantine artery

How well did you know this?
1
Not at all
2
3
4
5
Perfectly
503
Q

What is Little’s area?

A

AKA Kiesselbach’s plexus

Kiesselbach’s plexus, which lies in Kiesselbach’s area, Kiesselbach’s triangle, or Little’s area, is a region in the anteroinferior part of the nasal septum where four arteries anastomose to form a vascular plexus. The arteries are:[1]

Anterior ethmoidal artery and posterior ethmoidal artery (both from the ophthalmic artery)

Sphenopalatine artery (terminal branch of the maxillary artery)

Greater palatine artery (from the maxillary artery)

Septal branch of the superior labial artery (from the facial artery)

90% of nosebleeds occur here due to the drying effect of air

How well did you know this?
1
Not at all
2
3
4
5
Perfectly
504
Q

What are the red flags in epistaxis?

A

Age >50y

Nasal obstruction

Facial pain

Hearing loss

Proptosis/double vision

Lymphadenopathy, weight loss

How well did you know this?
1
Not at all
2
3
4
5
Perfectly
505
Q

Middle-aged Chinese people with epistaxis think

A

?Nasopharyngeal carcinoma as high incidence

How well did you know this?
1
Not at all
2
3
4
5
Perfectly
506
Q

Occupational exposure to dust/chemicals with epistaxis

A

Think nasopharyngeal carcinoma as RF

How well did you know this?
1
Not at all
2
3
4
5
Perfectly
507
Q

Epistaxis in child <2

A

Shouldn’t happen

?NAI

How well did you know this?
1
Not at all
2
3
4
5
Perfectly
508
Q

Risk factors for mastoiditis

A

Young children

Immunocompromised

Cholesteatoma

How well did you know this?
1
Not at all
2
3
4
5
Perfectly
509
Q

Organisms causing mastoiditis

A

Strep pneumonia, pyogenes, staph, pseudomonas, HiB

How well did you know this?
1
Not at all
2
3
4
5
Perfectly
510
Q

Unwell

Fever

Painful over mastoid process

Swollen and boggy

Young children: irritable, pinna protrusion +/- discharge

TM perforation

Hx of otitis media

Chronic: recurrent otalgia, headache, fever, OME/suppurative OM

A

?Mastoiditis

How well did you know this?
1
Not at all
2
3
4
5
Perfectly
511
Q
A

Mastoiditis

How well did you know this?
1
Not at all
2
3
4
5
Perfectly
512
Q

O/E

6th or 7th nerve palsy

Conductive deafness

Boggy swelling behind ear

A

?Mastoiditis

How well did you know this?
1
Not at all
2
3
4
5
Perfectly
513
Q

Ix in mastoiditis

A

FBC, CRP, blood cultures

Ear swab

CT/MRI

Audiogram

How well did you know this?
1
Not at all
2
3
4
5
Perfectly
514
Q

Mx of mastoiditis

A

High dose broad specrutm IV Abx

Analgesia

Emergency mastoidectomy for cholesteatoma and mastoid oteitits or intracranial spread or not improving

How well did you know this?
1
Not at all
2
3
4
5
Perfectly
515
Q

Dizziness what other questions should you ask?

A

N+V

Tinnitus

Hearing loss

Feeling of aural fullness

Headaches

Visual changes

Weakness

Numbness

How well did you know this?
1
Not at all
2
3
4
5
Perfectly
516
Q

Usually preceded by paranasal sinusitits

Medical emergency

Also caused by local trauma, insect or animal bites, FB, URTI

A

Periorbital cellulitis

How well did you know this?
1
Not at all
2
3
4
5
Perfectly
517
Q

Ocular pain

Eyelid swelling

Erythema

A

?Periorbital cellulitis

How well did you know this?
1
Not at all
2
3
4
5
Perfectly
518
Q

What differentiates between periorbital cellulitis and orbital cellulitis

A

Orbital also presents with painful eye movements, proptosis, opthalmoplegia, visual impairment, chemosis

How well did you know this?
1
Not at all
2
3
4
5
Perfectly
519
Q

Aetiology of SNHL

A

Usually idiopathic

How well did you know this?
1
Not at all
2
3
4
5
Perfectly
520
Q

Mx of idiopathic SNHL

A

Short-course high dose steroids (+/- PPI cover)

With repeat audiogram F/U

How well did you know this?
1
Not at all
2
3
4
5
Perfectly
521
Q

Def: acoustic neuroma

A

Benign slow-growing tumour of the vestibular nerve (vestibular schwanomma)

How well did you know this?
1
Not at all
2
3
4
5
Perfectly
522
Q

Sudden onset or progressive SNHL, tinnitus, balance issues, vertigo

A

?Acoustic neuroma

How well did you know this?
1
Not at all
2
3
4
5
Perfectly
523
Q

Local effects of acoustic neuroma

A

5th trigeminal causing facial numbness and tingling

Facial nerve can also be affected

If very large can compress the brainstem or cuse raised ICP

How well did you know this?
1
Not at all
2
3
4
5
Perfectly
524
Q

Most common site of acoustic neuroma growth

A

Cerebellopontine angle

How well did you know this?
1
Not at all
2
3
4
5
Perfectly
525
Q

Mx of acoustic neuroma

A

Watchful waiting with MRI/audiogram

Stereotactic radiotherapy

How well did you know this?
1
Not at all
2
3
4
5
Perfectly
526
Q

Cx of acoustic neuroma sx?

A

Loss of hearing

Damage to other cranial nerves esepcially the 7th

How well did you know this?
1
Not at all
2
3
4
5
Perfectly
527
Q
A

Bell’s palsy

How well did you know this?
1
Not at all
2
3
4
5
Perfectly
528
Q

Sudden onset unilateral facial droop

+/- hyperacusis

Loss of sensation in anterior 2/3rds of tongue

A

Bell’s palsy

How well did you know this?
1
Not at all
2
3
4
5
Perfectly
529
Q

What differentiates between LMN and UMN Bell’s palsy

A

UMN lesions have forehead sparing as frontalis receives innervation from both hemispheres)
For it to be true Bell’s it must not have forehead sparing

How well did you know this?
1
Not at all
2
3
4
5
Perfectly
530
Q

Why should Lyme disease serology be done in Bell’s palsy in endemic areas

A

As LD can lead to facial nerve palsy

How well did you know this?
1
Not at all
2
3
4
5
Perfectly
531
Q

Mx of Bell’s palsy

A

Short term high dose (60-80mg) steroids + eye care (drops with taping at night)

How well did you know this?
1
Not at all
2
3
4
5
Perfectly
532
Q

Bell’s palsy px

A

Incomplete lesions have better prognosis

How well did you know this?
1
Not at all
2
3
4
5
Perfectly
533
Q

What is Ludwig’s angina

A

Submandibular space infection

Aggressive and rapidly spreading cellulits from which patients can become quickly septic with compromised airway

How well did you know this?
1
Not at all
2
3
4
5
Perfectly
534
Q

Unwell

Fever

Mouth pain

Drooling

Dysphagia

A

Ludwig’s angina

How well did you know this?
1
Not at all
2
3
4
5
Perfectly
535
Q

Mx of Ludwig’s angina

A

IV Abx with anaerobe cover

Close monitoring

Airway +++

How well did you know this?
1
Not at all
2
3
4
5
Perfectly
536
Q

Def: conductive hearing loss

A

Impaired conduciton anywhere between auricle and round window

How well did you know this?
1
Not at all
2
3
4
5
Perfectly
537
Q

How can the causes of conductive hearing loss be classified?

A

External canal obstruciton

TM perforation

Ossicle defects

Inadequate eustachian tube ventilation of midddle air

How well did you know this?
1
Not at all
2
3
4
5
Perfectly
538
Q

Causes of external canal obstruction Leading to conductive hearing loss

A

Wax

Pus

FB

How well did you know this?
1
Not at all
2
3
4
5
Perfectly
539
Q

Causes of TM perforation Leading to conductive hearing loss

A

Trauma

Infection

How well did you know this?
1
Not at all
2
3
4
5
Perfectly
540
Q

Causes of ossicle defects leading to conductive hearing loss?

A

Otosclerosis

Infection

Trauma

How well did you know this?
1
Not at all
2
3
4
5
Perfectly
541
Q

Def: sensorineural hearing loss

A

Defects of cochlea, cochlear nerve or brain

How well did you know this?
1
Not at all
2
3
4
5
Perfectly
542
Q

Which drugs can cause sensorineural hearing loss?

A

Aminoglycosides

Vancomycin

How well did you know this?
1
Not at all
2
3
4
5
Perfectly
543
Q

Which post-infective syndromes are associated with sensorineural hearing loss?

A

Meningitis

Measles

Mumps

Herpes

How well did you know this?
1
Not at all
2
3
4
5
Perfectly
544
Q

What are some miscellaneous causes of sensorineural hearing loss?

A

Meniere’s

Trauma

MS

CPA lesions e.g. acoustic neuroma

Low B12

How well did you know this?
1
Not at all
2
3
4
5
Perfectly
545
Q

Acoustic neuroma=

A

Benign slow-growing tumour of superior vestibular nerve

How well did you know this?
1
Not at all
2
3
4
5
Perfectly
546
Q

􀁸 Slow onset, unilat SNHL, tinnitus ± vertigo
􀁸 Headache (↑ICP)
􀁸 CN palsies: 5,7 and 8
􀁸 Cerebellar signs

A

?Acoustic neuroma

How well did you know this?
1
Not at all
2
3
4
5
Perfectly
547
Q

All patients with unilateral tinnitus/deafness should receive
an?

A

MRI

How well did you know this?
1
Not at all
2
3
4
5
Perfectly
548
Q

What syndrome associated with acoustic neuromas?

A

NF2

How well did you know this?
1
Not at all
2
3
4
5
Perfectly
549
Q

What accounts for 80% of CPA tumours?

A

Acoustic neuromas

How well did you know this?
1
Not at all
2
3
4
5
Perfectly
550
Q

Common cause of CPA syndrome?

A

Acoustic neuromas

How well did you know this?
1
Not at all
2
3
4
5
Perfectly
551
Q

Bruns nystagmus

A

Dancing eyes

Seen in large tumours due to compression of the flocculi in CPA syndrome

How well did you know this?
1
Not at all
2
3
4
5
Perfectly
552
Q

Features of CPA syndrome

A

Tumours within nerve cannaliculi: unilateral SNHL, tinnitus or disequilibirium

Tumours extending into the CPA may present with disequilibrium and ataxia

With brainstem extension, midfacial and corneal hypesthesia, hydrocephalies and other CN palsies become more prevalent

(speech discrimination out of proportion to hearing loss and difficulty talking on the telephone are frequent accompaniements)

How well did you know this?
1
Not at all
2
3
4
5
Perfectly
553
Q

DDx slow onset unilateral SNHL

A

Meningioma

Cerebellar astrocytoma

Mets

How well did you know this?
1
Not at all
2
3
4
5
Perfectly
554
Q

Otosclerosis

A

AD condition characterised by fixation of the stapes at the oval window

F>M 2:1

How well did you know this?
1
Not at all
2
3
4
5
Perfectly
555
Q

Begins early in adult life

Bilateral conductive deafness + tinnitus

HL improved in noisy places (Willis’ paracousis)

Worsened by pregnancy/menstruation

A

Otosclerosis

How well did you know this?
1
Not at all
2
3
4
5
Perfectly
556
Q

>65y

Bilateral slow onset hearing loss

+/- tinnitus

A

Age-related hearing loss i.e. presbyacussis

How well did you know this?
1
Not at all
2
3
4
5
Perfectly
557
Q

Congenital causes of conductive hearing loss in children

A

Anomalies of pinna, external auditory canal, TM or ossicles

Congenital cholesteatoma

Pierre-Robin

How well did you know this?
1
Not at all
2
3
4
5
Perfectly
558
Q

Congenital causes of SNHL in children

A

AD: Waardenburgs: SNHL, heterochromia + telecanthus

AR: Alports (SNHL + haematuria), Jewell-Lange-Nielson

X-Linked: Alports

Infections: CMV, rubella, HSV, toxo, GBS

Ototoxic drugs

How well did you know this?
1
Not at all
2
3
4
5
Perfectly
559
Q

Perinatal causes of paediatric hearing loss

A

Anoxia

CP

Kernicterus

Infeciton: meningitis

How well did you know this?
1
Not at all
2
3
4
5
Perfectly
560
Q

Acquired causes of paediatric hearing loss

A

OM/OME

Infection: meningitis, measles

Head injury

How well did you know this?
1
Not at all
2
3
4
5
Perfectly
561
Q

What are some congenital anomalies seen in the ear

A

1st and 2nd branchial arches form auricle while 1st branchial groove forms external auditory canal

Malfusion leads to accessory tags/auricles and preauricular pits, fistulae or sinuses

Sinuses may get infected

How well did you know this?
1
Not at all
2
3
4
5
Perfectly
562
Q

Features of pinna heamatoma

A

Blunt trauma leading to subperichondrial haematoma

Can lead to ischaemic necrosis of cartilage and subsequent fibrosis to cauliflower ears

How well did you know this?
1
Not at all
2
3
4
5
Perfectly
563
Q

Mx of pinna haematoma

A

Aspiration and firm packing to auricle contour

How well did you know this?
1
Not at all
2
3
4
5
Perfectly
564
Q

Def: exostoses

A

Smooth, symmetrical bony narrowing of external canals

How well did you know this?
1
Not at all
2
3
4
5
Perfectly
565
Q

Pathology of exostoses

A

Bony hypertrophy due to cold exposure e.g. from swimming/surfing

Asymptomatic unless narrowing occludes the canal leading to conductive deafness

How well did you know this?
1
Not at all
2
3
4
5
Perfectly
566
Q

Causes of TM perforation

A

OM

Trauma

Barotrauma

FB

How well did you know this?
1
Not at all
2
3
4
5
Perfectly
567
Q

What are the classifications of allergic rhinosinusitis?

A

Seasonal

Perennial

How well did you know this?
1
Not at all
2
3
4
5
Perfectly
568
Q

Pathology of allergic rhunosinusitis?

A

T1HS Ig-E mediated inflammation from allergen exposure leading to mediator relesase from mast cells

Allergens include pollen, house dust mites

How well did you know this?
1
Not at all
2
3
4
5
Perfectly
569
Q

Sneezing

Pruritus

Rinorrhoea

A

?Allergic rhinosinusitis

How well did you know this?
1
Not at all
2
3
4
5
Perfectly
570
Q

Swollen, pale and boggy nasal turbinates

Nasal polyps

A

?Allergic rhinosinusitis

How well did you know this?
1
Not at all
2
3
4
5
Perfectly
571
Q

External nose parameters

A

Extends from the nasal bones and surrounding parts of the maxilla and frontal bone

Supported by the central septal cartilage and its lateral processes

At the apex there are two major alar cartilages

Near the maxilla there are smaller minor alar cartilages

How well did you know this?
1
Not at all
2
3
4
5
Perfectly
572
Q
A
How well did you know this?
1
Not at all
2
3
4
5
Perfectly
573
Q

Internal nose parameters

A

Comprises two paired nasal cavities which extend and expand supero-posteriorly from the anterior nares

Nasal septum separates left and right

Hard palate makes up the floor of the cavities and separates them from the oral cavity

Posterior nares open posteriorly into the nasopharynx

How well did you know this?
1
Not at all
2
3
4
5
Perfectly
574
Q

What are the three regions of the internal nose?

A

Vestibular: just inside nostrils

Respiratory: ciliated epithelium making up the bulk of the cavity walls

Olfactory: specialised olfactory epithelium on the roofs of the cavities

How well did you know this?
1
Not at all
2
3
4
5
Perfectly
575
Q

What are found on the lateral walls of each nasal cavities?

A

Superior middle and inferior conchae

How well did you know this?
1
Not at all
2
3
4
5
Perfectly
576
Q

Which bone do the superior and middle conchae arise from?

A

Ethmoid

How well did you know this?
1
Not at all
2
3
4
5
Perfectly
577
Q

Which bone do the inferior conchae extend from?

A

The maxilla

How well did you know this?
1
Not at all
2
3
4
5
Perfectly
578
Q

What do the conchae form?

A

Narrow air passages known as the superior, middle and inferior meatus

How well did you know this?
1
Not at all
2
3
4
5
Perfectly
579
Q

What is the major route for nerve entry into the nasal cavity?

A

Sphenopalatine formaen and is found on the postero-lateral wall of the superior meatus

How well did you know this?
1
Not at all
2
3
4
5
Perfectly
580
Q

What is the space above the superior concha called?

A

The spheno-ethmoidal recess

How well did you know this?
1
Not at all
2
3
4
5
Perfectly
581
Q

What are the 4 paranasal sinuses?

A

Ethmoidal

Sphenoidal

Maxillary

Frontal

How well did you know this?
1
Not at all
2
3
4
5
Perfectly
582
Q
A
How well did you know this?
1
Not at all
2
3
4
5
Perfectly
583
Q
A
How well did you know this?
1
Not at all
2
3
4
5
Perfectly
584
Q

Features of the ethmoidal sinus

A

Collection of specialised ethmoidal air cells in the respiratory region of the nose

Openings in the bulla ethmoidalis on the lateral wall of the middle meatus

Cells between medial and lateral plates of the ethmoid labyrinths

How well did you know this?
1
Not at all
2
3
4
5
Perfectly
585
Q

Features of the sphenoidal sinuses?

A

Posterior to the nasal cavity within the sphenoid bone

Openings in the posterior wall of the spheno-ethmoidal recess

Immediately antero-inferior to the pituitary fossa

How well did you know this?
1
Not at all
2
3
4
5
Perfectly
586
Q

What is the largest of the nasal sinuses?

A

Maxillary

How well did you know this?
1
Not at all
2
3
4
5
Perfectly
587
Q

Features of the maxillary sinus

A

Beyond the lateral walls of the nasal cavity within the maxillae

Openings in the hiatus semilunaris floor on the lateral wall of the middle meatus

Drain at the apex- prone to filling with fluids that are hard to drain

How well did you know this?
1
Not at all
2
3
4
5
Perfectly
588
Q

Features of the frontal sinus

A

Superior to the nasal cavities within the frontal bone

Openings in the hiatus semilunaris roof on the lateral wall of the middle meatus

Drains through the frontonasal duct to the ethmoidal infundibulum

How well did you know this?
1
Not at all
2
3
4
5
Perfectly
589
Q

What is the course of the nasolacrimal duct?

A

Carries tears from the corner of the eye opening in the inferior meatus, inerfo-anterior to the hiatus semilunaris

How well did you know this?
1
Not at all
2
3
4
5
Perfectly
590
Q

Whence does the blood supply of the nose come?

A

Internal and external carotid arteries

How well did you know this?
1
Not at all
2
3
4
5
Perfectly
591
Q

What are the significant arteries supplying the nose?

A

Sphenopalantine artery

Greater palantine artery

Anterior and posterior ethmoidal arteries

How well did you know this?
1
Not at all
2
3
4
5
Perfectly
592
Q
A
How well did you know this?
1
Not at all
2
3
4
5
Perfectly
593
Q

Features of the sphenopalatine artery?

A

Terminal branch of the maxillary artery- branch of external carotid

Enters the cavity through the sphenopalatine formaen

Gives lateral branches supplying most of the lateral wall and medial branches to medial wall

How well did you know this?
1
Not at all
2
3
4
5
Perfectly
594
Q

Features of the greater palatine artery

A

Branch of maxillary artery: branch of external carotid

Enters the anterior floor of the nasal cavity through the incisive canal and supplies the anterior septum and floor

Anastomoses with septal branches of the sphenopalatine artery

How well did you know this?
1
Not at all
2
3
4
5
Perfectly
595
Q

From which artery do the ethmoidal arteries branch and what carotid does it originate from?

A

Ophthalmic artery

Internal carotid

How well did you know this?
1
Not at all
2
3
4
5
Perfectly
596
Q

How do the ethmoidal arteries enter the nasal cavity?

A

Through the cribiform plate

How well did you know this?
1
Not at all
2
3
4
5
Perfectly
597
Q

Course of the anterior and posterior ethmoidal arteries

A

Anterior: anastomoses with branches of the sphenopalatine artery and terminates as the external nasal artery

Posterior division supplies the upper lateral and medial walls

How well did you know this?
1
Not at all
2
3
4
5
Perfectly
598
Q

Whence comes the general sensory innervation of the nose?

A

Ophthalmic and maxillary division of V

How well did you know this?
1
Not at all
2
3
4
5
Perfectly
599
Q

What innervates the mucous glands in the nose?

A

Parasympathetic fibres of the facial nerve: arise form the pterygopalatine ganglion and run with V2 fibres

How well did you know this?
1
Not at all
2
3
4
5
Perfectly
600
Q

Branches of the facial nerve

To Zanzibar By Motor Car

A

Temporal

Zygomatic

Buccal

Mandibular

Cervical

How well did you know this?
1
Not at all
2
3
4
5
Perfectly
601
Q

Branches of the trigeminal nerve

A

Ophthalmic

Maxillary

Mandibular

How well did you know this?
1
Not at all
2
3
4
5
Perfectly
602
Q

Cranial exits of trigeminal nerve branches

Standing

Room

Only

A

Superior orbital fissure

Foramen rotundum

Foramen ovale

How well did you know this?
1
Not at all
2
3
4
5
Perfectly
603
Q

Features of ophthalmic nervous supply of the nose

A

Two key branches: ethmoidal nerves

Anterior branch: travels with anterior ethmoidal artery and supplies the anterior medial and lateral walls before terminating as the external nasal nerve

Posterior branch supplies the ethmoidal air cells and does not enter the nasal cavity

How well did you know this?
1
Not at all
2
3
4
5
Perfectly
604
Q

Features of maxillary nerve supply of nasal cavity

A

Lateral branches supply the lateral wall

Nasopalantine nerve supplies the medial wall before terminating in the oral muscoa

How well did you know this?
1
Not at all
2
3
4
5
Perfectly
605
Q
A
How well did you know this?
1
Not at all
2
3
4
5
Perfectly
606
Q

Ix in allergic rhinosinusitis

A

Skin-prick testing to find allergnes

RAST

How well did you know this?
1
Not at all
2
3
4
5
Perfectly
607
Q

Mx of allergic rhinitis

A

Allergen avoidance: regulalry washing bedding, avoid pollen

Rx:

1st Line:

PRN: Oral antihistamine e.g. cetirazine, desloratidine or intranasal e.g. azelastine

Preventative: intranasal corticosteroid e.g. beclometasone (if nasal blockage or polyps) or oral antihistamine

2nd line:

Intranasal steroid + antihistamine (oral)

3rd line:

Zafirlukast (leukotriene antag)

4th line:

Immunotherapy: to induce desensitisation to allergen

Oral corticosteroids can be considered for severe symptoms

How well did you know this?
1
Not at all
2
3
4
5
Perfectly
608
Q

What are some adjuvant nasal decongestants?

A

Pseudoephedrine, otrivine

How well did you know this?
1
Not at all
2
3
4
5
Perfectly
609
Q

Pathophysiology of sinusitis

A

Viruses-> mucosal oedema and decreased mucosal ciliary actions leading to mucus retention and secondary bacterial infection

How well did you know this?
1
Not at all
2
3
4
5
Perfectly
610
Q

Causative organisms acute bacterial sinusitis?

A

Pneumococcus, Haemophilus, Moraxella

How well did you know this?
1
Not at all
2
3
4
5
Perfectly
611
Q

Causative organisms chronic bacterial sinusitis

A

S. aureus, anaerobes

How well did you know this?
1
Not at all
2
3
4
5
Perfectly
612
Q

Causes of sinusitis

A

Majority are bacterial infection 2o to viral

5% due to dental root infections

Diving/swimming in infected water

Anatomical variation may leave individuals susceptile e.g. deviated septum, nasal polyps

Systmic disease e.g. PCD/Kartagener’s

How well did you know this?
1
Not at all
2
3
4
5
Perfectly
613
Q

Pain increasing on bending/straining

Discharge from nose-> foul taste

Nasal obstruciton/congestion

Anosmia or cacosmia (bad smell without external source)

Systemic symptoms

A

?Bacterial sinusitis

How well did you know this?
1
Not at all
2
3
4
5
Perfectly
614
Q

Which sinus liklely to be invovled if cheek/teeth pain in sinusitis

A

Maxillary

How well did you know this?
1
Not at all
2
3
4
5
Perfectly
615
Q

Which sinus liklely to be invovled if pain between eyes in sinusitis

A

Ethomidal

How well did you know this?
1
Not at all
2
3
4
5
Perfectly
616
Q

Ix in sinusitis

A

Nasendoscopy +/- CT

How well did you know this?
1
Not at all
2
3
4
5
Perfectly
617
Q

Mx of acute single episode of sinusitis

A

Bed-rest, decongestants, analgesia

Nasal douching and topical steroids

Abx of uncertain benefit

How well did you know this?
1
Not at all
2
3
4
5
Perfectly
618
Q

Mx of chronic/recurrent sinusitis?

A

Usually a structural or drainage problem

Stop smoking and fluticasone nasal spray

Functional endoscopic sinus sx if medical therapy fails

How well did you know this?
1
Not at all
2
3
4
5
Perfectly
619
Q

Complications of sinusits (rare)

A

Mucoceles-> pyoceles

Orbital cellulitis/abscess

Ostemoyelitis e.g. staph in frontal bone

Intracranial infection e.g. meningitis, encephalitis, abscess, CVST

How well did you know this?
1
Not at all
2
3
4
5
Perfectly
620
Q

What are the sites of nasal polyps

A

Middle turbinates

Middle meatus

Ethmoids

How well did you know this?
1
Not at all
2
3
4
5
Perfectly
621
Q

Water, anterior rhinorrhoea

Purulent post-nasal drip

Nasal obstruciton

Sinusitis

Headaches

Snoring

A

?Nasal polyps

How well did you know this?
1
Not at all
2
3
4
5
Perfectly
622
Q

Mobile pale insensitive growths in nasal cavity

A

?Nasal polyps

How well did you know this?
1
Not at all
2
3
4
5
Perfectly
623
Q

What are the associations of nasal polyps?

A

Allergic/non-allergic rhinitis

CF

Aspirin hypersensitvity

Asthma

How well did you know this?
1
Not at all
2
3
4
5
Perfectly
624
Q

Single unilateral nasal polyp

A

May be a sign of rare but sinister pathology e.g.

Nasopharyngela Ca

Glioma

Lymphoma

Neuroblastoma

Sarcoma

CT + histology

How well did you know this?
1
Not at all
2
3
4
5
Perfectly
625
Q

Nasal polyps in children

A

Rare in <10, must consider neoplastic disease or CF

How well did you know this?
1
Not at all
2
3
4
5
Perfectly
626
Q

Mx of nasal polyps

A

Drugs:

Betamethasone for 2/7

Short course of oral steroids

Endoscopic polypectomy

How well did you know this?
1
Not at all
2
3
4
5
Perfectly
627
Q

No direct response but intact conseunsual response to light

Cannot initiate consensual response in contralteral eye

Dilatation on moving light from normal to abnormal eye

What is the defect?

A

Afferent defect

How well did you know this?
1
Not at all
2
3
4
5
Perfectly
628
Q

Cause of afferent pupillary defect?

A

Total CNII lesion

How well did you know this?
1
Not at all
2
3
4
5
Perfectly
629
Q

Marcus-Gunn pupil=

A

RAPD

How well did you know this?
1
Not at all
2
3
4
5
Perfectly
630
Q

Minor constriction to direct light

Dilatation on moving light from normal to abnormal eye

A

RAPD- Marcus Gunn pupil

How well did you know this?
1
Not at all
2
3
4
5
Perfectly
631
Q

Causes of RAPD

A

Optic neuritis

Optic atrophy

Retinal disease

How well did you know this?
1
Not at all
2
3
4
5
Perfectly
632
Q

Dilated pupil does not react to light

Initiates consensual response in contralateral pupil

Ophthalmoplegia and ptosis

A

Efferent defect

How well did you know this?
1
Not at all
2
3
4
5
Perfectly
633
Q

Cause of efferent pupillary defect

A

3rd nerve palsy

How well did you know this?
1
Not at all
2
3
4
5
Perfectly
634
Q

Medical 3rd nerve palsy

A

Pupil sparing as the visceral constrictive fibres run on the outisde of the nerve so are spared in vascular aetiologies

How well did you know this?
1
Not at all
2
3
4
5
Perfectly
635
Q

Complete 3rd nerve palsy=

A

“surgical third”

How well did you know this?
1
Not at all
2
3
4
5
Perfectly
636
Q

Ptosis

Down and out pupil

Dilated pupil

A

?3rd nerve palsy

How well did you know this?
1
Not at all
2
3
4
5
Perfectly
637
Q

Causes of third nerve palsy

A

DM (75% pupil sparing)

Temporal arteritis

SLE

MS

Cavernous sinus thrombosis

Amyloid

PCA aneurysm

Tumour

How well did you know this?
1
Not at all
2
3
4
5
Perfectly
638
Q

Ipsilateral third nerve palsy with contralateral hemiplegia

A

Weber syndrome= midbrain stroke

How well did you know this?
1
Not at all
2
3
4
5
Perfectly
639
Q

Whence does the CNIII arise?

A

Rostral midbrain

How well did you know this?
1
Not at all
2
3
4
5
Perfectly
640
Q

What are the nuclei of CNIII?

A

Oculomotor nucleus (somatic fibres)- eye movements

Edinger-Westphal nucleus (visceral fibres)- pupillary constriciton

How well did you know this?
1
Not at all
2
3
4
5
Perfectly
641
Q

Course of the oculomotor nerve?

A

Passes between the posterior cerebral and superior cerebellar arteries and then through the cavernous sinus and out through the superior orbital fissure

How well did you know this?
1
Not at all
2
3
4
5
Perfectly
642
Q

Branches of CNIII

A

Superior branch- levator palpebrae

Inferior branch- MR, IR and IO muscles and carries the visceral fibres

How well did you know this?
1
Not at all
2
3
4
5
Perfectly
643
Q
A

3rd nerve palsy

ptosis

‘down and out’ pupil

dilated pupil

How well did you know this?
1
Not at all
2
3
4
5
Perfectly
644
Q

DDx of a fixed, dilated pupil

A

Mydriatics e.g. tropicamide

Iris trauma

Acute glaucoma

CN3 compression: tumour, coning

How well did you know this?
1
Not at all
2
3
4
5
Perfectly
645
Q
A
How well did you know this?
1
Not at all
2
3
4
5
Perfectly
646
Q
A

RAPD

How well did you know this?
1
Not at all
2
3
4
5
Perfectly
647
Q

Young woman with sudden blurring of near vision

Initially unilateral and then bilateral pupil dilatation

Dilated pupil has no response to light and sluggish response to accomodation

= a tonic pupil

A

Holmes-Adie pupil

How well did you know this?
1
Not at all
2
3
4
5
Perfectly
648
Q

Iris shows spontaneous wormy movmenets on slit-lamp examination

Iris streaming

A

Holmes-Adie pupil

How well did you know this?
1
Not at all
2
3
4
5
Perfectly
649
Q

Aetiology of Holmes-Adie pupil

A

Damage to postganglionic parasympathetic fibres

Idiopathic: may have viral aetiology

How well did you know this?
1
Not at all
2
3
4
5
Perfectly
650
Q
A

Homes-Adie pupil

How well did you know this?
1
Not at all
2
3
4
5
Perfectly
651
Q

Tonic pupil + absent knee/ankle jerks + reduced BP=

A

Holmes-Adie syndrome

How well did you know this?
1
Not at all
2
3
4
5
Perfectly
652
Q

Johann Horner

A

Swiss opthalmologist

How well did you know this?
1
Not at all
2
3
4
5
Perfectly
653
Q

Features of Horner’s syndrome

PEAS

A

Ptosis: partial (superior tarsal muscle)

Enophthalmos

Anhydrosis

Small pupil

How well did you know this?
1
Not at all
2
3
4
5
Perfectly
654
Q
A

Horner’s syndrome

How well did you know this?
1
Not at all
2
3
4
5
Perfectly
655
Q

How can the causes of Horner’s syndrome be classified?

A

Central

Pre-ganglionic

Post-ganglionic

How well did you know this?
1
Not at all
2
3
4
5
Perfectly
656
Q

What are the central causes of Horner’s

A

MS

Wallenberg’s Lateral Medullary Syndrome

How well did you know this?
1
Not at all
2
3
4
5
Perfectly
657
Q

Small, irregular pupils

Accomodate but don’t react to light

Atrophied and depigmented Iris

A

Argyll-Robertson

How well did you know this?
1
Not at all
2
3
4
5
Perfectly
658
Q

Argyll-Robertson pupil

“prostitutes pupil”

A

Accomodates but doens’t react

How well did you know this?
1
Not at all
2
3
4
5
Perfectly
659
Q

Causes of Argyll Robertson

A

DM

Quaternary syphillis

How well did you know this?
1
Not at all
2
3
4
5
Perfectly
660
Q

Reduced acuity

Reduced colour vision (especially red)

Central scotoma

Pale optic disc

RAPD

A

?Optic atrophy/opticneuropathy

How well did you know this?
1
Not at all
2
3
4
5
Perfectly
661
Q

What are the most common causes of optic atrophy?

A

MS and glaucoma

How well did you know this?
1
Not at all
2
3
4
5
Perfectly
662
Q

How can the causes of optic neuropathy be classified?

CAC VISION

A

Congenital

Alcohol and other toxins

Compression

Vascular

Inflammatory

Sarcoid

Infection

Oedema

Neoplastic infiltration

How well did you know this?
1
Not at all
2
3
4
5
Perfectly
663
Q

What are the congenital causes of optic atrophy?

A

Leber’s hereditary optic neuropathy

Hereditary motor and sensory neuropathy/ Charcot Marie Tooth

Friedrich’s ataxia

Wolfram (DIDMOAD)

Retinitis pigmentosa

How well did you know this?
1
Not at all
2
3
4
5
Perfectly
664
Q

Attacks of acute visual loss, sequential in each eye

+/- ataxia and cardiac defects

FHx

Onset in 20-30s

A

Leber’s hereditary optic neuropathy

Mitochondrial disease

How well did you know this?
1
Not at all
2
3
4
5
Perfectly
665
Q

What are the toxins causing optic neuropathy?

A

Ethambutol

Pb

B12 deficiency

How well did you know this?
1
Not at all
2
3
4
5
Perfectly
666
Q

What are the compressive causes of optic neuropathy

A

Neoplasia: optic glioma, pituitary adenoma

Glaucoma

Paget’s

How well did you know this?
1
Not at all
2
3
4
5
Perfectly
667
Q

What are the vascular causes of optic atrophy

A

DM, GCA, thromboembolic

How well did you know this?
1
Not at all
2
3
4
5
Perfectly
668
Q

What are the inflammatory causes of optic atrophy

A

MS, Devic’s, DM

How well did you know this?
1
Not at all
2
3
4
5
Perfectly
669
Q

Devic’s disease

A

Neuromyelitis optica (NMO), also known as Devic’s disease or Devic’s syndrome, is a heterogeneous condition consisting of the simultaneous inflammation and demyelination of the optic nerve (optic neuritis) and the spinal cord (myelitis). It can be monophasic or recurrent.

How well did you know this?
1
Not at all
2
3
4
5
Perfectly
670
Q

What are the infective causes of optic atrophy?

A

Herpes zoster, TB, syphillis

How well did you know this?
1
Not at all
2
3
4
5
Perfectly
671
Q

oedematous causes of optic atrophy

A

Papilloedema

How well did you know this?
1
Not at all
2
3
4
5
Perfectly
672
Q

Anteroposterior anatomy of the eyeball

A

Cornea

Anterior chamber (aqueous humours)

Uvea (Iris, ciliary body, choroid plexus)

Posterior chamber

Lens

Vitreous humour

How well did you know this?
1
Not at all
2
3
4
5
Perfectly
673
Q

What are the two muscles responsible for eyelid movement

A

Obicularis oculi- closes the eyelids, innervated by CN7 (Bell’s palsy)

Levator palpebrae- opens the eyelid, innervated by CN3 (Horner’s)

How well did you know this?
1
Not at all
2
3
4
5
Perfectly
674
Q

What is the conjunctiva

A

Mucus membrane covering the eyeball

Semi-transparent hence the white colouring

Starts at the edge of the cornea (imbus) and loops forward to form the inner surface of the eyelid.

How well did you know this?
1
Not at all
2
3
4
5
Perfectly
675
Q

Where are the majority of tears produced?

A

Accessory tear glands located within the eyelid and the conjunctiva

Lacrimal gland itself is primarily responsible for reflexive tearing.

How well did you know this?
1
Not at all
2
3
4
5
Perfectly
676
Q

What is a consideration for lid laceration in the nasal quadrant of the lid?

A

Compromising the canalicular tear-drainage pathway.

How well did you know this?
1
Not at all
2
3
4
5
Perfectly
677
Q

What is a consideration re drug administration and nasolacrimal duct

A

Nasal absorption of drugs can have profound effects.

hence why squeezing the medial canthus during administration of eye drops can help mitigate htis

How well did you know this?
1
Not at all
2
3
4
5
Perfectly
678
Q

Features of the sclera

A

White fibrous outer layer of the eyeball, continuous with the cornea anteriorly and the optic nerve sheath posteriorly

How well did you know this?
1
Not at all
2
3
4
5
Perfectly
679
Q

Anterior chamber of the eye=

A

Between cornea and the iris

How well did you know this?
1
Not at all
2
3
4
5
Perfectly
680
Q

Posterior chamber of the eye

A

Between the iris and the lens

How well did you know this?
1
Not at all
2
3
4
5
Perfectly
681
Q

Viterous chamber of the eye

A

Extends from the lens to the retina

How well did you know this?
1
Not at all
2
3
4
5
Perfectly
682
Q

What is PVD in the context of ophthalmology?

A

Posterior viterous detachment

usually benign but can cause retinal tears

How well did you know this?
1
Not at all
2
3
4
5
Perfectly
683
Q

Where in the eye is the aqueous humor produced?

A

Posterior chamber, flowing through the pupil into the anterior chamber where it drains into the venous circulation via the Canal of schlemm

How well did you know this?
1
Not at all
2
3
4
5
Perfectly
684
Q

Features of the cornea

A

Avascular receiving its nutrition from tears on the outside and aqueous fluid internally with peripheral blood vessels

How well did you know this?
1
Not at all
2
3
4
5
Perfectly
685
Q

What are the layers of the cornea

A

Epithelium

Bowman’s layer (belfry)

Stroma

Descemet’s membrane (deep)

Endothelium

How well did you know this?
1
Not at all
2
3
4
5
Perfectly
686
Q

Damage to which layer of the cornea can lead to scar formation?

A

Corneal stroma

How well did you know this?
1
Not at all
2
3
4
5
Perfectly
687
Q

What forms the anterior chamber angle

A

Angle formed by the inner cornea and the root of the iris

Location of the Schlemms canal

How well did you know this?
1
Not at all
2
3
4
5
Perfectly
688
Q

Def: uvea

A

Iris

Ciliary body (secretes aqueous fluid and controls the shape of the lens)

Choroid plexus

How well did you know this?
1
Not at all
2
3
4
5
Perfectly
689
Q

Components of the lens

A

Capsule

Nucleus

Cortex

How well did you know this?
1
Not at all
2
3
4
5
Perfectly
690
Q

How are cataracts described?

A

Described by where they occur

I.e. nucleur, cortical, subcapsular

How well did you know this?
1
Not at all
2
3
4
5
Perfectly
691
Q

What does contraction of the ciliary muscle lead to?

A

Causes the zonule ligaments holding the lens in place to relax

Allowing the lens to become rounder

How well did you know this?
1
Not at all
2
3
4
5
Perfectly
692
Q

Presbyopia

A

Difficulty focusing on nearby objects

How well did you know this?
1
Not at all
2
3
4
5
Perfectly
693
Q

What is the macula

A

Pigmented area of the retina responsible for central vision

Within the macula lies the fovea (susceptible to injury during retinal detachments)

How well did you know this?
1
Not at all
2
3
4
5
Perfectly
694
Q

What are the bones making up the orbital walls?

A

Frontal

Sphenoid (greater wing)

Ethmoid

Lacrimal

Palatine

Maxillary

Zygomatic

How well did you know this?
1
Not at all
2
3
4
5
Perfectly
695
Q

What is the entry point for the nerves and vessels supplying the orbit

A

Orbital apex

How well did you know this?
1
Not at all
2
3
4
5
Perfectly
696
Q

Nerves passing through the orbital fissure

Live Frankly To See Absolutley No Insult

A

Lacrimal and Frontal divisions of V1

Trochlear nerve

Superior division fo the oculomotor nerve

Abducens

Nasociliary branch of V1

Inferior division of III

How well did you know this?
1
Not at all
2
3
4
5
Perfectly
697
Q

What is the thinnest bone in the orbit?

A

Ethmoid

Mose likely to perforate from an eroiding sinus infection in children

How well did you know this?
1
Not at all
2
3
4
5
Perfectly
698
Q
A
How well did you know this?
1
Not at all
2
3
4
5
Perfectly
699
Q

Funciton of LR

A

Abduction

How well did you know this?
1
Not at all
2
3
4
5
Perfectly
700
Q

Function of MR

A

Adduction

How well did you know this?
1
Not at all
2
3
4
5
Perfectly
701
Q

Extraocular muscle lesion, patients report

A

Diplopia when looking in the affected direction

How well did you know this?
1
Not at all
2
3
4
5
Perfectly
702
Q

Featuers of Horner’s syndrome

PAMEL

A

Ptosis

Anhydrosis

Miosis

Enophthalmos

Loss of ciliospinal reflex

How well did you know this?
1
Not at all
2
3
4
5
Perfectly
703
Q

Ciliospinal reflex

A

The ciliospinal reflex (pupillary-skin reflex) consists of dilation of the ipsilateral pupil in response to pain applied to the neck, face, and upper trunk. If the right side of the neck is subjected to a painful stimulus, the right pupil dilates (increases in size 1-2mm from baseline). This reflex is absent in Horner’s syndrome and lesions involving the cervical sympathetic fibers. The enhanced ciliospinal reflex in asymptomatic patients with cluster headache is due to preganglionic sympathetic mechanisms.

How well did you know this?
1
Not at all
2
3
4
5
Perfectly
704
Q

Mydriasis

A

Excessive dilataion of the pupil

How well did you know this?
1
Not at all
2
3
4
5
Perfectly
705
Q

Crossed eye

Nerve palsy

A

6th nerve palsy

How well did you know this?
1
Not at all
2
3
4
5
Perfectly
706
Q

Causes of 6th nerve palsy

A

Vasculopathic

Tumour

False localising sign i.e. raised ICP

How well did you know this?
1
Not at all
2
3
4
5
Perfectly
707
Q

Action of Trochlear nerve

A

SO

Causes eye depression and intorsion

Looking towards nose: more up down

Looking laterally: more rotational

How well did you know this?
1
Not at all
2
3
4
5
Perfectly
708
Q

4th nerve palsy

A

Nasal upshoot

How well did you know this?
1
Not at all
2
3
4
5
Perfectly
709
Q

Nasal upshoot nerve palsy causing?

A

4th nerve palsy

How well did you know this?
1
Not at all
2
3
4
5
Perfectly
710
Q

Causes of 4th nerve palsy

A

Vasculopathic

Tumour

Congenital

Trauma

How well did you know this?
1
Not at all
2
3
4
5
Perfectly
711
Q

Why do pupils blow first in 3rd nerve compression

A

As the fibres controlling pupillary constriction run on the external surface of the nerve

How well did you know this?
1
Not at all
2
3
4
5
Perfectly
712
Q

Pseudotumour cerebri

A

=Idiopathic intracranial HTN

How well did you know this?
1
Not at all
2
3
4
5
Perfectly
713
Q

What are the components of visual history?

A

Vision

Sensation

Appearance

Discharge

How well did you know this?
1
Not at all
2
3
4
5
Perfectly
714
Q

What are some significant symptoms in a red eye history

Vision

A

Blurred

Distorted

Diplopia

Field defect/scotoma

Floaters/flashes

How well did you know this?
1
Not at all
2
3
4
5
Perfectly
715
Q

What are some significant symptoms in a red eye history:

Sensation

A

Irritation

Pain

Itching

Photophobia

FB

How well did you know this?
1
Not at all
2
3
4
5
Perfectly
716
Q

What are some significant symptoms in a red eye history

Appearance

A

Red: ?distribution

Lump

Puffy lids

How well did you know this?
1
Not at all
2
3
4
5
Perfectly
717
Q

What are some significant symptoms in a red eye history

Discharge

A

Watering

Sticking

Stringy

How well did you know this?
1
Not at all
2
3
4
5
Perfectly
718
Q

What are the key questions in the examination of a red eye?

A

Inspect A-P

Is acuity affected

Is the globe painful

Pupils equal and reactive?

Corena: intact, cloudy? Use fluorescein

How well did you know this?
1
Not at all
2
3
4
5
Perfectly
719
Q

What are the signs of serious disease in a red eye history?

A

Photophobia

Poor vision

Corneal fluorescein staining

Abnromal pupil

How well did you know this?
1
Not at all
2
3
4
5
Perfectly
720
Q

Red eye

++++ pain

No photoboia

Reduced acuity

Hazy/cloudy cornea

Large pupil

Raised IOP

A

?Acute glaucoma

How well did you know this?
1
Not at all
2
3
4
5
Perfectly
721
Q
A

?Acute glaucoma

How well did you know this?
1
Not at all
2
3
4
5
Perfectly
722
Q

Eclipse sign

A

The eclipse sign using an oblique flashlight has been well described as a tool to detect shallow anterior chamber. A beam of light shone from the temporal aspect of the cornea towards the root of nose produces a semicircular shadow of the iris in the nasal area. The width of the semicircular shadow gives an indication of the depth of the anterior chamber. A shallow anterior chamber produces a broader shadow compared to an anterior chamber of normal depth as it is the eclipse of the light.

How well did you know this?
1
Not at all
2
3
4
5
Perfectly
723
Q

Red eye

++ pain

++ photophobia

Reduced acuity

Pain on accomodation

Normal cornea

Small pupil

Normal IOP

A

?Anterior uveitis

How well did you know this?
1
Not at all
2
3
4
5
Perfectly
724
Q

Red eye

Abdominal pain

A

?Acute closed angle glauocma

How well did you know this?
1
Not at all
2
3
4
5
Perfectly
725
Q

Red eye

+/- pain

+ photobia

Normal acuity

Normal cornea

Normal pupil

Normal IOP

A

?Conjunctivitis

How well did you know this?
1
Not at all
2
3
4
5
Perfectly
726
Q

How can the ddx of red eye be classified?

A

Site:

Lids

Conjunctiva

Sclera

Cornea

Anterior chamber

How well did you know this?
1
Not at all
2
3
4
5
Perfectly
727
Q

What are the mechanical causes of red eye

A

Lids:

Ectropion, entropion, FB, trichiasis

Conjunctiva:

Sub cojunctival haemorrhage

Sclera:

Perforation

Cornea:

Foreign body

Abrasion

Anterior chamber;

Acute glaucoma

How well did you know this?
1
Not at all
2
3
4
5
Perfectly
728
Q

What are the inflammatory causes of red eye?

A

Lids:

Blepharitis, Chalazion

Conjunctiva:

Allergic conjunct

Sclera:

Chemical burn, episcleritis, scleritis

Anterior chamber:

Iritis/uveitis

How well did you know this?
1
Not at all
2
3
4
5
Perfectly
729
Q

What are the infective causes of red eye?

A

Lids:

Perioribital cellulitis, orbital cellulitis

Conjunctiva:

Conjunctivitis

Cornea:

Keratitis

Ant chamber:

Endophthalmitis

How well did you know this?
1
Not at all
2
3
4
5
Perfectly
730
Q
A

Anterior uveitis

How well did you know this?
1
Not at all
2
3
4
5
Perfectly
731
Q
A

Scleritis

How well did you know this?
1
Not at all
2
3
4
5
Perfectly
732
Q

Aetiology of acute closed angle glaucoma?

A

Blocked drainage of aqueous from anterior chamber vis the canal of Schlemm

Pupilar dilatation worsens the blockage

IOP rises from 15-20–> 60mmHg

How well did you know this?
1
Not at all
2
3
4
5
Perfectly
733
Q

RFs for acute closed angle glaucoma

A

Hypermetropia

Shallow anterior chamber

Female

FHx

Increased age

Drugs:

anticholinergics

sympathomimetics

TCAs

Anti-histmaines

How well did you know this?
1
Not at all
2
3
4
5
Perfectly
734
Q

Prodrome: rainbow haloes around lights at night time

Severe pain with n+v

Reduced acuity and blurred vision

A

Acute closed angle glaucoma

How well did you know this?
1
Not at all
2
3
4
5
Perfectly
735
Q

Cloudy cornea with circumcorneal injection

Fixed dilated, irregular pupil

Raised IOP makes eye feel hard

A

Acute cloesd angle glaucoma

How well did you know this?
1
Not at all
2
3
4
5
Perfectly
736
Q

Ix in acute closed angle glaucoma

A

Tonometry: raised IOP, usually >40mmHg

How well did you know this?
1
Not at all
2
3
4
5
Perfectly
737
Q

Mx of acute closed angle glaucoma

A

Refer to ophthalmology

Lie the person flat

Pilocarpine eye drops (2% blue, 4% brown) (miosis opens blockage)

Acetazolamide 500mg IV stat: reduces aqueous formation

Analgesia

+/- antiemetic

How well did you know this?
1
Not at all
2
3
4
5
Perfectly
738
Q

What are the aims of the treatment of angle closure glauocma

A

Reduce intraocular pressure

Ease symptoms

Prevent development/further progression of glaucoma

How well did you know this?
1
Not at all
2
3
4
5
Perfectly
739
Q

Definitive treatment for angle closure glaucoma

A

Laser iridotomy to allow aqueous humour to freely flow into posterior chamber.

How well did you know this?
1
Not at all
2
3
4
5
Perfectly
740
Q

Pathophysiology of uveitis

A

Inflammation of uvea (iris, ciliary body and choroid)

If just iris and ciliary body= anterior uveitis

How well did you know this?
1
Not at all
2
3
4
5
Perfectly
741
Q

Acute pain and photophobia

Pain on accomodation

Blurred vision

A

?Acute uveitis

How well did you know this?
1
Not at all
2
3
4
5
Perfectly
742
Q

Small pupil initially, irregular later

Circumcorneal injection

Hypopyon (pus in anterior chamber)

Keratic precipitates on back of cornea

Talbots test: pain on convergence

A

Anterior uveitis

How well did you know this?
1
Not at all
2
3
4
5
Perfectly
743
Q

What are the systemic associations of anterior uveitis

A

Seronegative arthritits: AS, psoriatic, Reiter’s

Stills’/JIA

IBD

Sarcoidosis

Behcet’s

Infections: TB, leprosy, syphilis, HSV, CMV, toxo

How well did you know this?
1
Not at all
2
3
4
5
Perfectly
744
Q

Mx of anterior uveitis

A

Refer to ophthalmology

Prednisolone

Cyclopentolate drops: dilates pupil and prevents adhesions between the iris and lens

How well did you know this?
1
Not at all
2
3
4
5
Perfectly
745
Q

Def: episcleritis

A

Inflammation below conjunctiva in the episclera

How well did you know this?
1
Not at all
2
3
4
5
Perfectly
746
Q

Localised reddening that can be moved over sclera

Painless/mild discomfort

Acuity preserved

A

Episcleritis

How well did you know this?
1
Not at all
2
3
4
5
Perfectly
747
Q

Causes of episcleritis

A

Usually idiopathic

May complicate RA or SLE

How well did you know this?
1
Not at all
2
3
4
5
Perfectly
748
Q

Rx in episcleritis

A

Topical or systemic NSAIDs

How well did you know this?
1
Not at all
2
3
4
5
Perfectly
749
Q

What is the most common cause of red eye?

A

Conjunctivitis

How well did you know this?
1
Not at all
2
3
4
5
Perfectly
750
Q

Def: scleritis

A

Vasculitis of the sclera

How well did you know this?
1
Not at all
2
3
4
5
Perfectly
751
Q

Severe pain worse on eye movement

Vessels won’t move over sclera

Conjunctival oedema (chemosis)

A

Scleritis

How well did you know this?
1
Not at all
2
3
4
5
Perfectly
752
Q

Causes of scleritis

A

Wegener’s

RA

SLE

Vasculitis

How well did you know this?
1
Not at all
2
3
4
5
Perfectly
753
Q

Scleritis, what should you do?

A

RFTs as may be first presentation of Wegner’s

How well did you know this?
1
Not at all
2
3
4
5
Perfectly
754
Q

Cx of scleritis

A

Scleromalacia leading to globe perforation

How well did you know this?
1
Not at all
2
3
4
5
Perfectly
755
Q

Mx of scleritis?

A

Refer to specialist

Most need corticosteroids or immunosuppressants

How well did you know this?
1
Not at all
2
3
4
5
Perfectly
756
Q

Often bilateral with purulent discharge

Bacterial: sticky (staph, strep, haemophilus)

viral: watery

Discomfort

Vessels may be moved over slcera

Acuity unaffected

A

Conjunctivitis

How well did you know this?
1
Not at all
2
3
4
5
Perfectly
757
Q

What are the classes of conjunctivitis?

A

Viral

Bacterial

Allergic

How well did you know this?
1
Not at all
2
3
4
5
Perfectly
758
Q

Rx bacterial conjunctivitis

A

Chloramphenicol 0.5% ointment

How well did you know this?
1
Not at all
2
3
4
5
Perfectly
759
Q

Rx allergic conjunctivitis

A

Anti-histamine drops e.g. emedastine

How well did you know this?
1
Not at all
2
3
4
5
Perfectly
760
Q

What are the divisions of the pharynx

A

Nasopharynx

Oropharynx

Laryngopharynx

How well did you know this?
1
Not at all
2
3
4
5
Perfectly
761
Q

Extent of the nasopharynx

A

From the posterior nares to the soft palate

How well did you know this?
1
Not at all
2
3
4
5
Perfectly
762
Q

Extent of the oropharynx

A

From the soft palate to the superior margin of the epiglottis

How well did you know this?
1
Not at all
2
3
4
5
Perfectly
763
Q

Extent of the laryngopharynx

A

From the superior margin of the epiglottis to the oesophagus

How well did you know this?
1
Not at all
2
3
4
5
Perfectly
764
Q
A
How well did you know this?
1
Not at all
2
3
4
5
Perfectly
765
Q

How are the layers of the neck arranged

A

In various fascial layers: superficial fascia which surrounds the anterior neck and is innervated by CNVII

Deep fascia: surrounds rest of the neck, several different layers

How well did you know this?
1
Not at all
2
3
4
5
Perfectly
766
Q

What are the layers of the deep fascia in the neck?

A

Investing layer: surrounds all structures within the neck

Carotid sheaths: laterally surrounds the neurovascular components

Pre-treacheal layer: ventrally, surrounds the viscera

Pre-vertebral layer: dorsally: surrounds the vertebrae and associated muscles

How well did you know this?
1
Not at all
2
3
4
5
Perfectly
767
Q
A
How well did you know this?
1
Not at all
2
3
4
5
Perfectly
768
Q

What are the four longitudinal compartments in the neck

A

2 vascular compartments: bilaterally, covered by the carotid sheath

Visceral compartment: ventrally: covered by the pre-tracheal fascia

Vertebral compartment: dorsally: covered by the pre-vertebral fascia

How well did you know this?
1
Not at all
2
3
4
5
Perfectly
769
Q

What are the contents of the vascular compartment of the neck?

A

Internal jugular vein

Common carotid

Vagus nerve

How well did you know this?
1
Not at all
2
3
4
5
Perfectly
770
Q

What are the contents of the visceral compartment of the neck?

A

Contains the pharynx, larynx, trachea, oesophagus and thyroid gland

How well did you know this?
1
Not at all
2
3
4
5
Perfectly
771
Q

What are the contents of the vertebral compartment of the neck?

A

Contains the vertebral column and associated deep muscles

How well did you know this?
1
Not at all
2
3
4
5
Perfectly
772
Q

What are the branches of the external carotid artery?

Some Anatomists Like Fucking, Others prefer S and M

A

Superior thyroid: supplies thyroid gland

Ascending pharygneal: small posterior branch supplying around the pharynx

Lingual: anterior branch to the tongue

Facial: supplies the front of the face, soft palate and submandibular glands

Occipital: supplies the posterior scalp

Posterior auricular: ssupplies the ear

Terminates at the parotid gland, bifurcating into two terminal branches:

Superficial temporal: supplies the parotid gland, lateral face and temple region

Maxillary: supplies numerous structures around the maxilla and mandible

How well did you know this?
1
Not at all
2
3
4
5
Perfectly
773
Q

Where does the common carotid bifurcate?

A

At the superior margin of the thyroid cartilage

How well did you know this?
1
Not at all
2
3
4
5
Perfectly
774
Q

How can the muscles of the neck be divided?

A

Supra and infrahyoid (strap)

How well did you know this?
1
Not at all
2
3
4
5
Perfectly
775
Q

What are the suprahyoid muscles?

A

Digastric: both insert on the medial hyoid: anterior belly (innervated by nerve to mylohyoid (of V3), posterior belly (arises from mastoid process and innervated by CNVII

Stylohyoid: arses from the base of the sytloid process and inserts on the lateral hyoid (CNVII)

How well did you know this?
1
Not at all
2
3
4
5
Perfectly
776
Q

What is the innervation of the infra-hyoid strap muscles?

A

C1-C3

How well did you know this?
1
Not at all
2
3
4
5
Perfectly
777
Q

What are the infra-hyoid muscles?

A

Omohyoid

Sternohyoid

Thyrohyoid

Sternothyroid

How well did you know this?
1
Not at all
2
3
4
5
Perfectly
778
Q

Action of the suprahyoid muscles

A

Depress the mandible or raise the hyoid

How well did you know this?
1
Not at all
2
3
4
5
Perfectly
779
Q

Actions of the infrahyoid

A

Depress the hyoid or fix the hyoid so that the suprahyoid muscles can act

How well did you know this?
1
Not at all
2
3
4
5
Perfectly
780
Q

What are the constricter muscles of the pharynx?

A

Superior: from the pterygomandibular raphe to the pharyngeal tubercle

Middle: from the hyoid bone

Inferior: from the thyroid cartilage

All overlap to make up the pharyngeal walls

How well did you know this?
1
Not at all
2
3
4
5
Perfectly
781
Q

What innervates the constrictor muscles of the pharynxx?

A

Vagus nerve

How well did you know this?
1
Not at all
2
3
4
5
Perfectly
782
Q

What are the three longitudinal muscles of the pharynx?

A

Stylopharyngeus

Salpingopharyngeus

Palatopharyngeus

All innervated by vagus except stylopharyngeus which is innervated by the glossopharyngeal

How well did you know this?
1
Not at all
2
3
4
5
Perfectly
783
Q

Which muscle of the pharynx is innervated by glossopharyngeal nerve rather than vagus?

A

Stylopharyngeus

How well did you know this?
1
Not at all
2
3
4
5
Perfectly
784
Q
A
How well did you know this?
1
Not at all
2
3
4
5
Perfectly
785
Q

def: corneal abrasion

A

Epithelial breach w/o keratitis

Cause: trauma

How well did you know this?
1
Not at all
2
3
4
5
Perfectly
786
Q

Pain

Photophobia

Blurred vision

Following trauma

A

?Corneal abrasion

How well did you know this?
1
Not at all
2
3
4
5
Perfectly
787
Q

Ix in corneal abrasion

A

Slit lamp: fluorescein stains defect green

How well did you know this?
1
Not at all
2
3
4
5
Perfectly
788
Q

Rx in corneal abrasion

A

Chloramphenicol ointment for infection prohpylaxis

How well did you know this?
1
Not at all
2
3
4
5
Perfectly
789
Q

Causes of corneal ulcer + keratitis

A

Bacterial, herpetic, fungal, protozoa, vasculitic (RA)

How well did you know this?
1
Not at all
2
3
4
5
Perfectly
790
Q
A

Corneal ulcer

How well did you know this?
1
Not at all
2
3
4
5
Perfectly
791
Q

Dendritic ulcer=

A

Herpes simplex keratitis

How well did you know this?
1
Not at all
2
3
4
5
Perfectly
792
Q
A

Dendritic ulcer

Herpes simplex keratitis

How well did you know this?
1
Not at all
2
3
4
5
Perfectly
793
Q

Acanthomoeba

A

Protozoal infection affecting contact lens wearers in swimming pools

How well did you know this?
1
Not at all
2
3
4
5
Perfectly
794
Q
A

Acanthomoeba

How well did you know this?
1
Not at all
2
3
4
5
Perfectly
795
Q

Pain, photophobia

Conjuncitval hyperaemia

Reduced acuity

White corneal opacity

A

?Corneal ulcer

How well did you know this?
1
Not at all
2
3
4
5
Perfectly
796
Q

RF for corneal ulcer?

A

Contact lens wearers

How well did you know this?
1
Not at all
2
3
4
5
Perfectly
797
Q

Ix in corneal ulcer

A

Fluorescein on slit lamp

How well did you know this?
1
Not at all
2
3
4
5
Perfectly
798
Q

Rx in corneal ulcer

A

Refer to specialist who will

Take smears and cultures

Abx drops, oral/topical aciclovir

Cyclopegics/mydriatics to ease photophobia

NB steroids may worsen symptoms

How well did you know this?
1
Not at all
2
3
4
5
Perfectly
799
Q

Complications of corneal ulcer

A

Scarring and visual loss

How well did you know this?
1
Not at all
2
3
4
5
Perfectly
800
Q

Def: ophthalmic shingles

A

VZV of CNV1

20% of all shingles

How well did you know this?
1
Not at all
2
3
4
5
Perfectly
801
Q

Pain in CNV1 dermatome preceding blistering rash

keratitis, iritis

Hutchinson’s sign

Ophthalmic involvement: keratitis + corneal ulceration

A

?Opthalmic shingles

How well did you know this?
1
Not at all
2
3
4
5
Perfectly
802
Q

Hutchinson’s sign

A

Nose-tip zoster due to involvement of nasociliary branch

Increased risk of globe involvement as nasociliary involvement also supplies globe

How well did you know this?
1
Not at all
2
3
4
5
Perfectly
803
Q
A

Ophthalmic shingles

How well did you know this?
1
Not at all
2
3
4
5
Perfectly
804
Q
A

Hutchinson’s sign

Nose tip zoster due to involvement of nasociliary branch

Increased risk of globe involvement as nasociliary branch also supplies globe

How well did you know this?
1
Not at all
2
3
4
5
Perfectly
805
Q

Key questions in sudden loss of vision

HELLP

A

Headache associated? GCA

Eye movements hurt? Optic neuritis

Lights/flashes preceding visual loss? Retinal detachment

Like curtain descending? TIA/GCA

Poorly controlled DM? Vitreous bleed from new vessels

How well did you know this?
1
Not at all
2
3
4
5
Perfectly
806
Q

Anterior ischaemic optic neuropathy

A

Optic nerve damage due to occlusion of posterior ciliary arteries, etiher by inflammation or atheroma

Pale/swollen optic disc

How well did you know this?
1
Not at all
2
3
4
5
Perfectly
807
Q

Causes of anterior ischaemic optic neuropathy?

A

Arteritic: GCA

Non-arteritic: HTN, DM, raised lipids, smoking

How well did you know this?
1
Not at all
2
3
4
5
Perfectly
808
Q
A
How well did you know this?
1
Not at all
2
3
4
5
Perfectly
809
Q

Unilateral loss of acuity over hours-days

Reduced colour discrimination (dyschromatopsia)

Eye movements may hurt

Reduced acuity

Reduced colour vision

Enlarged scotoma

Optic disc may be: swollen, normal, blurred

Afferent defect

A

?Optic neuritis

How well did you know this?
1
Not at all
2
3
4
5
Perfectly
810
Q

Causes of optic neuritis?

A

MS (45-80% >15y)

DM

Drugs: ethambutol, chloramphenicol

Vitamin deficiency

Infection: Zoster, Lyme disease

How well did you know this?
1
Not at all
2
3
4
5
Perfectly
811
Q

Rx Optic neuritis

A

High dose IV methyl predniosolone for 72h

Then oral pred for 11/7

How well did you know this?
1
Not at all
2
3
4
5
Perfectly
812
Q

What are the sources of vitreous haemorrhage

A

New vessles: DM

Retinal tears/detachment/trauma

How well did you know this?
1
Not at all
2
3
4
5
Perfectly
813
Q
A

Optic neuritis

How well did you know this?
1
Not at all
2
3
4
5
Perfectly
814
Q

Small black dots/ring floaters

Obscured vision

A

?Viterous haemrorrhage

How well did you know this?
1
Not at all
2
3
4
5
Perfectly
815
Q
A

Viterous haemorrhage

How well did you know this?
1
Not at all
2
3
4
5
Perfectly
816
Q

What can happen in large vitreous bleeds

A

Can obscure vision-> no red reflex

Difficulty visualising retina

How well did you know this?
1
Not at all
2
3
4
5
Perfectly
817
Q

Mx of vitreous haemorrhage

A

Vitrectomy may be performed in dense VH

Usually undergoes spontaneous absorption

How well did you know this?
1
Not at all
2
3
4
5
Perfectly
818
Q

Dramatic unilateral visual loss in seconds

Afferent pupil defect may precede retinal changes

Pale retina with cherry-red macula

A

Central retinal artery occlusion

How well did you know this?
1
Not at all
2
3
4
5
Perfectly
819
Q

Causes of central retinal artery occlusion

A

GCA

Thromboembolism: clot, infective, tumour

How well did you know this?
1
Not at all
2
3
4
5
Perfectly
820
Q

Rx in retinal artery occlusion

A

If seen <6h aim is to increas retinal blood flow by reducing IOP:

Ocular massage, surgical removal of aqueous, local and systemic antihypertensives

How well did you know this?
1
Not at all
2
3
4
5
Perfectly
821
Q

What is more common retinal vein or artery occlusion

A

Retinal vein occlusion

How well did you know this?
1
Not at all
2
3
4
5
Perfectly
822
Q
A

Retinal artery occlusion

Pale retina with cherry red macula

How well did you know this?
1
Not at all
2
3
4
5
Perfectly
823
Q

Causes of central retinal vein occlusion?

A

Arteriosclerosis, raised BP, DM, polycythaemia

How well did you know this?
1
Not at all
2
3
4
5
Perfectly
824
Q

Sudden unilateral visual loss with RAPD

A

?Central Retinal vein occlusion

How well did you know this?
1
Not at all
2
3
4
5
Perfectly
825
Q

Fundus:

Stormy sunset apperaance

Torutous dilated vessels

Haemorrhages

Cotton wool spots

A

?Central Retinal vein occlusion

How well did you know this?
1
Not at all
2
3
4
5
Perfectly
826
Q
A

Stormy sunset apperaance

Torutous dilated vessels

Haemorrhages

Cotton wool spots

Central retinal vein occlusion

How well did you know this?
1
Not at all
2
3
4
5
Perfectly
827
Q

Cx of central retinal vein occlusion?

A

Glaucoma

Neovascularisation

How well did you know this?
1
Not at all
2
3
4
5
Perfectly
828
Q

Px for central retinal vein occlusion

A

Possibel improvement for 6mo-1 year

How well did you know this?
1
Not at all
2
3
4
5
Perfectly
829
Q

Unilateral visual loss

Segemental fundal changes

A

?Retinal vein branch occlusion

How well did you know this?
1
Not at all
2
3
4
5
Perfectly
830
Q

Blood supply of the retina

A

Central retinal artery branches off the ophthalmic artery

How well did you know this?
1
Not at all
2
3
4
5
Perfectly
831
Q

What supplies blood to the fovea?

A

The choroid

How well did you know this?
1
Not at all
2
3
4
5
Perfectly
832
Q

Of which cartoid is the ophhthalmic artery a branch?

A

Internal

How well did you know this?
1
Not at all
2
3
4
5
Perfectly
833
Q

What is the aetiology of cherry red sport in central retinal artery occlusion

A

Retina supplied by retinal artery

Fovea supplied by the vascular choroid that surrounds the eye

Thus in retinal artery occlusion the retinal blood supply is compromised and becomes pale and swollen, while the central fovea appears reddish as the choroid colour shows throguh

How well did you know this?
1
Not at all
2
3
4
5
Perfectly
834
Q

Complications of branch artery occlusion?

A

Retinal ischaemia-> VEGF release and neovascularisation

Treated with anti-VEGFR monoclonal

How well did you know this?
1
Not at all
2
3
4
5
Perfectly
835
Q

Def: retinal detachment

A

Holes/tears in retina which allow fluid to separate sensory retina from retinal pigmented epithelium.

May be secondary to cataract surgery, trauma, DM

How well did you know this?
1
Not at all
2
3
4
5
Perfectly
836
Q

4Fs

Floaters: numerous, acute onset, spiders web

Flashes

Field loss

Fall in acuity

Painless

A

?Retinal detachment

How well did you know this?
1
Not at all
2
3
4
5
Perfectly
837
Q
A

Grey, opalescent retina ballooning forwards

?Retinal detachment

How well did you know this?
1
Not at all
2
3
4
5
Perfectly
838
Q

Rx in retinal detachment

A

Urgent surgery

Vitrectomy and gas tamponade with laser coagulation to secure the retina

How well did you know this?
1
Not at all
2
3
4
5
Perfectly
839
Q

What are the causes of transient visual loss?

A

Vascular: TIA, migraine

MS

Subacute glaucoma

Papilloedema

How well did you know this?
1
Not at all
2
3
4
5
Perfectly
840
Q

What are common causes of gradual visual loss?

A

Diabetic retinopathy

ARMD

Cataracts

Open-angle glaucoma

How well did you know this?
1
Not at all
2
3
4
5
Perfectly
841
Q

What are rarer causes of gradual visual loss?

A

Genetic retinal disease: retinitis pigmentosa

HTN

Optic atrophy

How well did you know this?
1
Not at all
2
3
4
5
Perfectly
842
Q

What is the commonest cause of blindness >60y?

A

ARMD

How well did you know this?
1
Not at all
2
3
4
5
Perfectly
843
Q

What are the risk factors for ARMD?

A

Smoking

Increasing age

Genetic factors

How well did you know this?
1
Not at all
2
3
4
5
Perfectly
844
Q

Elderly patient

Central visual loss

A

ARMD

How well did you know this?
1
Not at all
2
3
4
5
Perfectly
845
Q

What are the two types of ARMD

A

Dry: geographic atrophy and Wet: subretinal neovascularisation

How well did you know this?
1
Not at all
2
3
4
5
Perfectly
846
Q

Drusen: fluffy white spots around macula

Macula degeneration

Slow visual decline over 1-2y

A

Dry ARMD

How well did you know this?
1
Not at all
2
3
4
5
Perfectly
847
Q

Aberrant vessels growing into retina from choroid with haemorrhage

Rapid visual decline (sudden/days/weeks) with distortion

Fundoscopy shows macular haemorrhage + scarring

A

Wet ARMD

How well did you know this?
1
Not at all
2
3
4
5
Perfectly
848
Q

What can be used to detect wet ARMD

A

Amsler grid

How well did you know this?
1
Not at all
2
3
4
5
Perfectly
849
Q

What is OCT?

A

Optic coherence tomography

Used to give high resolution images of the retina

How well did you know this?
1
Not at all
2
3
4
5
Perfectly
850
Q

Rx for Wet AMRD

A

Intravitreal inhibitors

e.g. Bevacizumab

Ranibizumab

Antioxidant vitamines (C,E) and zinc may help ARMD

How well did you know this?
1
Not at all
2
3
4
5
Perfectly
851
Q

What are Bevacizumab and ranibizumab?

A

Intravitreal VEGF inhibitors

How well did you know this?
1
Not at all
2
3
4
5
Perfectly
852
Q

Optic atrophy

Loss of red/green discrimination

Scotoma

Due to toxic effects of cyanide radicals when combined with thiamine deficiency

A

Tobacco-alcohol amyblopia

How well did you know this?
1
Not at all
2
3
4
5
Perfectly
853
Q

Rx in tobacco-aclohol ambylopia

A

Vitamins may help

How well did you know this?
1
Not at all
2
3
4
5
Perfectly
854
Q

Pathogenesis of chronic simple (open-angle) glaucoma

A

Depends on susceptibility of patients retina and optic nerve to raised IOP damage

IOP >21mmHg: reduced blood flow and damage to optic nerve leading to optic disc atrophy + cupping

How well did you know this?
1
Not at all
2
3
4
5
Perfectly
855
Q

Optic disc atrophy and cupping

A

Chronic simple glaucoma

How well did you know this?
1
Not at all
2
3
4
5
Perfectly
856
Q
A
How well did you know this?
1
Not at all
2
3
4
5
Perfectly
857
Q

Peripheral visual field defect: superior nasal first

Central field is intact with acuity maintained until late

Presentation is thus delayed until optic nerve damage is irreversible

A

Chronic Simple Glaucoma

How well did you know this?
1
Not at all
2
3
4
5
Perfectly
858
Q

Def: glaucoma

A

Optic neuropathy +/- pressure (pressure is not part of definition)

How well did you know this?
1
Not at all
2
3
4
5
Perfectly
859
Q

What population are at high risk of glaucoma?

A

>35y

Afro-carribean

FHx

Drugs: steroids

Comorbidities: DM, HTN, migraines

Myopia

Screening

How well did you know this?
1
Not at all
2
3
4
5
Perfectly
860
Q

Tonometry: IOP >21mmHg

Cupping of optic disc

Visual field loss: peripheral in arcuate pattern

A

Chronic simple glaucoma

How well did you know this?
1
Not at all
2
3
4
5
Perfectly
861
Q

What are the commonest causes of blindness worldwide?

A

Trachoma

Cataracts

Glaucoma

Keratomalacia: vitamin A deficiency

Onchoceriasis

Diabetic retinopathy

How well did you know this?
1
Not at all
2
3
4
5
Perfectly
862
Q

Mx of open angle glaucoma

A

Life long F/U

Set target pressure related to degree of damage

Medical therapy first line

1st line:

Prostaglandin analogue

Latanoprost, travoprost

2nd line:

Beta blockers:

timolol, betaxalol

muscarinic alpha agonsts e.g. brominidine, apraclonidine

Carbonic anyhdrase inhibitors: dorzolamide, drops, acetazolmide PO

Miotics

Non-medical options include laser trabeculoplasty, generally if drugs fail

How well did you know this?
1
Not at all
2
3
4
5
Perfectly
863
Q

Action in treatment of chronic glaucoma:

Prostaglandin analogues

A

Increase aqueous outflow via the uveoscleral route

How well did you know this?
1
Not at all
2
3
4
5
Perfectly
864
Q

Action in treatment of chronic glaucoma:

Beta blockers

A

Reduce aqueous secretion by inhibiting beta=adrenoreceptors on the ciliary body

How well did you know this?
1
Not at all
2
3
4
5
Perfectly
865
Q

Action in treatment of chronic glaucoma:

Carbonic anhydrase inhibitors

A

Reduce aqueous secretion by ciliary body

How well did you know this?
1
Not at all
2
3
4
5
Perfectly
866
Q

Action in treatment of chronic glaucoma:

Sympathomimetics

A

Reduce aqueous secretion and increase outflow through trabecular meshwork

How well did you know this?
1
Not at all
2
3
4
5
Perfectly
867
Q

Action in treatment of chronic glaucoma:

Miotics

A

Open up the drainage channels in the trabecular meshwork by ciliary muscle contraction

How well did you know this?
1
Not at all
2
3
4
5
Perfectly
868
Q

Issue with beta blockers in treating glaucoma

A

Caution in asthma/heart failure

How well did you know this?
1
Not at all
2
3
4
5
Perfectly
869
Q

Side-effects of prostaglandin analogues in chronic glaucoma

A

Change in eye colour and lash lengthening

How well did you know this?
1
Not at all
2
3
4
5
Perfectly
870
Q

Issue with DM and the eye

A

Leading cause of blindness up to 60y

30% have ocular probelms at presentation

Good BP and sugar control reduces diabetic retinopathy

How well did you know this?
1
Not at all
2
3
4
5
Perfectly
871
Q

Pathophysiology of diabetic eye disease

A

Cataract:

DM accelerates cataract formation, lens absorbs glucose which is converted to sorbitol by aldose reducatse

Retinopathy:

Microangiopathy-> occlusion

Occlusion-> ischaemia and new vessel formation in retina: bleed-> viterous haemorrhage, carry fibrous tissue with them-> retinal detachment

Occlusion also leads to cotton wool spots (ischaemia

Vascular leakage-> oedema and lipid exudates

Rupture of micro-aneurysms-> blot haemorrhage

How well did you know this?
1
Not at all
2
3
4
5
Perfectly
872
Q

Screening of DM in context of eye disease

A

All diabetics should be screened annually

Fundus photogrophay

Refer those with maculopathy, NPDR and pDR to opathlmologist

How well did you know this?
1
Not at all
2
3
4
5
Perfectly
873
Q

Conversion rate of NPDR to DPR?

A

30% in 1y

How well did you know this?
1
Not at all
2
3
4
5
Perfectly
874
Q

Ix in DM eye disease

A

Fluorescein angiography

How well did you know this?
1
Not at all
2
3
4
5
Perfectly
875
Q

Mx of diabetic eye disease

A

Good BP and glycaemic control

Rx concurrent disease: HTN, dyslipidaemia, renal disease, smoking, anaemia

Laser photocoagulation: maculopathy: focal or grid, proliferative disease: pan-retinal

How well did you know this?
1
Not at all
2
3
4
5
Perfectly
876
Q

What EOM palsies occur in DM?

A

CNII and VI

In diabetic CNIII the pupil may be spared as its nerve fibres run peripherally and receive blood from plial vessels

How well did you know this?
1
Not at all
2
3
4
5
Perfectly
877
Q

DM eye grading

A

Background retinopathy: leakage

Pre-proliferative retinopathy: ischaemia

Proliferative retinoapthy

Maculopathy

How well did you know this?
1
Not at all
2
3
4
5
Perfectly
878
Q

Dots and blots with hard exudates

A

Background DM retinopathy

How well did you know this?
1
Not at all
2
3
4
5
Perfectly
879
Q

Dots in DM retinopathy

A

Microaneurysms

How well did you know this?
1
Not at all
2
3
4
5
Perfectly
880
Q

Blots in DM retinopathy

A

Haemorrhages

How well did you know this?
1
Not at all
2
3
4
5
Perfectly
881
Q

Hard exudates in DM retinopathy

A

Yellow lipid patches

How well did you know this?
1
Not at all
2
3
4
5
Perfectly
882
Q

Cotton wool spots

Venous beading

Dark haemorrhages

Intra-retinal microvascular abnormalities

A

Pre-proliferative retinopathy (ischaemia)

How well did you know this?
1
Not at all
2
3
4
5
Perfectly
883
Q

New vessels

Pre-retinal or vitreous haemorrhage

Retinal detachment

A

Proliferative Retinopathy

How well did you know this?
1
Not at all
2
3
4
5
Perfectly
884
Q

Macular oedema

Reduced acuity

Hard exudates within one disc width of macula

A

Maculopathy

How well did you know this?
1
Not at all
2
3
4
5
Perfectly
885
Q
A
How well did you know this?
1
Not at all
2
3
4
5
Perfectly
886
Q
A

Diabetic maculopathy

How well did you know this?
1
Not at all
2
3
4
5
Perfectly
887
Q

Anatomy relevant to nose #

A

Upper 3rd of nose has bony support

Lower 2/3rd and septum are cartilaginous

How well did you know this?
1
Not at all
2
3
4
5
Perfectly
888
Q

Aspects of #nose hx

A

Time of injury

LOC

CSF rhinorrhoea

Epistaxis

Previous nose injury

Obstruction

Consider facial# check for teech malocclusion, diplopia (orbital floor#)

How well did you know this?
1
Not at all
2
3
4
5
Perfectly
889
Q

Ix in nose #

A

Cartilaginous injury won’t show and radiographs don’t alter management

How well did you know this?
1
Not at all
2
3
4
5
Perfectly
890
Q

Mx of nose #

A

Exclude septal haematoma

Re-examine after 1w once swelling reduced

Reduction under GA with post-op splinting best within 2w

How well did you know this?
1
Not at all
2
3
4
5
Perfectly
891
Q

Features of septal haematoma

A

Septal necrosis and nasal collapse if untreated as cartilage blood supply comes from the mucosa

Boggy swelling and nasal obstruction

Needs evacuation under GA with packing and suturing

How well did you know this?
1
Not at all
2
3
4
5
Perfectly
892
Q
A

Septal haematoma

How well did you know this?
1
Not at all
2
3
4
5
Perfectly
893
Q

Causes of epistaxis

A

80% unknown

Trauma: nose picking/fractures

Local infection: URTI

Pyogenic granuloma: overgrowth of tissues on Little’s area due to irritation or hormonal factors

Osler-Weber-Rendu/HHT

Coagulopathy: Warfarin, NSAIDs, haemophilia, thrombocytopenia, vWD, EToH

Neoplasm

How well did you know this?
1
Not at all
2
3
4
5
Perfectly
894
Q

Initial mx of epistaxis

A

Wear PPE

Assess for shock and manage accordingly

If bleeding not controlled, remove clots with suction or by blowing and try to visualise bleeding through rhinoscopy

How well did you know this?
1
Not at all
2
3
4
5
Perfectly
895
Q

Post epistaxis advice

A

Don’t pick nose

Sit upright, out of sun

Avoid bending, lifting or straining

Sneeze through mouth

No hot food or drink

Avoid EtOH and tobacco

How well did you know this?
1
Not at all
2
3
4
5
Perfectly
896
Q

Inheritance of OWR/HHT

A

Autosomal dominant with 5 genetic subtypes

How well did you know this?
1
Not at all
2
3
4
5
Perfectly
897
Q

Telangectasias in mucosae: recurrent spontaneous epistaxis, painless GI bleeds

Internal telangectasias and AVMs in lungs, liver, brain

Pulmonary HTN

Colonic polyps-> CRC

A

Osler-Weber-Rendu/HHT

How well did you know this?
1
Not at all
2
3
4
5
Perfectly
898
Q
A

Osler-Weber-Rendu/HHT

How well did you know this?
1
Not at all
2
3
4
5
Perfectly
899
Q

Sore throat, fever, malaise

Lymphadenopathy esp. jugulodigastric node

Inflamed tonsils and oropharynx

Exudates

A

Tonsillitis

How well did you know this?
1
Not at all
2
3
4
5
Perfectly
900
Q

Most common cause of tonsilitis

A

Viruses (consider EBV)

How well did you know this?
1
Not at all
2
3
4
5
Perfectly
901
Q

Organisms causing tonsillitis

A

Viruses (EBV)
GAS: pyogenes

Staph

Moraxella

How well did you know this?
1
Not at all
2
3
4
5
Perfectly
902
Q

Mx of tonsiltis

A

Swabbing superficial bacteria is irrelevant and can lead to overdiagnosis

Analgesia: ibuprofen/paracetamol +/- diffiam gargle

Consider Abx only if ill, use Centor criteria:

Pen V 250mg QDS (125mg TDS in children) or erythromycin for 5/7

How well did you know this?
1
Not at all
2
3
4
5
Perfectly
903
Q

Why is amoxicillin not used in tonsilitis

A

EBV-> maculopapular rash

How well did you know this?
1
Not at all
2
3
4
5
Perfectly
904
Q

Centor criteria

A

Fever >38 by history

Tender anterior cervical adenopathy

Tonsilar exudates

Absence of cough

How well did you know this?
1
Not at all
2
3
4
5
Perfectly
905
Q

Cut offs for Centor criteria

A

0-1: no Abx (risk of strep infection <10%)

2: consider rapid Ag test + Rx if +ve
3: Abx

How well did you know this?
1
Not at all
2
3
4
5
Perfectly
906
Q

Indications for tonsillectomy

A

Recurrent tonsilitis if all of the below criteria are met:

Caused by tonsilitis

5+ epsidoes/y

Symptoms >1y

Episodes are disabling and prevent normal functioning

Quinsy

Suspicion of carcinoma: unilateral enlargement or ulceration

How well did you know this?
1
Not at all
2
3
4
5
Perfectly
907
Q

Methods of tonsillectomy

A

Cold steel

Cautery

How well did you know this?
1
Not at all
2
3
4
5
Perfectly
908
Q

Cx of tonsillectomy

A

Reactive haemorrhage

Tonsillar gag may damage teeth, TMJ or posterior pharyngeal wall

Mortality is 1/30000

How well did you know this?
1
Not at all
2
3
4
5
Perfectly
909
Q
A

EBV tonsillitis

How well did you know this?
1
Not at all
2
3
4
5
Perfectly
910
Q
A

Strep tonsillitis

How well did you know this?
1
Not at all
2
3
4
5
Perfectly
911
Q

Trismus, odonophagia, halitosis

Tonsillitis, unilateral tonsilar enlargement, contralateral uvula displacement, cervical lymphadenopathy

Typically occuring in adults

A

?Quinsy

How well did you know this?
1
Not at all
2
3
4
5
Perfectly
912
Q
A

Quinsy

How well did you know this?
1
Not at all
2
3
4
5
Perfectly
913
Q

Mx of Quinsy abscess

A

Admit

IV Abx

I&D under LA or tonsillectomy under GA

How well did you know this?
1
Not at all
2
3
4
5
Perfectly
914
Q

Unwell child with stiff extended neck who refuses to eat and drink

Fails to improve with IV Abx

Unilateral swelling of tonsil and neck

A

?Retropharyngeal absceess

How well did you know this?
1
Not at all
2
3
4
5
Perfectly
915
Q

Ix in retropharyngeal abscess

A

Lateral neck XR show soft tissue swelling

CT skull and thorax

How well did you know this?
1
Not at all
2
3
4
5
Perfectly
916
Q

Rx in Retropharyngeal abscess

A

IV Abx

I&D

How well did you know this?
1
Not at all
2
3
4
5
Perfectly
917
Q

Throbophlebitis of the internal jugular vein occuring as a complication of bacterial sore throat infection

Most commonly affect lungs

A

Lemierre’s syndrome

How well did you know this?
1
Not at all
2
3
4
5
Perfectly
918
Q

Which bacteria is associated with Lemierre’s syndrome?

A

Fusobacterium necrophorum

How well did you know this?
1
Not at all
2
3
4
5
Perfectly
919
Q

Rx in Lemierre’s syndrome

A

IV Abx: pen G, clindamycin, metronidazole

How well did you know this?
1
Not at all
2
3
4
5
Perfectly
920
Q

Sandpaper like rash on chest, axillae or behind ears 12-48h after pharyngotonsillits

Circumoral pallor

Strawberry tongue

A

Scarlet fever

How well did you know this?
1
Not at all
2
3
4
5
Perfectly
921
Q

Rx in Scarlet fever

A

Start Pen V/G and notify HPA

How well did you know this?
1
Not at all
2
3
4
5
Perfectly
922
Q

Rheumatic fever

CASES

A

Carditis

Arthritis

Subcutaenous nodules

Erythema marginatum

Sydenham’s chorea

How well did you know this?
1
Not at all
2
3
4
5
Perfectly
923
Q

Malaise and smoky urine occuring 1-2w after pharyngitis

A

?Post-streptococcal GN

How well did you know this?
1
Not at all
2
3
4
5
Perfectly
924
Q

What are the functions of the larynx?

A

Phonation

Positive thoracic pressure including auto-PEEP

Respiration

Prevention of aspiriation

How well did you know this?
1
Not at all
2
3
4
5
Perfectly
925
Q

Features of Laryngitis

A

Usually viral and self-limiting

2o bacterial infection may develop

How well did you know this?
1
Not at all
2
3
4
5
Perfectly
926
Q

Pain, hoarseness and fever

Redness and swelling of the vocal cords

A

?Laryngitis

How well did you know this?
1
Not at all
2
3
4
5
Perfectly
927
Q

Rx in laryngitis

A

Supportive, Pen V if necessary

How well did you know this?
1
Not at all
2
3
4
5
Perfectly
928
Q

Pedunculated vocal cord swellings caused by HPV

Present with hoarsness

Usually occur in children

Rx laser removal

A

Laryngeal papilloma

How well did you know this?
1
Not at all
2
3
4
5
Perfectly
929
Q

Hoarseness

Breathy voice with bovine cough

Repeated coughing from aspiration due to reduced supraglottic sensation

Exertional dyspnoea due to narrow glottis

A

Recurrent laryngeal nerve palsy

How well did you know this?
1
Not at all
2
3
4
5
Perfectly
930
Q

Action of the recurrent laryngeal nerve

A

Supplies all of the intrinsic muscles of the larynx except cricothyroideus

Exterior branch of superior laryngeal nerve

Responsible for ab and adduction of the vocal folds

How well did you know this?
1
Not at all
2
3
4
5
Perfectly
931
Q

Causes of laryngeal nerve palsy

A

30% are cancers: larynx, thyroid, oesophagus, hypopharynx, bronchus

25% iatrogenic: para-/thyroidectomy, carotid endarterectomy

Other: aortic aneurysm, bulbar/pseudobulbar palsy

How well did you know this?
1
Not at all
2
3
4
5
Perfectly
932
Q
A
How well did you know this?
1
Not at all
2
3
4
5
Perfectly
933
Q

Associations of laryngeal SCC

A

Smoking, EtoH

How well did you know this?
1
Not at all
2
3
4
5
Perfectly
934
Q

Male smoker

Progressive hoarseness-> stridor

Dys/odono-phagia

Weight loss

A

?Laryngeal SCC

How well did you know this?
1
Not at all
2
3
4
5
Perfectly
935
Q

Trismus

A

spasm of the jaw muscles, causing the mouth to remain tightly closed, typically as a symptom of tetanus.

Origin

How well did you know this?
1
Not at all
2
3
4
5
Perfectly
936
Q

Ix in laryngeal SCC

A

Laryngoscopy + biopsy

MRI staging

How well did you know this?
1
Not at all
2
3
4
5
Perfectly
937
Q

Mx of laryngeal SCC

A

Based on stage

RTx

Laryngectomy

How well did you know this?
1
Not at all
2
3
4
5
Perfectly
938
Q

Post total laryngectomy

A

Patients have a permanent tracheostomy

  • speech valve
  • electrolarynx
  • oesophageal speech

Regular f/u for recurrence

How well did you know this?
1
Not at all
2
3
4
5
Perfectly
939
Q

Immature and floppy aryepiglottic folds and glottis leading to laryngeal collapse on inspiration

Stridor

Seen in neonates

A

Laryngomalacia

How well did you know this?
1
Not at all
2
3
4
5
Perfectly
940
Q

What is the commonest cause of stridor in children?

A

Laryngomalacia

How well did you know this?
1
Not at all
2
3
4
5
Perfectly
941
Q

Stridor

Presenting within first weeks of life

Noticeable at certain times: lying on back, feeding, excited/ upset

Problems can occur with concurrent laryngeal infections

Conservative management

A

Laryngomalacia

How well did you know this?
1
Not at all
2
3
4
5
Perfectly
942
Q

Sudden onset

Continuous stridor

Drooling

Toxic

A

?Epiglottits

How well did you know this?
1
Not at all
2
3
4
5
Perfectly
943
Q

Pathogens causing epiglottitis

A

Haemophilus

GAS

How well did you know this?
1
Not at all
2
3
4
5
Perfectly
944
Q

Rx in epiglottitis

A

Don’t examine

Consult anaesthetist and ENT

O2 + nebulised adrenaline

IV dexamethasone

Cefotaxime

Take to theatre for airway securing

How well did you know this?
1
Not at all
2
3
4
5
Perfectly
945
Q

Sudden onset stridor in a previously normal child

A

?FB

How well did you know this?
1
Not at all
2
3
4
5
Perfectly
946
Q

Mx of foreign body in throat

A

Encourage cough

Back slaps

Needle cricothyrotomy if necessary

FB in bronchus can only be excluded through bronchoscopy

How well did you know this?
1
Not at all
2
3
4
5
Perfectly
947
Q

Stridor

FTT

A

?Subglottic stenosis

How well did you know this?
1
Not at all
2
3
4
5
Perfectly
948
Q

Causes of subglottic stenosis

A

Prolonged intubation

Congential abnormalities

How well did you know this?
1
Not at all
2
3
4
5
Perfectly
949
Q

Def: Bell’s palsy

A

Inflammatory oedema from entrapment of CNVII in narrow facial canal

Probably of viral origin

75% of facial palsy

How well did you know this?
1
Not at all
2
3
4
5
Perfectly
950
Q

Sudden onset (e.g. overnight)

Complete, unilateral facial weakness

Failure of eye closure-> dryness and conjunctivitis

Drooling/speech difficulty

Numbness or pain around the ear

Reduced taste

Hyperacusis

A

Bell’s palsy

How well did you know this?
1
Not at all
2
3
4
5
Perfectly
951
Q

Ix in Bell’s palsy

A

Serology: borrelia or VZV Abs

MRI: SOL, stroke, MS

LP

How well did you know this?
1
Not at all
2
3
4
5
Perfectly
952
Q

Mx of Bell’s palsy

A

Protect eye: dark glasses, artifical tears, tape closed at night

Give high dose prednisolone within 72h: 60mg/d PO for 5/7 followed by tapering

Valaciclovir if ?zoster, otherwise antivirals don’t help

Plastic surgery if no recovery

How well did you know this?
1
Not at all
2
3
4
5
Perfectly
953
Q

Complications of Bell’s palsy

A

Aberrant neural connections:

Synkinesis e.g. blinking causes up-turning of mouth

Crocodile tears: eating stimulates unilateral lacrimation rather than salivation

How well did you know this?
1
Not at all
2
3
4
5
Perfectly
954
Q

Px of Bell’s palsy

A

Incomplete paralysis usually recovers completely within weeks

With complete lesions, 80% get full recovery but the remainder may have delayed or permanent neurological/cosmetic abnormalities

How well did you know this?
1
Not at all
2
3
4
5
Perfectly
955
Q

Hx of Ramsay Hunt syndrome

A

American neurologist James Ramsay Hunt in 1907

How well did you know this?
1
Not at all
2
3
4
5
Perfectly
956
Q

Def: Ramsay Hunt Syndrome

A

Reactivation of VZV in geniculate ganglion of CNVII

How well did you know this?
1
Not at all
2
3
4
5
Perfectly
957
Q

Preceding ear pain or stiff neck

Vesciular rash in auditory canal +/- TM, pinna, tongue, hard palate

Ipsilateral facial weakness

May also affect CNVIII-> vertigo, tinnitus, deafness

A

Ramsay Hunt Syndrome

How well did you know this?
1
Not at all
2
3
4
5
Perfectly
958
Q

What is Ramsay Hunt syndrome without the rash known as?

A

Zoster sine herpete

How well did you know this?
1
Not at all
2
3
4
5
Perfectly
959
Q

Mx of Ramsay Hunt Syndrome

A

Valaciclovir and prednisolone within first 72h

How well did you know this?
1
Not at all
2
3
4
5
Perfectly
960
Q

Px of Ramsay Hunt?

A

Rxed within 72h: 75% recovery

Otherwise: 1/3 full, 1/3 partial, 1/3 poor

How well did you know this?
1
Not at all
2
3
4
5
Perfectly
961
Q

What features may suggest an alternative cause of facial palsy?

A

Bilateral symptoms (Lyme, GBS, leukaemia, sarcoid)

UMN signs: sparing of frontalis and obicularis oculi

Other CN palsies (also seen in 8% of Bell’s)

Limb weakness

Rahses

How well did you know this?
1
Not at all
2
3
4
5
Perfectly
962
Q
A

Ramsay Hunt Snydrome

How well did you know this?
1
Not at all
2
3
4
5
Perfectly
963
Q

How can the causes of facial nerve palsy be classified?

A

Intracranial

Intratemporal

Infratemporal

Systemic

How well did you know this?
1
Not at all
2
3
4
5
Perfectly
964
Q

What are the intracranial lesions causing facial nerve palsy?

A

Vascular, MS, SOL: motor cortex= UMN signs. Brainstem nuclei= LMN signs

Cerebellopontine angle lesion: may be accompanied by 5th, 6th and 8th CN palsy

How well did you know this?
1
Not at all
2
3
4
5
Perfectly
965
Q

What are the intratemporal lesions causing facial nerve palsy?

A

Otitis media

Cholesteatoma

Ramsay Hunt

How well did you know this?
1
Not at all
2
3
4
5
Perfectly
966
Q

What are the infratemporal lesions causing Facial nerve palsy

A

Parotid tumours

Trauma

How well did you know this?
1
Not at all
2
3
4
5
Perfectly
967
Q

What are the potential systemic causes of facial nerve palsy?

A

Peripheral neuropathy e.g. GBS (demyelinating) or DM, Lyme, HIB, Sarcoid (axonal)

Pseudopalsy e.g. MG, botulism

How well did you know this?
1
Not at all
2
3
4
5
Perfectly
968
Q

What are the components of CNVII function

A

Motor: innervates the muscles of facial expression, the posterior belly of digastric, the stylohyoid and stapedius muscles

Sensory: nil

Special sensory: taste sensation to anterior 2/3rds of the tongue

PNS: supplies the submandibular, sublingual, nasal, palatine, lacrimal and pharyngeal glands

How well did you know this?
1
Not at all
2
3
4
5
Perfectly
969
Q

What provides taste sensation to anterior 2/3rds of tongue?

A

Facial nerve

How well did you know this?
1
Not at all
2
3
4
5
Perfectly
970
Q

Intracranial course of the facial nerve

A

Arises in the pons, begins as two roots a large motor and small sensory root (intermediate nerve is the part of the facial nerve that arises from the sensory root)

The two roots travel through the internal acoustic meatus, a 1cm opening in the petrous part of the temporal bone

Leaves the internal acoustic meatus and enters the facial canal where:

The two roots fuse to form the facial nerve.

The nerve forms the geniculate ganglion

The nerve gives rise to the greater petrosal nerve, the nerve to staepdius and the chorda tympani

Exits via the facial canal, posterior to the styloid process of the temporal bone

How well did you know this?
1
Not at all
2
3
4
5
Perfectly
971
Q

Greater petrosal nerve

A

Branch of facial nerve, PNS fibres to glands

How well did you know this?
1
Not at all
2
3
4
5
Perfectly
972
Q

Nerve to stapedius

A

Facial nerve branch

Motor fibres to stapedius muscle

How well did you know this?
1
Not at all
2
3
4
5
Perfectly
973
Q

Chorda tympani

A

Branch of the facial nerve, special sensory fibres to the anterior 2/3rds of the tongue

How well did you know this?
1
Not at all
2
3
4
5
Perfectly
974
Q

What innervates the parotid gland

A

Sensory and autonomic innervation

Sensory innervation is supplied by the auriculotemporal nerve (V3)

Parasympathetic innervation begins with CNIX which then synapses with the otic ganglion. The auriculotemporal nerve then carries fibres from the otic ganglion to the parotid

Sympathetic innervation originates from the superior cervical ganglion.

How well did you know this?
1
Not at all
2
3
4
5
Perfectly
975
Q
A
How well did you know this?
1
Not at all
2
3
4
5
Perfectly
976
Q

Extracranial course of the facial nerve

A

First branch is the posterior aruicular nerve- provides motor innervation to some of the muscles around the ear, immediately dital to this it sends branches to the posterior belly of digastric and the stylohyoid muscle

Main trunk, now termed the motor root continuse anteriorly and inferiorly to the parotid (which it does not innervate)

Within the parotid gland, the nerve splits into its 5 terminal branches

TZBMC

How well did you know this?
1
Not at all
2
3
4
5
Perfectly
977
Q

Posterior auricular nerve

A

Branch of the facial nerve

Ascends in front of the mastoid process and innervates the intrinsic and extrinsic muscles of the outer ear

How well did you know this?
1
Not at all
2
3
4
5
Perfectly
978
Q

What is innervated by (of facial nerve)

Temporal

A

Innervates the frontalis, obicularis oculi and corrugator supercili

How well did you know this?
1
Not at all
2
3
4
5
Perfectly
979
Q

What is innervated by (of facial nerve)

Zygomatic

A

Innervates obicularis ori

How well did you know this?
1
Not at all
2
3
4
5
Perfectly
980
Q

What is innervated by (of facial nerve)

Buccal

A

Innervates the obicularis oris, buccinator and zygomaticus

How well did you know this?
1
Not at all
2
3
4
5
Perfectly
981
Q

What is innervated by (of facial nerve)

Marignal mandibular branch

A

Innervates the mentalis muscle

How well did you know this?
1
Not at all
2
3
4
5
Perfectly
982
Q

What is innervated by (of facial nerve)

Cervical branch

A

Innervates the platysma

How well did you know this?
1
Not at all
2
3
4
5
Perfectly
983
Q

Increasing myopia

Blurred vision-> gradual visual loss

Dazzling in sunshine/bright lights

Monocular diplopia

A

?Cataracts

How well did you know this?
1
Not at all
2
3
4
5
Perfectly
984
Q

Causes of cataracts?

A

Age: >75% of >65

DM

Steroids

Congenital:

Idiopathic

Infeciton: rubella

Metabolic: Wilson’s, galactosaemia

Myotonic dystrohpy

How well did you know this?
1
Not at all
2
3
4
5
Perfectly
985
Q

Ix in cataracts

A

Visual acuity

DIlated fundoscopy

Tonometry

Blood glucose to exclude DM

How well did you know this?
1
Not at all
2
3
4
5
Perfectly
986
Q

Mx of cataracts

A

Conservative: glasses

Medical: mydriatic drops and sunglasses may give some relief

Sx:

Consider if symptoms affect lifestyle or driving

Day case surgery

Pacoemulsion and lens implant

How well did you know this?
1
Not at all
2
3
4
5
Perfectly
987
Q

Cx of cataracts sx

A

1% risk of serious complication

Anterior uveitis/iritis

VH

Retinal detachment

Secondary glaucoma

Endophthalmitis (blindness in 0.1%)

Post op capsule thickening is common and treated with laser capsulotomy

Post-op eye irritation is common and requires drops

How well did you know this?
1
Not at all
2
3
4
5
Perfectly
988
Q

Def: The retina

A

Outer pigmented layer in contact with the choroid

Inner sensory layer in contact with vitreous

At the centre is the fovea

How well did you know this?
1
Not at all
2
3
4
5
Perfectly
989
Q

Colour of the optic disc

A

Pale pink

How well did you know this?
1
Not at all
2
3
4
5
Perfectly
990
Q

Pale optic disc

A

?Optic atrophy

How well did you know this?
1
Not at all
2
3
4
5
Perfectly
991
Q

Blurred margins of optic disk

A

?Papilloedema and optic neuritis

How well did you know this?
1
Not at all
2
3
4
5
Perfectly
992
Q

What are the features of the optic disk

A

Colour

Contour

Cup: physiological- 1/3rd. Cup widening and deepening seen in glaucoma

How well did you know this?
1
Not at all
2
3
4
5
Perfectly
993
Q
A

Normal retina

How well did you know this?
1
Not at all
2
3
4
5
Perfectly
994
Q
A

Optic atrophy

How well did you know this?
1
Not at all
2
3
4
5
Perfectly
995
Q
A
How well did you know this?
1
Not at all
2
3
4
5
Perfectly
996
Q

What is the most prevalent inherited degeneration of the macula?

A

Retinitis pigmentosa

Various modes of inheritance

Mostly AR, AD has best prognosis

X-linked has worst prognosis

Affects 1/2000

How well did you know this?
1
Not at all
2
3
4
5
Perfectly
997
Q

Night blindness

Reduced visual fields-> tunnel vision

Most registered blind by mid 30s

A

?Retinitis pigmentosa

How well did you know this?
1
Not at all
2
3
4
5
Perfectly
998
Q

Pale optic disc

Peripheral retinal pigmentation

A

?Retinitis pigmentosa

How well did you know this?
1
Not at all
2
3
4
5
Perfectly
999
Q
A

Classic Retinitis pigmentosa, with mid-peripheral bony spicules, baring of RPE, vessel attenuation, and sparing of central macula

How well did you know this?
1
Not at all
2
3
4
5
Perfectly
1000
Q
A

DM retinopathy post laser treatment

How well did you know this?
1
Not at all
2
3
4
5
Perfectly
1001
Q

What conditions are associated with retinitis pigmentosa?

A

Friedrich’s ataxia

Refsum’s disease

Kearns-Sayre syndrome

Usher’s syndrome

How well did you know this?
1
Not at all
2
3
4
5
Perfectly
1002
Q

Inheritance of retinoblastoma

A

Herediatry type differs from non-hereditary type

AD mutations of RB gene

Patients typically have on mutant allele in every retinal cell, if the other alleles mutates-> retinoblastoma

2 HIT hypothesis

How well did you know this?
1
Not at all
2
3
4
5
Perfectly
1003
Q

With what conditions are retinoblastomas associated?

A

5% occur with pineal or other tumour

Increased risk of osteosarcoma and rhabdomyosarcoma

How well did you know this?
1
Not at all
2
3
4
5
Perfectly
1004
Q

Strabismus

Leukocoria

A

?Retinoblastoma

How well did you know this?
1
Not at all
2
3
4
5
Perfectly
1005
Q
A

Leucokoria

?Retinoblastoma

How well did you know this?
1
Not at all
2
3
4
5
Perfectly
1006
Q

An abscess/infection in a lash follicle which points outwards

Rx with local Abx e.g. fusidic acid

A

Stye or horeolum externum

How well did you know this?
1
Not at all
2
3
4
5
Perfectly
1007
Q
A

Hordeolum externum (stye)

How well did you know this?
1
Not at all
2
3
4
5
Perfectly
1008
Q

Abscess of the Melbomian glands which point inwards onto the conjunctiva

A

Chalazion or hoedolum internum

How well did you know this?
1
Not at all
2
3
4
5
Perfectly
1009
Q
A

Hordeolum internum/ Chalazion

How well did you know this?
1
Not at all
2
3
4
5
Perfectly
1010
Q

Def: Blepharitis

A

Chronic inflammation of the eyelid

How well did you know this?
1
Not at all
2
3
4
5
Perfectly
1011
Q

Red eyes

Gritty/itchy sensation

Scales on lashes

Rosacea

A

?Blepharitis

How well did you know this?
1
Not at all
2
3
4
5
Perfectly
1012
Q

Causes of Blepharitis

A

Seborrhoic dermatitis

Staphs

How well did you know this?
1
Not at all
2
3
4
5
Perfectly
1013
Q

Rx in Blepharitis

A

Clean crusts of lashes with warm soaks

May need fusidic acid drops

How well did you know this?
1
Not at all
2
3
4
5
Perfectly
1014
Q
A

Blepharitis

How well did you know this?
1
Not at all
2
3
4
5
Perfectly
1015
Q

Entropion

A

Lid inversion-> corneal irritation

Degeneration of the lower lid fascia

How well did you know this?
1
Not at all
2
3
4
5
Perfectly
1016
Q

Ectropion

A

Low lid eversion-> watering and exposure keratitis

Associated with ageing and facial N. palsy

How well did you know this?
1
Not at all
2
3
4
5
Perfectly
1017
Q

Def: true ptosis

A

Intrinisc LPS weakness

How well did you know this?
1
Not at all
2
3
4
5
Perfectly
1018
Q

Causes of bilateral ptosis

A

Congenital

Senile

MG

Myotonic dystrophy

How well did you know this?
1
Not at all
2
3
4
5
Perfectly
1019
Q

Causes of unilateral ptosis

A

3rd Nerve palsy

Horner’s (partial)

Mechanical: xanthelasma, trauma

How well did you know this?
1
Not at all
2
3
4
5
Perfectly
1020
Q

Def: lagophthalmos

A

Difficulty in lid closure over the globe which may lead to exposure keratitis

How well did you know this?
1
Not at all
2
3
4
5
Perfectly
1021
Q

Causes of lagophthalmos

A

Exophthalmos

Facial palsy

Injury

How well did you know this?
1
Not at all
2
3
4
5
Perfectly
1022
Q

Rx of lagophthalmos

A

Lubricate eyes

Temporary tarsorrhaphy may be needed if corneal ulcers develop

How well did you know this?
1
Not at all
2
3
4
5
Perfectly
1023
Q

Pinguecula

A

Yellow vascular nodules either side of the cornea

How well did you know this?
1
Not at all
2
3
4
5
Perfectly
1024
Q
A

Pinguecula

How well did you know this?
1
Not at all
2
3
4
5
Perfectly
1025
Q

Pterygium

A

Similar to pinguecula but grows over the cornea-> reduced vision

Benign growth of conjunctiva

Associated with dusty, wind-blown life styles, sun exposure

How well did you know this?
1
Not at all
2
3
4
5
Perfectly
1026
Q
A

Pterygium

How well did you know this?
1
Not at all
2
3
4
5
Perfectly
1027
Q

Pathophysiology of orbital cellulitis

A

Infection spreading locally e.g from paranasal sinuses, eyelid or external eye

Staphs, pneumococcus, GAS

How well did you know this?
1
Not at all
2
3
4
5
Perfectly
1028
Q

Inflammation of the orbit and lid swelling

Pain and reduced range of eye movement

Exopthalmos

Systemic signs e.g. fever

Tenderness over the sinuses

A

Orbital cellulitis

How well did you know this?
1
Not at all
2
3
4
5
Perfectly
1029
Q
A

Orbital cellulitis

How well did you know this?
1
Not at all
2
3
4
5
Perfectly
1030
Q

Rx in orbital cellulitis

A

IV Abx: Cefuroxime (20mg/kg/8h IV)

How well did you know this?
1
Not at all
2
3
4
5
Perfectly
1031
Q

Cx of orbital cellulitis

A

Local extension-> meningitis and cavernous sinus thrombosis

Blindness due to optic nerve pressure

How well did you know this?
1
Not at all
2
3
4
5
Perfectly
1032
Q

Features of carotico-cavernous fistula

A

May fallow carotid aneurysm rupture with reflex of blood into cavernous sinus

Spontaneous/trauma

How well did you know this?
1
Not at all
2
3
4
5
Perfectly
1033
Q

Engorgement of eye vessels

Lid and conjunctival oedema

Pulsatile exophthalmos

Eye bruit

A

Carotico-cavernous fistula

How well did you know this?
1
Not at all
2
3
4
5
Perfectly
1034
Q
A

Carotid-cavernous fistula

How well did you know this?
1
Not at all
2
3
4
5
Perfectly
1035
Q

Exophthalmos/proptosis

A

Protrusion of one or both eyes

How well did you know this?
1
Not at all
2
3
4
5
Perfectly
1036
Q

Common causes of proptosis

A

Graves: 25-50%, increased in smoker’s, anti-TSH Abs-> retro-orbital inflammation and lymphocyte infiltration leading to swelling

Orbital cellulitis

Trauma

How well did you know this?
1
Not at all
2
3
4
5
Perfectly
1037
Q

Other causes of proptosis

A

Idiopathic orbital inflammatory disease

Vasculitis- Wegener’s

Neoplasm: lymhpoma, optic glioma, capillary haemangioma, metastatic

Carotico-cavernous fistula

How well did you know this?
1
Not at all
2
3
4
5
Perfectly
1038
Q

What is used to classify hypertensive retinopathy?

A

Keith-Wagener Classification

How well did you know this?
1
Not at all
2
3
4
5
Perfectly
1039
Q

Keith-Wagener classification of hypertensive retinopathy

A
  1. Tortuosity and silver wiring
  2. AV nipping
  3. Flame haemorrhages and soft/cotton wool spots
  4. Papilloedema

Grades 3 and 4= malignant HTN

How well did you know this?
1
Not at all
2
3
4
5
Perfectly
1040
Q
A

A. Earliest sign of hypertension showing only generalized narrowing of the arterioles with no change in the vessel wall thickness or reflex. B. Grade II hypertension showing generalized narrowing plus focal constriction and grade I arteriolar sclerosis with widening of the reflex stripe. C. Grade III hypertension showing generalized narrowing, focal constriction, hemorrhages, and exudate, and grade I arteriolar sclerosis with widening of the light reflex. D. Grade IV hypertension showing generalized narrowing, focal constriction, hemorrhages, and exudates and edema of the disc with grade I arteriolar sclerosis

How well did you know this?
1
Not at all
2
3
4
5
Perfectly
1041
Q

What are the granulomatous disorders causing uveitis and choroidoretinitis

A

TB, sarcoid, toxoplasmosis, leprosy, brucella

How well did you know this?
1
Not at all
2
3
4
5
Perfectly
1042
Q

What inflammatory diseases are associated with:

Conjunctivitis

A

SLE, reactive arthritis, IBD

How well did you know this?
1
Not at all
2
3
4
5
Perfectly
1043
Q

What inflammatory diseases are associated with:

Scleritis/episcleritis

A

RA, vasculitis, SLE, IBD

How well did you know this?
1
Not at all
2
3
4
5
Perfectly
1044
Q

What inflammatory diseases are associated with:

Iritis

A

Ank spond

IBD

Sarcoid

How well did you know this?
1
Not at all
2
3
4
5
Perfectly
1045
Q

What inflammatory diseases are associated with:

Retinopathy

A

Dermatomyositis

How well did you know this?
1
Not at all
2
3
4
5
Perfectly
1046
Q

Reduced tear production

Dry eyes and ry mouth

1o

or 2o: SLE< RA, sarcoid

A

Keratoconjuncitivities SIcca/ Sjogren’s

How well did you know this?
1
Not at all
2
3
4
5
Perfectly
1047
Q

Roth spots associated with?

A

Infective endocarditis

As a consequence of microemboli

How well did you know this?
1
Not at all
2
3
4
5
Perfectly
1048
Q
A

Boat shaped haemorrhage with pale centre

Roth spot

Infective endocarditis

How well did you know this?
1
Not at all
2
3
4
5
Perfectly
1049
Q
A

Kayser-Fleischer Rings

Wilson’s

How well did you know this?
1
Not at all
2
3
4
5
Perfectly
1050
Q
A

Exophthalmos

Grave’s disease

How well did you know this?
1
Not at all
2
3
4
5
Perfectly
1051
Q
A

Corneal calcification

Hyperparathyroidism

How well did you know this?
1
Not at all
2
3
4
5
Perfectly
1052
Q
A

CMV retinitis

Pizza-pie fundus + flames

How well did you know this?
1
Not at all
2
3
4
5
Perfectly
1053
Q
A

HIV retinopathy

Cotton wool spots

How well did you know this?
1
Not at all
2
3
4
5
Perfectly
1054
Q

What are the mydriatics?

A

Anti-muscarinics

Sympathomimetics

How well did you know this?
1
Not at all
2
3
4
5
Perfectly
1055
Q

Indications for mydriatics

A

Eye examination

Prevention of snechiae in anterior uveitis/iritis

How well did you know this?
1
Not at all
2
3
4
5
Perfectly
1056
Q

Caution with mydriatics?

A

May lead to acute glaucoma if shallow anterior chamber

How well did you know this?
1
Not at all
2
3
4
5
Perfectly
1057
Q

Class of drug:

Tropicamide, cyclopentolate

A

Anti-muscarinic

Tropicamide -3h

Cyclopentolate- 24h

How well did you know this?
1
Not at all
2
3
4
5
Perfectly
1058
Q

Effects of anit-muscarinics on eye

A

Pupil dilatation and loss of light reflex

Cyclopegia-> blurred vision

How well did you know this?
1
Not at all
2
3
4
5
Perfectly
1059
Q

What class of drug is pilocarpine?

A

Muscarinic agonist

How well did you know this?
1
Not at all
2
3
4
5
Perfectly
1060
Q

Use of pilocarpine

A

Acute closed-angle galucoma

How well did you know this?
1
Not at all
2
3
4
5
Perfectly
1061
Q

What is tetracaine

A

Anaesthetic used to permit examination of a painful eye

How well did you know this?
1
Not at all
2
3
4
5
Perfectly
1062
Q

What are emedastine and antazoline

A

Topical anti-histamines used in ophthalmology

How well did you know this?
1
Not at all
2
3
4
5
Perfectly
1063
Q

What is hypomellose and carbomer

A

Eye lubricants

How well did you know this?
1
Not at all
2
3
4
5
Perfectly
1064
Q

Whence do refractive errors arise?

A

Disorders of the size and shape of the eye

How well did you know this?
1
Not at all
2
3
4
5
Perfectly
1065
Q

Myopia=

A

Short-sightedness

How well did you know this?
1
Not at all
2
3
4
5
Perfectly
1066
Q

Problem in myopia

A

Eye is too long, distant objects are focussed too far forward

Genetic or excessive close work in early decades

How well did you know this?
1
Not at all
2
3
4
5
Perfectly
1067
Q

Solution to myopia

A

Concave lenses

How well did you know this?
1
Not at all
2
3
4
5
Perfectly
1068
Q

Problem in astigmatism

A

Cornea or lens doesn’t have the same degree of curvature in horizontal and vertical planes

Image of object is distorted longitudinally or vertically

How well did you know this?
1
Not at all
2
3
4
5
Perfectly
1069
Q

Hypermetropia=

A

Long-sightedness

How well did you know this?
1
Not at all
2
3
4
5
Perfectly
1070
Q

Problem in hypermetropia

A

Eye is too short

When the eye is relaxed and not accomodating, objects are focussed behind the retina

Contraction of ciliary muscles to focus image-> tiredness of gage and psosible a convergent squint in children

How well did you know this?
1
Not at all
2
3
4
5
Perfectly
1071
Q

Solution to hypermetropia

A

Convex lenses

How well did you know this?
1
Not at all
2
3
4
5
Perfectly
1072
Q

Presbyopia

A

With age, the lens becomes stiff and less easy to deform

Start at about 40y and are complete by 60y

Use convex lenses to correct

How well did you know this?
1
Not at all
2
3
4
5
Perfectly
1073
Q

Esotropia=

A

Convergent squint

May be idiopathic or due to hypermetropia

How well did you know this?
1
Not at all
2
3
4
5
Perfectly
1074
Q
A

Esotropia

How well did you know this?
1
Not at all
2
3
4
5
Perfectly
1075
Q

Exotropia

A

Divergent squint

common in older children, often divergent

How well did you know this?
1
Not at all
2
3
4
5
Perfectly
1076
Q
A

Exotropia

How well did you know this?
1
Not at all
2
3
4
5
Perfectly
1077
Q

Diagnosis of non-paralytic squint

A

Corneal reflection: should fall centrally and symmetrically on each cornea

Cover test: movement of uncovered eye to take up fixation demonstrates manifest squint

How well did you know this?
1
Not at all
2
3
4
5
Perfectly
1078
Q

Mx of non-paralytic squint

3Os

A

Optical: correct refractive errors

Orthoptic: patching good eye encourages use of squinting eye

Operation: e.g. resection and recession of rectus muscles

How well did you know this?
1
Not at all
2
3
4
5
Perfectly
1079
Q

Diplopia most on looking in direction of pull of paralysed muscle

Eye won’t fixate on covering

A

Paralytic squint

How well did you know this?
1
Not at all
2
3
4
5
Perfectly
1080
Q

How to determine which eye is malfunctioning in paralytic squint

A

Cover each eye in turn, whichever eye sees the outer image is malfunctioning

How well did you know this?
1
Not at all
2
3
4
5
Perfectly
1081
Q

In paralytic squint, how can you determine which muscle is affect

A

Diplopia most on looking in direction of pull of paralysed muscle

How well did you know this?
1
Not at all
2
3
4
5
Perfectly
1082
Q

Ptosis

Fixed dilated pupil

Down and out

A

CNIII lesion

How well did you know this?
1
Not at all
2
3
4
5
Perfectly
1083
Q

Causes of CNIII palsy

A

Medical: DM, MS, infarction

Surgical: raised ICP, cavernous sinus thrombosis, posterior communicating artery aneurysm

How well did you know this?
1
Not at all
2
3
4
5
Perfectly
1084
Q

Diplopia, espsecially on going down stairs

Head tilt

A

?CNVI palsy

How well did you know this?
1
Not at all
2
3
4
5
Perfectly
1085
Q

Causes of CNIV palsy

A

Peripheral: DM (30%), trauma (30%)- nerve exits posterior of brainstem, compression

Central: MS, vascular, SOL

How well did you know this?
1
Not at all
2
3
4
5
Perfectly
1086
Q

Eye is medially deviated and cannot abduct

Diplopia in the horizontal plane

A

?CNVI palsy

How well did you know this?
1
Not at all
2
3
4
5
Perfectly
1087
Q

Causes of CNVI palsy

A

Peripheral: DM, compression, trauma

Central: MS, vascular, SOL

How well did you know this?
1
Not at all
2
3
4
5
Perfectly
1088
Q

Mx of eye trauma

A

Record acuity of both eyes

Take detailed Hx of event

If unable to open injured eye, instill LA e.g. tetracaine

How well did you know this?
1
Not at all
2
3
4
5
Perfectly
1089
Q

Ix in ?FB in orbit

A

XR orbit if metal suspected

Fluorescein may show corneal abrasions

How well did you know this?
1
Not at all
2
3
4
5
Perfectly
1090
Q

Mx of eye trauma

A

Chloramphenicol drops 0.5% to prevent infection, usually with coag negative staph

Eye patch

Cycloplegic drops may reduce pain e.g. tropicamide, cyclopentolate

How well did you know this?
1
Not at all
2
3
4
5
Perfectly
1091
Q

Features of intra-ocular haemorrhage

A

Blood in anterior chamber- hyphaema

Small amounts clear spontaneously but some may need evacuation

Complicated by corneal staining and glaucoma

Keep IOP down and monitor

How well did you know this?
1
Not at all
2
3
4
5
Perfectly
1092
Q

Features of orbital blowout #

A

Blunt injury-> sudden increase in orbital pressure with herniation of orbital contents into maxillary sinus

How well did you know this?
1
Not at all
2
3
4
5
Perfectly
1093
Q

Opthalmoplegia and diplopia (tethering of IR and IO)

Loss of sensation to lower lid skin (infraorbital nerve injury)

Ipsilateral epistaxis (damage to anterior ethmoidal artery)

Reduced acuity

Irregular pupil that reacts slowly to light

A

?Orbital blowout fracture

How well did you know this?
1
Not at all
2
3
4
5
Perfectly
1094
Q

Mx of chemical injury to the eye

A

Alkaline solutions are particulary damaging due to saponification

Mx with copious irrigation and specialist referral

How well did you know this?
1
Not at all
2
3
4
5
Perfectly
1095
Q

Small dark spots in the visual field

A

Floaters

How well did you know this?
1
Not at all
2
3
4
5
Perfectly
1096
Q

Sudden showers of floaters in one eye may be due to?

A

Blood or retinal detachment

How well did you know this?
1
Not at all
2
3
4
5
Perfectly
1097
Q

Causes of floaters

A

Retinal detachment

VH

Diabetic retinopathy/ HTN

Old retinal branch vein occlusion

Syneresis (degenerative opacities in the vitreous)

How well did you know this?
1
Not at all
2
3
4
5
Perfectly
1098
Q

Flashes in vision=

A

Photopsia

How well did you know this?
1
Not at all
2
3
4
5
Perfectly
1099
Q

Causes of flashes

A

Either from intraocular or intracerebral pathology

Headached, NV: migraine

Flashes and floater= retinal detachment

How well did you know this?
1
Not at all
2
3
4
5
Perfectly
1100
Q

What can cause haloes

A

May be caused by hazy ocular media- cataract, corneal oedema, acute glaucoma

How well did you know this?
1
Not at all
2
3
4
5
Perfectly
1101
Q

Haloes + eye pain=

A

Acute glaucoma

How well did you know this?
1
Not at all
2
3
4
5
Perfectly
1102
Q

Jagged haloes which change shape are usually

A

Migrainous

How well did you know this?
1
Not at all
2
3
4
5
Perfectly
1103
Q

Rx in seasonal allergical conjuncitivitis

A

Antazoline: antihistamine drops

Cromoglycate: inhibits mast cell degranulation

How well did you know this?
1
Not at all
2
3
4
5
Perfectly
1104
Q

Small papillae on tarsal conjuncitvae

A

?Seasonal allergic/ perenial allergic conjuncitivites

How well did you know this?
1
Not at all
2
3
4
5
Perfectly
1105
Q
A
How well did you know this?
1
Not at all
2
3
4
5
Perfectly
1106
Q

Rx of perennial conjuncitivis

A

Olopatadine (anti-histamine and mast cell stabiliser)

How well did you know this?
1
Not at all
2
3
4
5
Perfectly
1107
Q

Causes of giant papillary conjuncitvitis

Mx

A

Iatrogenic FBs: contact lenses, prostheses, sutures

Removal of FB, mast cell stabilisers

How well did you know this?
1
Not at all
2
3
4
5
Perfectly
1108
Q
A

Giant papillary conjunctivitis

How well did you know this?
1
Not at all
2
3
4
5
Perfectly
1109
Q

Mx of allergic eye disorders

A

Remove the allergen responsible where possible

General measures: cold compress, artificial tears, oral antihistamine (loratadine 10mg/pd PO)

Eye drops:

Antihistamines: antazoline, azelastine

Mast cell stabilisers: cromoglycate, lodoxamide

Steroids: dexamethasone (NB of inducing glaucoma)

NSAIDs: Diclofenac

How well did you know this?
1
Not at all
2
3
4
5
Perfectly
1110
Q

Pathophysiology of trachoma

A

Caused by Chlamydia trachomatis (A, B, C)

Spread by flies

Inflammatory reaction under the lids-> scarring-> entropion-> eyelashes scratching cornea-> ulceration-> blindness

How well did you know this?
1
Not at all
2
3
4
5
Perfectly
1111
Q

Rx Trachoma

A

Tetracycline 1% +/- PO

How well did you know this?
1
Not at all
2
3
4
5
Perfectly
1112
Q

Pathophysiology of onchoceriasis

A

Caused by microfilariae or nematode Onchocerca volvulus

Spread by flies

Fly bites-> microfilariae infeciton-> invade the eye-> inflammation-> fibrosis-> corneal opacities and synechiae

How well did you know this?
1
Not at all
2
3
4
5
Perfectly
1113
Q

Rx in onchoceriasis

A

Ivermectin

How well did you know this?
1
Not at all
2
3
4
5
Perfectly
1114
Q

Xerophthalmia and keratomalacia are manifestations of?

A

Vitamin A deficiency

How well did you know this?
1
Not at all
2
3
4
5
Perfectly
1115
Q

Night blindness and dry conjunctivae

Corneal ulceration and perforation

A

?Vitamin A deficiency

How well did you know this?
1
Not at all
2
3
4
5
Perfectly
1116
Q

Rx in Vitamin A related Xerophthalmia?

A

Vitamin A/palmitate reverses early corneal changes

How well did you know this?
1
Not at all
2
3
4
5
Perfectly
1117
Q

Which two cranial nerves originate from the cerebrum?

A

Olfactory and optic

How well did you know this?
1
Not at all
2
3
4
5
Perfectly
1118
Q

What is the origin of the trochlear nerve?

A

The midbrain, comes from the posterior side of the midbrain and has the longest intracranial length

How well did you know this?
1
Not at all
2
3
4
5
Perfectly
1119
Q

What is the origin of the oculomotor nerve?

A

Midbrain-pontine junction

How well did you know this?
1
Not at all
2
3
4
5
Perfectly
1120
Q

What is the origin of the trigeminal?

A

Pons

How well did you know this?
1
Not at all
2
3
4
5
Perfectly
1121
Q

What is the origin of the abducens, facial and vestibulocochlear nerves?

A

Pontine medulla junction

How well did you know this?
1
Not at all
2
3
4
5
Perfectly
1122
Q

What is the origin of glossopharyngeal, vagus, accessory nerve?

A

Posterior to the olive in the medulla oblongata

How well did you know this?
1
Not at all
2
3
4
5
Perfectly
1123
Q

What is the origin of the hypoglossal?

A

Anterior to the olive in the medulla oblongata

How well did you know this?
1
Not at all
2
3
4
5
Perfectly
1124
Q
A
How well did you know this?
1
Not at all
2
3
4
5
Perfectly
1125
Q

What cranial nerve exits through the cribiform plate?

A

Olfactory

How well did you know this?
1
Not at all
2
3
4
5
Perfectly
1126
Q

What cranial nerve exits through the optic canal?

A

III

How well did you know this?
1
Not at all
2
3
4
5
Perfectly
1127
Q

What CNs exit through the superior orbital fissure?

A

III

IV

V1

VI

How well did you know this?
1
Not at all
2
3
4
5
Perfectly
1128
Q

What cranial nerve exits through the foramen rotundum

A

V2

How well did you know this?
1
Not at all
2
3
4
5
Perfectly
1129
Q

What CN exits through the formaen ovale?

A

V3

How well did you know this?
1
Not at all
2
3
4
5
Perfectly
1130
Q

What cranial nerve exits through the internal acoustic meatus?

A

VII

VIII

How well did you know this?
1
Not at all
2
3
4
5
Perfectly
1131
Q

What cranial nerves exit through the jugular foramen?

A

IX

X

XI

How well did you know this?
1
Not at all
2
3
4
5
Perfectly
1132
Q

What CN exits through the hypoglossal canal?

A

XII

How well did you know this?
1
Not at all
2
3
4
5
Perfectly
1133
Q

What modality is carried by:

CNI

A

Special visceral sensory: smell

How well did you know this?
1
Not at all
2
3
4
5
Perfectly
1134
Q

What modality is carried by:

CNII

A

Special somatic sensory (Vision)

How well did you know this?
1
Not at all
2
3
4
5
Perfectly
1135
Q

What modality is carried by:

III

A

General somatic motor: skeletal muscles

General visceral motor: pupillary sphincter (autonomic)

How well did you know this?
1
Not at all
2
3
4
5
Perfectly
1136
Q

What modality is carried by:

IV

A

General sensory motor: SO

How well did you know this?
1
Not at all
2
3
4
5
Perfectly
1137
Q

What modality is carried by:

V1

A

General somatic sensory: scalp, forehead and nose

How well did you know this?
1
Not at all
2
3
4
5
Perfectly
1138
Q

What modality is carried by:

V2

A

General somatic sensory: cheeks, lower eye lid, nasal mucosa, upper lip, upper teeth, palate

How well did you know this?
1
Not at all
2
3
4
5
Perfectly
1139
Q

What modality is carried by:

V3

A

General somatic sensory: anterior 2/3rd tongue, skin over mandible and lower teeth

Special visceral motor: Muscles of mastication

How well did you know this?
1
Not at all
2
3
4
5
Perfectly
1140
Q

What modality is carried by:

VI

A

General somatic motor: LR

How well did you know this?
1
Not at all
2
3
4
5
Perfectly
1141
Q

What modality is carried by:

VII

A

General somatic sensory: sensation to part of external ear

Special visceral sensory: taste from anterior 2/3rds of tongue and soft palate

General somatic motor: muscles of facial expression

General visceral motor: lacrimal, submandibular, sublingual glands and mucous glands of mouth and nose

How well did you know this?
1
Not at all
2
3
4
5
Perfectly
1142
Q

What modality is carried by:

VIII

A

Special somatic sensory: hearing and balance

How well did you know this?
1
Not at all
2
3
4
5
Perfectly
1143
Q

What modality is carried by:

IX

A

General somatic sensory: posterior 1/3rd of the tongue, external ear and middle ear cavity

General visceral sensory: carotid body and sinus

Special visceral sensory: taste from posterior 1/3rd of tonuge

General visceral motor: parotid gland

Special visceral motor: stylopharyngeus

How well did you know this?
1
Not at all
2
3
4
5
Perfectly
1144
Q

What modality is carried by:

X

A

General somatic sensory: external ear, larynx, pharynx

General visceral motor: larynx, pharynx and thoracic and abdominal viscera

Special visceral sensory: taste from epiglottis region of tongue

General visceral motor: smooth muscles of the pharynx larynx and most of the GIT

Special visceral motor: most muscles of the pharynx and larynx

How well did you know this?
1
Not at all
2
3
4
5
Perfectly
1145
Q

What modality is carried by:

Accessory nerve?

A

General somatic motor: trapezius and sternocleidomastoid

Special visceral motor: few fibres run with CNX to visceral

How well did you know this?
1
Not at all
2
3
4
5
Perfectly
1146
Q

What modality is carried by:

XII

A

Intrinsic and extrinsic tongue muscles (except palatoglossus)

How well did you know this?
1
Not at all
2
3
4
5
Perfectly
1147
Q

What innervates palatoglossus?

A

CNX

Only muscle of the tongue not innervated by XII

How well did you know this?
1
Not at all
2
3
4
5
Perfectly
1148
Q

What is the composition of the olfactory nerve?

A

Peripheral olfactory processes (in the olfactory mucosa)

Central processes that return the information to the brain

How well did you know this?
1
Not at all
2
3
4
5
Perfectly
1149
Q

Course of the olfactory nerve

A

Enters the cranial cavity through the cribiform plate of the ethmoid bone

Once in the cranial cavity the fibres enter the olfacory bulb which lies in the olfactory groove in the anterior cranial fossa.

Pass posteriorly to the olfactory tract

As the tract reaches the anterior perforated substance it divides into lateral and medial stria

Lateral stria carries axons to the primary olfactory cortex

Medial stria carry axons to the olfactory bulb on the other side

How well did you know this?
1
Not at all
2
3
4
5
Perfectly
1150
Q

How does the optic nerve enter the cranial cavity

A

Through the optic canal, a passage in the sphenoid bone running along the middle cranial fossa in close proximity to the pituitary gland

How well did you know this?
1
Not at all
2
3
4
5
Perfectly
1151
Q

Intracranial course of the optic nerve

A

Within the middle cranial fossa the optic nerves from each eye unite to form the optic chiasm

At the optic chiasm the nerves cross over and form the optic tracts

Each tract travels to its corresponding cerebral hemisphere to reach the lateral geniculate nucleus, a relay system in the thalamus where the fibres synpase

Axons from the LGN carry information via optic radiation

The upper optic radiation carries fibres from the superior retinal quadrants (corresponding to the inferior visual field quadrants), travelling through the parietal lobe to reach the visual cortex

The lower optic radiation carries fibres from the inferior retinal quadrants (superior visual field quadrants) and travels through the temporal lobe via Meyer’s loop.

How well did you know this?
1
Not at all
2
3
4
5
Perfectly
1152
Q

Course of the oculomotor nerve

A

Originates from the anterior branch of the midbrain, moving anteriorly

Passes below the posterior cerebral artery and above the superior cerebellar artery

Pierces the dura mater and enters the cavernous sinus

Within the cavernous sinus it receives sympathetic branches from the internal carotid plexus, that travel within its sheath

Leaves the cranial cavity via the superior orbital fissure, where it divides into the superior and inferior branches

How well did you know this?
1
Not at all
2
3
4
5
Perfectly
1153
Q

What does the superior branch of oculomotor innervate?

A

SR and LPS

Sympathetic fibres run with the superior branch to innervate the superior tarsal muscle

How well did you know this?
1
Not at all
2
3
4
5
Perfectly
1154
Q

What does the inferior branch of III innervate?

A

IR, MR IO

PNS fibres run to the ciliary ganglion which ultimately innervates the sphincter pupillae and ciliary muscles

How well did you know this?
1
Not at all
2
3
4
5
Perfectly
1155
Q

Which SO do the IV neurones innervate?

A

. As the fibres from the trochlear nucleus cross in the midbrain before they exit, the trochlear neurones innervate the contralateral superior oblique.

How well did you know this?
1
Not at all
2
3
4
5
Perfectly
1156
Q

What is unique about the trochlear nerve?

A

The trochlear nucleus is unique in that its axons run dorsally and cross the midline before emerging from the brainstem posteriorly. Thus a lesion of the trochlear nucleus affects the contralateral eye. Lesions of all other cranial nuclei affect the ipsilateral side.

Also the only CN to emerge from the posterior of the brainstem

How well did you know this?
1
Not at all
2
3
4
5
Perfectly
1157
Q

Action of SO

A

Depress and intort the eyeball- down and in

How well did you know this?
1
Not at all
2
3
4
5
Perfectly
1158
Q

What is innervated by the motor component of trigeminal

A

Only the mandibular branch of CN V has motor fibres. It innervates the muscles of mastication: medial pterygoid, lateral pterygoid, masseter and temporalis. The mandibular nerve also supplies other 1st pharyngeal arch derivatives: anterior belly of digastric, tensor veli palatini and tensor tympani.

How well did you know this?
1
Not at all
2
3
4
5
Perfectly
1159
Q

Parasympathetic supply and the trigeminal

A

The post-ganglionic neruones of the PNS ganglia travel with branches of the trigeminal but the trigeminal nerve is not part of the cranial outflow of PNS supply

How well did you know this?
1
Not at all
2
3
4
5
Perfectly
1160
Q

Anatomical course of V

A

Originates from three sensory nuclei (mesencephalic, principal sensory, spinal nuceli) and one motor nucleus extending from the midbrain to the medulla

At the level of the pons, the sensory nuclei merge to form a sensory root

The motor nucleus continues to form a motor root

In the middle cranial fossa the sensory root expands into the trigeminal ganglion which is located lateral to the cavernous sinus. In a depression of the temporal bone- the trigeminal cave

Trigeminal gives rise to V1, V2, V3. Motor root passes inferiorly to the sensory root and its fibres are only carried in V3

How well did you know this?
1
Not at all
2
3
4
5
Perfectly
1161
Q

Function of the ophthalmic nerve

A

Gives rise to 3 terminal branches, frontal, lacrimal and nasociliary which innervate the skin and mucous membrane of derivatives of the frontonasal prominence

Also provides PNS supply to the lacrimal gland from the pterygopalatine ganglion (facial nerve derivative) travel with V2 then join the lacrimal branch of V1 providing PNS innervation to the lacrimal gland

How well did you know this?
1
Not at all
2
3
4
5
Perfectly
1162
Q

Corneal reflex arms

A

Afferent: V1

Efferent: facial nerve

How well did you know this?
1
Not at all
2
3
4
5
Perfectly
1163
Q

V2 function

A

Sensory

PNS: initially carries post ganglionic fibres from the pterygopalatine ganglion (VII derivative) and travel with the zygomatic branch of V2 then join the lacrimal branch of V1

Nasal glands: PNS fibres are also carried to the mucous glands of the nasal mucosa travelling with branches of V2

How well did you know this?
1
Not at all
2
3
4
5
Perfectly
1164
Q

What is the motor funciton of V

A

Carried by V3

Muscles of mastication: medial and lateral pterygoids, massester, temporalis

Anterior belly of digastric and mylohyoid muscles

Tensor veli palatini

Tensory tympani

PNS: post ganglionic fibres from the submandibular ganglion travel with the lingual nerve to innervate submandibular and sublingual glands

Parotid gland: post ganglionic fibres from the otic ganglion (derived from CN IX) travel with the auriculotemporal branch of V3 to innervate the parotid gland

How well did you know this?
1
Not at all
2
3
4
5
Perfectly
1165
Q

Anatomical course of the VIII

A

Vestibular and cochlear portions of the VIII are functionally discrete so orignate from different nuclei in the brain

Vestibular: vesitublar nuclei complex in the pon and medulla

Cochlear: from the ventral and dorsal cochlear nuclei in the inferior cerebellar peduncle.

Combine in the pons, nerve emerges from the brain at the CPA and exits the cranium via the internal acoustic meatus of the temporal bone. Splits distal to the internal acoustic meatus.

How well did you know this?
1
Not at all
2
3
4
5
Perfectly
1166
Q

Clinical relevance of basilar skull # to VIII

A

Basilar skull fracture can cause damage to VIII in the internal acoustic meatus producing symptoms of vestibular and cochlear nerve damage.

How well did you know this?
1
Not at all
2
3
4
5
Perfectly
1167
Q

Course of CN IX

A

Arises from the medulla oblongata

Leaves the cranium via the jugular foramen at which point the tympanic nerve arises.

Immediately outside the jugular foramen lie two ganglia known as the superior and inferior (or petrous) ganglia that contain the cell bodies of the sensory fibres in the glossopharyngeal nerve.

Descends to give innervation to stylopharyngeus and the carotid sinus nerve which provides sensation to the carotid sinus and body.

How well did you know this?
1
Not at all
2
3
4
5
Perfectly
1168
Q
A
How well did you know this?
1
Not at all
2
3
4
5
Perfectly
1169
Q

What are the components of the gag reflex?

A

Afferent= CNIX

Efferent= CNX

How well did you know this?
1
Not at all
2
3
4
5
Perfectly
1170
Q

Course of the vagus nerve in the head

A

Originates from the medulla and exits via the jugular foramen

Within the cranium the auricular branch arises which supplies sensation to the posterior part of the external auditory canal and external ear

How well did you know this?
1
Not at all
2
3
4
5
Perfectly
1171
Q

Anatomical course of the vagus nerve in the neck

A

Passes into carotid sheath, travelling inferiorly with the internal jugular vein and common carotid.

Right vagus passes anterior to the subclavian artery and posterior to the sternoclavicular joint, entering the thorax

Left vagus passes inferiorly between the left common carotid and the left subclavian arteries, posterior to the sternoclavicular joint.

How well did you know this?
1
Not at all
2
3
4
5
Perfectly
1172
Q

Branches of the vagus nerve in the neck?

A

Pharyngeal branches: motor innervation to the muscles of the pharynx and soft palate

Superior laryngelal nerve: internal and external branches, external innervate the cricothyroid muscle, the internal providing sensory innervation to the laryngopharynx and superior part of the larynx

Recurrent laryngeal nerve (on the right only) hooks underneath the subclavian artery and ascends to the larynx, innervating the majority of the intrinsic muscles of the larynx

How well did you know this?
1
Not at all
2
3
4
5
Perfectly
1173
Q

Course of the vagus nerve in the thorax

A

Right vagus forms the posterior vagal trunk

The left forms the anterior vagal trunk

Branches from these trunks contribute to the oesophageal plexus

Two other branches arise: the left recurrent laryngeal nerve (hooks under the arch of the aorta)

Cardiac branches

Vagal trunks enter the abdomen via the oesophageal hiatus

How well did you know this?
1
Not at all
2
3
4
5
Perfectly
1174
Q

Lesion to one recurrent laryngeal nerve causes?

Both?

A

Dysphonia

Aphonia

How well did you know this?
1
Not at all
2
3
4
5
Perfectly
1175
Q

Anatomical course of the accessory nerve (spinal)

A

Spinal portion arises from neurones of the upper SC (C1-C5/C6) of the spinal nerve roots.

Coalesce to form the spinal part of the accessory nerve which then runs superiorly to enter the cranium via the foramen magnum

It travels to the posterior cranial fossa to reach the jugular foramen where it meets the cranial portion of the accessory nerve before exiting the skull.

Descends along the internal carotid to reach sternocleidomastoid which it innervates.

Extracranial course is relatively superficial leaving it vulnerable to damage.

How well did you know this?
1
Not at all
2
3
4
5
Perfectly
1176
Q

Cranial course of the accessory nerve

A

Much smaller and arises from the lateral aspect of the medulla.

Leaves the cranium via the jugular foramen where it briefly contacts the spinal part of the accessory nerve.

Immediately after leaving the skull, the cranial portion combines with the vagus at the inferior ganglion of the vagus.

The fibres are then distributed through the vagus nerve.

How well did you know this?
1
Not at all
2
3
4
5
Perfectly
1177
Q

What is the most common cause of accessory nerve damage?

A

Iatrogenic e.g. cervical LN biopsy or cannulation of the IJV.

Clinical features include muscle wasting and partial paralysis of the sternocleidomastoid, resulting in the inability to rotate the head or weakness in shrugging the shoulders. Damage to the muscles may also result in an asymmetrical neckline.

How well did you know this?
1
Not at all
2
3
4
5
Perfectly
1178
Q
A

Accessory nerve palsy

Asymmetrical neck line

How well did you know this?
1
Not at all
2
3
4
5
Perfectly
1179
Q

What muscles are innervated by the hypoglossal nerve?

A

All muscles of the tongue except palatoglossus (innervated by the vagus nerve)

How well did you know this?
1
Not at all
2
3
4
5
Perfectly
1180
Q

What are the extrinsic muscles of the tongue?

A

Genioglossus

Hypoglossus

Styloglossus

[Palatoglossus- vagus nerve]

How well did you know this?
1
Not at all
2
3
4
5
Perfectly
1181
Q

What are the intrinsic muscles of the tongue?

A

Superior longitudinal

Inferior longitudinal

Transverse

Vertical

How well did you know this?
1
Not at all
2
3
4
5
Perfectly
1182
Q

Classification of hypoglossal nerve palsy

A

Can be supranuclear (UMN), nuclear, or infranuclear (LMN)

UMN: tongue will deviate away from injured side. Only seen during the initial days after the injury, after which it will not deviate.

Infranuclear lesions lead to parlaysis of the hpyoglossal nerve leading to atrophy of muscles of the tonuge, tongue will be deviated TOWARDS lesion. Due to weaker geniglossal muscle.

How well did you know this?
1
Not at all
2
3
4
5
Perfectly
1183
Q

What may be seen in supranuclear damage to the left and right tracts of CNXII

A

Damage to facial and trigemnial nerve dysfunction as a consequence of damage to the brainstem following arteriosclerosis of the vertebrobasilar artery

How well did you know this?
1
Not at all
2
3
4
5
Perfectly
1184
Q

CN palsy + pain

A

?Cartoid dissection

How well did you know this?
1
Not at all
2
3
4
5
Perfectly
1185
Q

What is the only muscle of the tongue not innervated by hypoglossal?

A

Extrinsic, palatoglossus (Vagus)

How well did you know this?
1
Not at all
2
3
4
5
Perfectly
1186
Q
A

Right subclavian arises from the brachiocephalic trunk

Left subclavian directly from the arota

How well did you know this?
1
Not at all
2
3
4
5
Perfectly
1187
Q

When do the subclavian arteries become the axillary arteries?

A

When they cross the lateral edge of the first 1 rib and enter the axilla

How well did you know this?
1
Not at all
2
3
4
5
Perfectly
1188
Q

Course of the axillary artery

A

Passes through the axilla underneath pectoralis minor enclosed in the axillary sheath

At the humeral surgical neck the posterior and anterior circumflex humeral arteries arise

Circle posteriorly around the humerus to supply the shoulder region

The subscapsular artery also arises here (largest brach of the axillary artery)

At the level of the teres major the axillary artery becomes the brachial artery

How well did you know this?
1
Not at all
2
3
4
5
Perfectly
1189
Q

Where does the axillary artery become the brachial

A

At the lower margin of teres major

How well did you know this?
1
Not at all
2
3
4
5
Perfectly
1190
Q
A
How well did you know this?
1
Not at all
2
3
4
5
Perfectly
1191
Q

What in the axilla can compress the brachial plexus producing neurological symptoms and can be treated surgically

A

Axillary artery aneurysm

How well did you know this?
1
Not at all
2
3
4
5
Perfectly
1192
Q

Course of the brachial artery

A

Immediately distal to teres major gives the profunda brachii- deep artery of the arm
which travels along the posterior surface of the humerus in the radial groove, supplying the posterior aspect of the arm

Brachial artery descends down the arm immediately posterior to the median nerve, as it crosses the cubital fossa underneath brachialis it terminates by bifurcating into the radial and ulnar arteries

How well did you know this?
1
Not at all
2
3
4
5
Perfectly
1193
Q
A
How well did you know this?
1
Not at all
2
3
4
5
Perfectly
1194
Q

Course of the arteries in the forearm

A

Radial- posterior aspect of the forearm

Ulnar- anterior

Areteries anastomose by forming the superficial palmar arch and the deep palmar arch

How well did you know this?
1
Not at all
2
3
4
5
Perfectly
1195
Q

What are the major superifical veins of the upper limb

A

The cephalic and basilic veins

How well did you know this?
1
Not at all
2
3
4
5
Perfectly
1196
Q

Course of the superficial veins of the upper limb

A

Basilic vein originates from the dorsal venous network of the hand ascending the medial aspect of the upper limb. At the border of teres major the vein moves deep into the arm where it combines with brachial veins to form the axillary vein

The cephalic vein arises from the dorsal venous network of the ascending ascending on the anterolateral aspect of the upper limb, passing anteriorly at the elbow.

At the shoulder the cephalic vein travels between the delotid and pec major muscles in the deltopectoral groove and enters the axilla via the clavipectoral triangle.

It terminates here by joining the axillray vein.

At the elbow the cephalic and basilic veins are connected by the median cubital vein

How well did you know this?
1
Not at all
2
3
4
5
Perfectly
1197
Q
A
How well did you know this?
1
Not at all
2
3
4
5
Perfectly
1198
Q

What is the course of the deep veins of the upper limb

A

Paired veins that accompany and lie either side of an artery

Brachial veins are the largest in size and are situated either side of the brachial artery

These are vena comitantes

Perforating veins run between the deep and superficail veins of the upper limb, connecting the two systems

How well did you know this?
1
Not at all
2
3
4
5
Perfectly
1199
Q

What are at risk of damage in axillary LN dissection?

A

Long thoracic nerve (winged scapula)

Or the thoracodorsal nerve

How well did you know this?
1
Not at all
2
3
4
5
Perfectly
1200
Q

Describe the course of the femoral artery

A

Continuation of the EIA which becomes the femoral artery when it crosses under the inguinal ligament and enters the femoral triangle

In the femoral triangle the profunda femoris artery arises from the posterolateral aspect. It travels posteriorly giving off three main branches:

Lateral femoral circumflex: wraps around the anterior, lateral side of the femur supplying muscles in the lateral side of the thigh

Medial circumflex artery: wraps around the posterior side of the femur supplying the head and neck (avascular necorsis in intracapsular #)

Perforating branches: 3/4 arteries that perforate the adductor magnus

After exiting the femoral triangle the femoral artery continues down the anterior surface of the thigh via the adductor canal.

The adductor canal ends at the adductor hiatus at which point the femoral artery enters the posterior compartment of the thigh proximal to the knee

How well did you know this?
1
Not at all
2
3
4
5
Perfectly
1201
Q

Borders of the femoral triangle

A

Superior border: inguinal ligament

Lateral border: sartorius

Medial border: medial border of adductor longus

Anteriorly the roof of the femoral triangle is the fascia lata

Posteriorly the base is formed by pectineus, ilopsoas and adductor longus muscles.

How well did you know this?
1
Not at all
2
3
4
5
Perfectly
1202
Q

Contents of the femoral triangle

NAVEL

A

Femoral nerve: innervates the anterior compartment of the thigh

Femoral artery

Femoral vein

Empty space

Lymph canal

Contained in the femoral sheath

How well did you know this?
1
Not at all
2
3
4
5
Perfectly
1203
Q
A
How well did you know this?
1
Not at all
2
3
4
5
Perfectly
1204
Q

Location of the femoral pulse

A

Midway between the pubic symphysis and ASIS

How well did you know this?
1
Not at all
2
3
4
5
Perfectly
1205
Q
A
How well did you know this?
1
Not at all
2
3
4
5
Perfectly
1206
Q

Describe the course of the other arteries supplying the thigh

A

Obturator artery arises from the IIA in the pelvic region, descends via the obturator canal to enter the medial thigh bifurcating into the canal.

Gluteal region is largely supplied by the superior and inferior gluteal arteries which exit the pelvis via the GSF.

The IGA also contirbutes to the vasculature of the posterior thigh

How well did you know this?
1
Not at all
2
3
4
5
Perfectly
1207
Q

What differentiates between the SGA and the IGA

A

Both exit the GSF

SGA superior to piriformis

IGA inferior to piriformis

How well did you know this?
1
Not at all
2
3
4
5
Perfectly
1208
Q

Describe the arterial supply of the leg

A

Popliteal artery descends down the posterior thigh giving off genicular branches that supply the knee joint.

Moves through the popliteal fossa between gastrocnemius and popliteus.

At the lower border of popliteus it terminates by dividing into anterior and posterior tibial artery.

Posterior tibial continues inferiorly along the surface of the deep muscles accompanying the tibial nerve entering the sole of the foot via the tarsal tunnel.

During the descent of the posterior tibial the fibular artery arises which moves laterally penetrating the lateral compartment of the leg.

The anterior tibial artery passes anteriorly between the tibia and fibular through a gap in the interoesseous membrane. Moving inferiorly down the leg and into the foot where it becomes dorsalis pedis.

How well did you know this?
1
Not at all
2
3
4
5
Perfectly
1209
Q
A
How well did you know this?
1
Not at all
2
3
4
5
Perfectly
1210
Q

What is the clinical revelance of the fact that the popliteal fascia is tough and non-extensable

A

Aneurysm of the popliteal artery has consequences for other contents of the popliteal fossa.

The tibial nerve is particularly susceptible to compression from the popliteal artery

Major features are absent/weakened plantar fleixion, paraesthesia of the foot and posterolateral leg.

Can be detected through obvious expansile pulsatile mass and an arterial bruit

How well did you know this?
1
Not at all
2
3
4
5
Perfectly
1211
Q

Describe the arterial supply of the foot

A

Dorsalis pedis- continuation of the anterior tibial artery

Posterior tibial enters through the sole of the foot through the tarsal tunnel and supplies the lateral and medial plantar surfaces

How well did you know this?
1
Not at all
2
3
4
5
Perfectly
1212
Q

Where can the femoral pulse be palpated?

A

be palpated as it enters the femoral triangle, midway between the anterior superior iliac spine of the pelvis, and the pubis synthesis (the mid-inguinal point).

How well did you know this?
1
Not at all
2
3
4
5
Perfectly
1213
Q

Where can the dorsalis pedis be palpated?

A

Dorsum of the foot in the fist intermetatarsal space, lateral to the tendon of the great toe

How well did you know this?
1
Not at all
2
3
4
5
Perfectly
1214
Q

Where can the posterior tibial pulse be palpated

A

Behind and below the medial malleolus

How well did you know this?
1
Not at all
2
3
4
5
Perfectly
1215
Q

Where can the popliteal pulse be palpated

A

In the midline in the popliteal fossa with gentle knee flexion

How well did you know this?
1
Not at all
2
3
4
5
Perfectly
1216
Q

Describe the course of the deep venous drainage of the lower limb

A

Dorsal arch drains mainly into the superficial veins, some penetrate deep into the leg forming the anterior tibial vein.

On the plantar aspect of the foot the medial and lateral plantar veins arise and combine to form the posterior tibial and fibular vein.

The posterior tibial vein accompanies the posterior tibial artery, entering the leg posteriorly to the medial malleolus.

On the posterior surface of the knee the anterior tibial, posterior tibial and fibular veins unite to form the popliteal vein which enters the thigh through the adductor canal

How well did you know this?
1
Not at all
2
3
4
5
Perfectly
1217
Q

Course of the deep venous drainage of the thigh

A

The popliteal vein becomes the femoral vein once it enters the thigh and is situated anteriorly.

The profunda femoris vein is the other main venous structure. Empties into the distal section of the femoral vein.

The femoral vein leaves the thigh to become the EIV

How well did you know this?
1
Not at all
2
3
4
5
Perfectly
1218
Q

Outline the superficial venous drainage of the leg

A

Two major veins, the great and small saphenous

Great saphenous is formed by the dorsal venous arch of the foot. Ascends on the medial side of the leg, passing anteriorly to the medial malleolus and posterior to the medial condyle at the knee.

As the vein moves up the leg it receives tributaries from other small superficial veins, and terminates by draining into the femoral vein immediately inferior to the inguinal ligament.

The small saphenous vein moves up the posterior side of the leg, passing posterior to the lateral malleolus along the border of the calcaneal tendon. It moves between the two heads of gastrocnemius and empties into the popliteal vein in the popliteal fossa

How well did you know this?
1
Not at all
2
3
4
5
Perfectly
1219
Q

What is the location of the saphenofemoral junction

A

2.5cm below and 2.5cm lateral to the pubic tubercle

How well did you know this?
1
Not at all
2
3
4
5
Perfectly
1220
Q
A
How well did you know this?
1
Not at all
2
3
4
5
Perfectly
1221
Q

Incidence of chronic limb ischaemia

A

5% of males >50 years have intermittent claudication

How well did you know this?
1
Not at all
2
3
4
5
Perfectly
1222
Q

Def: chronic limb ischaemia

A

Ankle artery pressure <50mmHg (toe <30)

And either:

persistent rest pain requiring analgesia for >2w

or

Ulceration or gangrene

How well did you know this?
1
Not at all
2
3
4
5
Perfectly
1223
Q

Cause of chronic limb ischaemia

A

Atherosclerosis: typically asymptomatic until 50% stenosis

Vasculitis and fibromuscular dysplasia are v rare causes

How well did you know this?
1
Not at all
2
3
4
5
Perfectly
1224
Q

Summary of atherosclerosis

A

Endothelial injury: haemodynamic, HTN, raised lipids

Chronic inflammation: lipid-laden foam cells produce GFs, cytokines, ROS and MMPs-> lymphocyte and SMC recruitment

SM proliferation: conversion of fatty streak to atherosclerotic plaque

How well did you know this?
1
Not at all
2
3
4
5
Perfectly
1225
Q

Difference betweeen arteriosclerosis and atherosclerosis

A

Arteriosclerosis= general arterial hardening

Atherosclerosis= arterial hardening specifically due to atheroma

How well did you know this?
1
Not at all
2
3
4
5
Perfectly
1226
Q

Atheroma pathology

A

Fibrous cap: SM cells, lymphocytes, collagen

Necrotic centre: cell debris, cholesterol, Ca, foam cells

How well did you know this?
1
Not at all
2
3
4
5
Perfectly
1227
Q

Modifiable risk factors for chronic limb ischaemia

A

Smoking

BP

DM

Hyperlipiademia

Reduced exercise

How well did you know this?
1
Not at all
2
3
4
5
Perfectly
1228
Q

Non modifiable RFs for chronic limb ischaemia

A

FH and PMHx

Male

Increased age

Genetic

How well did you know this?
1
Not at all
2
3
4
5
Perfectly
1229
Q

What is the association of vascular disease with chronic limb ischaemia

A

IHD: 90%

Carotid stenosis; 15%

AAA

Renovascular disease

DM microvascular disease

How well did you know this?
1
Not at all
2
3
4
5
Perfectly
1230
Q

cramping pain after walking a fixed distance

Pain rapidly relieved by rest

Either calf or buttock

A

Intermittent claudication

How well did you know this?
1
Not at all
2
3
4
5
Perfectly
1231
Q

Calf pain in intermittent claudication

A

Superifical femoral disease (commonest)

How well did you know this?
1
Not at all
2
3
4
5
Perfectly
1232
Q

Buttock pain in context of intermittent claudication

A

Iliac disease (internal or common)

How well did you know this?
1
Not at all
2
3
4
5
Perfectly
1233
Q

Def: critical limb ischaemia

A

Fontaine 3 or 4

Rest pain

Especially at night, usually felt in the foot

Patient hangs foot out of bad

Due to reduced CO and loss of gravity

Ulceration

Gangrene

How well did you know this?
1
Not at all
2
3
4
5
Perfectly
1234
Q

Rest pain

Especially at night, usually felt in the foot

Patient hangs foot out of bad

Due to reduced CO and loss of gravity

Ulceration

Gangrene

A

?Critical limb ischaemia

How well did you know this?
1
Not at all
2
3
4
5
Perfectly
1235
Q

Leriche’s syndrome

A

Aortoiliac occlusive disease

Atherosclerotic occlusion of abdominal aorta and iliacs

How well did you know this?
1
Not at all
2
3
4
5
Perfectly
1236
Q

Triad in Leriche’s syndrome

A

Buttock claudication and wasting

Erectile dysfunction

Absent femoral pulses

How well did you know this?
1
Not at all
2
3
4
5
Perfectly
1237
Q

Buerger’s disease=

A

Thromboangitis obliterans

How well did you know this?
1
Not at all
2
3
4
5
Perfectly
1238
Q

Young, male, heavy smoker

Acute inflammation and thrombosis of arteries and veins in hands and feet-> ulceration and gangrene

A

Buerger’s disease

How well did you know this?
1
Not at all
2
3
4
5
Perfectly
1239
Q
A

Buerger’s Disease

Thrombangitis obliterans

How well did you know this?
1
Not at all
2
3
4
5
Perfectly
1240
Q

Weak pulses and CRT >2

Ulcers: painful, punched out on pressure points

Nail dystrophy/ onchylosis

Skin: cold, white, atrophy, absent hair

Venous guttering

Muscle atrophy

A

?Chornic limb ischaemia

How well did you know this?
1
Not at all
2
3
4
5
Perfectly
1241
Q

What is Buerger’s test?

A

The vascular angle, which is also called Buerger’s angle, is the angle to which the leg has to be raised before it becomes pale, whilst in supine decubitus. In a limb with a normal circulation the toes and sole of the foot, stay pink, even when the limb is raised by 90 degrees.

How well did you know this?
1
Not at all
2
3
4
5
Perfectly
1242
Q

Reduced Buerger’s angle seen in?

A

Chronic limb iscahemia

How well did you know this?
1
Not at all
2
3
4
5
Perfectly
1243
Q

Buerger’s angle cut offs?

A

>90= normal

20-30= ischaemia

<20= severe ischaemia

How well did you know this?
1
Not at all
2
3
4
5
Perfectly
1244
Q

What is positive Buerger’s sign?

A

Pale-> reactive hyperaemia due to accumulation of deoxygenated blood in dilated capillaries

How well did you know this?
1
Not at all
2
3
4
5
Perfectly
1245
Q

How can chronic limb ischaemia be classified?

A

Fontaine

or

Rutherford

How well did you know this?
1
Not at all
2
3
4
5
Perfectly
1246
Q

Fontaine classification of chronic limb ischaemia

A
  1. Asymptomatic
  2. Intermittent claudication

a >200m

b <200m

  1. Iscahemic rest pain
  2. Ulceration/ gangrene
How well did you know this?
1
Not at all
2
3
4
5
Perfectly
1247
Q

Rutherford classification of chronic limb iscahemia

A
  1. Mild claudication
  2. Moderate claudication
  3. Severe claudication
  4. Ischaemic rest pain
  5. Minor tissue loss
  6. Major tissue loss
How well did you know this?
1
Not at all
2
3
4
5
Perfectly
1248
Q

Ix in Chronic limb iscahemia

A

ABPI

Doppler

Walk test

Bloods

Imaging

Other: ECG

How well did you know this?
1
Not at all
2
3
4
5
Perfectly
1249
Q

Doppler waveforms in Chronic limb ischaemia

A

Normal: triphasic

Mild stenosis: biphasic

Severe stenosis: monophasic

How well did you know this?
1
Not at all
2
3
4
5
Perfectly
1250
Q

ABPI >1.4

A

Calcification: CRF, DM

How well did you know this?
1
Not at all
2
3
4
5
Perfectly
1251
Q

ABPI >1

A

Normal

How well did you know this?
1
Not at all
2
3
4
5
Perfectly
1252
Q

ABNPI 0.8-0.9

A

Asymptomatic

Fontaine 1

How well did you know this?
1
Not at all
2
3
4
5
Perfectly
1253
Q

ABPI 0.6-08

A

Fontaine 2

Claudication

How well did you know this?
1
Not at all
2
3
4
5
Perfectly
1254
Q

ABPI 0.3-0.6

A

Fontaine 3

Rest pain

How well did you know this?
1
Not at all
2
3
4
5
Perfectly
1255
Q

ABPI <0.3

A

Ulceration and gangrene

Fontaine 4

How well did you know this?
1
Not at all
2
3
4
5
Perfectly
1256
Q

Why might false ABPI results be obtained in DM/CRF?

A

Due to calcification of vessels: mediasclerosis

Use toe pressure with small cuff, <30mmHg is the cut off

How well did you know this?
1
Not at all
2
3
4
5
Perfectly
1257
Q

What is the walk test in chronic limb ischaemia Ix

A

Walk on treadmill @ certain speed and incline to establish maximum claudication distance

ABPI measured before and after, 20% reduction is signifcant

How well did you know this?
1
Not at all
2
3
4
5
Perfectly
1258
Q

Role of bloods in chronic limb ischaemia Ix

A

FBC + U+Es: anaemia, renovascular disease

Lipids and glucose

ESR: arteritis

G+S: possible procedure

How well did you know this?
1
Not at all
2
3
4
5
Perfectly
1259
Q

Role of imaging in chronic limb iscahemia Ix

A

Assess site, extent and distal run-off

Colour duplex US

CT/MR angiogram with gadolinium contrast

Digital subtraction angiography: invasive, not commonly used for Dx only but used when performing therapeutic angioplasty or stenting

How well did you know this?
1
Not at all
2
3
4
5
Perfectly
1260
Q

Why perform an ECG in chronic limb ischaemia?

A

Look for evidence of coronary artery pathology

How well did you know this?
1
Not at all
2
3
4
5
Perfectly
1261
Q

Mx of chronic limb ischaemia: Conservative

A

Most patients with claudication can be managed conservatively

Increase exercise and employ exercise programs

Stop smoking

Weight loss

Foot care

How well did you know this?
1
Not at all
2
3
4
5
Perfectly
1262
Q

Prognosis with conservative management of chronic limb iscahemia

A

1/3 improve

1/3 stay the same

1/3 deteriorate

How well did you know this?
1
Not at all
2
3
4
5
Perfectly
1263
Q

Mx of chronic limb iscahemia: Medical

A

Modification of cardiovascular risk fators: BP, lipis, DM

Beta blockers don’t worsen intermittent caludication but use with caution

Parenteral prostanoids reduces pain in patients unfit for surgery

Antiplatelet drugs: clopidogrel/aspirin

Peripheral vasodilators: naftidrofuryl oxalate

How well did you know this?
1
Not at all
2
3
4
5
Perfectly
1264
Q

NB re: ACEI intermittent claudication

A

BP medication may initially worsen pain of claudication but improve LT CV risk

However, 25% of patients with PAD also have RAS so ACEI should be used with caution

How well did you know this?
1
Not at all
2
3
4
5
Perfectly
1265
Q

Management of chronic limb ischaemia: endovascular

A

Percutaneous transluminal angioplasty

Good for short stenosis in big vessels e.g. iliacs, SFA

Lower risk for patient as performed under LA

Improved inflow can reduce pain but restoration of foot pulses is required for Rx of ulceration/gangrene

How well did you know this?
1
Not at all
2
3
4
5
Perfectly
1266
Q

Indications for surgical reconstruction in chronic limb ischaemia

A

V. short claudication distance e.g. <100m

Symptoms greatly affecting patients QoL

Development of rest pain

NB pre-op assessment as patient likely to have cardiorespiratory co-morbidities

How well did you know this?
1
Not at all
2
3
4
5
Perfectly
1267
Q

Practicalities of surgery for chronic limb ischaemia

A

Need good proximal supply and distal run-off

Saphenous vein grafts are preferred below the IL

More distal grafts have increased rates of thrombosis

How well did you know this?
1
Not at all
2
3
4
5
Perfectly
1268
Q

Classification of surgeries for chronic limb iscahemia

A

Anatomical: fem-pop, fem-distal, aortobifemoral

Extra-anatomical: axillo-femoral, bifemoral, fem-fem crossover

How well did you know this?
1
Not at all
2
3
4
5
Perfectly
1269
Q

Other surgical options for chronic limb ischaemia

A

Endarterectomy

Sympathectomy: chemical or surgical

Amputation

How well did you know this?
1
Not at all
2
3
4
5
Perfectly
1270
Q

Px 1yr after critical limb ischaemia

A

50% alive w/o amputation

25% will have had major amputation

25% dead (usually MI or stroke)

How well did you know this?
1
Not at all
2
3
4
5
Perfectly
1271
Q

Px following amputation in CLI

A

1/3 complete autonomy

1/3 partial autonomy

1/3 dead

How well did you know this?
1
Not at all
2
3
4
5
Perfectly
1272
Q

Def: acute limb iscahemia

A

Ischaemia <14d

How well did you know this?
1
Not at all
2
3
4
5
Perfectly
1273
Q

Def: acute on chronic limb iscahemia

A

Worsening symptoms and signs <14d

How well did you know this?
1
Not at all
2
3
4
5
Perfectly
1274
Q

Def: chronic limb iscahemia

A

Ischaemia stable for >14d

How well did you know this?
1
Not at all
2
3
4
5
Perfectly
1275
Q

Incomplete acute limb iscahemia

A

Limb not threatened

How well did you know this?
1
Not at all
2
3
4
5
Perfectly
1276
Q

Complete acute limb ischaemia

A

Limb threatened, loss of limb unless intervention within 6hrs

How well did you know this?
1
Not at all
2
3
4
5
Perfectly
1277
Q

Irreversible acute limb ischaemia

A

Requires amputation

How well did you know this?
1
Not at all
2
3
4
5
Perfectly
1278
Q

Causes of acute limb ischaemia

A

Thrombosis in situ: 60%

Embolism: 30%

Graft/stent occlusion

Trauma

Aortic dissection

How well did you know this?
1
Not at all
2
3
4
5
Perfectly
1279
Q

Thrombosis in situ, acute limb iscahemia

A

A previously stenosed vessel with plaque rupture, usually incomplete iscahemia

How well did you know this?
1
Not at all
2
3
4
5
Perfectly
1280
Q

Embolism in acute limb ischaemia

A

80% from LA in AF

Valvular disease

Can be iatrogenic/surgery

Cholesterol in long bone #

Paradoxical via PFO

Typically lodge at femoral bifurcation causing complete limb ischaemia

How well did you know this?
1
Not at all
2
3
4
5
Perfectly
1281
Q

6Ps of acute limb ischaemia

A

Pale

Pulseless

Perishingly cold

Painful

Paraesthesia

Paralysis

How well did you know this?
1
Not at all
2
3
4
5
Perfectly
1282
Q

Acute limb ischaemia:

Onset in hours to days

Less severe collaterals

Claudication history present

Contralateral pulses absent

Diagnosed with angiography

Rx with thrombolysis/bypass surgery

A

Thrombotic cause

How well did you know this?
1
Not at all
2
3
4
5
Perfectly
1283
Q

Acute limb ischaemia:

Sudden onset

Profound ischaemia

AF

Absent claudication history

Present contralateral pulses

Clinical diangosis

Rx with embolectomy + warfarin

A

Embolic acute limb iscahemia

How well did you know this?
1
Not at all
2
3
4
5
Perfectly
1284
Q

Ix in acute limb ischaemia

A

Bloods: FBC, U+E, INR, G+S, CK

ECG

Imaging: CXR, Duplex doppler

How well did you know this?
1
Not at all
2
3
4
5
Perfectly
1285
Q

General management of acute limb ischaemia

A

SENIOR

NBM

Rehydration: IV fluids

Analgesia: morphine + metoclopramide

Abx if signs of infection

Unfractionated heparin IV: prevent extension

If occlusion is complete: urgent surgery

If incomplete: angiogram + observe for deterioriation

How well did you know this?
1
Not at all
2
3
4
5
Perfectly
1286
Q

Angiography in acute limb iscahemia

A

Not performed if there is complete occlusion as it introduces delay

If occlusion is incomplete, pre-op angio will guide any distal bypass

How well did you know this?
1
Not at all
2
3
4
5
Perfectly
1287
Q

Mx of embolic acute limb iscahemia

A
  1. Embolectomy
  2. Thrombolysis
  3. Other options
How well did you know this?
1
Not at all
2
3
4
5
Perfectly
1288
Q

Embolectomy in acute limb ischaemia

A

Under LA or GA

Wire fed through embolus

Fogarty catheter fed over the top

Balloon inflated and catheter withdrawn removing the embolism

Send embolism for histology to exclude atrial myxoma

Adequacy can be confirmed by on-table angiography

How well did you know this?
1
Not at all
2
3
4
5
Perfectly
1289
Q

Thrombolysis in acute limb ischaemia

A

Consider if embolectomy unsuccessful

E.g. local injection of TPA

How well did you know this?
1
Not at all
2
3
4
5
Perfectly
1290
Q

Other options in acute management of embolic acute limb ischaemia

A

Emergency reconstruction

Amputation

How well did you know this?
1
Not at all
2
3
4
5
Perfectly
1291
Q

Mx of acute limb post embolectomy

A

Anticoagulate: IV heparin-> warfarin

ID embolic source: ECG, echo, US aorta, femoral and pop

Complications

How well did you know this?
1
Not at all
2
3
4
5
Perfectly
1292
Q

Complications post embolectomy in acute limb ischaemia

A

Reperfusion injury:

Local swelling-> compartment syndrome

Acidosis and arrhythmia 2o to raised K

ARDS

GI oedema-> endotoxic shock

Chronic pain syndromes

How well did you know this?
1
Not at all
2
3
4
5
Perfectly
1293
Q

Mx of thrombotic acute limb ischaemia

A

Emergency reconstruction if complete occlusion

Angiography and angioplasty

Thrombolysis

Amputation

How well did you know this?
1
Not at all
2
3
4
5
Perfectly
1294
Q

Pathogenesis of carotid artery diesease

A

Turbulent flow-> reduced shear stress at carotid bifurcation promoting atherosclerosis and plaque formation

Plaque rupture-> complete occlusion or distal emboli

Causes 15-25% of CVA/TIA

How well did you know this?
1
Not at all
2
3
4
5
Perfectly
1295
Q

CVA/TIA

Bruit

A

?Carotid artery disease

How well did you know this?
1
Not at all
2
3
4
5
Perfectly
1296
Q

Ix in carotid artery disease?

A

Duplex carotid Doppler

MRA

How well did you know this?
1
Not at all
2
3
4
5
Perfectly
1297
Q

Mx of carotid artery disease

A

Aspirin or clopidogrel

Control RFs

Surgical: endarterectomy

Symptomatic >70% benefit (5% stroke risk per year)

>50% benefit if low risk

Performed within 2w of presentation

Asymptomatic:

>60% benefit if low risk

How well did you know this?
1
Not at all
2
3
4
5
Perfectly
1298
Q

What studies have been used to look at the impact of endarterectomy in symptomatic caroit dartery disease?

A

ECST vs optimal medical therapy

NASCET

How well did you know this?
1
Not at all
2
3
4
5
Perfectly
1299
Q

Cx of endarterectomy

A

Stroke or death: 3%

HTN: 60%

Haematoma

MI

Nerve injury

How well did you know this?
1
Not at all
2
3
4
5
Perfectly
1300
Q

What nerves can be damaged in carotid endarterectomy

A

Hypoglossal: ipsilateral tongue deviation

Great auricular: numb ear lobe

Recurrent larygneal: hoarse voice, bovine cough

How well did you know this?
1
Not at all
2
3
4
5
Perfectly
1301
Q

Stenting vs carotid endarterectomy

A

Less invasive: reduced hospital stay, reduced infection, reduced CN injury

Concern over increased stroke risk, esp. in pts >70y

Meta-analysis shows no sig difference in mortality vs CEA

How well did you know this?
1
Not at all
2
3
4
5
Perfectly
1302
Q

Def: Aneurysm

A

Abnormal dilatation of a blood vessel >50% of its normal diameter

How well did you know this?
1
Not at all
2
3
4
5
Perfectly
1303
Q

Def: true aneurysm

A

Dilatation of a blood vessel involving all layers of the wall and that is >50% of its normal diameter

Two different morphologies:

Fusiform e.g. AAA

Saccular: e.g. Berry aneurysm

How well did you know this?
1
Not at all
2
3
4
5
Perfectly
1304
Q

Difference between fusiform and saccular aneurysm

A

The shape of an aneurysm is described as being fusiform or saccular, which helps to identify a true aneurysm. The more common fusiform-shaped aneurysm bulges or balloons out on all sides of the blood vessel. A saccular-shaped aneurysm bulges or balloons out only on one side.

How well did you know this?
1
Not at all
2
3
4
5
Perfectly
1305
Q

Def: false aneurysm

A

Collection of blood around a vessel wall that communicates with the vessel lumen

Usually iatrogenic e.g. puncture, cannulation

How well did you know this?
1
Not at all
2
3
4
5
Perfectly
1306
Q

Def: dissection

A

Vessel dilatation caused by blood splaying apart the media to form a channel within the vessel wallt

How well did you know this?
1
Not at all
2
3
4
5
Perfectly
1307
Q

Layers of a blood vessel

A

Lumen

Intima

Media

Advenititia

How well did you know this?
1
Not at all
2
3
4
5
Perfectly
1308
Q

Congenital causes of aneurysms

A

ADPKD-> Berry aneurysms

Marfan’s, Ehlers Danlos

How well did you know this?
1
Not at all
2
3
4
5
Perfectly
1309
Q

Acquired causes of aneurysms

A

Atherosclerosis

Trauma e.g. penetrating trauma

Inflammatory: Takayasu’s, HSP, Kawasaki

Infection:

Mycotic- SBE

Tertiary syphillis (esp. thoracic)

How well did you know this?
1
Not at all
2
3
4
5
Perfectly
1310
Q

Complications of aneurysms

A

Rupture

Thrombosis

Distal embolisation

Pressure: DVT, oesophagus, nutcracker syndrome

Fistula (IVC, intestine)

How well did you know this?
1
Not at all
2
3
4
5
Perfectly
1311
Q

Nutcracker syndrome

A

Nutcracker syndrome is a vascular compression disorder, and refers to the compression of the left renal vein between the superior mesenteric artery (SMA) and aorta. This can lead to renal venous hypertension, resulting in rupture of thin-walled veins into the collecting system with resultant haematuria.

How well did you know this?
1
Not at all
2
3
4
5
Perfectly
1312
Q

Relationship between popliteal aneurysms and AAA

A

50% of patients with popliteal aneurysm also have AAA

AAA is more common than popliteal aneurysm

How well did you know this?
1
Not at all
2
3
4
5
Perfectly
1313
Q

Very easily palpable popliteal pulse

50% bilateral

Expansile and pulsatile mass

A

Popliteal aneurysm

How well did you know this?
1
Not at all
2
3
4
5
Perfectly
1314
Q

Main complication of popliteal aneurysm

A

Thrombosis and distal embolism-> acute limb ischaemia

How well did you know this?
1
Not at all
2
3
4
5
Perfectly
1315
Q

Acute Mx of popliteal aneurysm

A

Embolectomy or fem-distal bypass

How well did you know this?
1
Not at all
2
3
4
5
Perfectly
1316
Q

Mx of stable popliteal aneurysm

A

Elective grafting + tie off vessel

How well did you know this?
1
Not at all
2
3
4
5
Perfectly
1317
Q

Epidemiology of AAA

A

Prevalence: 5% >50y

Mortality: 10,000 deaths/yr

M>F 3:1

How well did you know this?
1
Not at all
2
3
4
5
Perfectly
1318
Q

Def: AAA

A

Dilatation of abdominal aorta >3cm

90% infrarenal

30% involving iliac arteries

How well did you know this?
1
Not at all
2
3
4
5
Perfectly
1319
Q

Usually asymptomatic: discovered incidentally

May-> back pain or umbilical pain radiating to groin

Acute limb ischaemia

Blue toe syndrome: distal embolisation

Acute rupture

A

?AAA

How well did you know this?
1
Not at all
2
3
4
5
Perfectly
1320
Q

Expansile mass just above the umbilicus

Bruits heard

Tenderness and shock suggests?

A

AAA

Rupture

How well did you know this?
1
Not at all
2
3
4
5
Perfectly
1321
Q

Ix in AAA

A

Bloods:

FBC, clotting screen, renal function and liver function

Cross match if sx

ESR and or CRP if an inflammatory cause suspected

AXR: calcification may be seen

Abdo US: screening and monitoring

CT/MRI: gold standard

Angiography: won’t show true extent of aneurysm due to endoluminal thrombus but useful to delineate relationship of renal arteries

How well did you know this?
1
Not at all
2
3
4
5
Perfectly
1322
Q
A

AAA

Calcified

How well did you know this?
1
Not at all
2
3
4
5
Perfectly
1323
Q

Conservative Mx of AAA

A

Manage CV RFs esp BP

UK small aneurysm trial suggested that AAA <5.5cm in maximum diameter can be monitored by US

<4cm: yearly monitoring

4.5-5cm: 6 monthly monitoring

How well did you know this?
1
Not at all
2
3
4
5
Perfectly
1324
Q

Surgical Mx of nonacute AAA

A

Aim to treat aneurysm before it ruptures

Elective mortality: 5%, emergency mortality: 50%

How well did you know this?
1
Not at all
2
3
4
5
Perfectly
1325
Q

Indications for surgical intervention in AAA

A

Symptomatic: back pain= imminent rupture

Diameter >5.5cm

Expanding >1cm per year

Causing complications e.g. emboli

How well did you know this?
1
Not at all
2
3
4
5
Perfectly
1326
Q

Open vs EVAR in AAA repair

A

EVAR has reduced perioperative mortality

No reduction in mortality by 5 years due to fatal endograft failures

EVAR not better than medical Rx in unfit patients

EVAR requires that the aneurysm should have an adequate 1.2cm neck below the renal arteries for stent fixation, currently 65% with AAA are suitable for endovascular repair.

How well did you know this?
1
Not at all
2
3
4
5
Perfectly
1327
Q

AAA screening in UK

A

MASS trial revealed 50% reduction in aneurysm-related mortality in males aged 65-74 screened with US.

UK- one time screen at 65y

How well did you know this?
1
Not at all
2
3
4
5
Perfectly
1328
Q

Rupture rates of AAA:

<5.5cm

A

1%/year

How well did you know this?
1
Not at all
2
3
4
5
Perfectly
1329
Q

Rupture rates of AAA:

>6cm

A

25%/year

How well did you know this?
1
Not at all
2
3
4
5
Perfectly
1330
Q

Increased risk of AAA rupture if?

A

Raised BP

Smoker

Female

Strong FHx

How well did you know this?
1
Not at all
2
3
4
5
Perfectly
1331
Q

Sudden onset severe abdominal pain

Intermittent or continuous, radiating to back or flanks

Collapse-> shock

Expansile abdominal mass

A

?AAA rupture

How well did you know this?
1
Not at all
2
3
4
5
Perfectly
1332
Q

Mx of AAA rupture

A

ABC

High flow O2

2x large bore cannulae:

Give fluid if shocked but keep SBP<100mHg (permissive hypotension)

Instigate major haemorrhage protocol

Call vascular surgeon, anaesthetist and warn theatre

Abx prophylaxis: cef+met

Analgesia

Urinary catheter + CVP line

If stable + Dx uncertain: US or CT may be feasible

Take to theatre, clamp neck, insert dacron graft

How well did you know this?
1
Not at all
2
3
4
5
Perfectly
1333
Q

Mortality in AAA rupture

A

100% without surgery

50% with surgery

How well did you know this?
1
Not at all
2
3
4
5
Perfectly
1334
Q

Def: thoracic aortic dissection

A

Blood splays apart the laminar planes of the media to form a channel within the aortic wall

How well did you know this?
1
Not at all
2
3
4
5
Perfectly
1335
Q

Aetiology of thoracic aortic dissection

A

Atherosclerosis and HTN cause 90%

Minority caused by underlying CTD e.g. Marfan’s, Ehlers Danlos

Vit CD

How well did you know this?
1
Not at all
2
3
4
5
Perfectly
1336
Q

Sudden onset, tearing chest pain

Radiating through to back

Tachycardia and hypertension (1o and sympathetic)

A

?Aortic dissection

How well did you know this?
1
Not at all
2
3
4
5
Perfectly
1337
Q

Propagation in thoracic aortic dissection

A

Distal: sequential occlusion of branches

Left hemiplegia

Unequal arm pulses and BP

Paraplegia

Anuria

Proximal propagation: AR, tamponade

Rupture into pericardial, pleural or peritoneal cavities commonest cause of death

How well did you know this?
1
Not at all
2
3
4
5
Perfectly
1338
Q

What is the classification system for aortic dissection

A

Stanford

How well did you know this?
1
Not at all
2
3
4
5
Perfectly
1339
Q

Outline Stanford classification

A

Type A: proximal

70%

involves ascending aorta +/- descending

Higher mortality due to probable cardiac involvement

Usually require Sx

Type B: distal

30%

Involves descending only, distal to left subclavian artery

Usually managed conservatively

How well did you know this?
1
Not at all
2
3
4
5
Perfectly
1340
Q

Ix in thoracic aortic dissection

A

ECG: exclude MI

TTE/TOE can be used in haemodynamically unstable pts

CT/MRI not suitable for unstable paitents

How well did you know this?
1
Not at all
2
3
4
5
Perfectly
1341
Q

Mx of thoracic aortic dissection

A

ABC

Bloods:

X-match 10u, FBC, U+E, clotting, amylase

ECG: 20% show ischaemia due to involvement of the coronary ostia

Ix:

CXR, CT/MRI, TOE if haemodynamically unstable

Treat:

Analgesia

Reduce SBP (labetalol or esmolol) 100-110mmHg

Type A: open repair, acute mortality <25%

Type B: conservative, sx if persistent pain or complications, consider TEVAR if uncomplicated

How well did you know this?
1
Not at all
2
3
4
5
Perfectly
1342
Q

Def: gangrene

A

Death of tissue from poor vascular supply

How well did you know this?
1
Not at all
2
3
4
5
Perfectly
1343
Q

Classification of gangrene

A

Wet: tissue death + infection

Dry: tissue death only

Pregangrene: tissue on brink of gangrene

How well did you know this?
1
Not at all
2
3
4
5
Perfectly
1344
Q

Black tissues +/- slough

May be suppuration +/- sepsis

A

?Gangrene

How well did you know this?
1
Not at all
2
3
4
5
Perfectly
1345
Q
A

Wet gangrene

How well did you know this?
1
Not at all
2
3
4
5
Perfectly
1346
Q
A

Dry gangrene

How well did you know this?
1
Not at all
2
3
4
5
Perfectly
1347
Q
A

Pregangrene

How well did you know this?
1
Not at all
2
3
4
5
Perfectly
1348
Q

Cause of gas gangrene

A

Clostridium perfringes myositis

How well did you know this?
1
Not at all
2
3
4
5
Perfectly
1349
Q

RFs for gas gangrene

A

DM

Trauma

Malignancy

How well did you know this?
1
Not at all
2
3
4
5
Perfectly
1350
Q

Toxaemia

Haemolytic jaundice

Oedema

Creptius from surgical emphysema

Bubbly brown pus

A

Gas gangrene

How well did you know this?
1
Not at all
2
3
4
5
Perfectly
1351
Q

Rx of gas gangrene

A

Debridement: may need amputation

Benzylpenicllin and metronidazole

Hyperbaric O2

How well did you know this?
1
Not at all
2
3
4
5
Perfectly
1352
Q

What is synergistic gangrene?

A

Involves aerobes and anaerobes

Can progress rapidly to nec fasc and myositis

How well did you know this?
1
Not at all
2
3
4
5
Perfectly
1353
Q

Meleney Gangrene

A

Synergistic gangrene

Chronic undermining burrowing ulcer (also known as Meleney gangrene, or Meleney’s ulcer) is a cutaneous condition that is a postoperative, progressive bacterial gangrene.[1]:269 It is seen in immunocompromised individuals, mostly after post abdominal surgery and rapidly spreads to involve a large area.[citation needed]

How well did you know this?
1
Not at all
2
3
4
5
Perfectly
1354
Q

Fournier’s gangrene

A

Synergistic gangrene

Fournier gangrene is a type of necrotizing fasciitis or gangreneaffecting the perineum.

How well did you know this?
1
Not at all
2
3
4
5
Perfectly
1355
Q

Mx of gangrene

A

Take cultures

Debridement

Benzlpenicllin +/- clindamycin

How well did you know this?
1
Not at all
2
3
4
5
Perfectly
1356
Q

Def: varicose veins

A

Tortuous dilated veins of the superficial venous system

How well did you know this?
1
Not at all
2
3
4
5
Perfectly
1357
Q

Pathophysiology of varicose veins

A

One-way flow from superifical-> deep venous system maintained by valves

Valve failure-> increased pressure in superficial veins-> varicosity

How well did you know this?
1
Not at all
2
3
4
5
Perfectly
1358
Q

What are the 3 main sites of valve incompetence in varicose veins

A

SFJ: 3cm below and 3cm lateral to pubic tubercle

SPJ: popliteal fossa

Perforators: draining GSV

How well did you know this?
1
Not at all
2
3
4
5
Perfectly
1359
Q

What are the 3 medial calf perforators?

A

Cockett’s

How well did you know this?
1
Not at all
2
3
4
5
Perfectly
1360
Q

What is the 1 medial thigh perforator called?

A

Hunter’s

How well did you know this?
1
Not at all
2
3
4
5
Perfectly
1361
Q
A
How well did you know this?
1
Not at all
2
3
4
5
Perfectly
1362
Q

Primary causes of varicose veins

A

Idiopathic: congenitally weak valves

Prolonged standing, pregnancy, obesity, OCP, FHx

Congenital valve absence (v. rare)

How well did you know this?
1
Not at all
2
3
4
5
Perfectly
1363
Q

Secondary causes of varicose veins

A

Valve destruction-> reflux: DVT, thrombophlebitis

Obstruction: DVT, foetus, pelvic mass

Constipation

AVM

Overactive pumps e.g. cyclists

Klippel-Trenaunay: PWS, varicose veins, limb hypertrophy

How well did you know this?
1
Not at all
2
3
4
5
Perfectly
1364
Q

Trendelenberg test

A

Torniquet
Start 3cm below and 3cm lateral to pubic tubercle

Lift patient’s leg as high as comfortable and milk leg to empty veins.

Once varicosities are empty place thumb or torniquet over SFJ

Ask patient to stand while pressure is maintained

If varicosities rapidly fill, suggests the incompetent perforator veins lie below the SFJ

Now repeat moving down 3cm each time, when varicosities do not refill, the incompetent perforator is above the torniquet but below the previous one.

How well did you know this?
1
Not at all
2
3
4
5
Perfectly
1365
Q

Cosmetic defect

Pain, cramping, heaviness in the legs

Tingling

Bleeding; may be severe

Swelling

A

Varicose veins

How well did you know this?
1
Not at all
2
3
4
5
Perfectly
1366
Q

Venous stars

Haemosiderin deposition

Venous eczma

Lipodermatosclerosis

Atrophie blanche

Ulcer in medial malleolus/gaiter area

Oedema

Thrombophlebitis

A

Varicose veins

How well did you know this?
1
Not at all
2
3
4
5
Perfectly
1367
Q
A

Venous stars

How well did you know this?
1
Not at all
2
3
4
5
Perfectly
1368
Q
A

Haemosiderin deposition

Varicose veins

How well did you know this?
1
Not at all
2
3
4
5
Perfectly
1369
Q
A

Venous eczema

Varicose veins

How well did you know this?
1
Not at all
2
3
4
5
Perfectly
1370
Q
A

Lipodermatosclerosis

Varicose veins

How well did you know this?
1
Not at all
2
3
4
5
Perfectly
1371
Q
A

Atrophie blanche

Venous eczema

How well did you know this?
1
Not at all
2
3
4
5
Perfectly
1372
Q
A

Venous ulcer

How well did you know this?
1
Not at all
2
3
4
5
Perfectly
1373
Q

Ix in varicose veins

A

Duplex ultrasonography: anatomy, presence of incompetence, caused by obstruction or reflux

Sx: FBC, U&E, clotting, G+S, CXR, ECG

How well did you know this?
1
Not at all
2
3
4
5
Perfectly
1374
Q

Referral criteria for varicose veins

A

Bleeding

Pain

Ulceration

Superficial thrombophlebitis

Severe impact on QoL

How well did you know this?
1
Not at all
2
3
4
5
Perfectly
1375
Q

Classification of varicose veins is with?

A

CEAP classification

How well did you know this?
1
Not at all
2
3
4
5
Perfectly
1376
Q

CEAP classification

A

Chronic venous disease can be classified according to

Clinical signs (1-6 + Symptomatic or Asymptomatic) i.e. 1S or 4A

Etiology

Anatomy

Pathophysiology

How well did you know this?
1
Not at all
2
3
4
5
Perfectly
1377
Q

Conservative Mx of varicose veins

A

Treat any contributing factors: lose weight, relieve constipation

Education: avoid prolonged standing, regular walks

Class II Graduated compression stockings: symptomatic relief and slows progression

Skin care: maintain hydration with emollients and treat ulcers rapidly

How well did you know this?
1
Not at all
2
3
4
5
Perfectly
1378
Q

Indications for minimally invasive therapies in varicose veins treatment

A

Small below knee varicosities not involving GSV or SSV

How well did you know this?
1
Not at all
2
3
4
5
Perfectly
1379
Q

Techniques of minimally invasive varicose vein therapy

A

Local or GA

Injection scleropathy: 1% Na tetradecyl sulphate

Endovenous laser or radiofrequency ablation

Post-operatively:

Compression bandage for 24h

Compression stockings for 1m

How well did you know this?
1
Not at all
2
3
4
5
Perfectly
1380
Q

Indications for sx management of varicose veins

A

SFJ incompetence

Major perforator incompetence

Symptomatic: ulceration, skin changes, pain

How well did you know this?
1
Not at all
2
3
4
5
Perfectly
1381
Q

Procedures for varicose veins

A

Trendelenberg: SFJ ligation

SSV ligation: popliteal fossa

LSV stripping: no longer performed due to risk of saphenous nerve damage

Multiple avulsions

Perforator ligation: Cockett’s operation

Subfascial endoscopic perforator surgery

How well did you know this?
1
Not at all
2
3
4
5
Perfectly
1382
Q

Post-op care for varicose vein patients

A

Bandage tightly

Elevate for 24h

Discharge with compression stockings and instruct to walk daily

How well did you know this?
1
Not at all
2
3
4
5
Perfectly
1383
Q

Cx of varicose vein surgery

A

Haematoma (esp. groin)

Wound sepsis

Damage to cutaneous nerve e.g. long saphenous

Superficial thrombophlebitis

DVT

Recurrence: may approach 50%

How well did you know this?
1
Not at all
2
3
4
5
Perfectly
1384
Q

Def: ulcer

A

Interruption in the continuity of an epithelial surface

How well did you know this?
1
Not at all
2
3
4
5
Perfectly
1385
Q

What is the commonest cause of leg ulcer?

A

Venous

How well did you know this?
1
Not at all
2
3
4
5
Perfectly
1386
Q

What are the different forms of leg ulcers?

A

Venous

Arterial

Neuropathic

Traumatic

Systemic disease

Neoplastic

How well did you know this?
1
Not at all
2
3
4
5
Perfectly
1387
Q

Painless, sloping, shallow ulcer

Usually on gaiter area

Associated with lipodermatosclerosis

RFs: venous insufficiency, varicosities, DVT, obesity

A

?Venous ulcer

How well did you know this?
1
Not at all
2
3
4
5
Perfectly
1388
Q

Proportion of leg ulcers that are venous

A

75%

How well did you know this?
1
Not at all
2
3
4
5
Perfectly
1389
Q

Proportion of leg ulcers that are arterial

A

2%

How well did you know this?
1
Not at all
2
3
4
5
Perfectly
1390
Q

Hx of vasculopathy and risk factors

Painful, deep, punched-out lesions

Occur at pressure points: heel, tips of and between toes, metatarsal heads (esp. 5th)

Other signs of chronic leg ischaemia

A

Arterial ulcer

How well did you know this?
1
Not at all
2
3
4
5
Perfectly
1391
Q
A

Arterial ulcer

How well did you know this?
1
Not at all
2
3
4
5
Perfectly
1392
Q

Painless ulcer with insensate surrounding skin

Warm foot with good pulses

A

Neuropathic ulcer

How well did you know this?
1
Not at all
2
3
4
5
Perfectly
1393
Q
A

Neuropathic ulcer

How well did you know this?
1
Not at all
2
3
4
5
Perfectly
1394
Q

Cx of ulcers

A

Osteomyelitis

Development of SCC in ulcer (Marjolin’s)

How well did you know this?
1
Not at all
2
3
4
5
Perfectly
1395
Q

Marjolin’s ulcer=

A

Development of SCC in ulcer

How well did you know this?
1
Not at all
2
3
4
5
Perfectly
1396
Q

Ix in venous ulcers?

A

ABPI if possible

Duplex ultrasonography

Biopsy may be necessary to look for malignant change

How well did you know this?
1
Not at all
2
3
4
5
Perfectly
1397
Q

Mx of venous ulcers

A

Refer to leg ulcer community clinic

Focus on prevention: graduated compression stockings, venous uclers

Optimise risk factors: nutrition, smoking

How well did you know this?
1
Not at all
2
3
4
5
Perfectly
1398
Q

Specific Rx for venous ulcers

A

Analgesia

Bed Rest and Elevate leg

4 layer graded compression bandage if ABPI >0.8

Pentoxyfylline PO: increases microcirculatory blood flow, improves healing rates

How well did you know this?
1
Not at all
2
3
4
5
Perfectly
1399
Q

DDx fo bilateral leg swelling

A

Increased venous pressure:

RHF

Venous insufficiency

Drugs e.g. nifedipine

Reduced oncotic pressure:

Nephrotic syndrome

Hepatic failure

Protein losing enteropathy

Lymphoedema

Myxoedema: hyper/hypothyroidism

How well did you know this?
1
Not at all
2
3
4
5
Perfectly
1400
Q

DDx of unilateral leg swelling

A

Venous insufficiency

DVT

Infection or inflammation

Lymphoedema

How well did you know this?
1
Not at all
2
3
4
5
Perfectly
1401
Q

Def: Lymphoedema

A

Collection of interstitial fluid due to blockage or absence of lymphatics

How well did you know this?
1
Not at all
2
3
4
5
Perfectly
1402
Q

Primary lymphoedema

A

Congenital absence of lymphatics

May or may not be familial

How well did you know this?
1
Not at all
2
3
4
5
Perfectly
1403
Q

Different types of lymphoedema and presentations

A

Congenital: evident from birth

Praecox: after birth but <35

Tarda: >35

How well did you know this?
1
Not at all
2
3
4
5
Perfectly
1404
Q

Milroy’s syndrome

A

Familial AD subtype of congenital lymphoedema: F>M

How well did you know this?
1
Not at all
2
3
4
5
Perfectly
1405
Q

Secondary causes of lymphoedema

FIIT

A

Fibrosis; e.g. post RTx

Infiltration:

Ca: prostate, lymphoma

Filariasis

Infection: TB

Trauma: block dissection of lymphatics

How well did you know this?
1
Not at all
2
3
4
5
Perfectly
1406
Q

Ix in lymphoedema

A

Doppler US

Lymphoscintigraphy

CT/MRI

How well did you know this?
1
Not at all
2
3
4
5
Perfectly
1407
Q

Mx of lymphoedema

A

Conservative:

Skin care, compression stockings, physio, treat or prevent comorbid infections

Surgical: debulking

How well did you know this?
1
Not at all
2
3
4
5
Perfectly
1408
Q

Epidemiology of DVT post surgically

A

DVTs occur in 25-50% of surgical patients

How well did you know this?
1
Not at all
2
3
4
5
Perfectly
1409
Q

Risk factors for DVT post-Sx

A

Virchow’s triad

Blood:

Surgery-> increased plt and fibrinogen

Dehydration

Malignancy

Increased age

Blood flow:

Sx

Immobility

Obesity

Vessel wall:

damage to veins: esp. pelvic veins

Previous VTE

How well did you know this?
1
Not at all
2
3
4
5
Perfectly
1410
Q

Peak incidence @5-10d

65% below knee are asymptomatic

Calf warmth, tenderness, erythema, swelling

Low grade pyrexia

Pitting oedema

A

?DVT

How well did you know this?
1
Not at all
2
3
4
5
Perfectly
1411
Q
A

DVT

How well did you know this?
1
Not at all
2
3
4
5
Perfectly
1412
Q

DDx for DVT

A

Cellulitis

Ruptured Baker’s cyst

How well did you know this?
1
Not at all
2
3
4
5
Perfectly
1413
Q

Ix in DVT

A

D-dimers: sensitive but not specific

Compression US: clot will be incompressible

Thrombophilia screen if no precipitating factors, recurrent DVT or family Hx

How well did you know this?
1
Not at all
2
3
4
5
Perfectly
1414
Q

Dx of DVT

A

Assess probability using Well’s Score

Low probability: perform D-dimers

-ve= excludes DVT

+ve= compression US

Medium/high probability: compression USS

How well did you know this?
1
Not at all
2
3
4
5
Perfectly
1415
Q

What score is used to assess likelihood of DVT?

A

Well’s score

How well did you know this?
1
Not at all
2
3
4
5
Perfectly
1416
Q

Well’s criteria

A

Active cancer

Paralysis, paresis or recent plaster immobilisation of legs

Recently bedridden for >3d or major sx within last 12w

Localised tenderness along the distribution of the deep venous system

Entire leg is swollen

Calf swelling by more than 3cm compared with asymptomatic leg (10cm below the tibial tuberosity)

Piiting oedema (greater than on the asymptomatic leg)

Pitting oedema

Collateral superficial veins

Perviously documented DVT

Subtract two points if an alternative cause is considered more likely than DVT

>2= likely, <2 unlikely

How well did you know this?
1
Not at all
2
3
4
5
Perfectly
1417
Q

Rx of DVT

A

Antiocagulate:

Therapeutic LMWH: enoxaparin 1.5mg/kg/24h SC

Start warfrain

Stop LMWH when INR 2.5

Duration depends on cause

How well did you know this?
1
Not at all
2
3
4
5
Perfectly
1418
Q

Below knee DVT duration of anticoag

A

6-12w

How well did you know this?
1
Not at all
2
3
4
5
Perfectly
1419
Q

Above knee DVT duration of anticoag

A

3-6m

How well did you know this?
1
Not at all
2
3
4
5
Perfectly
1420
Q

Initiation of warfarin treatment (rapid anticoagulation)

A

5mg OD for 2 days, measure INR on day 3

subsequent doses depend on INR

How well did you know this?
1
Not at all
2
3
4
5
Perfectly
1421
Q

What is important about timing of Warfarin dose

A

Should be taken at the same time each day

How well did you know this?
1
Not at all
2
3
4
5
Perfectly
1422
Q

What should be considered for patients post-DVT?

A

Graduated compression stockings to prevent post-phlebitic syndrome

How well did you know this?
1
Not at all
2
3
4
5
Perfectly
1423
Q

Pre-op prevention of DVT

A

Pre-op VTE risk assessment

TED stockings

Aggressive optimisation esp. hydration

Stop OCP 4w pre-op

How well did you know this?
1
Not at all
2
3
4
5
Perfectly
1424
Q

Intra-operative prevention of DVT

A

Minimise length of sx

Use minimal access surgery where possible

Intermittent pneumatic compression boots

How well did you know this?
1
Not at all
2
3
4
5
Perfectly
1425
Q

Post-op prevention of DVT

A

LMWH

Early mobilisation

Good analgesia

Physio

Adequate hydration

How well did you know this?
1
Not at all
2
3
4
5
Perfectly
1426
Q

Causes of post-operative dyspnoea

A

Previous lung disease

Atelectasis, aspiration, pneumonia

LVF

PE

Pneumothorax (e.g. due to CVP line insertion)

Pain-> hypoventilation

How well did you know this?
1
Not at all
2
3
4
5
Perfectly
1427
Q

Ix in post-op dyspnoea

A

FBC, ABG

CXR

ECG

How well did you know this?
1
Not at all
2
3
4
5
Perfectly
1428
Q

Rx in post-op dyspnoea

A

Sit up, give O2, monitor SpO2

Rx cause

How well did you know this?
1
Not at all
2
3
4
5
Perfectly
1429
Q

Causes of reduced UO post-op

A

Post renal:

Most common cause- blocked/malsited catheter, acute urinary retention

Pre-renal:

Hypovolaemia

Renal:

NSAIDs, gentamicin

How well did you know this?
1
Not at all
2
3
4
5
Perfectly
1430
Q

Anuria post-op usually

A

Blocked or malsited catheter

How well did you know this?
1
Not at all
2
3
4
5
Perfectly
1431
Q

Oliguria post op usually

A

Inadequate fluid replacement

How well did you know this?
1
Not at all
2
3
4
5
Perfectly
1432
Q

Mx of reduced UO post-operatively

A

Information:

Operation history, obs chart: UO

Drugs chart: nephrotoxins

Examination:

Assess fluid status

Examine for palpable bladder

Inspect drips, drains, stomas, CVP

Action

Flush with 50ml NS and aspirate back

Fluid challenge

How well did you know this?
1
Not at all
2
3
4
5
Perfectly
1433
Q

Causes of post-op N+V

A

Obstruction

Ileus

Emetic drugs e.g. opioids

How well did you know this?
1
Not at all
2
3
4
5
Perfectly
1434
Q

Mx of post-op N+V

A

Consider NGT, AXR and ondansetron 4mg IV TDS

How well did you know this?
1
Not at all
2
3
4
5
Perfectly
1435
Q

Post op hyponatraemia

A

What was the pre-op level

Common causes: SIADH: pain, nausea, opioids, stress

Overadministration of IV fluids

How well did you know this?
1
Not at all
2
3
4
5
Perfectly
1436
Q

Immediate mx of hypotension post-sx

A

Tilt bed head down give O2

Assess fluid status

How well did you know this?
1
Not at all
2
3
4
5
Perfectly
1437
Q

Causes of post-operative hypotension

CHOD

A

Cardiogenic: MI, fluid overload

Hypovolaemia: inadequate replacement of fluids losses, haemorrhage

Obstructive: PE

Distributive: sepsis, neurogenic shock

How well did you know this?
1
Not at all
2
3
4
5
Perfectly
1438
Q

Mx of hypovolaemic hypotension

A

Fluid challenge

250-500ml fluid challenge STAT

How well did you know this?
1
Not at all
2
3
4
5
Perfectly
1439
Q

Mx of haemorrhagic hypotension post-op

A

Return to theatre

How well did you know this?
1
Not at all
2
3
4
5
Perfectly
1440
Q

Mx of septic hypotension post-op

A

Fluid challenge, start Abx

How well did you know this?
1
Not at all
2
3
4
5
Perfectly
1441
Q

Mx of neurogenic hypotension post-op

A

Nadr infusion

How well did you know this?
1
Not at all
2
3
4
5
Perfectly
1442
Q

HTN and sx

A

Continue anti-hypertensives during peri-operative period

How well did you know this?
1
Not at all
2
3
4
5
Perfectly
1443
Q

Causes of post-operative HTN

A

Pain

Urinary retention

Previous HTN

How well did you know this?
1
Not at all
2
3
4
5
Perfectly
1444
Q

Rx of post-operative HTN

A

Rx cause

May use labetalol 50mg IV every 5 mins (200mg max)

How well did you know this?
1
Not at all
2
3
4
5
Perfectly
1445
Q

Def: dermatome

A

An area of skin that is mainly supplied by a single spinal nerve

How well did you know this?
1
Not at all
2
3
4
5
Perfectly
1446
Q
A
How well did you know this?
1
Not at all
2
3
4
5
Perfectly
1447
Q

Which dermatome is the thumb?

A

C6

How well did you know this?
1
Not at all
2
3
4
5
Perfectly
1448
Q
A
How well did you know this?
1
Not at all
2
3
4
5
Perfectly
1449
Q

What dermatome at level of the nipples?

A

T4

T4 at teat pore

How well did you know this?
1
Not at all
2
3
4
5
Perfectly
1450
Q

What dermatome at level of xihpoid process?

A

T7 at bottom of sternum

How well did you know this?
1
Not at all
2
3
4
5
Perfectly
1451
Q

What dermatome at level of umbilicus?

A

T10 at belly butTEN

How well did you know this?
1
Not at all
2
3
4
5
Perfectly
1452
Q

What dermatome at level of the inguinal ligament?

A

L1 for IL

How well did you know this?
1
Not at all
2
3
4
5
Perfectly
1453
Q

What dermatome at patella/medial malleolus?

A

L4 to the floor

How well did you know this?
1
Not at all
2
3
4
5
Perfectly
1454
Q
A
How well did you know this?
1
Not at all
2
3
4
5
Perfectly
1455
Q

What are the KLM sounds?

A

Series of sounds used to examine cranial nerve function

Each tests a specific muscle and nerve pair

How well did you know this?
1
Not at all
2
3
4
5
Perfectly
1456
Q

Kuh-kuh-kuh

Muscle tested

Nerve tested

A

Levator palatini: responsible for palatal elevation

Nerve required: CNX

How well did you know this?
1
Not at all
2
3
4
5
Perfectly
1457
Q

La-La-La

Muscle required

Nerve required

A

Glossus muscles

CNXII

How well did you know this?
1
Not at all
2
3
4
5
Perfectly
1458
Q

Mi-mi-mi

Muscle required

Nerve required

A

Perioral muscles

CNVII

How well did you know this?
1
Not at all
2
3
4
5
Perfectly
1459
Q

What are the most common causes of sudden painless loss of vision?

A

ischaemic optic neuropathy (e.g. temporal arteritis or atherosclerosis)

occlusion of central retinal vein

occlusion of central retinal artery

vitreous haemorrhage

retinal detachment

How well did you know this?
1
Not at all
2
3
4
5
Perfectly
1460
Q

Flashes of light (photopsia) - in the peripheral field of vision
Floaters, often on the temporal side of the central vision

A

Posterior vitreous detachment

How well did you know this?
1
Not at all
2
3
4
5
Perfectly
1461
Q

Dense shadow that starts peripherally progresses towards the central vision
A veil or curtain over the field of vision
Straight lines appear curved
Central visual loss

A

Retinal detachment

How well did you know this?
1
Not at all
2
3
4
5
Perfectly
1462
Q

Large bleeds cause sudden visual loss
Moderate bleeds may be described as numerous dark spots
Small bleeds may cause floaters

A

Vitreous haemorrhage

How well did you know this?
1
Not at all
2
3
4
5
Perfectly
1463
Q

Red eye

Severe pain (ocular or headache)

Reduced acuity + haloes

Semi-dilated pupil

Hazy cornea

A

Acute angle closure glaucoma

How well did you know this?
1
Not at all
2
3
4
5
Perfectly
1464
Q

Red eye

Acute onset

Pain

Blurred vision and photophobia

Small, fixed, oval pupil, ciliary flush

A

Anterior uveitis

How well did you know this?
1
Not at all
2
3
4
5
Perfectly
1465
Q

Red eye

Severe pain and tenderness

May be underlying autoimmune disease e.g. RA

A

Scleritis

How well did you know this?
1
Not at all
2
3
4
5
Perfectly
1466
Q

Red eye

Purulent discharge (bacterial)

Clear discharge (viral)

A

Conjunctivitis

How well did you know this?
1
Not at all
2
3
4
5
Perfectly
1467
Q

Red eye

History of trauma of coughing bouts

A

Subconjunctival haemorrhage

How well did you know this?
1
Not at all
2
3
4
5
Perfectly
1468
Q

Mx of herpes zoster ophthalmicus

A

Oral antiviral treatment for 7-0 days, ideally started within 72h

Oral corticosteroids may reduce duration of pain

Ophthalmology review

How well did you know this?
1
Not at all
2
3
4
5
Perfectly
1469
Q

Red eye

Serous discharge
Recent URTI
Preauricular lymph nodes

A

Viral conjunctivitis

How well did you know this?
1
Not at all
2
3
4
5
Perfectly
1470
Q

Red eye

Bilateral symptoms
Itch is prominent
May be history of atopy
May be seasonal (due to pollen) or perennial (due to dust mite, washing powder or other allergens)

A

Allergic conjunctivitis

How well did you know this?
1
Not at all
2
3
4
5
Perfectly
1471
Q

Drusen=

A

Dry macular degeneration

How well did you know this?
1
Not at all
2
3
4
5
Perfectly
1472
Q

Characterised by choroidal neovascularisation. Leakage of serous fluid and blood can subsequently result in a rapid loss of vision. Carries worst prognosis

A

Wet macular degeneration

How well did you know this?
1
Not at all
2
3
4
5
Perfectly
1473
Q

characterised by drusen - yellow round spots in Bruch’s membrane

A

Dry macular degneration

How well did you know this?
1
Not at all
2
3
4
5
Perfectly
1474
Q

intravitreal ranibizumab

A

Used to treat wet macular degeneration

Anti-VEGF

How well did you know this?
1
Not at all
2
3
4
5
Perfectly
1475
Q

What are the ocular manifestations of RA?

A

keratoconjunctivitis sicca (most common)

episcleritis (erythema)

scleritis (erythema and pain)

corneal ulceration

keratitis

Iatrogenic

steroid-induced cataracts

chloroquine retinopathy

How well did you know this?
1
Not at all
2
3
4
5
Perfectly
1476
Q

Which one of the following is not a risk factor for primary open-angle glaucoma?

Diabetes mellitus

Family history

Hypertension

Afro-Caribbean ethnicity

Hypermetropia

A

Acute angle closure glaucoma is associated with hypermetropia, where as primary open-angle glaucoma is associated with myopia

How well did you know this?
1
Not at all
2
3
4
5
Perfectly
1477
Q

What differentiates between central, preganglionic and post-ganglionic lesions in Horners?

A

Anhydrosis of the face, arm and trunk= central

Anhydrosis of the face= preganglionic

No anhydrosis= post-ganglionic

How well did you know this?
1
Not at all
2
3
4
5
Perfectly
1478
Q

Central causes of horner’s

S

A

Stroke

Syringomelia

MS

Tumour
Encephalitis

How well did you know this?
1
Not at all
2
3
4
5
Perfectly
1479
Q

Pre-ganglionic causes of Horner’s

A

Pancoast

Thyroidectomy

Trauma

Cervical rib

How well did you know this?
1
Not at all
2
3
4
5
Perfectly
1480
Q

Post-ganglionic causes of horners

A

Carotid artery dissection

Carotid aneurysm

CVST

Cluster headache

How well did you know this?
1
Not at all
2
3
4
5
Perfectly
1481
Q

What is the most likely diagnosis?

Ciliary body rupture with lens dislocation

Vitreous haemorrhage

Ischaemic optic neuropathy

Central retinal artery

Retinal detachment

A

Retinal detachment

How well did you know this?
1
Not at all
2
3
4
5
Perfectly
1482
Q

Causes of papilloedema

A

Causes of papilloedema

space-occupying lesion: neoplastic, vascular

malignant hypertension

idiopathic intracranial hypertension

hydrocephalus

hypercapnia

Rare causes include

hypoparathyroidism and hypocalcaemia

vitamin A toxicity

How well did you know this?
1
Not at all
2
3
4
5
Perfectly
1483
Q

venous engorgement: usually the first sign

loss of venous pulsation: although many normal patients do not have normal pulsation

blurring of the optic disc margin

elevation of optic disc

loss of the optic cup

Paton’s lines: concentric/radial retinal lines cascading from the optic disc

A

Papilloedema

How well did you know this?
1
Not at all
2
3
4
5
Perfectly
1484
Q

Theme: Visual field defects

A.Left homonymous hemianopia

B.Unilateral peripheral visual field loss

C.Bitemporal hemianopia, upper quadrant defect

D.Unilateral central visual field loss

E.Left homonymous hemianopia with macula sparing

F.Right homonymous hemianopia

G.Bitemporal hemianopia, lower quadrant defect

For each one of the following please select the associated visual field defect:

Pituitary gland tumour

Primary open angle glaucoma in right eye

Patient who has had an extensive stroke with right-sided hemiplegia

A

Pit- upper, cranipharyngioma- lower

Bitemporal hemianopia, upper quadrant defect

Unilateral peripheral visual field loss

Right homonymous hemianopia

Remember: the homonymous hemianopia is always on the same side as the paresis.

How well did you know this?
1
Not at all
2
3
4
5
Perfectly
1485
Q

Upper quadrant vs lower quadrant bitemporal hemianopia

A

upper quadrant defect > lower quadrant defect = inferior chiasmal compression, commonly a pituitary tumour

lower quadrant defect > upper quadrant defect = superior chiasmal compression, commonly a craniopharyngioma

How well did you know this?
1
Not at all
2
3
4
5
Perfectly
1486
Q

A 35-year-old man presents with visual problems. He has had very poor vision in the dark for a long time but is now worried as he is developing ‘tunnel vision’. He states his grandfather had a similar problem and was registered blind in his 50’s. What is the most likely diagnosis?

Leber’s congenital amaurosis

Vitelliform macular dystrophy

Central serous retinopathy

Primary open angle glaucoma

Retinitis pigmentosa

A

Retinitis pigmentosa primarily affects the peripheral retina resulting in tunnel vision

Features

night blindness is often the initial sign

tunnel vision due to loss of the peripheral retina (occasionally referred to as funnel vision)

fundoscopy: black bone spicule-shaped pigmentation in the peripheral retina, mottling of the retinal pigment epithelium

Associated diseases

Refsum disease: cerebellar ataxia, peripheral neuropathy, deafness, ichthyosis

Usher syndrome

abetalipoproteinemia

Lawrence-Moon-Biedl syndrome

Kearns-Sayre syndrome

Alport’s syndrome

How well did you know this?
1
Not at all
2
3
4
5
Perfectly
1487
Q

A 35-year-old female who has recently being diagnosed with Grave’s disease presents for review 3 months after starting a ‘block and replace’ regime with carbimazole and thyroxine. She is concerned about developing thyroid eye disease. What is the best way that her risk of developing thyroid eye disease can be reduced?

Reduce alcohol intake

A diet rich in omega-3 fatty acids

Regular exercise

Stop smoking

Lose weight

A

Smoking is the most important modifiable risk factor for the development of thyroid eye disease

How well did you know this?
1
Not at all
2
3
4
5
Perfectly
1488
Q

What are the indications for urgent opthalmology r/v in thyroid eye disease?

A

unexplained deterioration in vision

awareness of change in intensity or quality of colour vision in one or both eyes

history of eye suddenly ‘popping out’ (globe subluxation)

obvious corneal opacity

cornea still visible when the eyelids are closed

disc swelling

How well did you know this?
1
Not at all
2
3
4
5
Perfectly
1489
Q

A 34-year-old man with a history of ankylosing spondylitis presents with a painful right eye associated with mild photophobia:

Cycloplegic drops have recently been given. What is the most likely diagnosis?

Scleritis

Acute angle closure glaucoma

Anterior uveitis

Conjunctivitis

Episcleritis

A

Ankylosing spondylitis is associated with anterior uveitis. The history of pain and photophobia combined with the examination findings of a red eye confirm the diagnosis

Note that the pupil in this image is dilated - this is not a typical finding at diagnosis but may reflect the fact that cycloplegic drops are commonly given to patients for pain relief

How well did you know this?
1
Not at all
2
3
4
5
Perfectly
1490
Q

What is the normal range for intraocular pressure?

A

10-21 mmHg

How well did you know this?
1
Not at all
2
3
4
5
Perfectly
1491
Q

A 64-year-old woman presents with bilateral sore eyelids. She also complains of her eyes being dry all the time. On examination her eyelid margins are erythematous at the margins but are not swollen. Of the given options, what is the most appropriate initial management?

Topical chloramphenicol + mechanical removal of lid debris

Hot compresses + topical steroids

Topical chloramphenicol + topical steroids

Hot compresses + mechanical removal of lid debris

Topical chloramphenicol + hot compresses

A

Hot compresses + mechanical removal of lid debris

How well did you know this?
1
Not at all
2
3
4
5
Perfectly
1492
Q

symptoms are usually bilateral

grittiness and discomfort, particularly around the eyelid margins

eyes may be sticky in the morning

eyelid margins may be red. Swollen eyelids may be seen in staphylococcal blepharitis

styes and chalazions are more common in patients with blepharitis

secondary conjunctivitis may occur

A

Blepharitis

How well did you know this?
1
Not at all
2
3
4
5
Perfectly
1493
Q

What eye condition is more common in patients with rosacea?

A

Blepharitis

How well did you know this?
1
Not at all
2
3
4
5
Perfectly
1494
Q

Mx of blepharitis

A

softening of the lid margin using hot compresses twice a day

mechanical removal of the debris from lid margins - cotton wool buds dipped in a mixture of cooled boiled water and baby shampoo is often used*

artificial tears may be given for symptom relief in people with dry eyes or an abnormal tear film

How well did you know this?
1
Not at all
2
3
4
5
Perfectly
1495
Q

A 67-year-old man presents as he has developed a painful blistering rash around his right eye. On examination a vesicular rash covering the right trigeminal nerve dermatome is seen. Currently he has no eye symptoms or signs. Which one of the following is most likely to predict future eye involvement?

Presence of the rash on the tip of his nose

Smoking history

Increasing age

Previous courses of corticosteroids

Presence of the rash in the ear canal

A

This is Hutchinson’s sign which is strongly predictive for ocular involvement.

How well did you know this?
1
Not at all
2
3
4
5
Perfectly
1496
Q

A 60-year-old woman who has recently started treatment for polymyalgia rheumatica presents with a five day history of headaches and reduced vision on the right side since this morning There is no eye pain but the there is a ‘large, dark shadow’ covering the superior visual field on the right side. On examination she has a tender, palpable right temporal artery. What is the most likely explanation for the reduced vision?

Anterior ischemic optic neuropathy

Central retinal vein occlusion

Optic neuritis

Acute angle closure glaucoma

Central retinal artery occlusion

A

Anterior ischemic optic neuropathy

How well did you know this?
1
Not at all
2
3
4
5
Perfectly
1497
Q

What causes the visual disturbances in temporal arteritis?

A

Anterior ischaemic optic neuropathy

How well did you know this?
1
Not at all
2
3
4
5
Perfectly
1498
Q

A 54-year-old woman presents with a persistent watery left eye for the past 4 days. On examination there is erythema and swelling of the inner canthus of the left eye. What is the most likely diagnosis?

Blepharitis

Acute angle closure glaucoma

Meibomian cyst

Dacryocystitis

Pinguecula

A

Dacryocystitis is infection of the lacrimal sac

Features

watering eye (epiphora)

swelling and erythema at the inner canthus of the eye

Management is with systemic antibiotics. Intravenous antibiotics are indicated if there is associated periorbital cellulitis

How well did you know this?
1
Not at all
2
3
4
5
Perfectly
1499
Q

At what age would the average child be expected to have visual acuity similar to that of an adult?

3 months

6 months

9 months

12 months

2 years

A

2 years

A newborn’s visual acuity is only about 6/200. This improves to 6/60 at 3 months but does no reach adult levels until about 2 years of age.

The table below summarises the vision tests which may be performed when assessing children:

AgeTest

BirthRed reflex

6 weeksFix and follow to 90 degrees (e.g. Red ball 90cm away)

3 monthsFix and follow to 180 degrees
No squint

12 monthsCan pick up ‘hundreds and thousands’ with pincer grip

> 3 yearsLetter matching test

> 4 yearsSnellen charts
Ishihara plates for colour vision

How well did you know this?
1
Not at all
2
3
4
5
Perfectly
1500
Q

A 24-year-old man presents to the emergency department complaining of left eye pain. He has not been able to wear his contact lenses for the past 24 hours due to the pain. He describes the pain as severe and wonders whether he has ‘got something stuck in his eye’. On examination there is diffuse hyperaemia of the left eye. The left cornea appears hazy and pupillary reaction is normal. Visual acuity is reduced on the left side and a degree of photophobia is noted. A hypopyon is also seen. What is the most likely diagnosis?

Acute angle closure glaucoma

Viral conjunctivitis

Keratitis

Episcleritis

Anterior uveitis

A

Whilst a hypopyon can of course be seen in anterior uveitis the combination of a normal pupillary reaction and contact lens use make a diagnosis of keratitis more likely.

How well did you know this?
1
Not at all
2
3
4
5
Perfectly
1501
Q

A 70-year-old man presented with a watering and sore left eye:

What is the diagnosis?

Left-sided ectropion

Left-sided pinguecula

Stroke

Bell’s palsy

Left-sided entropion

A

Left sided ectropion

How well did you know this?
1
Not at all
2
3
4
5
Perfectly
1502
Q

What is the most common cause of blindness in adults 35-65 years old?

A

Diabetic retinopathy

How well did you know this?
1
Not at all
2
3
4
5
Perfectly
1503
Q

A 54-year-old man is noted to have papilloedema on examination. Which one of the following may be responsible?

Vitamin D toxicity

Hypercapnia

Hyperkalaemia

Hypercalcaemia

Hypoglycaemia

A

Hyperventilation to induce hypocapnia may be used in the emergency setting to reduce intracranial pressure

How well did you know this?
1
Not at all
2
3
4
5
Perfectly
1504
Q

A 23-year-old female presents with recurrent headaches. Examination of her cranial nerves reveals the right pupil is 3 mm whilst the left pupil is 5 mm. The right pupil constricts to light but the left pupil is sluggish. Peripheral neurological examination is unremarkable apart from difficult to elicit knee and ankle reflexes. What is the most likely diagnosis?

Acute angle closure glaucoma

Migraine

Multiple sclerosis

Holmes-Adie syndrome

Argyll-Roberson syndrome

A

olmes-Adie pupil is a benign condition most commonly seen in women. It is one of the differentials of a dilated pupil.

Overview

unilateral in 80% of cases

dilated pupil

once the pupil has constricted it remains small for an abnormally long time

slowly reactive to accommodation but very poorly (if at all) to light

Holmes-Adie syndrome

association of Holmes-Adie pupil with absent ankle/knee reflexes

How well did you know this?
1
Not at all
2
3
4
5
Perfectly
1505
Q

A 65-year-old man with a history of primary open-angle glaucoma presents with sudden painless loss of vision in his right eye. On examination of the right eye the optic disc is swollen with multiple flame-shaped and blot haemorrhages. What is the most likely diagnosis?

Diabetic retinopathy

Vitreous haemorrhage

Ischaemic optic neuropathy

Occlusion of the central retinal vein

Occlusion of the central retinal artery

A

Central retinal vein occlusion - sudden painless loss of vision, severe retinal haemorrhages on fundoscopy

How well did you know this?
1
Not at all
2
3
4
5
Perfectly
1506
Q

heme: Primary open-angle glaucoma: management

A.Timolol

B.Pilocarpine

C.Brimonidine

D.Dorzolamide

E.Acetylcysteine

F.Ranibizumab

G.Atropine

H.Latanoprost

I.Cyclopentolate

J.Tropicamide

For each one of the following questions please select the correct answer:

Adverse effects include brown pigmentation of the iris

Should be avoided in patients taking MAOI drugs

Causes pupillary constriction, blurred vision and headaches

A

Latanoprost

Brimonidine

Pilocarpine

How well did you know this?
1
Not at all
2
3
4
5
Perfectly
1507
Q

A man is recovering after having an operation to remove a meningioma in his left temporal lobe. What sort of visual field defect is he at risk of having following the procedure?

Right inferior homonymous quadrantanopia

Right superior homonymous quadrantanopia

Left inferior homonymous quadrantanopia

Right homonymous hemianopia with macula sparing

Left superior homonymous quadrantanopia

A

homonymous quadrantanopias: PITS (Parietal-Inferior, Temporal-Superior)

How well did you know this?
1
Not at all
2
3
4
5
Perfectly
1508
Q

A 70-year-old man is investigated for blurred vision. Fundoscopy reveals drusen, retinal epithelial and macular neovascularisation. A diagnosis of age related macular degeneration is suspected. What is the most appopriate next investigation?

Vitreous fluid sampling

MRI orbits

Ocular tonometry

Fluorescein angiography

Kinetic perimetry

A

optical coherence tomography: provide cross-sectional views of the macula

if neovascularisation is present fluorescein angiography is performed

How well did you know this?
1
Not at all
2
3
4
5
Perfectly
1509
Q

73-year-old man complains of a sore right eye:

What is the diagnosis?

Dacryocystitis

Chalazion

Entropion

Blepharitis

Ectropion

A

If left untreated this patient may develop a corneal ulcer. The definitive management of entropion is surgical although eye lubricants and tape (to pull the eyelid outwards) may be used whilst awaiting surgery.

How well did you know this?
1
Not at all
2
3
4
5
Perfectly
1510
Q

Theme: Visual defects

A.Right homonymous hemianopia

B.Left homonymous hemianopia

C.Right superior quadranopia

D.Left superior quadranopia

E.Right inferior quadranopia

F.Left inferior quadranopia

G.Upper bitemporal hemianopia

H.Lower bitemporal hemianopia

A 42-year-old woman is admitted to the vascular ward for an endarterectomy. Her CT report confirms a left temporal lobe infarct.

A 22-year-old man is referred to urology with possible urinary retention. He is passing huge amounts of urine. Post void bladder ultrasound is normal.

A 53-year-old man is admitted to the vascular ward for a carotid endarterectomy. His CT head report confirms a left parietal lobe infarct.

A

Right superior quadranopia

Lower bitemporal hemianopia

Right inferior quadranopia

How well did you know this?
1
Not at all
2
3
4
5
Perfectly
1511
Q

Conditions associated with anterior uveitits?

A

ankylosing spondylitis

reactive arthritis

ulcerative colitis, Crohn’s disease

Behcet’s disease

How well did you know this?
1
Not at all
2
3
4
5
Perfectly
1512
Q

Each one of the following is a cause of a mydriatic pupil, except:

Third nerve palsy

Atropine

Holmes-Adie pupil

Argyll-Robertson pupil

Traumatic iridoplegia

A

Argyll-Robertson pupil is one of the classic pupillary syndrome. It is sometimes seen in neurosyphilis and is often said to be the prostitute’s pupil - accommodates but doesn’t react. Another mnemonic used for the Argyll-Robertson Pupil (ARP) is Accommodation Reflex Present (ARP) but Pupillary Reflex Absent (PRA)

Features

small, irregular pupils

no response to light but there is a response to accommodate

Causes

diabetes mellitus

syphilis

How well did you know this?
1
Not at all
2
3
4
5
Perfectly
1513
Q

This man presents with unilateral visual loss. On examination he has a relative afferent pupillary defect. Fundoscopy shows the following:

What is the most likely diagnosis?

Central retinal vein occlusion

Subhyaloid haemorrhage

Solar retinopathy

Central retinal artery occlusion

Optic neuritis

A

Central retinal artery occlusion

The pale retina is the most obvious sign in this slide.

How well did you know this?
1
Not at all
2
3
4
5
Perfectly
1514
Q

Which one of the following best describes the action of latanoprost in the management of primary open-angle glaucoma?

Carbonic anhydrase inhibitor

Reduces aqueous production + increases outflow

Opens up drainage pores

Increases uveoscleral outflow

Reduces aqueous production

A

Increases uveoscleral outflow

How well did you know this?
1
Not at all
2
3
4
5
Perfectly
1515
Q

What are the acquired causes of optic atrophy?

A

MS

Longstanding papilloedema

Raised IOP

Retinal damage e.g. choroiditis, retinitis pigmentosa

Ischaemia

Toxins: tobacco ambylopia, quinine, methanol, arsenic, lead

Nutritional: vitamin B1, B2, B6 and B12 deficiency

How well did you know this?
1
Not at all
2
3
4
5
Perfectly
1516
Q

What are the congenital causes of optic atrophy?

A

Friedrich’s ataxia

Mitochondrial disorders

DIDMOAD (cranial DI, DM, OA, deafness- Wolfram’s)

How well did you know this?
1
Not at all
2
3
4
5
Perfectly
1517
Q

Which one of the following statements regarding the Holmes-Adie pupil is incorrect?

May be associated with absent ankle/knee reflexes

Bilateral in 80% of cases

It is a benign condition

Slowly reactive to accommodation but very poorly (if at all) to light

Causes a dilated pupil

A

The Holmes-Adie pupil is unilateral, rather than bilateral, in 80% of patients

How well did you know this?
1
Not at all
2
3
4
5
Perfectly
1518
Q

Theme: Primary open-angle glaucoma: management

A.Timolol

B.Pilocarpine

C.Brimonidine

D.Dorzolamide

E.Acetylcysteine

F.Ranibizumab

G.Atropine

H.Latanoprost

I.Cyclopentolate

J.Tropicamide

Both reduces aqueous production and increases outflow

First-line treatment in a patient with a history of heart block

Systemic absorption may cause sulphonamide-like reactions

A

Brimonidine

Latanoprost

Dorzolamide

How well did you know this?
1
Not at all
2
3
4
5
Perfectly
1519
Q

Causes of tunnel vision

A

papilloedema

glaucoma

retinitis pigmentosa

choroidoretinitis

optic atrophy secondary to tabes dorsalis

hysteria

How well did you know this?
1
Not at all
2
3
4
5
Perfectly
1520
Q

A 71-year-old man who has recently been diagnosed with macular degeneration asks for advice regarding antioxidant dietary supplements. Which one of the following may contraindicate the prescription of such supplements?

Current smoker

Pernicious anaemia

Treated hypertension

History of depression

Previous episodes of tendonitis

A

Beta-carotene has been found to increase the risk of lung cancer and hence antioxidant dietary supplements are not recommended for smokers.

How well did you know this?
1
Not at all
2
3
4
5
Perfectly
1521
Q

Theme: Visual field defects

A.Optic nerve

B.Optic chiasm

C.Retina

D.Occipital cortex

E.Optic tract

For each one of the following please select where the lesion is likely to be:

A 72-year-old man develops visual problems. He is noted to have a left homonymous hemianopia with some macula sparing.

A 54-year-old man complains of sweating, headaches and ‘tunnel vision’.

A 30-year-old man with a family history of early blindness is concerned that he is developing ‘tunnel vision’.

A

Occipital cortex

The macula sparing suggests the lesion is most likely to be in the occipital cortex rather than the optic tract.

Optic chiasm

This patient has a pituitary tumour causing compression of the optic chiasm and bitemporal hemianopia. The sweating and headaches are consistent with acromegaly.

Retina

This is a common presentation of retinitis pigmentosa. Extensive pigmentation would normally be noted on fundoscopy.

How well did you know this?
1
Not at all
2
3
4
5
Perfectly
1522
Q

Each one of the following predisposes to cataract formation, except:

Down’s syndrome

Hypercalcaemia

Diabetes mellitus

Long-term steroid use

Congenital rubella infection

A

Hypocalcaemia, rather than hypercalcaemia, predisposes to cataract formation.

How well did you know this?
1
Not at all
2
3
4
5
Perfectly
1523
Q

Causes of cataracts

Majority

Systemic

Ocular

A

Majority

age related

UV light

Systemic

diabetes mellitus

steroids

infection (congenital rubella)

metabolic (hypocalcaemia, galactosaemia)

myotonic dystrophy, Down’s syndrome

Ocular

trauma

uveitis

high myopia

topical steroids

How well did you know this?
1
Not at all
2
3
4
5
Perfectly
1524
Q

Causes of lens dislocation?

A

Marfan’s: upwards

Homocystinuria: downwards

Ehlers-Danlos

Trauma

Uveal tumours

Autosomal recessive ectopia lentis

How well did you know this?
1
Not at all
2
3
4
5
Perfectly
1525
Q

A 47-year-old female with a history of rheumatoid arthritis presents with a painful and red left eye. Visual acuity is normal. What is the most likely diagnosis?

Scleritis

Episcleritis

Glaucoma

Anterior uveitis

Keratoconjunctivitis sicca

A

A key way to discriminate between scleritis and episcleritis is the presence of pain. Keratoconjunctivitis sicca is usually bilateral and associated more with dryness, burning and itch

How well did you know this?
1
Not at all
2
3
4
5
Perfectly
1526
Q

A 62-year-old woman presents with sudden loss of vision in her left eye. Fundoscopy reveals the following:

What is the diagnosis?

Retinal detachment

Ischaemic optic neuropathy

Vitreous haemorrhage

Central retinal vein occlusion

Central retinal artery occlusion

A

Central retinal vein occlusion - sudden painless loss of vision, severe retinal haemorrhages on fundoscopy

This appearance is sometimes compared to a cheese and tomato pizza.

How well did you know this?
1
Not at all
2
3
4
5
Perfectly
1527
Q

Composition of bone matrix?

A

Organic= osteoid (40%)

Collagent Type 1

Resists tension, twisting and bending

Inroganice (60%)

Calcium hydroxapatite

Resits compressive forces

How well did you know this?
1
Not at all
2
3
4
5
Perfectly
1528
Q

How can bone be classified?

A

Woven bone: disorganised bone that forms the embryonic skeleton and fracture callus

Lamellar bone:

mature bone that can be of two types:

Cortical/compact= dense outer layer

Cancellous/trabecular: porous central layer

How well did you know this?
1
Not at all
2
3
4
5
Perfectly
1529
Q

Process of intramembranous ossification

A

Direct ossification of mesenchymal bone models formed during embryonic development

Skull bones, mandible and clavicle

How well did you know this?
1
Not at all
2
3
4
5
Perfectly
1530
Q

Process of endochondral ossification

A

Mesenchyme-> cartilage-> bone

Most bones ossify this way

How well did you know this?
1
Not at all
2
3
4
5
Perfectly
1531
Q

What are the phases of fracture healing?

A

Reactive phase

Reparative phase

Remodelling phase

How well did you know this?
1
Not at all
2
3
4
5
Perfectly
1532
Q

Features of reactive phase of bone healing

A

Injury-48h

Bleeding into #site-> haematoma

Inflammation-> cytokine, GF and vasoactive mediator release-> recruitment of leukocytes and fibroblasts-> granulation tissue

How well did you know this?
1
Not at all
2
3
4
5
Perfectly
1533
Q

Features of the reparative phase of bone healing

A

2d-2w

Proliferation of osteoblasts and fibroblasts-> cartilage and woven bone production-> callus formation

Consolidation-> endochondral ossificaiton of woven bone-> lamellar bone

How well did you know this?
1
Not at all
2
3
4
5
Perfectly
1534
Q

Features of the remodelling phase of bone healing

A

1w-7y

Remodelling of lamellar bone to cope with mechanical forces applied to it

Follows Wolff’s law

How well did you know this?
1
Not at all
2
3
4
5
Perfectly
1535
Q

What is Wolff’s Law

A

Form follows function

Wolff’s law, developed by the German anatomist and surgeon Julius Wolff (1836–1902) in the 19th century, states that bone in a healthy person or animal will adapt to the loads under which it is placed.[1] If loading on a particular bone increases, the bone will remodel itself over time to become stronger to resist that sort of loading.[2][3] The internal architecture of the trabeculae undergoes adaptive changes, followed by secondary changes to the external cortical portion of the bone,[4] perhaps becoming thicker as a result. The inverse is true as well: if the loading on a bone decreases, the bone will become less dense and weaker due to the lack of the stimulus required for continued remodeling.[5] This reduction in bone density (osteopenia) is known as stress shielding and can occur as a result of a hip replacement (or other prosthesis).[6] The normal stress on a bone is shielded from that bone by being placed on a prosthetic implant.

How well did you know this?
1
Not at all
2
3
4
5
Perfectly
1536
Q

Healing time for a closed, paediatric, metaphyseal, upper limb #

A

3w

How well did you know this?
1
Not at all
2
3
4
5
Perfectly
1537
Q

What are the complicating factors for #s and what do they do?

A

Double healing time

Adult

LL

Diaphyseal

Open

How well did you know this?
1
Not at all
2
3
4
5
Perfectly
1538
Q

Anatomy of a long bone proximal to distal

A

Diaphysis= shaft

Metaphysis: growth plate region

(physis- growth plate)

Epiphysis

Atrciular cartilage over joint surfaces

How well did you know this?
1
Not at all
2
3
4
5
Perfectly
1539
Q

Anatomy of a long bone

Deep to superficial

A

Medullary cavity

Endosteum

Cortical bone

Periosteum

How well did you know this?
1
Not at all
2
3
4
5
Perfectly
1540
Q

How can fractures be classified?

A

Traumatic

Stress

Pathological

How well did you know this?
1
Not at all
2
3
4
5
Perfectly
1541
Q

E.g. of traumatic #

A

Direct e.g. assualt with metal bar

Indirect e.g. FOOSH-> #clavicle

Avulsion

How well did you know this?
1
Not at all
2
3
4
5
Perfectly
1542
Q

Features of stress#

A

Bone fatigue due to repetitive strain

E.g. foot #s in marathon runners

How well did you know this?
1
Not at all
2
3
4
5
Perfectly
1543
Q

E.g. of pathological #s

A

Normal forces but disease bone

Local- tumours

General: osteoporosis, Cushing’s Pagets

How well did you know this?
1
Not at all
2
3
4
5
Perfectly
1544
Q

XR of #s

A

Radiographs must be orthogonal request AP and lat films

Need images of joint above and below #

How well did you know this?
1
Not at all
2
3
4
5
Perfectly
1545
Q

Structure for describing fracture

PAID

A

Demographics

Pattern

Anatomical

Intra/extra-articular

Deformity

Soft tissues

Specific #classification/type

How well did you know this?
1
Not at all
2
3
4
5
Perfectly
1546
Q

Components of pattern when describing #

A

Transverse

Oblique

Spiral

Multifragmentary

Crush

Greenstick

Avulsion

How well did you know this?
1
Not at all
2
3
4
5
Perfectly
1547
Q

Deformity when describing XR #

A

Translation

Angulation or tilt

Rotation

Impaction (shortening)

Describe distal relative to proximal

How well did you know this?
1
Not at all
2
3
4
5
Perfectly
1548
Q

Features of soft tissue when describing XR#

A

Open or closed

Neurovascular status

Compartment syndrome

How well did you know this?
1
Not at all
2
3
4
5
Perfectly
1549
Q

What are the 4Rs of fracture management

A

Resuscitation

Reduction

Restriction

Rehabilitation

How well did you know this?
1
Not at all
2
3
4
5
Perfectly
1550
Q

What are the principles of resuscitation in # management

A

Follow ATLS guidelines

Trauma series in 1o survey: C-spine, chest and pelvis

usually assessed in 2o survey

Assess neurovascular status and look for dislocations

Consider reduction and splinting before imaging when: reduced pain, reduced bleeding, risk of neurovascular imaging

XR once stable

How well did you know this?
1
Not at all
2
3
4
5
Perfectly
1551
Q

When should #reduction be considered before imaging?

A

To reduce pain

To reduce bleeding

To reduce risk of neurovascular injury

How well did you know this?
1
Not at all
2
3
4
5
Perfectly
1552
Q

What are the 6As of open #

A

Analgesia

Assess: NV status, soft tissues, photograph

Antisepsis: wound swab, copious irrigation, cover with betadine-soaked dressign

Alignment: align # and splint

Anti-tetanus: check status

Abx: fluxclox 500mg IV/IM and benpen 600mg IV/IM

Or augmentin 1.2g IV

How well did you know this?
1
Not at all
2
3
4
5
Perfectly
1553
Q

Mx of open #

A

Debridement and fixation in theatre

How well did you know this?
1
Not at all
2
3
4
5
Perfectly
1554
Q

What is the most dangerous complication of open# and why?

A

Clostridium perfrigens

Wound infections and gas gangrene +/- shock and renal failure

How well did you know this?
1
Not at all
2
3
4
5
Perfectly
1555
Q

Rx in c. perfringens infection of open#?

A

Debride, benpen, clindamycin

How well did you know this?
1
Not at all
2
3
4
5
Perfectly
1556
Q

What can be used to classify open #s?

A

Gustillo

How well did you know this?
1
Not at all
2
3
4
5
Perfectly
1557
Q

Components of Gustillo classification of open #s

A
  1. Wound <1cm in length
  2. Wound >1cm with minimal soft tissue damage
  3. Extensive soft tissue damage
How well did you know this?
1
Not at all
2
3
4
5
Perfectly
1558
Q

What are the principles for redcution of #s?

A

Displaced #s should be reduced unless no effect on outcome e.g. ribs

Aim for anatomical reduction, especially if articular surfaces involved

Alignment is more important than opposition

How well did you know this?
1
Not at all
2
3
4
5
Perfectly
1559
Q

What are the methods of # reduction?

A

Manipulation/closed reduction

Traction

Open reduction and internal fixation

How well did you know this?
1
Not at all
2
3
4
5
Perfectly
1560
Q

Features of manipulation/closed reduction

A

Under local, regional or general anaesthetic

Traction to disimpact

Manipulation to align

How well did you know this?
1
Not at all
2
3
4
5
Perfectly
1561
Q

Features of traction for # reduction

A

Not typically used now

Employed to overcome contraction of large muscles e.g. femoral #s

Skeletal traction vs skin traction

How well did you know this?
1
Not at all
2
3
4
5
Perfectly
1562
Q

Features of open reduction and internal fixation

A

Accurate reduction weighed up vs risks of surgery

Intra-articular #s

Open #s

2 #s on 1 limb

Failure in conservative mx

Bilateral identical fractures

How well did you know this?
1
Not at all
2
3
4
5
Perfectly
1563
Q

What are the principles of restriction in # management

A

Interfragmentary strain hypothesis dictates that tissue is formed at site dependent on strain it experiences

Fixation reduces strain -> bone formation

Fixation also reduces pain, increases stability and ability to function

How well did you know this?
1
Not at all
2
3
4
5
Perfectly
1564
Q

What are the methods of restriction in # mx

A

Non-rigid

Plaster

Functional bracing

Continuous traction

Ex-fix

Internal fixation

How well did you know this?
1
Not at all
2
3
4
5
Perfectly
1565
Q

What are some examples of non-rigid fixation?

A

Slings

Elastic supports

How well did you know this?
1
Not at all
2
3
4
5
Perfectly
1566
Q

What are some methods for plaster restriction of #s

What is an important consideration?

A

POP

In first 24-48h use back-slab or split cast due to risk of compartment syndrome

How well did you know this?
1
Not at all
2
3
4
5
Perfectly
1567
Q

What is functional bracing of #

A

Joints free to move but bone shafts supported in cast segments

How well did you know this?
1
Not at all
2
3
4
5
Perfectly
1568
Q

What is an example of continuous traction

A

Collar and cuff sling

How well did you know this?
1
Not at all
2
3
4
5
Perfectly
1569
Q

Features of external fixation

A

Fragments held in position by pins/wires which are then connected to an external frame

Intervention is away from field of injury

Useful in open #s, burns, tissue loss to allow wound access and reduce infection risk

However there is a risk of pin-site infections

How well did you know this?
1
Not at all
2
3
4
5
Perfectly
1570
Q

Features of internal fixation

A

Pins, plates, screws, IM nails

Usually perfect anatomical alignement

Increased stability

Aid early mobilisation

How well did you know this?
1
Not at all
2
3
4
5
Perfectly
1571
Q

What are the principles in # rehabilitation

A

Immobility-> reduced muscle and bone mass, joint stiffness

Need to maximise return to function to reduce later morbidity

How well did you know this?
1
Not at all
2
3
4
5
Perfectly
1572
Q

What are some methods of # rehabilitation

A

PT: exercises to improve mobility

OT: splints, mobility aids, home modification

Social services: meals on wheels, home help

How well did you know this?
1
Not at all
2
3
4
5
Perfectly
1573
Q

What are the general complications of fractures?

A

Tissue damage:

Haemorrhage and shock

Infection

Muscle damage-> rhabdomyolysis

Anaesthesia:

Anaphylaxis

Damage to teeth

Aspiration

Prolonged bed rest:

Chest infection, UTI

Pressure sores and muscle wasing

DVT, PE

Decreased bone mineral density

How well did you know this?
1
Not at all
2
3
4
5
Perfectly
1574
Q

What are the immediate complications of fractures?

A

Neurovascular damage

Visceral damage

How well did you know this?
1
Not at all
2
3
4
5
Perfectly
1575
Q

What are the early complications of fractures?

A

Compartment syndrome

Infection (worse if associated with metalwork)

Fat embolism-> ARDS

How well did you know this?
1
Not at all
2
3
4
5
Perfectly
1576
Q

What are the late complications of fractures?

A

Problems with union

AVN

Growth disturbance

Post-traumatic osteoarthritis

Complex regional pain syndromes

Myositis ossificans

How well did you know this?
1
Not at all
2
3
4
5
Perfectly
1577
Q

What is myositis ossificans

A

far most common type, nonhereditary myositis ossificans (commonly referred to simply as “myositis ossificans”, as in the remainder of this article), calcifications occur at the site of injured muscle, most commonly in the arms or in the quadriceps of the thighs.

The term myositis ossificans traumatica is sometimes used when the condition is due to trauma.[1][2] Also Myositis ossificans circumscripta is another synonym of myositis ossificans traumatica refers to the new extraosseous bone that appears after traum

How well did you know this?
1
Not at all
2
3
4
5
Perfectly
1578
Q

What is the second, rarer type of myositis ossificans?

A

The second condition, myositis ossificans progressiva (also referred to as fibrodysplasia ossificans progressiva) is an inherited affliction, autosomal dominant pattern, in which the ossification can occur without injury, and typically grows in a predictable pattern. Although this disorder can be passed to offspring by those afflicted with FOP, it is also classified as nonhereditary, as it is most often attributed to a spontaneous genetic mutation upon conception.

How well did you know this?
1
Not at all
2
3
4
5
Perfectly
1579
Q
A

Myositis ossificans

How well did you know this?
1
Not at all
2
3
4
5
Perfectly
1580
Q

What are the features of neurological complications following #

A

Severance is rare, stretching over bone edge commoner

Seddon classification describes three types of injury

How well did you know this?
1
Not at all
2
3
4
5
Perfectly
1581
Q

What are the components of the Seddon Classification

A

Descrbie neurological damage post #

Neuropraxia

Axonotmesis

Neurotmesis

How well did you know this?
1
Not at all
2
3
4
5
Perfectly
1582
Q

What is Neuropraxia

A

Temprorary interruption of conduction without loss of axonal continuity

How well did you know this?
1
Not at all
2
3
4
5
Perfectly
1583
Q

What is axonotmesis?

A

Disruption of nerve axon-> distal Wallerian degeneration

Connective tissue framework of nerve is preserved

Regeneration occurs and recovery is possible

How well did you know this?
1
Not at all
2
3
4
5
Perfectly
1584
Q

Wallerian degeneration

A

Wallerian degeneration is a process that results when a nerve fiber is cut or crushed, in which the part of the axon separated from the neuron’s cell body degenerates distal to the injury. This is also known as anterograde or orthograde degeneration.

How well did you know this?
1
Not at all
2
3
4
5
Perfectly
1585
Q

What is neurotmesis?

A

Disruption of the entire nerve fibre

Surgery is required and recovery is not usually complete

How well did you know this?
1
Not at all
2
3
4
5
Perfectly
1586
Q

Ant. shoulder dislocation
Humeral surgical neck

Nerve damage

Result?

A

Axillary nerve

Numb chevron

Weak abduction

How well did you know this?
1
Not at all
2
3
4
5
Perfectly
1587
Q

humeral shaft

Nerve affected

Clinical manifestation

A

Radial nerve

Waiter’s tip

How well did you know this?
1
Not at all
2
3
4
5
Perfectly
1588
Q

Elbow dislocation

Nerve affected

Clinical manifestation

A

Ulnar nerve

Claw hand

How well did you know this?
1
Not at all
2
3
4
5
Perfectly
1589
Q

Hip dislocation

Nerve affected

Clinical manifestation

A

Sciatic nerve

Foot drop

How well did you know this?
1
Not at all
2
3
4
5
Perfectly
1590
Q

neck of fibula

Knee dislocation

Nerve affected

Clinical manifestation

A

Fibular nerve

Foot drop

How well did you know this?
1
Not at all
2
3
4
5
Perfectly
1591
Q

Features of compartment syndrome

A

Osteofascial membranes divide limbs into separate comparments of muscles

Oedema following # can lead to an increase in compartment pressure, this reduces venous drainage increasing compartment pressure further

If compartment pressure > capillary pressure-> ischaemia

Muscle infarction->

Rhabdomyolsis and ATN

Fibrosis-> Volkman’s ischaemic contractrue

How well did you know this?
1
Not at all
2
3
4
5
Perfectly
1592
Q

Pain > clinical findings

Pain on passive muscle stretching

Warm erythematous, swollen limb

Prolonged CRT and weak/absent peripheral pulses

A

?Compartment syndrome

How well did you know this?
1
Not at all
2
3
4
5
Perfectly
1593
Q

Rx of compartment syndrome

A

Elevate limb

Remove all bandages and split/remove cast

Fasciotomy

How well did you know this?
1
Not at all
2
3
4
5
Perfectly
1594
Q

Def: delayed union

A

Union takes longer than expected

How well did you know this?
1
Not at all
2
3
4
5
Perfectly
1595
Q

Non-union of fracture is when

A

fails to unite

How well did you know this?
1
Not at all
2
3
4
5
Perfectly
1596
Q

What are the causative factors for problems with bone union?

5Is

A

Ischaemia: poor blood supply or AVN

Infection

Increased interfragmentary strain

Interposition of tissue between fragments

Intercurrent disease: e.g. malignancy or malnutrition

How well did you know this?
1
Not at all
2
3
4
5
Perfectly
1597
Q

How can non-union be classified?

A

Hypertrophic: bone end is rounded, dense and sclerotic

Atrophic: bone looks osteopenic

How well did you know this?
1
Not at all
2
3
4
5
Perfectly
1598
Q

Management of non-union

A

Optimise biology: infection, blood supply, bone graft, BMPs

Optimise mechanics: ORIF

How well did you know this?
1
Not at all
2
3
4
5
Perfectly
1599
Q

Def: malunion

A

healed in an imperfect position

poor position and or function

e.g. gunstock deformity

How well did you know this?
1
Not at all
2
3
4
5
Perfectly
1600
Q

Gunstock deformity

A

Cubitus varus (varus means a deformity of a limb in which part of it is deviated towards the midline of the body) is a common deformity in which the extended forearm is deviated towards midline of the body.

Cubitus varus is often referred to as “Gunstock deformity”, due to the crooked nature of the healing.

A common cause is the supracondylar fracture of humerus.

How well did you know this?
1
Not at all
2
3
4
5
Perfectly
1601
Q

What is Pelllegrini-Stieda disease

A

Form of myositis ossificans

Calcification of the superior attachment of MCL at the knee following traumatic injury

How well did you know this?
1
Not at all
2
3
4
5
Perfectly
1602
Q
A

Pellegrini-Stieda disease
􀁸 form of MO
􀁸 Calcification of the superior attachment of MCL @
the knee following traumatic injury.

How well did you know this?
1
Not at all
2
3
4
5
Perfectly
1603
Q

Def: Complex Regional Pain Syndrome Type 1

A

=Reflex Sympathetic Dystrophy, Sudek’s atrophy

Complex disorder of pain, sensory abnormalities, abnormal blood flow, sweating and trophic changes in superficial or deep tissues

No evidence of nerve injury

How well did you know this?
1
Not at all
2
3
4
5
Perfectly
1604
Q

Causes of Complex Regional Pain Syndrome Type 1

A

Injury: #s, carpal tunnel release, ops for Dupuytren’s

Zoster, MI, Idiopathic

How well did you know this?
1
Not at all
2
3
4
5
Perfectly
1605
Q

Weeks-Months after injury

Not traumatised area that is affected, affects a neighbouring area

Lancing pain, hyperalgesia or allodynia

Vasomotor: hot and sweaty or cold and cyanosed

Skin: swollen or atrophic and shiny

NM: weakness, hyper-reflexia, dystonia, contractures

A

Complex Regional Pain Syndrome Type 1

(Reflex Sympathetic Dystrophy, Sudek’’s atrophy)

How well did you know this?
1
Not at all
2
3
4
5
Perfectly
1606
Q

Allodynia

A

Pain caused by something that wouldn’t normally cause pain

How well did you know this?
1
Not at all
2
3
4
5
Perfectly
1607
Q

Rx of CRPS Type I

A

Usually self-limiting

Refer to pain team

Amitryptilline, gabapentin

Sympathetic nerve blocks can be tried

How well did you know this?
1
Not at all
2
3
4
5
Perfectly
1608
Q

What is CRPS type II?

A

Persistent pain following an injury caused by nerve lesions

How well did you know this?
1
Not at all
2
3
4
5
Perfectly
1609
Q

What can happen in children with fractures

A

Damage to the physis can result in abnormal bone growth

Salter-Harris classification categorises growth plate injuries

How well did you know this?
1
Not at all
2
3
4
5
Perfectly
1610
Q

What is the Salter-Harris Classification

SALT C

A

Used to classify growth plate injuries

  1. Straight across
  2. Above
  3. Lower
  4. Through
  5. Crush
How well did you know this?
1
Not at all
2
3
4
5
Perfectly
1611
Q

Px of SH1

A

e.g. SUFE

Normal growth with good reduction

How well did you know this?
1
Not at all
2
3
4
5
Perfectly
1612
Q

Px of SH4

A

Union across physis may interfere with bone growth

How well did you know this?
1
Not at all
2
3
4
5
Perfectly
1613
Q

Px of SH5

A

Crush-> physis injury-> growth arrest

How well did you know this?
1
Not at all
2
3
4
5
Perfectly
1614
Q
A
How well did you know this?
1
Not at all
2
3
4
5
Perfectly
1615
Q

What is the most common point of # clavicle

And mechanism?

A

Junction of the medial 2/3 and lateral 1/3

After the # the lateral end of the clavicle is displaced inferiorly be the weight of the arm and medially by pectoralis major

Medial end is pulled superiorly by sternocleidomastoid

FOOSH

How well did you know this?
1
Not at all
2
3
4
5
Perfectly
1616
Q

Winging of the scapula caused by?

A

Serratus anterior muscle originates from ribs 2-8 and attaches to the costal face of the scapula pulling it against the ribcage

This is innervated by the LTN

If the LTN becomes damaged the scapula protrudes out of the back when pushing with the arm.

How well did you know this?
1
Not at all
2
3
4
5
Perfectly
1617
Q

What rotator cuff muscles attach to the greater tubercle?

A

Greater tubercle is located laterally on the humerus

Serves as attachment site for 3 of the rotator cuff muscles, supraspinatus, infraspinatus, teres minor.

How well did you know this?
1
Not at all
2
3
4
5
Perfectly
1618
Q

What rotator cuff muscle is attached to the lesser tubercle of the humerus

A

Subscapularis

How well did you know this?
1
Not at all
2
3
4
5
Perfectly
1619
Q

Action of supraspinatus

A

Initiator of abduction up to first 25

How well did you know this?
1
Not at all
2
3
4
5
Perfectly
1620
Q

What is the action of infraspinatus and teres minor

A

External rotators of the shoulder

Infraspinatus acting when the arm is in neutral and teres minor when the arm is in 90% of abduction

How well did you know this?
1
Not at all
2
3
4
5
Perfectly
1621
Q

What is the action of subscapularis

A

Main internal rotator of the shoulder

Largest and strongest cuff muslce

How well did you know this?
1
Not at all
2
3
4
5
Perfectly
1622
Q

Actions of rotator cuff muscles

A

SITS

Supraspinatus: abduction

Infraspinatus: external rotation

Teres minor: external rotation (when arm is abducted to 90 deg)

Subscapularis: internal rotation

How well did you know this?
1
Not at all
2
3
4
5
Perfectly
1623
Q

Articulation of the distal humerus with ulna and radius

A

Trochlea articulates with ulnar: located medially and extends onto the posterior of the bone

Capitulum articulates with the radius

How well did you know this?
1
Not at all
2
3
4
5
Perfectly
1624
Q

Monteggia’s fracture

A

Usually caused by a force from behind the ulna

Proximal shaft of the ulna is fractured and the head of the radius dislocates anteriorly at the elbow

How well did you know this?
1
Not at all
2
3
4
5
Perfectly
1625
Q
A

Monteggia fracture

The Monteggia fracture is a fracture of the proximal third of the ulna with dislocation of the head of the radius. It is named after Giovanni Battista Monteggia.[1][2]

Fall on an outstretched hand with the forearm in excessive pronation (hyper-pronation injury). The Ulna fractures in the proximal one-third of the shaft due to extreme dislocation. Depending on the impact and forces applied in each direction, degree of energy absorption determines pattern, involvement of the radial head and whether or not open soft tissue occurs.

Direct blow on back of upper forearm would be a very uncommon cause. In this context, isolated ulnar shaft fractures are most commonly seen in defence against blunt trauma (e.g. nightstick injury). Such an isolated ulnar shaft fracture is not a Monteggia fracture.[citation needed] It is called a ‘nightstick fracture’.

How well did you know this?
1
Not at all
2
3
4
5
Perfectly
1626
Q
A

Galeazzi’s Fracture – A fracture to the distal radius, with the ulna head dislocating at the distal radio-ulnar joint.

How well did you know this?
1
Not at all
2
3
4
5
Perfectly
1627
Q

Colle’s fracture

A

Most common types of radial fracture

Fall onto an outstretched hand causing a fracture of the distal radius.

Structures distal to the fracture (wrist and hand are displaced posteriorly to produce the dinner fork deformity)

How well did you know this?
1
Not at all
2
3
4
5
Perfectly
1628
Q
A

Most common type of radial fracture

FOOSH causing a fracture of the distal radius.

Structures distal to the fracture (wrist and hand are displaced posteriorly to produce the dinner fork deformity)

How well did you know this?
1
Not at all
2
3
4
5
Perfectly
1629
Q

Smith’s fracture

A

Smith’s Fracture – A fracture caused by falling onto the back of the hand. It is the opposite of a Colles’ fracture, as the distal fragment is now placed anteriorly.

How well did you know this?
1
Not at all
2
3
4
5
Perfectly
1630
Q
A

Smith’s Fracture – A fracture caused by falling onto the back of the hand. It is the opposite of a Colles’ fracture, as the distal fragment is now placed anteriorly.

How well did you know this?
1
Not at all
2
3
4
5
Perfectly
1631
Q
A
How well did you know this?
1
Not at all
2
3
4
5
Perfectly
1632
Q

What are the carpal bones

SLTP

TTCH

A

Scaphoid

Lunate

Triquetrum

Pisiform

Trapezium

Trapezoid

Capitate

Hamate

How well did you know this?
1
Not at all
2
3
4
5
Perfectly
1633
Q

Which carpal bones are most commonly fractured?

A

Scaphoid and lunate

FOOSH

How well did you know this?
1
Not at all
2
3
4
5
Perfectly
1634
Q

Features of scaphoid fracture

A

Classical clinical features is pain and tenderness in the anatomical snuffbox

Fracture needs to be reduced quickly as the blood supply to the proximal part of the bone can be cut off leading to an avascular necrosis

How well did you know this?
1
Not at all
2
3
4
5
Perfectly
1635
Q

Features of a lunate fracture

A

Occurs when there is hyperextension at the wrist

Associated with damage to the median nerve

How well did you know this?
1
Not at all
2
3
4
5
Perfectly
1636
Q

What is Boxer’s fracture

A

A fracture of the 5th metacarpal neck

Usually caused by a clenched fist striking a hard object

The distal part of the fracture is displaced posteriorly producing shortening of the affected finger

How well did you know this?
1
Not at all
2
3
4
5
Perfectly
1637
Q
A

Boxer’s fracture

How well did you know this?
1
Not at all
2
3
4
5
Perfectly
1638
Q

Bennet’s fracture

A

Fracture of the 1st metacarpal base extending into the carpometacarpal joint

It is caused by hyperabduction of the thumb

How well did you know this?
1
Not at all
2
3
4
5
Perfectly
1639
Q
A

Bennet’s fracture

How well did you know this?
1
Not at all
2
3
4
5
Perfectly
1640
Q

Innervation of the rotator cuff muscles

A

Supraspinatus and infraspinatus both innervated by suprascapular

Teres minor innervated by axillary nerve

Subscapularis upper and lower subscapular nerve

How well did you know this?
1
Not at all
2
3
4
5
Perfectly
1641
Q
A
How well did you know this?
1
Not at all
2
3
4
5
Perfectly
1642
Q

Abduction of the upper limb

A

Abduction (upper limb away from midline in coronal plane)
The first 0-15 degrees of abduction is produced by the supraspinatus. The middle fibres of the deltoid are responsible for the next 15-90 degrees. Past 90 degrees, the scapula needs to be rotated to achieve abduction – that is carried out by the trapezius and serratus anterior.

How well did you know this?
1
Not at all
2
3
4
5
Perfectly
1643
Q
A

Acromioclavicular joint dislocation

How well did you know this?
1
Not at all
2
3
4
5
Perfectly
1644
Q

What is the difference between tennis elbow and golfer’s elbow?

A

Most of the flexor and extensor muscles in the forearm have a common tendinous origin. The flexor muscles originate from the medial epicondyle, and the extensor muscles from the lateral. Sportspersons can develop an overuse strain of the common tendon – which results in pain and inflammation around the area of the affected epicondyle.

Typically, tennis players experience pain in the lateral epicondyle from the common extensor origin. Golfers experience pain in the medial epicondyle from the common flexor origin.

How well did you know this?
1
Not at all
2
3
4
5
Perfectly
1645
Q

Which carpal bones articulate with the radius and ulna?

A

Scaphoid, lunate, triqeutrum

How well did you know this?
1
Not at all
2
3
4
5
Perfectly
1646
Q
A
How well did you know this?
1
Not at all
2
3
4
5
Perfectly
1647
Q
A
How well did you know this?
1
Not at all
2
3
4
5
Perfectly
1648
Q
A
How well did you know this?
1
Not at all
2
3
4
5
Perfectly
1649
Q
A
How well did you know this?
1
Not at all
2
3
4
5
Perfectly
1650
Q

Epidemiology of hip fracture

A

80/100,000

50% >80 y

F>M 3:1

How well did you know this?
1
Not at all
2
3
4
5
Perfectly
1651
Q

Pathophysiology of hip #

A

Old= osteoporosis with minor trauma e.g. fall

Young= major trauma

How well did you know this?
1
Not at all
2
3
4
5
Perfectly
1652
Q

What are the risk factors for osteoporosis?

Age

+

Shattered

A

Age

Steroids

Hyper para/thyroidism

Alcohol and cigarettes

Thin (BMI <22)

Testosterone low

Early menopause

Renal/ liver failure

Ersoive/inflammatory bone disease e.g. RA, myeloma

Dietary Ca low/malabsorption DM

How well did you know this?
1
Not at all
2
3
4
5
Perfectly
1653
Q

Key questions in hip#?

A

Mechanism

RFs for osteoporosis/pathological #

Premorbid mobility

Premorbid independence

Comorbidities

MMSE

How well did you know this?
1
Not at all
2
3
4
5
Perfectly
1654
Q

Hip # O/E

A

Shortened and externally rotated

How well did you know this?
1
Not at all
2
3
4
5
Perfectly
1655
Q

Initial management of hip #

A

Resuscitate: dehydration, hypothermia

Analgesia: M + M

Assess neurovascular status of limb

Imaging: AP and lateral films

Prep for theatre

How well did you know this?
1
Not at all
2
3
4
5
Perfectly
1656
Q

Orthopaedic prep for theatre

ABCDEFG

A

Anaesthetist and book theatre

Bloods: FBC, U+Es, clotting, X-match (2u)

CXR

DVT prophylaxis: TEDS, LMWH

ECG

Films: orthogonal XR

Get consent

How well did you know this?
1
Not at all
2
3
4
5
Perfectly
1657
Q

Imaging in hip XR

A

Ask for AP and lateral film

Look at Shenton’s lines

Intra or extracapsular

Displaced or non-displaced

Osteopenic?

How well did you know this?
1
Not at all
2
3
4
5
Perfectly
1658
Q

What is Shenton’s line?

A

Shenton’s line is formed by the medial edge of the femoral neck and the inferior edge of the superior pubic ramus

Loss of contour of Shenton’s line is a sign of a fractured neck of femur

IMPORTANT NOTE: Fractures of the femoral neck do not always cause loss of Shenton’s line

How well did you know this?
1
Not at all
2
3
4
5
Perfectly
1659
Q

What is the extent of the joint capsule of the hip?

A

Capsule envelopes the femoral head and neck

Subcapital, transcervical and basicervical fractures are intracapsular hip fractures

Intertrochanteric and subtrochanteric fractures do not involve the neck of the femur

How well did you know this?
1
Not at all
2
3
4
5
Perfectly
1660
Q
A
How well did you know this?
1
Not at all
2
3
4
5
Perfectly
1661
Q
A
How well did you know this?
1
Not at all
2
3
4
5
Perfectly
1662
Q

What is the blood supply of the femoral head

A

Most of the bloody suppply to the femoral head ariseses from the lateral femoral circumflex artery which gives rise to the retinacular vessels. These run posteriorly until they reach the cartilaginous border of the femoral head.

The obturator artery gives rise to the vessels within the ligemantum teres (artery of ligamentum teres)

An ascending branch of the medial femoral circumflex artery supplies the greater trochanter and anastomoses with the lateral femoral circumflex artery

How well did you know this?
1
Not at all
2
3
4
5
Perfectly
1663
Q

Key anatomy of the hip joint

A

Capsular attaches proximally to the acetabular margin and distally to the intertrochanteric line

If retinacular vessels are damaged theres is a risk of AVN of the femoral head -> pain, stiffness and OA

How well did you know this?
1
Not at all
2
3
4
5
Perfectly
1664
Q

How can hip fractures be classified

A

Intracapsular:

Subcapital, transcervical, basicervical

Extracapsular:

Intertrochanteric, subtrochanteric

How well did you know this?
1
Not at all
2
3
4
5
Perfectly
1665
Q

What can be used to classify intracapsular hip #s?

A

Garden Classification

How well did you know this?
1
Not at all
2
3
4
5
Perfectly
1666
Q

Components of the Garden Classification of Intracapsular fractures?

A
  1. Incomplete # undisplaced
  2. Complete fracture, undisplaced
  3. Complete fracture, partially dispalced
  4. Complete fracture, completely dispalced
How well did you know this?
1
Not at all
2
3
4
5
Perfectly
1667
Q

Surgical management of intracapsular hip #

A

Depends on Garden classification

1,2. ORIF with cancellous screws

3,4.

<55: ORIF with screws, FU in OPD and do arthroplasty if AVN develops (30%)

55-75: THR

>75: hemiarthroplasty

Mobilises: cemented Thompson’s

Non-mobiliser: uncemented Austin Moore

How well did you know this?
1
Not at all
2
3
4
5
Perfectly
1668
Q

Surgical management of extracapsular hip fracture

A

ORIF with DHS

How well did you know this?
1
Not at all
2
3
4
5
Perfectly
1669
Q

Discharge of patients with hip #

A

Involve OT and physios

Discharge when mobilisation and social circumstances permit

How well did you know this?
1
Not at all
2
3
4
5
Perfectly
1670
Q

What are the specific complications of hip #

A

AVN of femoral head in displaced #s (30%)

Non/malunion

Infection

OA

How well did you know this?
1
Not at all
2
3
4
5
Perfectly
1671
Q

Px of hip #

A

30% mortality at 1y

50% never regain pre-morbid functioning

>10% unable to return to premorbid residence

Majority will have some residual pain or disability

How well did you know this?
1
Not at all
2
3
4
5
Perfectly
1672
Q

Features of Colles fracture

A

FOOSH

Most common elderly females with osteoporosis

Dinner fork deformity

How well did you know this?
1
Not at all
2
3
4
5
Perfectly
1673
Q

Extra-articular # of distal radius within 1.5” of joint

Dorsal displacement of distal fragment

Dorsal angulation of distal fragment (normally 11 degrees volar tilt)

Reduced radial height (norm= 11m)

Reduced radial inclination (norm= 22 deg)

+/- avulsion of ulna styloid. +/- impaction

A

Colles fracture

How well did you know this?
1
Not at all
2
3
4
5
Perfectly
1674
Q

Management of Colles

A

Examine for neurovascular injury as median nerve and radial artery lie close

If there is much displacement: reduction

Under haematoma block, IV regional anaesthesia or GA

Disimpact and correct angulation

Position: ulnar deviation + some wrist flexion

Apply dorsal backslab: provide 3- point pressure

Re-XR: satisfactory position

No: ortho review and consider MUA +/- K wires

Yes: home with clinic follow up witihin 48h for completion of POP

6w in POP + physio

If comminuted: intra-articular or redisplaces: surgical fixation with ex-fix, Kirschner-wries or ORIF and plates

How well did you know this?
1
Not at all
2
3
4
5
Perfectly
1675
Q

What are the specific complications of Colles fractures?

A

Median nerve injury

Frozen shoulder/adhesive capsulitis

Tendon rupture especially EPL

Carpal tunnel syndrome

Mal/nonunion

Sudek’s atrophy/CRPS

How well did you know this?
1
Not at all
2
3
4
5
Perfectly
1676
Q

Fall onto back of flexed wrist

Fracture of distal radius with volar (palmar) displacement and angulation of distal fragment

Reduce to restore anatomy and POP for 6w

A

Smith’s/Reverse Colle’s

How well did you know this?
1
Not at all
2
3
4
5
Perfectly
1677
Q

Oblique intra-articular fracture involving the dorsal aspect of the distal radius and dislocation of the radiocarpal joint

A

Barton’s fracture

How well did you know this?
1
Not at all
2
3
4
5
Perfectly
1678
Q
A

Barton fractures are fractures of the distal radius. It is also sometimes termed the dorsal type Barton fracture to distinguish it from the volar type or reverse Barton fracture.

Barton fractures extend through the dorsal aspect to the articular surface but not to the volar aspect. Therefore, it is similar to a Colles fracture. There is usually associated dorsal subluxation/dislocation of the radiocarpal joint.

How well did you know this?
1
Not at all
2
3
4
5
Perfectly
1679
Q

FOOSH

Pain in anatomical snuffbox

Pain on telescoping the thumb

A

Scaphoid fracture

How well did you know this?
1
Not at all
2
3
4
5
Perfectly
1680
Q

Specific management of scaphoid fracture

A

Request scaphoid XR view

If clinical hx and exam suggest a scaphoid fracture it should be initially treated even if the XR is normal as # may take 10d to become apparent

Place wrist in scaphoid blaster (beer glass position)

If initial XR is negative F/U in clinic in 10d for repeat XR

Fracture visible-> plaster for 6 w

No visible fracture but clinically tender-> 2w

No visible fracture not tender, no plaster

How well did you know this?
1
Not at all
2
3
4
5
Perfectly
1681
Q

Fracture of proximal 3rd of ulna shaft

Anterior dislocation of radial head at the capitulum

May lead to palsy of deep branch of radial nerve- weak finger extension but no senosry loss

A

Monteggia

How well did you know this?
1
Not at all
2
3
4
5
Perfectly
1682
Q

Fractutre of radial shaft between mid and distal 3rds

Dislocation of distal radio-ulna joint

A

Galleazi fracture

How well did you know this?
1
Not at all
2
3
4
5
Perfectly
1683
Q

Management of radial and ulna shaft fractures

A

Unstable: adults: ORIF, children: MUA and above elbow plaster

Fractures of forearm should be plastered in most stable position

Proximal: supination

Distal: pronation

Mid-shaft: neutral

How well did you know this?
1
Not at all
2
3
4
5
Perfectly
1684
Q

How can shoulder dislocation be classified?

A

Anterior

Posterior

How well did you know this?
1
Not at all
2
3
4
5
Perfectly
1685
Q

Features of anterior shoulder dislocation

A

95% of shoulder dislocations

Direct trauma or falling on hand

Humeral head dislocates antero-inferiorly

How well did you know this?
1
Not at all
2
3
4
5
Perfectly
1686
Q

Features of posterior shoulder dislocation

A

Caused by direct trauma or muscle contraction (electrocution, epileptics)

How well did you know this?
1
Not at all
2
3
4
5
Perfectly
1687
Q
A

Anterior shoulder dislocation

How well did you know this?
1
Not at all
2
3
4
5
Perfectly
1688
Q
A

Posterior dislocation

Lightbulb sign: internally rotated humeral head takes on a rounded appearance

How well did you know this?
1
Not at all
2
3
4
5
Perfectly
1689
Q

What are the lesions associated with shoulder dislocation?

A

Bankart lesion

Hill-Sachs lesion

How well did you know this?
1
Not at all
2
3
4
5
Perfectly
1690
Q

What is a Bankart lesion

A

Damage to anteroinferior glenoid labrum

How well did you know this?
1
Not at all
2
3
4
5
Perfectly
1691
Q

What is Hill-Sachs lesion

A

Cortical depression in the posteriolateral part of the humeral head following impaction against the glenoid rim during anterior dislocaiton

Occurs in 35-40% of anterior dislocaiton

How well did you know this?
1
Not at all
2
3
4
5
Perfectly
1692
Q
A

Hill Sachs

How well did you know this?
1
Not at all
2
3
4
5
Perfectly
1693
Q
A

Bankart Lesion

How well did you know this?
1
Not at all
2
3
4
5
Perfectly
1694
Q

Shoulder contour loss: appears square

Bulge in infraclavicular fossa

Arm spported in opposite hand

Severe pain

A

?Shoulder dislocation

How well did you know this?
1
Not at all
2
3
4
5
Perfectly
1695
Q

Specific management of shoulder dislocation

A

Assess for neurovascular deficit: especially axillary nerve- sensation over chevron area before and after reduction

Occurs in 5%

XR: AP and transcapular view

Reduction under sedation

Rest arm in a sling for 3-4w

Physio

How well did you know this?
1
Not at all
2
3
4
5
Perfectly
1696
Q

What are the methods for reduciton of shoulder dislocation?

A

Hippocratic

Kocher’s

How well did you know this?
1
Not at all
2
3
4
5
Perfectly
1697
Q

Features of hippocratic shoulder reduction

A

Longitudinal traction with arm in 30 degree abduction and counter traction at the axilla

How well did you know this?
1
Not at all
2
3
4
5
Perfectly
1698
Q

Features of Kocher’s reduction of dislocation

A

External rotation of adducted arm, anterior movement, internal rotation

How well did you know this?
1
Not at all
2
3
4
5
Perfectly
1699
Q

Complications of shoulder dislocation

A

Recurrent dislocation

90% of patietns <20y with traumatic dislocation

Axillary nerve injury

How well did you know this?
1
Not at all
2
3
4
5
Perfectly
1700
Q

TUBS Recurrent shoulder instability

A

Traumatic Unilateral dislocations with a Bankart lesion often require Surgery

Mostly young patients: 15-30y

Surgery involves a Bankart repair

How well did you know this?
1
Not at all
2
3
4
5
Perfectly
1701
Q

AMBRI Recurrent shoulder instability

A

Atraumatic Multidirectional Bilateral shoulder dislocation is treated with Rehabilitation but may require Inferior capsular shift

How well did you know this?
1
Not at all
2
3
4
5
Perfectly
1702
Q

Pathology of impingement syndrome/painful arc

A

Entrapment of supraspinatus tendon and subacromial bursa between acromion and greater tuberosity of humerus

-> subacromial bursitis and or supraspinatus tendonitis

How well did you know this?
1
Not at all
2
3
4
5
Perfectly
1703
Q

Painful arc: 60-120 degrees

Weakness and reduced ROM

+ve Hawkins test

A

Impingement syndrome/ painful arc

How well did you know this?
1
Not at all
2
3
4
5
Perfectly
1704
Q

Hawkins test

A

The patient is examined while sitting with their shoulder flexed to 90° and their elbow flexed to 90°. The examiner grasps and supports proximal to the wrist and elbow to ensure maximal relaxation, the examiner and the patient then quickly rotate the arm internally.[4][5] Pain located below the acromioclavicular joint with internal rotation is considered a positive test result

How well did you know this?
1
Not at all
2
3
4
5
Perfectly
1705
Q

Ix in impingement sydnrome

A

Plain radiographs may see bony spurs

US

MRI arthrogram

How well did you know this?
1
Not at all
2
3
4
5
Perfectly
1706
Q

Rx of impingement syndrome

A

Conservative: rest, PT

Medical: NSAIDs, subacromial bursa steroid +/- LA injeciton

Surgical: arthoroscopic acromioplasty

How well did you know this?
1
Not at all
2
3
4
5
Perfectly
1707
Q

DDx of painful arc

A

Impingement

Supraspinatus tear or partial tear

AC joint OA

How well did you know this?
1
Not at all
2
3
4
5
Perfectly
1708
Q

Progressive reduction in active and passive ROM

EXTERNAL ROTATION<30 deg

Abduction <90

Shoulder pain especially at night (can’t lie on affected side)

A

Frozen shoulder: adhesive capsulitis

How well did you know this?
1
Not at all
2
3
4
5
Perfectly
1709
Q

Stages of frozen shoulder

A

Freezing

Frozen

Unfreezing

How well did you know this?
1
Not at all
2
3
4
5
Perfectly
1710
Q

With what is frozen shoulder associated?

A

DM

How well did you know this?
1
Not at all
2
3
4
5
Perfectly
1711
Q

Mx of frozen shoulder

A

Conservative: rest, PT

Medical: NSAIDs, subacromial bursa steroid +/- LA injection

How well did you know this?
1
Not at all
2
3
4
5
Perfectly
1712
Q

Features of partial rotator cuff tear

A

Painful arc

How well did you know this?
1
Not at all
2
3
4
5
Perfectly
1713
Q

What is rotator cuff tear secondary to?

A

Degeneration, a sudden jolt or fall

1714
Q

Features of a complete rotator cuff tear

A

Shoulder tip pain

Full range of passive movement

Inability to abduct the arm

Active abduction possible following passive abduction to 90 degrees

Lowering the arm beneath this -> sudden drop: drop-arm sign

1715
Q

Drop arm test

A

A positive test is determined by the patient’s inability to smoothly control the lowering of their arm or the inability to hold the arm in 90 degrees of abduction. In a positive test that starts above 90 degrees of abduction, the patient will tend to have difficulty controlling the movement around 90 degrees of abduction. There may or may not be pain reported. Pain alone is not a positive test.

1716
Q

Mx of rotator cuff tear

A

Open or arthroscopic repair

1717
Q

Common in children after FOOSH

Elbow very swollen and held semi-flexed

Sharp edge of proximal humerus may injury brachial artery which lies anteriorly to it

A

Supracondylar fracture of the humerus

1718
Q

How can supracondylar fractures of the humerus be classified?

A

Extension: commonest type

Flexion

1719
Q

How can extension type supracondylar fractures of the humerus be further classified?

A

Gartland

Type 1: non-displaced

Type 2: angulated with intact posterior cortex

Type 3: displaced with no cortical contact

1720
Q
A

Supracondylar fracture (extension type)

1721
Q
A

Supracondylar fracture flexion type

1722
Q
A
1723
Q

Specific management of supracondylar fractures of the humerus

A

Ensure there is no neurovascular damage: if radial pulse absent or damage to brachial artery suspected, take urgently to theatre for reduciton +/- on table angiogram. Median nerve is also vulnerable

Restore the anatomy: no displacement-> flex the arm as fully as possible and apply a collar and cuff for 3w. Triceps acts as sling to stabilise fragments.

Displacement: MUA + fixation with K wires + collar and cuff with arm flexed for 3w

1724
Q

What are the specific complications to supracondylar fractures of the humerus?

A

Neurovascular injury

Compartment syndrome

Gunstock deformity

1725
Q

Features of neurovascular injury following supracondylar humeral fracture

A

Brachial artery

Radial nerve

Median nerve

All vulnerable to injury: esp anterior ossesus branch of the median nerve which supplies the deep forearm flexors (FPL, lateral half of FDP and pronator quadratus)

1726
Q

Features of compartment syndrome in Supracondylar fractures of humerus

A

Monitor closely during the first 24h

Pain on passive extension of the fingers (stretches flexor compartment) is early sign

Mx: try extension of the elbow though surgical Rx may be needed

Volkmann’s ischaemic contracture can result-> fibrosis of flexors-> claw hand

1727
Q

Features of Gunstock deformity following supracondylar humeral fracture

A

Valgus, varus and rotational deformities in the coronal plane do not remodel and can lead to cubitus varus deformity aka Gunstock deformity

1728
Q

A 19-year-old presents to his general practitioner (GP) with an asymptomatic right sided neck lump that he noticed 4 weeks previously. On examination the lump is soft and mobile, is within the dermal layer of the skin and on transillumination the area is equal to the surrounding tissue. There is no central punctum. What is the most likely diagnosis?

Branchial cleft cyst

Epidermoid cyst

Lipoma

Cystic hygroma

Deep cervical abscess

A

The correct answer is a lipoma.

Epidermoid cysts are common cutaneous cysts that result from the proliferation of epidermal cells within a circumscribed space of the dermis. They can occur at any age and are typically asymptomatic. On physical examination they are typically firm, round nodules of various sizes and a central punctum may be present.

Cystic hygromas are congenital lymphatic lesions that are typically found in the neck and axilla region with a predilection for the left side. The majority present by 2 years of age. On physical examination, cystic hygromas are often soft, painless and transilluminate brightly.

Branchial cleft cysts are congenital epithelial cysts that develop due to failure of obliteration of the second branchial cleft in embryonic development. They typically present by early adulthood. On physical examination, they are typically smooth, painless, and do not transilluminate.

Deep cervical abscesses typically occur following a source of infection such as recent dental work, upper respiratory tract infections, neck or oral cavity trauma. They can occur at any age. On physical examination, the patient typically presents with a painful and tender swelling that may be hot to touch. The patient may also have other signs of infection such as fever, chills, aches, and pains.

1729
Q

By far the most common cause of neck swellings. There may be a history of local infection or a generalised viral illness

A

Reactive lymphadenopathy

1730
Q

Rubbery, painless lymphadenopathy
The phenomenon of pain whilst drinking alcohol is very uncommon
There may be associated night sweats and splenomegaly

A

Lymphoma

1731
Q

May be hypo-, eu- or hyperthyroid symptomatically
Moves upwards on swallowing

A

Thyroid swelling

1732
Q

More common in patients < 20 years old
Usually midline, between the isthmus of the thyroid and the hyoid bone
Moves upwards with protrusion of the tongue
May be painful if infected

A

Thyroglossal cyst

1733
Q

More common in older men
Represents a posteromedial herniation between thyropharyngeus and cricopharyngeus muscles
Usually not seen but if large then a midline lump in the neck that gurgles on palpation
Typical symptoms are dysphagia, regurgitation, aspiration and chronic cough

A

Pharyngeal pouch

1734
Q

A congenital lymphatic lesion (lymphangioma) typically found in the neck, classically on the left side
Most are evident at birth, around 90% present before 2 years of age

A

Cystic hygroma

1735
Q

An oval, mobile cystic mass that develops between the sternocleidomastoid muscle and the pharynx
Develop due to failure of obliteration of the second branchial cleft in embryonic development
Usually present in early adulthood

A

Branchial cyst

1736
Q

More common in adult females
Around 10% develop thoracic outlet syndrome

A

Cervical rib

1737
Q

Pulsatile lateral neck mass which doesn’t move on swallowing

A

Carotid aneurysm

1738
Q

An 18 year old student presents to your GP practice after presenting to you a week ago with a supraclavicular neck lump. The last consultation reads that she had also been feeling unwell with night sweats and loss of appetite. On examination the neck lump was rubbery and non-tender.

A biopsy has been taken and she has come for the results. The report shows cells with a ‘mirror image nuceli’ .

What is the most likely diagnosis?

Non-Hodgkin’s lymphoma

Hodgkin’s lymphoma

Epstein- Barr virus

Thyroid adenoma

Tuberculosis

A

Mirror image nuclei = Reed-Sternberg cells

Reed-Sternberg cells are diagnostic of Hodgkin’s lymphoma.
Patient’s present with large, non-tender, rubbery lymph nodes. 25% will have constitutional upset with fever, night sweats, weight loss, lethargy and pruritus.

An exam favourite, but lesser seen symptom is alcohol induced lymph node pain.

1739
Q

lymphadenopathy (75%) - painless, non-tender, asymmetrical

systemic (25%): weight loss, pruritus, night sweats, fever (Pel-Ebstein)

alcohol pain in HL

normocytic anaemia, eosinophilia

LDH raised

A

Hodgkin’s lymphoma is a malignant proliferation of lymphocytes characterised by the presence of the Reed-Sternberg cell. It has a bimodal age distributions being most common in the third and seventh decades

1740
Q

A 23-year-old man is diagnosed as having nasal polyps. Sensitivity to which medication is associated with this condition?

Sulfa drugs

ACE inhibitors

Penicillins

Paracetamol

Aspirin

A

Around in 1% of adults in the UK have nasal polyps. They are around 2-4 times more common in men and are not commonly seen in children or the elderly.

Associations

asthma* (particularly late-onset asthma)

aspirin sensitivity*

infective sinusitis

cystic fibrosis

Kartagener’s syndrome

Churg-Strauss syndrome

Features

nasal obstruction

rhinorrhoea, sneezing

poor sense of taste and smell

Unusual features which always require further investigation include unilateral symptoms or bleeding.

Management

all patients with suspected nasal polyps should be referred to ENT for a full examination

topical corticosteroids shrink polyp size in around 80% of patients

1741
Q

What is Samter’s triad?

A

The association of asthma, aspirin sensitivity and nasal polyposis

1742
Q

A 27-year-old woman complains of recurrent ear discharge. Otoscopy is as follows:

What is the most likely diagnosis?

Otitis externa

Chronic suppurative otitis media

Mastoiditis

Cholesteatoma

Acute otitis media

A

A cholesteatoma consists of squamous epithelium that is ‘trapped’ within the skull base causing local destruction. It is most common in patients aged 10-20 years.

Main features

foul smelling discharge

hearing loss

Other features are determined by local invasion:

vertigo

facial nerve palsy

cerebellopontine angle syndrome

Otoscopy

‘attic crust’ - seen in the uppermost part of the ear drum

Management

patients are referred to ENT for consideration of surgical removal

1743
Q

Anaphylaxis doses

In adult and child >12 years

A

Adrenaline
500 mcg 0.5ml 1 in 1000

200mg Hydrocortisone

10mg Chlorphenamine

1744
Q

A 25 year-old lady with no significant past medical history presents with 2 days of right ear discomfort, discharge and reduced hearing. She is systemically well and has not had a recent cold. On examination the auditory canal appears inflamed and there is a small amount of debris, but you can still see the tympanic membrane. What is the best management from the options below?

Start topical acetic acid 2% spray

Start oral amoxicillin

Refer to ENT

Take an ear swab and start a topical antibiotic

Start a combination topical antibiotic and steroid

A

This lady likely has acute otitis externa.

The NICE Clinical Knowledge summary suggests that inflammation is more likely to be severe if there is:

a red, oedematous ear canal which is narrowed and obscured by debris

conductive hearing loss

discharge

regional lymphadenopathy

cellulitis spreading beyond the ear

fever

This lady has some features of inflammation with a red swollen ear canal and reduced hearing.
NICE recommend that for mild cases (mild discomfort and/or pruritus; no deafness or discharge), consider prescribing topical acetic acid 2% spray.
When features of more severe inflammation are present, such as in this case, they advise 7 days of a topical antibiotic with or without a topical steroid.

Taking swabs from the ear is not useful routinely as virtually all bacteria detected are sensitive to the high concentrations of antibiotic in topical medications. However, this should be done if there is no response to an initial course of treatment or infections are recurrent.

All patients should be advised to use simple analgesia if needed and to avoid getting water into the ear until the infection has resolved.

1745
Q

A 35-year-old man presents to his GP surgery as he is having some difficulties with his hearing. He now struggles to follow conversation and often has the TV volume turned up high. Otoscopy is normal. An audiogram is requested:

What does the audiogram show?

Bilateral mixed hearing loss

Right conductive hearing loss

Normal hearing

Bilateral conductive hearing loss

Bilateral sensorineural hearing loss

A

Audiograms are usually the first-line investigation that is performed when a patient complains of hearing difficulties. They are relatively easy to interpret as long as some simple rules are followed:

anything above the 20dB line is essentially normal (marked in red on the blank audiogram below)

in sensorineural hearing loss both air and bone conduction are impaired

in conductive hearing loss only air conduction is impaired

in mixed hearing loss both air and bone conduction are impaired, with air conduction often being ‘worse’ than bone

1746
Q

Indications for antibiotics in Sore throat

A

features of marked systemic upset secondary to the acute sore throat

unilateral peritonsillitis

a history of rheumatic fever

an increased risk from acute infection (such as a child with diabetes mellitus or immunodeficiency)

patients with acute sore throat/acute pharyngitis/acute tonsillitis when 3 or more Centor criteria are present

1747
Q

Elderly patient
Dizziness on extension of neck

A

Vertebrobasilar ischaemia

1748
Q

A 37-year-old man presents with nasal obstruction and loud snoring. He has noticed these symptoms get gradually worse for the past two months. His left nostril feels blocked whilst his right feels clear and normal. There is no history of epistaxis and he is systemically well. On examination a large nasal polyp can be seen in the left nostril. What is the most appropriate action?

Reassure + provide patient information leaflet on nasal polyps

Enquire about cocaine use

Refer to ENT

Trial of intranasal steroids

Nasal cautery

A

Given that his symptoms are unilateral it is important he is referred to ENT for a full examination.

1749
Q

A patient presents due to a ‘brown coating’ on his tongue. He is 34-years-old and has no significant medical history. The coating has been present for the past few weeks. He is asymptomatic other than a slight ‘tickling’ sensation on his tongue.

What is the most likely diagnosis?

Lichen Planus

Oral Candida

Iron-deficiency anaemia

Hairy leukoplakia

Black hairy tongue

A

Black hairy tongue is relatively common condition which results from defective desquamation of the filiform papillae. Despite the name the tongue may be brown, green, pink or another colour.

Predisposing factors

poor oral hygiene

antibiotics

head and neck radiation

HIV

intravenous drug use

The tongue should be swabbed to exclude Candida

Management

tongue scraping

topical antifungals if Candida

1750
Q

NICE urgent 2 week referral for lung cancer if

A

Have CXR findings that suggest lung cancer

Are aged 40 and over with unexplained haemoptysis

1751
Q

NICE urgent CX indications

A

People over 40 with 2 or more of:

cough

fatigue

shortness of breath

chest pain

weight loss

appetite loss

1752
Q

NICE consideration of urgent chest xr for

A

People aged 40 and over with any of

persistent or recurrent chest infection

finger clubbing

supraclavicular lymphadenopathy or persistent cervical lymphadenopathy

chest signs consistent with lung cancer

thrombocytosis

1753
Q

A 48-year-old female presents to the GP with a lump in throat. She can swallow foods and liquids normally if she tries, although she has noticed the discomfort is worse on swallowing saliva. She does not have any pain on swallowing, chest pain or heart burn. Her appetite is normal.

What is the most likely diagnosis?

Achalasia

Oesophageal spasm

Oesophageal carcinoma

Benign oesophageal stricture

Globus pharyngis

A

Globus pharyngis (also known as globus hystericus) is the persistent sensation of having a ‘lump in the throat’, when there is none. Symptoms are often intermittent and relieved by swallowing food or drink. Swallowing of saliva is often more difficult.

1754
Q

May be history of heartburn
Odynophagia but no weight loss and systemically well

A

Oesophagitis

1755
Q

Dysphagia may be associated with weight loss, anorexia or vomiting during eating
Past history may include Barrett’s oesophagus, GORD, excessive smoking or alcohol use

A

Oesophageal cancer

1756
Q

Dysphagia of both liquids and solids from the start
Heartburn
Regurgitation of food - may lead to cough, aspiration pneumonia etc

A

Achalasia

1757
Q

Other features of CREST syndrome may be present, namely Calcinosis, Raynaud’s phenomenon, oEsophageal dysmotility, Sclerodactyly, Telangiectasia

As well as oesophageal dysmotility the lower oesophageal sphincter (LES) pressure is decreased. This contrasts to achalasia where the LES pressure is increased

A

Systemic sclerosis

1758
Q

Other symptoms may include extraocular muscle weakness or ptosis
Dysphagia with liquids as well as solids

A

Myasthenia gravis

1759
Q

May be history of anxiety
Symptoms are often intermittent and relieved by swallowing
Usually painless - the presence of pain should warrant further investigation for organic causes

A

Globus hystericus

1760
Q

A 20-year-old girl presents with a thyroid cancer, she is otherwise well with no significant family history. On examination she has a nodule in the left lobe of the thyroid with a small discrete mass separate from the gland itself. Which of the following is the most likely cause?

Follicular carcinoma

Anaplastic carcinoma

Medullary carcinoma

Papillary carcinoma

B Cell Lymphoma

A

Papillary carcinoma is the most common subtype and may cause lymph node metastasis (mass separate from the gland itself) that is rare with follicular tumours. Anaplastic carcinoma would cause more local symptoms and would be rare in this age group

1761
Q

Thyroid malignancy:

Commonest sub-type

Accurately diagnosed on fine needle aspiration cytology

Histologically they may demonstrate psammoma bodies (areas of calcification) and so called ‘orphan Annie’ nuclei

They typically metastasise via the lymphatics and thus laterally located apparently ectopic thyroid tissue is usually a metastasis from a well differentiated papillary carcinoma.

A

Papillary carcinoma

1762
Q

Are less common than papillary lesions

Like papillary tumours they may present as a discrete nodule. Although they appear to be well encapsulated macroscopically there invasion on microscopic evaluation.

Lymph node metastases are uncommon and these tumours tend to spread haematogenously. This translates into a higher mortality rate.

Follicular lesions cannot be accurately diagnosed on fine needle aspiration cytology and thus all follicular FNA’s will require at least a hemi thyroidectomy.

A

Follicular carcinoma of thyroid

1763
Q

Thyroid malignancy

Less common and tend to occur in elderly females

Disease is usually advanced at presentation and often only palliative decompression and radiotherapy can be offered.

A

Anaplastic carcinoma

1764
Q

These are tumours of the parafollicular cells ( C Cells) and are of neural crest origin.

The serum calcitonin may be elevated which is of use when monitoring for recurrence.

They may be familial and occur as part of the MEN -2A disease spectrum.

Spread may be either lymphatic or haematogenous and as these tumours are not derived primarily from thyroid cells they are not responsive to radioiodine.

A

Medullary carcinoma

1765
Q

A 21-year-old man presents with halitosis and mouth pain. Examination reveals very poor dental hygiene with bleeding gums and widespread gingival ulceration. He has a temperature of 38.0ºC. You advise him to see a dentist. What other treatment options should be offered?

Paracetamol + oral phenoxymethylpenicillin

Paracetamol + oral phenoxymethylpenicillin + chlorhexidine mouthwash

Paracetamol + chlorhexidine mouthwash

Paracetamol + oral metronidazole + chlorhexidine mouthwash

Paracetamol + oral metronidazole

A

This man has acute necrotizing ulcerative gingivitis with systemic upset. Treatment should be commenced whilst he is awaiting to see a dentist.

Paracetamol + oral metronidazole + chlorhexidine mouthwash

1766
Q

A 53-year-old comes to the GP complaining of an ulcer in his mouth which has been present for 6 weeks. He believes it first started when he went out with his colleagues for some Indian food, which was quite spicy. He reports that the area is painful at times. He reports feeling otherwise well. On examination a 1cm by 1cm lesion is seen with irregular edging.

Which one of the following is the most suitable course of action?

Bonjela to be applied as needed, up to 4 times a day

Routine referral to dental hygienist

Urgent referral to dentist

Routine referral to maxillo-facial surgery

2 week wait referral to oral surgery

A

All mouth ulcers persisting for greater than 3 weeks should be sent to oral surgery as a 2 week wait referral.

2 week wait referrals to oral surgery should be done in all of the following cases:

Unexplained oral ulceration or mass persisting for greater than 3 weeks

Unexplained red, or red and white patches that are painful, swollen or bleeding

Unexplained one-sided pain in the head and neck area for greater than 4 weeks, which is associated with ear ache, but does not result in any abnormal findings on otoscopy

Unexplained recent neck lump, or a previously undiagnosed lump that has changed over a period of 3 to 6 weeks

Unexplained persistent sore or painful throat

Signs and symptoms in the oral cavity persisting for more than 6 weeks, that cannot be definitively diagnosed as a benign lesion

The level of suspicion should be higher in patients who are over 40, smokers, heavy drinkers and those who chew tobacco or betel nut (areca nut).

1767
Q

A 62 year old man presents 2 days after receiving a punch to his head on the right side. Since the injury he feels his hearing has been muffled on the right side. On examination there is no bruising. Both his ears are obscured by a thin translucent layer of wax. On the right, Rinne’s test demonstrates the tuning fork is easier to hear when pressed on the mastoid bone. On Weber’s test the sound is heard best on the right hand side. What is the most likely diagnosis?

Otosclerosis

Base of skull fracture

Otitis media

Earwax

Perforated eardrum

A

Tympanic membrane perforation is a relatively common complication from trauma to the skull. It is important to distinguish this from sensorineural hearing loss resulting from a base of skull fracture.

Rinne’s test her shows that there is a conductive hearing loss in the affected ear. Weber’s test confirms that there is no sensorineural hearing loss on the right.

1768
Q

A 45-year-old man presents with dizziness and right-sided hearing loss. Which one of the following tests would most likely indicate an acoustic neuroma?

Jerky nystagmus

Left homonymous hemianopia

Tongue deviated to the left

Fasciculation of the tongue

Absent corneal reflex

A

Loss of corneal reflex - think acoustic neuroma

1769
Q

A 74-year-old man presents with a 8-week history of right sided otalgia. This is associated with a sore throat and odynophagia. He smokes 20 cigarettes every day and is known to be a heavy drinker. On examination of the ear there are no abnormalities noted.

What is the most likely cause of the otalgia?

Eustachian tube dysfunction

Acute otitis media

Cholsteatoma

Referred pain from nasopharyngeal carcinoma

Mastoiditis

A

Otagia can be due to a primary or secondary cause. Primary otalgia is a result of pathology within the ear where as secondary otalgia occurs due to referred pain from an external source.

Referred otalgia can occurs due to one of five neural pathways (cranial nerve V, cranial nerve VII, cranial nerve IX, cranial nerve X, and via the second and third spinal segments, C2 and C3). Therefore, many parts of the head and neck can refer pain back to the ears.

Otalgia in the absence of any ear signs is a red flag for head and neck malignancy and must be investigated further. In this case the otalgia is likely to be explained by referred pain from the pharynx, and the pharynx must be assess for malignancy, particular due to the age of the gentleman and the risk factor profile.

1770
Q

Systemic: cervical lymphadenopathy

Local:

otalgia

unilateral serous otitis media

Nasal obstruction, dsicharge and or epistaxis

Cranial nerve palsies e.g. III-VI

A

?Nasopharyngeal carcinoma

Squamous cell carcinoma of the naasopharynx

Associated with EBV infection

1771
Q

Theme: Parotid gland disorders

A.Sialectasis

B.Pleomorphic adenoma

C.Bacterial parotitis

D.Viral parotitis

E.Sjogren’s syndrome

F.Adenoid cystic carcinoma

G.Mucoepidermoid carcinoma

H.Warthin’s tumour

I.Sarcoidosis

Please select the most likely diagnosis for the scenario given. Each option may be used once, more than once or not at all.

  1. A 50-year-old lady presents with symptoms of a dry mouth that has been present for the past few months. She also has a sensation of grittiness in her eyes. On examination she has a diffuse swelling of her parotid gland. There is no evidence of facial nerve palsy.
  2. A 50-year-old female presents with bilateral parotid gland swelling and symptoms of a dry mouth. On examination she has bilateral facial nerve palsies. This improved following steroid treatment.
  3. An 18-year-old boy presents with pancreatitis. He has bilateral painful parotid enlargement.
A

Sjogren’s syndrome

Most patients with Sjogren’s present in the post menopausal years. Multi-system involvement is common.

Sarcoidosis

Sarcoid occurs bilaterally in 70% of cases and facial nerve involvement is recognised. Treatment is conservative in most cases although individuals with facial nerve palsy will usually receive steroids with good effect.

Viral parotitis

In a young adult with parotid swelling and pancreatitis/orchitis/reduced hearing/meningoencephalitis suspect mumps.

1772
Q

Most common parotid neoplasm (80%)

Proliferation of epithelial and myoepithelial cells of the ducts and an increase in stromal components

Slow growing, lobular, and not well encapsulated

Recurrence rate of 1-5% with appropriate excision (parotidectomy)

Recurrence possibly secondary to capsular disruption during surgery

Malignant degeneration occurring in 2-10% of adenomas observed for long periods, with carcinoma ex-pleomorphic adenoma occurring most frequently as adenocarcinoma

A

Benign pleomorphic adenoma or benign mixed tumour

1773
Q

Second most common benign parotid tumor (5%)

Most common bilateral benign neoplasm of the parotid

Marked male as compared to female predominance

Occurs later in life (sixth and seventh decades)

Presents as a lymphocytic infiltrate and cystic epithelial proliferation

May represent heterotopic salivary gland epithelial tissue trapped within intraparotid lymph nodes

Incidence of bilaterality and multicentricity of 10%

Malignant transformation rare (almost unheard of)

A

Warthin tumor (papillary cystadenoma lymphoma or adenolymphoma)

1774
Q

Account for less than 5% of tumours Parotid

Slow growing

Consist of only one morphological cell type (hence term mono)

Include; basal cell adenoma, canalicular adenoma, oncocytoma, myoepitheliomas

A

Monomorphic adenoma

1775
Q

Should be considered in the differential of a parotid mass in a child

Accounts for 90% of parotid tumours in children less than 1 year of age

Hypervascular on imaging

Spontaneous regression may occur and malignant transformation is almost unheard of

A

Haemangioma

1776
Q

30% of all parotid malignancies

Usually low potential for local invasiveness and metastasis (depends mainly on grade)

A

Mucoepidermoid carcinoma

1777
Q

Parotid

Unpredictable growth pattern

Tendency for perineural spread

Nerve growth may display skip lesions resulting in incomplete excision

Distant metastasis more common (visceral rather than nodal spread)

5 year survival 35%

A

Adenoid-cystic carcinoma

1778
Q

Often a malignancy occurring in a previously benign parotid lesion

A

Mixed parotid tumour

1779
Q

Parotid tumour

Intermediate grade malignancy

May show perineural invasion

Low potential for distant metastasis

5 year survival 80%

A

Acinic cell carcinoma

1780
Q

Parotid

Develops from secretory portion of gland

Risk of regional nodal and distant metastasis

5 year survival depends upon stage at presentation, may be up to 75% with small lesions with no nodal involvement

A

Adenocarcinoma

1781
Q

Large rubbery lesion, may occur in association with Warthins tumours

Diagnosis should be based on regional nodal biopsy rather than parotid resection

Treatment is with chemotherapy (and radiotherapy)

A

Lymphoma

1782
Q

Diagnosing parotid masses

A

Plain x-rays may be used to exclude calculi

Sialography may be used to delineate ductal anatomy

FNAC is used in most cases

Superficial parotidectomy may be either diagnostic or therapeutic depending upon the nature of the lesion

Where malignancy is suspected the primary approach should be definitive resection rather than excisional biopsy

CT/ MRI may be used in cases of malignancy for staging primary disease

1783
Q

Autoimmune disorder characterised by parotid enlargement, xerostomia and keratoconjunctivitis sicca

90% of cases occur in females

Second most common connective tissue disorder

Bilateral, non tender enlargement of the gland is usual

Histologically, the usual findings are of a lymphocytic infiltrate in acinar units and epimyoepithelial islands surrounded by lymphoid stroma

Treatment is supportive

There is an increased risk of subsequent lymphoma

A

Sjogren’s

1784
Q

Parotid involvement occurs in 6% of patients

Bilateral in most cases

Gland is not tender

Xerostomia may occur

Management of isolated parotid disease is usually conservative

A

Sarcoidosis

1785
Q

Lymphoepithelial cysts occur almost exclusively in the parotid

Typically presents as bilateral, multicystic, symmetrical swelling

Risk of malignant transformation is low and management usually conservative

A

Parotid manifestation of HIV infection

1786
Q

A 24-year-old man who is suffering from sinusitis asks about using Sudafed (pseudoephedrine). Which one of the following medications would make the use of Sudafed contraindicated?

Sodium valproate

Monoamine oxidase inhibitor

Salbutamol

Triptan

Selective serotonin reuptake inhibitor

A

A monoamine oxidase inhibitor combined with pseudoephedrine could potentially cause a hypertensive crisis.

1787
Q

A 7-year-old girl is brought to surgery due to a sore throat. She has a temperature of 39.2ºC and is not eating due to the pain, although she is tolerating fluids. She has had no other related symptoms such as a cough or a rash. Her heart rate is 120/min and auscultation of the chest is unremarkable. The tonsils are covered in exudate bilaterally. Examination of the ears is unremarkable. Other than supportive treatment, what is the most appropriate management?

Erythromycin for 10 days

Amoxicillin for 7 days

Antibiotics are not indicated

Phenoxymethylpenicillin for 10 days

Aciclovir for 5 days

A

This girl has marked systemic upset and should be treated with antibiotics. A 7 or 10 day course of antibiotics is appropriate to ensure eradication of possible Streptococcus infection. Phenoxymethylpenicillin is the first-line antibiotic choice in the BNF

1788
Q

You see a 3-year-old boy as a follow-up appointment. Two weeks ago he presented with left-sided otalgia associated with a purulent discharge. You prescribed amoxicillin and arranged to see him today. His mum reports that he is much better and says she has managed to keep the ear dry. On examination of the left side a perforation of the tympanic membrane is noted. What is the most appropriate action?

Advise to keep ear dry and see in a further 4 weeks time

Prescribe gentamicin ear drops to prevent infection + see in a further 6 weeks time

Advise to keep ear dry and see in a further 12 weeks time

Refer to ENT

Prescribe prophylactic dose amoxicillin to prevent infection + see in a further 4 weeks time

A

When he presented initially with the perforation this boy was given amoxicillin which is consistent with NICE guidelines. There is no indication for continuing the antibiotics if the ear is dry.

If there is still a perforation when the boy is reviewed in 4 weeks time (i.e. 6 weeks since the perforation occurred) then ENT referral should be considered.

1789
Q

Management of perforated tympanic membrane

A

no treatment is needed in the majority of cases as the tympanic membrane will usually heal after 6-8 weeks. It is advisable to avoid getting water in the ear during this time

it is common practice to prescribe antibiotics to perforations which occur following an episode of acute otitis media. NICE support this approach in the 2008 Respiratory tract infection guidelines

myringoplasty may be performed if the tympanic membrane does not heal by itself

1790
Q

A 44-year-old man asks for advice. He is due to go on a long bus journey but suffers from debilitating motion sickness. Which one of the following medications is most likely to prevent motion sickness?

Cyclizine

Chlorpromazine

Metoclopramide

Prochlorperazine

Domperidone

A

Motion sickness - hyoscine > cyclizine > promethazine

1791
Q

Management of motion sickness

A

Management

the BNF recommends hyoscine (e.g. transdermal patch) as being the most effective treatment. Use is limited due to side-effects

non-sedating antihistamines such as cyclizine or cinnarizine are recommended in preference to sedating preparation such as promethazine

1792
Q

This 21-year-old woman has a history of recurrent epistaxis:

© Image used on license from DermNet NZ

What is the most likely underlying diagnosis?

Idiopathic thrombocytopenic purpura

Peutz-Jeghers syndrome

Anorexia nervosa

Combined oral contraceptive pill use

Hereditary haemorrhagic telangiectasia

A

Osler Weber Rendu

HHT

1793
Q

What are the diagnostic criteria for HHT?

A

2= possible diagnosis

3 or more= definite diagnosis

Epistaxis: spontaneous and recurrent nosebleeds

Telangiectasis: multiple at characteristic sites: lips, oral cavity, fingers, nose

Visceral lesions: e.g. GI, pulmonary AVM, hepatic AVM, spinal AVM

Fhx of first degree relative with HHT

1794
Q
A

The chest x-ray shows multiple pulmonary nodules representing arteriovenous malformations, the largest in the right mid-zone. The CT scan shows multiple hepatic arteriovenous malformations

1795
Q

Which one of the following medications is most useful for helping to prevent attacks of Meniere’s disease?

Promethazine

Prochlorperazine

Betahistine

Chlorphenamine

Cinnarizine

A

Management

ENT assessment is required to confirm the diagnosis

patients should inform the DVLA. The current advice is to cease driving until satisfactory control of symptoms is achieved

acute attacks: buccal or intramuscular prochlorperazine. Admission is sometimes required

prevention: betahistine may be of benefit

1796
Q

A 59-year-old man presents with recurrent attacks of vertigo and dizziness. These attacks are often precipitated by a change in head position and typically last around half a minute. Examination of the cranial nerves and ears is unremarkable. His blood pressure is 120/78 mmHg sitting and 116/76 mmHg standing. Given the likely diagnosis of benign paroxysmal positional vertigo, what is the most appropriate next step to help confirm the diagnosis?

Epley manoeuvre

Tilt table test

Tympanometry

MRI of the cerebellopontine angle

Dix-Hallpike manoeuvre

A

This patient has classical symptoms of benign paroxysmal positional vertigo. A positive Dix-Hallpike manoeuvre is an appropriate next step and would help support the diagnosis.

The change in blood pressure on standing is not significant.

1797
Q

You review a 25-year-old man who has allergic rhinits. He has been using intranasal oxymetazoline which he bought from the local chemist for the past 10 days. What is the main side-effect of using topical decongestants for prolonged periods?

Permanent loss of smell

Infective sinusitis

Post-nasal drip

Tachyphylaxis

Necrosis of the nasal septum

A

After using topical decongestants for prolonged periods increasing doses are needed to provide the same effect, a phenomenon known as tachyphylaxis.

1798
Q

Facial ‘fullness’ and tenderness
Nasal discharge, pyrexia or post-nasal drip leading to cough

A

Sinusitis

1799
Q

Unilateral facial pain characterised by brief electric shock-like pains, abrupt in onset and termination
May be triggered by light touch, emotion

A

Trigeminal neuralgia

1800
Q

Pain typical occurs once or twice a day, each episode lasting 15 mins - 2 hours
Clusters typically last 4-12 weeks
Intense pain around one eye
Accompanied by redness, lacrimation, lid swelling, nasal stuffiness

A

Cluster headache

1801
Q

Tender around temples
Raised ESR

A

Temporal arteritis

1802
Q

A 60 year-old man presents with a two month history of nasal blockage on the right side, which is now beginning to disrupt his sleep. He has not noticed any bleeding but has had postnasal drip. On examination you see a polyp on the right side and an inflamed mucosa bilaterally. What is the most appropriate management?

Start saline nasal douche and review in 6 weeks

Start intranasal steroids and review in 6 weeks

Start saline douche and intranasal steroid and review in 6 weeks

Start antihistamine and intranasal steroid and review in 6 weeks

Refer to ENT

A

This patient has a unilateral nasal polyp. Polyps due to rhinosinusitis are usually bilateral so it is important to refer this gentleman to be seen by an ENT doctor to exclude malignancy.

If small bilateral nasal polyps are seen these can be treated in primary care with a saline nasal douche and intranasal steroids, but if they are causing significant obstruction patients should be referred to ENT.

1803
Q

A 37 year old man presents to your clinic with otalgia. He was seen in the emergency department 2 days previously but was discharged with advice only. He has now had otalgia for 5 days.

On examination, he has a temperature of 38.5ºC, and he has a red bulging ear drum on the right. How should you manage this gentleman?

Advise on regular paracetamol and return if no better

Start erythromycin

Start penicillin V

Start amoxicillin

Start co-amoxiclav

A

This is a case of otitis media. Although 50% of these cases are viral, and 60% improve without antibiotics, guidelines would advocate treatment after a delay of 2-3 days if there is no improvement in symptoms. Especially in this case, where the gentleman has a temperature and therefore evidence of systemic involvement. Therefore, advising regular paracetamol is not correct in this case.

Erythromycin is a useful alternative to patients who are penicillin allergic, and especially as a syrup in children (as it costs less that some alternatives!) but would not be first line in someone who can take penicillin.

Penicillin V is the first line antibiotic for tonsillitis due to amoxicillin having the potential to cause a rash in cases of glandular fever. However, it is not generally used in otitis media.

Amoxicillin is the correct first line medication for treating otitis media at 500mg TDS for 7 days.

Co-amoxiclav is used as a second line agent if amoxicillin doesn’t work, but wouldn’t be used first line according to current guidelines.

1804
Q

A 23-year-old woman presents one week after being prescribed a combined antibiotic and steroid spray for otitis externa. There has been no improvement in her symptoms and the erythema seems to have extended to the ear itself. What is the most appropriate treatment?

Topical clotrimazole

Oral flucloxacillin

Topical ciprofloxacin

Oral fluconazole

Oral ciprofloxacin

A

The spreading erythema is an indication for oral antibiotics. Flucloxacillin is first-line.

1805
Q

A 25 year-old man presents with a history of sudden hearing loss on the right side. He had no preceding coryzal illness, fevers, headache or ear pain. On examination his ear canal and tympanic membrane appear normal. Weber testing localises to the left side. What is the appropriate management?

Refer routinely to ENT

Watch and wait

Refer urgently to ENT and start high dose oral steroids

Start oral aciclovir and high dose steroids

Start decongestant nasal spray and intranasal steroids

A

This man has sudden sensorineural hearing loss, which in the vast majority of cases is idiopathic.

There is some evidence that high dose steroids (60mg/day) for seven days improves prognosis, so all patients should start treatment as soon as possible. ENT assessment should be arranged as soon as possible to allow pure tone audiometry testing and to arrange an MRI to exclude an acoustic neuroma. Intra-tympanic steroids can also be given if there is no response to oral steroids.

Aciclovir is not routinely recommended as there is no evidence of benefit.

1806
Q

A.Peripheral arterial disease

B.Prolapsed disc

C.Facet joint pain

D.Perforated duodenal ulcer

E.Leaking abdominal aortic aneurysm

F.Pyelonephritis

G.Ankylosing spondylitis

H.Rheumatoid arthritis

I.Crush fracture

J.Spinal stenosis

A 34-year-old man reports the sudden onset of back pain after bending over to tie his shoe laces. There is tenderness over the lumbar spine on examination and leaning back worsens the pain. Neurological examination and straight leg raising is normal

A

Although patients often give a history of bending prior to disc prolapse the normal straight leg raising makes this diagnosis less likely.

1807
Q

May be acute or chronic
Pain worse in the morning and on standing
On examination there may be pain over the facets. The pain is typically worse on extension of the back

A

Facet joint

1808
Q

Usually gradual onset
Unilateral or bilateral leg pain (with or without back pain), numbness, and weakness which is worse on walking. Resolves when sits down. Pain may be described as ‘aching’, ‘crawling’.
Relieved by sitting down, leaning forwards and crouching down
Clinical examination is often normal
Requires MRI to confirm diagnosis

A

Spinal stenosis

1809
Q

What are the Ottawa ankle criteria

A

Ankle XR only reqired if there is any pain in malleolar zone + 1 of:

Bony tenderness at lateral malleolar zone

Bony tenderness at the medial malleolar zone

Inability to walk four weight bearing steps immediately after the injury and in the ED

1810
Q

A 7-year-old boy is taken to the Emergency Department after falling during football. He complains of pain in his left wrist.

The x-ray is shown below:

© Image used on license from Radiopaedia

What which one of the following best describes this injury?

Greenstick fracture

Buckle fracture

Salter-Harris type 1 fracture

Salter-Harris type 2 fracture

Salter-Harris type 3 fracture

A

A typical buckle fracture is shown in this radiograph. This is a common fracture pattern in children due to the pliable nature of the bone.

Buckle, or torus, fractures are incomplete fractures of the shaft of a long bone that is characterised by bulging of the cortex. They typically occur in children aged 5-10 years.

As they are typically self-limiting they do not usually require operative intervention and can sometimes be managed with splinting and immobilisation rather than a cast.

1811
Q

Softening of the cartilage of the patella
Common in teenage girls
Characteristically anterior knee pain on walking up and down stairs and rising from prolonged sitting
Usually responds to physiotherapy

A

Chondromalacia patellae

1812
Q

Seen in sporty teenagers
Pain, tenderness and swelling over the tibial tubercle

A

Osgood-Schlatter disease

(tibial apophysitis)

1813
Q

Pain after exercise
Intermittent swelling and locking

A

Osteochondritis dissecans

1814
Q

Medial knee pain due to lateral subluxation of the patella
Knee may give way

A

Patellar subluxation

1815
Q

More common in athletic teenage boys
Chronic anterior knee pain that worsens after running
Tender below the patella on examination

A

Patellar tendonitis

1816
Q

A 10-year-old girl is brought to the Emergency Department after falling from a tree swing. She is complaining of pain in the left forearm. An x-ray is requested:

© Image used on license from Radiopaedia

What type of fracture is seen?

Buckle fracture

Greenstick fracture

Salter-Harris type I

Salter-Harris type II

Salter-Harris type IV

A

The radiograph shows a fracture involving the palmar surface of the distal radius, with minimal dorsal angulation and no disruption of the dorsal cortex. The growth plate appears normal.

1817
Q

Both sides of cortex are breached

A

Complete fracture

1818
Q

Oblique tibial fracture in infants

A

Toddlers fracture

1819
Q

Stress on bone resulting in deformity without cortical disruption

A

Plastic deformity

1820
Q

Unilateral cortical breach only

A

Greenstick fracture

1821
Q

Incomplete cortical disruption resulting in periosteal haematoma only

A

Buckle fracture

1822
Q

Defective osteoid formation due to congenital inability to produce adequate intercellular substances like osteoid, collagen and dentine.

Failure of maturation of collagen in all the connective tissues.

Radiology may show translucent bones, multiple fractures, particularly of the long bones, wormian bones (irregular patches of ossification) and a trefoil pelvis.

A

Osteogenesis imperfecta

1823
Q

Type 1 Osteogenesis imperfecta

A

Collagen is normal quality but insufficient quantity

1824
Q

Type II OI

A

Poor collagen quantity and quality

1825
Q

Type III OI

A

Collagen poorly formed, normal quantity

1826
Q

Type IV OI

A

Sufficient collagen quantity but poor quality

1827
Q

Bones become harder and more dense.

Autosomal recessive condition.

It is commonest in young adults.

Radiology reveals a lack of differentiation between the cortex and the medulla described as marble bone.

A

Osteopetrosis

1828
Q

A 65-year-old man presents with bilateral leg pain that is brought on by walking. His past medical history includes peptic ulcer disease and osteoarthritis. He can typically walk for around 5 minutes before it develops. The pain subsides when he sits down. He has also noticed that leaning forwards or crouching improves the pain. Musculoskeletal and vascular examination of his lower limbs is unremarkable. What is the most likely diagnosis?

Inflammatory arachnoiditis

Peripheral arterial disease

Raised intracranial pressure

Spinal stenosis

Lumbar vertebral crush fracture

A

This is a classic presentation of spinal stenosis. Whilst peripheral arterial disease is an obvious differential the characteristic relieving factors of the pain and normal vascular examination point away from this diagnosis.

1829
Q

What are the red flags for LBP?

A

Age <20 or >50

Hx of previous malignancy

Night pain

History of trauma

Systemicallly unwell e.g. weight loss fever

Altered neurology

Incontinence or constipation

1830
Q

Muscles innervated by median nerve

LOAF

A

Lumbricals (third and fourth)

Opponens pollicis

Abductor pollicis brevis

Flexor pollicis brevis

1831
Q

He is a know intravenous drug user who has been commenced on a methadone rehabilitation programme around 2 weeks previously. He has no fixed abode and spends his night between various hostels.

He has no significant past medical history. Although has previous admissions following various injuries such as stabbings.

His observations are: heart rate 99/min, respiratory rate 16/min, blood pressure 98/75 mmHg, temperature 37.7º, Sats 99% on air.

On examination he appears to be in pain and is lying flat on his back with his knees flexed. He is very tender over his back at L1 and L2 levels. He actively resists passive movement from his position of comfort and is unable to weight bear due to pain.

Blood tests show raised inflammatory markers, lumbar spine and pelvic x-ray show no abnormality.

He is requesting analgesia for his pain.

What is the most likely cause of this mans symptoms?

Atypical femoral neck fracture

Discitis

Malingering

Vertebral osteomyelitits

Psoas abscess

A

Psoas abscess is a collection within the psoas muscle. It is commonly missed or diagnosed late due to its numerous differential diagnoses and a high index of suspicion is required in those with risk factors. The psoas muscle extends from T12 - L5 caudally, inserting on the lesser trochanter of the femur. It can be of primary origin or a result of spread from local sources such as pyelonephritis or inflammatory bowel disease. Left untreated it can lead to septicaemia and multi organ failure.

The most common causative organism is staphylococcus or streptococcus and risk factors for developing a primary abscess are related to causes of immunosupression such as HIV, cancer and diabetes. Being an intravenous drug user, previous surgery and tuberculosis also predispose to the condition.

Pain is usually non specific initially but increases over several days. It is worth suspecting if there is no history of trauma or injury. Fever may be present but not always. Psoas irritation is evidenced when the position of comfort is the patient lying on their back with slightly flexed knees. Inability to weight bear or pain when moving the hip is usually evident.

Investigations should include bloods to evidence infection and a complete septic screen if systemic inflammatory response syndrome criteria are met. Plain radiographs are not useful for identifying an abscess although are useful for ruling out differentials. CT abdomen may identify the abscess but MRI is the gold standard.

Management is with antibiotic therapy +/- drainage. Alongside managing any predisposing risk factors if appropriate.

1832
Q

What is the diagnostic intracompartmental pressure in compartment syndrome

A

>20mmHg is abnormal

>40mmHg is diagnostic

1833
Q

Treatment of compartment syndrome

A

This is essentially prompt and extensive fasciotomies

In the lower limb the deep muscles may be inadequately decompressed by the inexperienced operator when smaller incisions are performed

Myoglobinuria may occur following fasciotomy and result in renal failure and for this reason these patients require aggressive IV fluids

Where muscle groups are frankly necrotic at fasciotomy they should be debrided and amputation may have to be considered

Death of muscle groups may occur within 4-6 hours

1834
Q

Associations of trigger finger

A

More common in women than men

RA

DM

1835
Q

More common in thumb, middle or ring finger

Initially stiffness and snapping when extending a flexed digit

Nodule may be felt at base of affected finger

A

Tirgger finger

1836
Q

Mx of trigger finger

A

Steroid injeciton is successful in the majority

Surgery may be reserved for patients who have not responded to steroid injections

1837
Q

You see an 81 year old lady with a history of diabetes, osteoarthritis and hypertension. She twisted her leg whilst getting out of a car and developed increasing pain weight bearing which has eased with simple analgesia. She also tells you she has a lump under her knee. On examination she has a 4cm non-tender lump just below the popliteal fossa which becomes tense on extending the leg. She has full power throughout. What is the most likely diagnosis?

Deep vein thrombosis

Popliteal artery aneurysm

Sprain

Baker’s cyst

Ruptured head of gastrocnemius

A

This describes the typical patient with a Baker’s cyst. They are more likely to develop in patients with arthritis or gout and following a minor trauma to the knee. Foucher’s sign describes the increase in tension of the Baker’s cyst on extension of the knee.

A DVT (deep vein thrombosis) needs to be considered because it can mimic a Baker’s cyst. A DVT can also co-exist with a Baker’s cyst and a low threshold for ultrasound should be considered.

1838
Q

Knee pain.

Patient is typically > 50 years, often overweight
Pain may be severe
Intermittent swelling, crepitus and limitation of movement may occur

A

Osteoarthritis of the knee

1839
Q

Knee pain associated with kneeling

A

Infrapatellar bursitis: Clergyman’s knee

1840
Q

Knee pain associated with more upright kneeling

A

Prepatellar bursitis: Housemaid’s knee

1841
Q

Knee pain

May be caused by twisting of the knee, popping noise may have been noted

Rapid onset knee effusion

Positive draw test

A

ACL

1842
Q

Knee pain

May have been caused by anterior force applied to proximal tibia e.g. knee hitting dashboard during car accident

A

PCL

1843
Q

Knee pain

Tenderness over affected ligament

Knee effusion

A

?Collateral ligament injury

1844
Q

Knee pain

May have been caused by twisting of the knee

Locking and giving way are common feature

Tender joint line

A

Meniscal lesion

1845
Q

What is significant about anatomical vs surgical neck fractures of the proximal humerus

A

Anatomical neck fractures are rare, those that are displaced by >1cm carry a risk of avascular necrosis to the humeral head

1846
Q

What are the types of shoulder dislocation?

A

Glenohumeral dislocation

Acromioclavicular dislocation: clavicle loses all attachment with scapula

Sternoclavicular dislocation (uncommon)

1847
Q

What are the types of glenohumeral dislocation?

A

Anterior

Inferior

Posterior

Superior

1848
Q

External rotation and abduction

35-40% recurrent

Associated with greater tuberosity fracture, Bankart lesion, Hill-Sachs defect

A

Anterior dislocation

1849
Q

What is Trough line sign

A

The trough line sign is a sign of posterior shoulder dislocation on AP films.

In posterior dislocation, the anterior aspect of the humeral head becomes impacted against the posterior glenoid rim. With sufficient force, this causes a compression fracture on the anterior aspect of the humeral head. This compression fracture is analogous to the Hill-Sachs compression fracture seen with anterior shoulder dislocation of the glenohumeral joint.

Frontal radiographs reveal two nearly parallel lines in the superomedial aspect of the humeral head.

1850
Q
A

Trough sign

1851
Q

Luxatio erecta

A

An inferior shoulder dislocation is the least common form of shoulder dislocation. The condition is also called luxatio erecta because the arm appears to be permanently held upward or behind the head, in fixed abduction.

1852
Q
A

Luxatio erecta (inferior shoulder disloccation)

1853
Q

A 75-year-old female presents with weakness of her left hand. On examination wasting of the hypothenar eminence is seen and there is weakness of finger abduction. Thumb adduction is also weak. Where is the lesion most likely to be?

Common peroneal nerve

Median nerve

Radial nerve

Anterior interosseous nerve

Ulnar nerve

A

Ulnar nerve

1854
Q

pain and tenderness localised to the lateral epicondyle

pain worse on resisted wrist extension with the elbow extended or supination of the forearm with the elbow extended

episodes typically last between 6 months and 2 years. Patients tend to have acute pain for 6-12 weeks

A

Lateral epicondylitis (tennis elbow)

1855
Q

Features

pain and tenderness localised to the medial epicondyle

pain is aggravated by wrist flexion and pronation

symptoms may be accompanied by numbness / tingling in the 4th and 5th finger due to ulnar nerve involvement

A

Medial epicondylitis (golfer’s elbow)

1856
Q

Most commonly due to compression of the posterior interosseous branch of the radial nerve. It is thought to be a result of overuse.

Features

symptoms are similar to lateral epicondylitis making it difficult to diagnose

however, the pain tends to be around 4-5 cm distal to the lateral epicondyle

symptoms may be worsened by extending the elbow and pronating the forearm

A

Radial tunnel syndrome

1857
Q

Due to the compression of the ulnar nerve.

Features

initially intermittent tingling in the 4th and 5th finger

may be worse when the elbow is resting on a firm surface or flexed for extended periods

later numbness in the 4th and 5th finger with associated weakness

A

Cubital tunnel syndrome

1858
Q

Swelling over the posterior aspect of the elbow. There may be associated pain, warmth and erythema. It typically affects middle-aged male patients.

A

Olecranon bursitis

1859
Q

Def: De Quervain’s tenosynovitis

A

De Quervain’s tenosynovitis is a common condition in which the sheath containing the extensor pollicis brevis and abductor pollicis longus tendons is inflamed. It typically affects females aged 30 - 50 years old

1860
Q

Features

pain on the radial side of the wrist

tenderness over the radial styloid process

abduction of the thumb against resistance is painful

Finkelstein’s test: with the thumb is flexed across the palm of the hand, pain is reproduced by movement of the wrist into flexion and ulnar deviation

A

De Quervain’s tenosynovitis

1861
Q

Mx of tenosynovitis

A

Management

analgesia

steroid injection

immobilisation with a thumb splint (spica) may be effective

surgical treatment is sometimes required

1862
Q

A 30-year-old woman has left knee ligament reconstruction surgery for a torn lateral collateral ligament. During recovery, she notices persistent weakness in her left leg and foot. On examination, she has weakness of dorsiflexion and eversion of the left foot (MRC power grade 2/5). There is a small patch of sensory loss over the first and second toes. What is the most likely diagnosis?

L5 radiculopathy

Tibial nerve palsy

Compartment syndrome

Right-sided stroke

Common peroneal nerve palsy

A

Loss of dorsiflexion and eversion suggests a common peroneal nerve lesion as opposed to an L5 radiculopathy (in L5 radiculopathy, eversion tends to be spared while sensory involvement tends to be greater). This is before the recent risk factor of knee surgery is considered, making common peroneal nerve palsy the more obvious answer.

NB common peroneal palsy spares inversion whilst l5 radiculopathy will have weakness of ankle inversion

1863
Q

Foot drop

weakness of foot dorsiflexion

weakness of foot eversion

weakness of extensor hallucis longus

sensory loss over the dorsum of the foot and the lower lateral part of the leg

wasting of the anterior tibial and peroneal muscles

A

Common peroneal nerve palsy

1864
Q

A 57-year-old woman presents with a three month history of right-sided hip pain. This seems to have come on spontaneously without any obvious precipitating event. The pain is described as being worse on the ‘outside’ of the hip and is particularly bad at night when she lies on the right hand side.

On examination there is a full range of movement in the hip including internal and external rotation. Deep palpation of the lateral aspect of the right hip joint recreates the pain.

An x-ray of the right hip is reported as follows:

Right hip: Minor narrowing of the joint space otherwise normal appearance

What is the most likely diagnosis?

Fibromyalgia

Lumbar nerve root compression

Osteoarthritis

Greater trochanteric pain syndrome

Meralgia paraesthetica

A

Greater trochanteric pain syndrome is now the preferred term for trochanteric bursitis.

Whilst the x-ray shows joint space narrowing this is not an uncommon finding. Osteoarthritis would also be less likely given the palpable nature of the pain and relatively short duration of symptoms.

1865
Q

Hip pain

Pain exacerbated by exercise and relieved by rest
Reduction in internal rotation is often the first sign
Age, obesity and previous joint problems are risk factors

A

OA

1866
Q

Hip pain

Pain in the morning
Systemic features
Raised inflammatory markers

A

Inflammatory arthritis

1867
Q

Hip pain

Femoral nerve compression may cause referred pain in the hip
Femoral nerve stretch test may be positive - lie the patient prone. Extend the hip joint with a straight leg then bend the knee. This stretches the femoral nerve and will cause pain if it is trapped

A

Referred lumbar spine pain

1868
Q

Due to repeated movement of the fibroelastic iliotibial band
Pain and tenderness over the lateral side of thigh
Most common in women aged 50-70 years

A

Greater trochanteric pain syndrome (Trochanteric bursitis)

1869
Q

Caused by compression of lateral cutaneous nerve of thigh
Typically burning sensation over antero-lateral aspect of thigh

A

Meralgia paraesthetica

1870
Q

Symptoms may be of gradual or sudden onset
May follow high dose steroid therapy or previous hip fracture of dislocation

A

Avascular necrosis

1871
Q

Common in pregnancy
Ligament laxity increases in response to hormonal changes of pregnancy
Pain over the pubic symphysis with radiation to the groins and the medial aspects of the thighs. A waddling gait may be seen

A

Pubic symphysis dysfunction

1872
Q

An uncommon condition sometimes seen in the third trimester of pregnancy
Groin pain associated with a limited range of movement in the hip
Patients may be unable to weight bear
ESR may be elevated

A

Transient idiopathic osteoporosis

1873
Q

A 48-year-old woman presents with progressively worsening pain in the right shoulder over the past few weeks. She is generally fit and well but smokes 20 cigarettes/day.

On examination there is diffuse mild tenderness over the lateral aspect of the right shoulder. The pain is recreated when abducting the shoulder to around 70-80 degrees.

A shoulder x-ray is requested:

© Image used on license from Radiopaedia

What is the most likely diagnosis?

Pancoast tumour

Supraspinatus tendonitis

Adhesive capsulitis

Humeral head fracture

Avascular necrosis

A

The x-ray shows calcification of the supraspinatus tendon consistent with prolonged inflammation. On examination the patient exhibits the classical ‘painful arc’ associated with this condition.

1874
Q

What are the associations of talipes equinovarus?

A

Most commonly idiopathic. Associations include:

spina bifida

cerebral palsy

Edward’s syndrome (trisomy 18)

oligohydramnios

arthrogryposis

1875
Q

A 22-year-old male presents to the emergency room with pain in the left knee following a twisting injury during a rugby match. He states that it has gradually swollen over the past 24 hours, and he is unable to fully extend it. On examination you note tenderness over the medial joint line, a joint effusion, and the joint is held in a flexed position. There is no laxity on valgus stress test.

What is the most likely diagnosis?

Medial meniscus tear

Lateral meniscus tear

Anterior cruciate ligament (ACL) tear

Posterior cruciate ligament (PCL) tear

Medial collateral ligament (MCL) tear

A

Gradual swelling of the knee is suggestive of effusion which often occurs due to meniscal injury. Tenderness over the medial joint line suggests a medial meniscus tear.

An ACL or PCL tear would more commonly present with rapid joint swelling due to bleeding within the joint capsule (haemarthrosis).

Isolated MCL injuries rarely cause a large effusion. In addition, the lack of laxity on the valgus stress test makes an MCL injury less likely.

1876
Q

Sport injury

Mechanism: high twisting force applied to a bent knee

Typically presents with: loud crack, pain and RAPID joint swelling (haemoarthrosis)

Poor healing

Management: intense physiotherapy or surgery

A

Ruptured ACL

1877
Q

Mechanism: hyperextension injuries

Tibia lies back on the femur

Paradoxical anterior draw test

A

Ruptured posterior cruciate ligament

1878
Q

Mechanism: leg forced into valgus via force outside the leg

Knee unstable when put into valgus position

A

Rupture of medial collateral ligament

1879
Q

Rotational sporting injuries

Delayed knee swelling

Joint locking (Patient may develop skills to “unlock” the knee

Recurrent episodes of pain and effusions are common, often following minor trauma

A

Menisceal tear

1880
Q

Teenage girls, following an injury to knee e.g. Dislocation patella

Typical history of pain on going downstairs or at rest

Tenderness, quadriceps wasting

A

Chondromalacia patellae

1881
Q

Most commonly occurs as a traumatic primary event, either through direct trauma or through severe contraction of quadriceps with knee stretched in valgus and external rotation

Genu valgum, tibial torsion and high riding patella are risk factors

Skyline x-ray views of patella are required, although displaced patella may be clinically obvious

An osteochondral fracture is present in 5%

The condition has a 20% recurrence rate

A

Dislocation of the patella

1882
Q

2 types:

i. Direct blow to patella causing undisplaced fragments
ii. Avulsion fracture

A

Fractured patella

1883
Q

Occur in the elderly (or following significant trauma in young)

Mechanism: knee forced into valgus or varus, but the knee fractures before the ligaments rupture

Varus injury affects medial plateau and if valgus injury, lateral plateau depressed fracture occurs

Classified using the Schatzker system (see below)

A

Tibial plateau fracture

1884
Q

What can be used to classify tibial plateau fractures?

A

Schatzker system

1885
Q

Describe Schatzker Classification

A

Type 1:

Vertical split of lateral condyle

Fracture through dense bone, usually in the young, may be undisplaced or condylar fragment may be pushed in feriorly

Type 2:

Vertical split of the lateral condyle combined with an adjacent load bearing part of the condyle

The wedge fragment (which may be of variable size), is displaced laterally; the joint is widened. Untreated, a valgus deformity may develop

Type 3:

Depression of the articular surface with intact condylar rim

The split does not extend to the edge of the plateau. Depressed fragments may be firmly embedded in subchondral bone, the joint is stable

Type 4

Fragment of the medial tibial condyle

Two injuries are seen in this category; (1) a depressed fracture of osteoporotic bone in the elderly. (2) a high energy fracture resulting in a condylar split that runs from the intercondylar eminence to the medial cortex. Associated ligamentous injury may be severe

Type 5

Fracture of both condyles

Both condyles fractured but the column of the metaphysis remains in continuity with the tibial shaft

Type 6

Combined condylar and subcondylar fractures

High energy fracture with marked comminution

1886
Q

Which option is not recommended during the management of compartment syndrome?

Anticoagulation

Keep limb level with the body

Intravenous fluids

Pain control

Fasciotomy

A

Anticoagulation will worsen compartment syndrome.

1887
Q

Associations of Dupuytren’s

A

Dupuytren’s contracture has a prevalence of about 5%. It is more common in older male patients and around 60-70% have a positive family history

Specific causes include:

manual labour

phenytoin treatment

alcoholic liver disease

trauma to the hand

1888
Q

A 16-year-old male presents with lower back pain of 5 months duration, worse at night, with morning stiffness. He finds the pain improved with exercise. There is no history of trauma. He is given a clinical diagnosis of ankylosing spondylitis. Which of the following findings on examination would be most associated with this diagnosis?

Schober’s test 4.0cm

Development of scoliosis

Cervical kyphosis

Mitral valve incompetence

Osteomyelitis

A

Schober’s test <5cm is suggestive of ankylosing spondylitis. This is an indication of reduced lumbar flexion.

Schober’s test is performed by identifying L5, and then marking 10cm above and 5cm below this point whilst the patient is stood upright. The patient is then asked to bend forwards to touch their toes whilst keeping their knees straight. If the distance between the points does not increase by 5cm (or the distance between the points originally marked is not more than 20cm in total), then it can be said that there is reduced flexion of the lumbar spine, which is a sign of ankylosing spondylitis.

Thoracic kyphosis is an associated spinal deformity, alongside neck hyperextension. This may be referred to by some as the ‘question mark’ appearance.

Aortic valve incompetence is associated with <3% of cases.

1889
Q

As of ank spond

A

Apical fibrosis

Anterior uveitis

Aortic regurgitation

Achilles tendonitis

AV node block

Amyloidosis

and cauda equina syndrome

peripheral arthritis (25%, more common if female)

1890
Q

Specific management of femoral and tibial fractures

A

Resus and Mx life threatening injuries first

X-match

Assess neurovascular status: especially distal pulses

If open

Abx and ATT

Take to theatre urgently for debridement, washout and stabilisation

Fixation methods:

Intramedullary nail

External fixation

Plates and screws

MUA with fixed traction for 3-4 months

1891
Q

X-match how many units in tibial #?

A

2 units

1892
Q

X-match how many units in #femur

A

4 units

1893
Q

What are the specific complications of femoral and tibial fractures?

A

Hypovolaemic shock

Neurovascular:

SFA: swelling and check pulses

Sciatic nerve

Compartment syndrome

Respiratory complications:

Fat embolism

ARDS

Pneumonia

1894
Q

Features of ankle ligament strain

A

Typically twisting inversion injury

Strains anterior talofibular part of LCL

Medial deltoid ligament strains are rare

May be associated with malleolar avulsion fractures

1895
Q

What is Weber classification of ankle injuries?

A

Relation of fibula # to joint line

1896
Q

Outline Weber classificaiton of fibula #

A

A: below joint line

B: at joint line

C: above joint line

1897
Q

Significance of Weber classification

A

Weber’s B and C represent possible injury to the syndesmotic ligaments between tib and fib -> instability

1898
Q

What is this classificaiton system?

A

Weber Classification
􀁸 Relation of fibula # to joint line
􀁸 A: below joint line
􀁸 B: at joint line
􀁸 C: above joint line
􀁸 Weber’s B and C represent possible injury to the
syndesmotic ligaments between tib and fib → instability

1899
Q

Mx of Weber A

A

Boot or below knee POP

1900
Q

Mx of Non-displaced Weber B/C

A

Below knee POP

1901
Q

Mx of displaced Weber B/C

A

Closed reduction and POP if anatomical reduction achieved

ORIF if closed reduction fails

1902
Q

Important components of knee injury history

A

Mechanism

Swelling: immediate vs overnight

Pain/tenderness: joint line vs medial/lateral margins

Locking: meniscal tear

Giving way: instability following lgt injury

1903
Q

Immediate swelling following knee injury

A

= haemarthrosis. # or torn cruciates

1904
Q

Overnight swelling following knee injury

A

= effusion

Meniscus or other ligamentous injury

1905
Q

Locking following knee injury=

A

Meniscal tear

1906
Q

Pain/tenderness over joint line following knee injury

A

Meniscal

1907
Q

Pain/tenderness over med/lat knee margins=

A

Collateral ligaments

1908
Q

1o knee haemarthrosis

A

Spontaneous bleeding:

Coagulopathy: warfarin, haemophilia

1909
Q

2o Knee haemarthrosis

A

Trauma

ACL injury: 80%

Patella dislocation: 10%

Meniscal injury: outer third where it is vascularised (10%)

Osteophyte #

1910
Q

What is the unhappy triad of O’Donoghue

MMA

A

MCL

Medial meniscus

ACL

1911
Q

Mx of acutely injured knee

A

Full examination of acutely swollen knee after injury is difficult

Take XR to ensure no #s: fluid level indicates a lipohaemarthrosis and indicates either a # or torn cruciate

If no #-> RICE + later re-examination

If meniscal or cruciate injury suspected-> MRI

1912
Q

Features of knee arthroscopy

A

Direct vision of inside of knee joint by arthroscope

Can examine knee under anaesthesia (reduced muscle tone)

Meniscal tears can be trimmed or repaired

1913
Q

Mx of Ruptured ACL

A

Conservative:

Rest

PT to strengthen quads and hamstrings

Not enough stability for many sports

Surgical:

Gold-standard is autograft repair

Usually semintendonosus +/- gracillis (can use patella tendon)

Tendon threaded through heads of tibia and femur and held using screws

1914
Q

Pathophysiology of OA

A

Softening of articular cartilage-> fraying and fissuring of smooth surface-> underlying bone exposure

Subchondral bone becomes sclerotic with cysts

Proliferation and ossification of cartilage in unstressed areas-> osteophytes

Capsular fibrosis-> stiff joints

1915
Q

DDx for OA

A

Septic

Crystal

Trauma

1916
Q

XR changes in OA

A

Loss of joint space

Osteophytes

Subchondral cysts

Subchondral sclerosis

Deformity

1917
Q

Bloods in OA

A

CRP may be elevated

Ca, PO4, ALP all normal

1918
Q

Rx of OA

A

MDT: GP, physio, OT, dietician, orthopod

Conservative:

Lifestyle: reduce weight, increase exercise

PT: muscle strengthening

OT: walking aids, supportive footwear, home modifications

Medical:

Analgesia: paracetamol + topical NSAIDs e.g. voiltarel

Tramadol

Joint injeciton: LA and steroids

Surgical:

Arthroscopic washout

Realignment osteotomy: small area of bone cut out, useful in younger patients with medial knee OA. High tibial valgus osteotomy redistributes weight to lateral part of joint

Arthroplasty

Arthrodesis

Novel techniques

1919
Q

Mechanical back pain features

A

Soft tissue injury-> dysfunction of whole spine-> muscle spasm-> pain

May have inciting event e.g. lifting

Young patients with no sinister features

1920
Q

Mx of mechanical back pain

A

Conservative:

Max 2d bed rest

Education: keep active, how to lift/stoop

PT

Psychoscoial issues

Warmth

Medical:

Analgesia: paracetamol +/- NSAIDS +/- codeine

Muscle relaxant: low-dose diazepam (used ST)

1921
Q

Def: disc prolapse

A

Herniation of nucleeus pulposus through annulus fibrosus

1922
Q

Which spinal nerve roots most commonly compressed in disc herniation?

A

L5 (L4/L5 disc)

S1 (L5/S1 disc)

1923
Q

May present as severe pain on sneezing, coughing or twisting a few days after low back strain

Lumbago: low back pain

Sciatica: shooting radicular pain down buttock and thigh

A

=?Disc prolapse

1924
Q

Limited spinal flexion and extension

Free lateral flexion

Lesague’s sign

A

?Disc herniation

1925
Q

What is Lesague’s sign

A

Pain on straight leg raise

?Radiculopathy

1926
Q

Location of disc herniation and significance

A

Lateral herniation= radiculopathy

Central herniatoin= cauda equina syndrome

1927
Q

Weak hallux extension +/- foot drop

Weak inversion (helps distinguish from peroneal nerve palsy)

Reduced sensation on inner dorsum of foot

A

L4/L5 root compression

1928
Q

Weak floot plantarflexion and eversion

Loss of ankle jerk

Calf pain

Reduced sensation over sole of foot and back of calf

A

L5/S1-> S1 root compression

1929
Q

Ix in disc prolapse

A

MRI (emergency if cauda equina)

1930
Q

Mx of disc herniation

A

Brief rest, analgesia and mobilisation effective in >90%

Conservative:

Brief rest

Mobilisation/PT

Medical: analgesia, transforaminal steroid injection

Surgical:

discectomy or laminectomy may be needed in cauda equina syndrome, continuing pain or muscle weakness

1931
Q

Features of lumbar microdiscectomy?

A

Commonest procedure for disc prolapse

Microscopic resection of the protruding nucleus pulposus

Posterior approach with patient in prone position

May be performed endoscopically

1932
Q

Def: spondylolisthesis

A

Displacement of one lumbar vertebra on another

Usually forward

Usually L5 on S1

1933
Q
A

Spondylolisthesis

1934
Q

Causes of spondylolisthesis

A

Congenital malformation

Spondylosis

osteoarthritis

1935
Q

Onset of pain usually in adolescence or early adulthood

Worse on standing

+/- sciatic, hamstring tightness, abormal gait

A

Spondylolisthesis

1936
Q

Dx of sponylolisthesis

A

Plain radiography

1937
Q

Rx of spondylolisthesis

A

Corset

Nerve release

Spinal fusion

1938
Q

Def: spinal stenosis

A

Developmental predisposition +/- facet joint OA-> generalised narrowing of lumbar spinal canal

1939
Q

Aching or heavy buttock and lower limb pain on walking

Rapid onset

May c/o paraesthesia, numbness

Pain eased by leaning forward e.g. on a bike

Pain on extension

A

Spinal claudicaiton

1940
Q

Ix in spinal stenosis

A

MRI

1941
Q

Rx of spinal stenosis

A

Corsets

NSAIDs

Epidural steroid injection

Canal decompression surgery

1942
Q

Def: lordosis

A

Spine of a person with lordosis curves significantly inward at lower back

1943
Q
A
1944
Q

Def: kyphosis

A

Abnormally rounded upper back (more than 50 deg of curvature)

1945
Q

Def: scoliosis

A

Sideways curve to the spine, often S-shaped or C-shaped

1946
Q

Def: osteochondritis

A

Idiopathic condition in which bony centres of children/adolescents become temporarily softened due to osteonecrosis

Bone hardens in new deformed position

1947
Q

Radiography in osteochondritis

A

Initially: increased denisty/sclerosis

then: patchy appearance

1948
Q

Scheuermann’s disease

A

Self-limiting skeletal disorder of childhood.

Vertebrae grow unevenly in respect to the saggital plane. (posterior angle greater than anterior angle)

Uneven growth results in wedging shape of vertebrae causing kyphosis.

1949
Q
A

Scheueurmann’s disease

Autosomal dominant

Vertebral tenderness and kyphosis

XR: wedge-shaped thoracic vertebra

1950
Q

Kohler’s disease

A

Rare bone disorder of the foot found in young children (3-5).

Caused by the navicular bone temporarily losing its blood supply.

Undergoes avascular necrosis.

Pain in mid-tarsal region-> limp

1951
Q
A

Kohler’s disease

1952
Q

Kienbochs disease

A

Kienböck’s disease is a disorder of the wrist. It is named for Dr. Robert Kienböck, a radiologist in Vienna, Austria who described osteomalacia of the lunate in 1910.[1]

It is breakdown of the lunate bone, a carpal bone in the wrist that articulates with the radius in the forearm. Specifically, Kienböck’s disease is another name for avascular necrosis2 with fragmentation and collapse of the lunate. This has classically been attributed to arterial disruption, but may also occur after events that produce venous congestion with elevated interosseous pressure.

1953
Q
A

Kienboch’s disease

Lunate bone
􀁸 Adults
􀁸 Pain over lunate, esp. on active movement
􀁸 Impaired grip

1954
Q

Friedberg’s disease

A

Freiberg disease, also known as a Freiberg infraction, is a form of avascular necrosis in the metatarsal. It generally develops in the second metatarsal, but can occur in any metatarsal. Physical stress causes repeated microfractures where the middle of the metatarsal meets the growth plate. These restrict circulation to the end of the metatarsal, causing the necrosis. It is an uncommon condition, occurring most often in young women, athletes, and those with abnormally long metatarsals. Approximately 80% of those diagnosed are women.[1]

1955
Q
A

Friedberg’s disease

2nd/3rd metatarsal heads

Around puberty

Forefoot pain worse with pressure

1956
Q

Panner disease

A

Panner disease is an osteochondrosis of the capitellum of the elbow. It causes pain and stiffness in the affected elbow and may limit extension. On radiographs, the capitulum may appear irregular with areas of radioluceny. Treatment is symptomatic, with a good prognosis. It is primarily seen in children between five and ten years old. The disease is named after the Danish radiologist Hans Jessen Panner

1957
Q
A

Panner’s disease

Capitulum of humerus

1958
Q

Perthes Disease

A

Legg–Calvé–Perthes disease (LCPD) is a childhood hip disorder initiated by a disruption of blood flow to the ball of the femur called the femoral head. Due to the lack of blood flow, the bone dies (osteonecrosis or avascular necrosis) and stops growing. Over time, healing occurs by new blood vessels infiltrating the dead bone and removing the necrotic bone which leads to a loss of bone mass and a weakening of the femoral head.[1] The bone loss leads to some degree of collapse and deformity of the femoral head and sometimes secondary changes to the shape of the hip socket. It is also referred to as idiopathic avascular osteonecrosis of the capital femoral epiphysis of the femoral head since the cause of the interruption of the blood supply of the head of the femur in the hip joint is unknown.

1959
Q
A

Legg-Calve-Perthes Disease of right hip

1960
Q

Featurs of Osgood Schlatter’s

A

Tibial tuberosity apophysitis + patellar tendonitis

Seen in children 10-14y/o

M>F 3:1

Associated with physical activity

Pain below knee especially on quad contraction

1961
Q

Tibial tuberosity apophysitis + patellar tendonitis

Seen in children 10-14y/o

M>F 3:1

Associated with physical activity

Pain below knee especially on quad contraction

A

Osgood-Shlatter’s

1962
Q
A

Osgood-Shlatters

1963
Q

Tranction tendinopathy with calcification of proximal
attachment of patellar tendon
􀁸 Children 8-10yrs

A

Singind Larsen’s disease

1964
Q

Sinding Larsen’s disease

A

Tranction tendinopathy with calcification of proximal
attachment of patellar tendon
􀁸 Children 8-10yrs

1965
Q
A

Sinding Larsen’s disease

Tranction tendinopathy with calcification of proximal
attachment of patellar tendon
􀁸 Children 8-10yrs

1966
Q

Sever’s disease

A

Calcaneal apophysitis

8-13 years

Pain behind heel and limp

1967
Q

Calcaneal apophysitis

8-13 years

Pain behind heel and limp

A

Sever’s disease

1968
Q

Features of osteochondritis dissecans

A

Piece of bone overlying catrilage dissects off into the joint space

Commonly knee (medial femoral condyle), also elbow, hip and ankle

Young adult/adolescent

Pain, swelling, locking, reduced ROM

1969
Q

Piece of bone overlying catrilage dissects off into the joint space

Commonly knee (medial femoral condyle), also elbow, hip and ankle

Young adult/adolescent

Pain, swelling, locking, reduced ROM

A

Osteochondritis dissecans

1970
Q

Loose bodies, lucent crater on XR

Mx with arthroscopic removal

A

Osteochondritis Dissecans

1971
Q

Causes of avascular necrosis

A

or dislocation

SCD, thalassaemia

SLE

Gaucher’s

Drugs: steroids, NSAIDs

1972
Q

Apophysis

A

Natural swelling of bone for attachment of a muscle

1973
Q

Enthesis

A

Connective tissue between tendon or ligament and bone. There are two types of entheses: Fibrous entheses and fibrocartilaginous entheses.

1974
Q

Trimalleolar fracture

A

A trimalleolar fracture is a fracture of the ankle that involves the lateral malleolus, the medial malleolus, and the distal posterior aspect of the tibia, which can be termed the posterior malleolus. The trauma is sometimes accompanied by ligament damage and dislocation.[1]

1975
Q

Bimalleolar fracture

A

A bimalleolar fracture is a fracture of the ankle that involves the lateral malleolus and the medial malleolus. Studies have shown[1] that bimalleolar fractures are more common in women, people over 60 years of age, and patients with existing comorbidities.[1]

1976
Q

Borders of the anatomical snuffbox

A

Ulnar (medial border): tendon of extensor pollicis longus

Radial (lateral) border: tendons of abductor pollicis longus and extensor pollicis brevis

Proximal border: styloid process of the radius

Floor: carpal bones: scaphoid and trapezium

Roof: skin

1977
Q

Contents of the anatomical snuffbox

A

Radial artery

Superficial branch of radial nerve

Cephalic vein

1978
Q

Borders of the cubital fossa

A

Lateral border: medial border of brachioradialis

Medial border: lateral border of pronator teres

Superior border: line between the epicondyles of the humerus

1979
Q

Contents of the cubital fossa

Really Need Beer To Be At My Nicest

A

Lateral to medial:

Radial Nerve

Biceps Tendon

Brachial Artery

Median Nerve

1980
Q

Borders of the carpal tunnel

A

Formed by two layers:

a deep carpal arch and a superficial flexor retinaculum

Carpal arch:

Concave on palmar side

Formed laterally by the scaphoid and trapezium tubercles

Formed medially by the hook of hamate and pisiform

Flexor retinaculum:

Thick connective tissue

Turns the carpal arch into the carpal tunnel

Originates on the lateral side and inserts on the medial side of the carpal arch

1981
Q

Contents of the carpal tunnel

A

9 tendons and median nerve

1982
Q

Tendons in the carpal tunnel

A

1 FPL own sheath

4 FDP + 4 FDS shared sheath

1983
Q

Contents of carpal tunnel

A

Median nerve

1 FPL

4 FDP

4 FDS

1984
Q

Borders of the axilla

A

4 sides and a base with an opening at the apex

Apex: lateral border of first rib, superior border of scapula and posterior border of clavicle

Lateral wall: intertubercular groove of the humerus

Medial wall: serratus anterior and thoracic wall

Anterior wall: pec major and pec minor and subclavius

Posterior wall: subscap, teres major, lat dorsi

1985
Q

What are the passageways exiting the axilla

A

3 routes

Main route: into the upper limb, immediately inferiorly and laterally

Another via the quadrangular space: gap in the posterior wall of the axilla

Clavipectoral triangle: opening in the anterior wall of the axilla bounded by pectoralis major, deltoid and clavicle

1986
Q

Contents of the axilla

A

Axillary artery

Axillary vein

Brachial plexus

Biceps brachii and coarcobrachialis

Axillary lymph nodes

1987
Q

What are the four muscles in the pectoral region?

A

Pec major, minor, serratus anterior, subclavius

1988
Q

Function of pec major?

A

Adducts and medially rotates the upper limb

Draws scapula anteroinferiorly

1989
Q

Innervation of pec major

A

Lateral and medial pectoral nerves

1990
Q

Action of pec minor

A

Stabilises the scapula by drawing it anteroinferiorly against the thoracic wall

1991
Q

Innervation of pec minor

A

Medial pectoral nerve

1992
Q

Action of serratus anterior

A

Rotates the scapula allowing the arm to be raised over 90

Holds scapula against the ribcage

1993
Q

Innervation of serratus anterior

A

LTN

1994
Q

Actions of subclavius

A

Anchors and depresses the clavicle

1995
Q

Innervation of subclavius

A

Nerve to subclavius

1996
Q

What is the division of muscles of the shoulder?

A

Extrinsic: originate from the torso and attach to bones of the shoulder

Intrinsic: originate from scapula or clavicle and attach to the humerus

1997
Q

What are the intrinsic muscles of the shoulder?

A

Deltoid, teres major

Rotator cuff muscles

1998
Q

Deltoid actions

A

Anterior fibres flex arm at the shoulder

Posterior fibres extend arm at the shoulder

Middle fibres are the major abductor of the arm, taking over from supraspinatus which abducts the first 15 degrees

1999
Q

Innervation of Deltoid

A

Axillary nerve

2000
Q

Action of Teres major

A

Adducts at the shoulder and internally rotates arm

2001
Q

Innervation of teres major

A

Lower subscapular nerve

2002
Q

What muscle is innervated by the thoracodorsal nerve?

A

Lat dorsi

2003
Q

What muscles innervated by the dorsal scapular nerve?

A

Rhomboid

Levator scapulae

2004
Q

How are the extrinsic muscles of the shoulder organised?

A

Into a superficial and deep layer

2005
Q

What are the superifical extrinsic muscles of the shoulder?

A

Trapezius

Lat dorsi

2006
Q

Action of trapezius

A

Elevate scapula and rotate it during abduction of the arm.

Middle fibres retract scapula

Lower fibres pull the scapula inferiorly

2007
Q

Innervation of trapezius

A

Accessory nerve

2008
Q

Action of lat dorsi

A

Extends, adducts and medially rotates the upper limb

2009
Q

What are the deep muscles of the shoulder?

A

Levator scapulae

Rhomboids

2010
Q

What muscles are in the anterior compartment of the upper arm?

A

Biceps brachii

Brachialis

Coarcobrachialis

2011
Q

What muscles are in the posterior compartment of the upper arm?

A

triceps

2012
Q

Action of biceps brachii

A

Supination of forearm

Flexes arm at elbow and at the shoulder

2013
Q

Innervation of biceps brachii

A

Musculocutaneous nerve

2014
Q

What spinal cord segment is tested by the biceps reflex?

A

C6

2015
Q

Action of coracobrachialis

A

Flexion of the arm at the shoulder and weak adduciton

2016
Q

Innervation of coracobrachialis

A

Musculocutaneous nerve

2017
Q

Action of brachialis

A

Flexion at the elbow

2018
Q

Innervation of brachialis

A

Musculocutanous nerve

2019
Q
A
2020
Q

Popeye sign

A

Rupture of biceps tendon

2021
Q

Function of the triceps brachii

A

Extension of the arm at the elbow

2022
Q

Innervation of triceps brachii

A

Radial nerve

2023
Q

Triceps tendon tests which spinal segment?

A

C7

2024
Q

How are the muscles in the anterior compartment of the forearm arranged?

A

Superficial Intermediate Deep

2025
Q

General action of muscles in the anterior compartment of forearm?

A

Flexion at wrist and fingers, pronation

2026
Q

What are the muscles in the superifical compartment of the forearm?

A

Flexor carpi ulnaris

Palmaris Longus

Flexor carpi radialis

Pronator teres

2027
Q

Action of FCU

A

Flexion and adduciton at wrist

2028
Q

Innervation of FCU

A

Ulnar nerve

2029
Q

Action of palmaris longus

A

Flexion at wrist

NB this muscle is absent in 15% of the population

2030
Q

Innervation of PL

A

Median nerve

2031
Q

Action of FCR

A

Flexion and adduction at the wrist

2032
Q

Innervation of FCR

A

Median nerve

2033
Q

Action of pronator teres

A

Pronation of the forearm

2034
Q

Innervation of pronator teres

A

Median nerve

2035
Q
A
2036
Q

Muscles in the intermediate compartment of the forearm

A

Flexor digitorum superficialis

2037
Q

Action of flexor digitorum superficialis

A

Flexes the metacarpophalangeal joints and PIP at the 4 fingers and flexes the wrist

2038
Q

Muscles in the deep compartment of the anterior forearm

A

FDP

FPL

Pronator quadratus

2039
Q

Action of FDL

A

Flex the DIP of the fingers (only)

2040
Q

Innervation of FDL

A

Medial half is innervated by the ulnar nerve

Lateral half innervated by median nerve

2041
Q

Action of FPL

A

Flexes the interphalangeal joint and MCP of the thumb

2042
Q

Innervation of FPL

A

Median nerve (anterior interosseus branch)

2043
Q

Action of pronator quadratus

A

Pronates the forearm

2044
Q

Innervation of pronator quadratus

A

Median nerve (ant interosseus)

2045
Q

How are the muscles in the posterior compartment of the forearm arranged?

A

Deep and superficial

2046
Q

What are the superficial muscles of the posterior forearm

A

Extensor carpi radialis brevis and longus,

extensor digitorum,

extensor carpi ulnaris,

extensor digitis minimi,

brachioradialis,

anconeus

2047
Q

Action of brachioradialis

A

Flexes forearm

2048
Q

Innervation of muscles in posterior compartment of forearm

A

Radial nerve

2049
Q

What are the deep muscles of the posterior compartment of the forearm?

A

Supinator

APL

EPB

EPL

Extensor indices

2050
Q

Action of supinator

A

Supinates the forearm

2051
Q

How can muscles of the hand be classified?

A

The extrinsic muscles are located in the anterior and posterior compartments of the forearm. They control crude movements and produce a forceful grip.

The intrinsic muscles of the hand are located within the hand itself. They are responsible for the fine motor functions of the hand.

2052
Q

What is the innervation of the thenar muscles?

A

Median nerve

2053
Q

What are the thenar muscles?

A

Opponens polllicis

Abductor Pollicis brevis

Flexor pollicis brevis

2054
Q

Innervation of the hypothenar muscles?

A

Ulnar nerve

2055
Q

What are the hypothenar muscles?

A

Opponens digiti minimi

Abductor digiti minimi

Flexor digiti minimi brevis

2056
Q

Action of the lumbricals

A

Flex at the MCP joint and extend at the IP joints

2057
Q

Innervation of the lumbricals

A

Medial two: ulnar nerve

Lateral two: median nerve

2058
Q

Action of dorsal interossei?

A

Abduct the fingers at the MCP

2059
Q

Innervation of the interossei?

A

Ulnar nerve

2060
Q

Action of the palmar interossei

A

Adducts the fingers at the MCP

2061
Q

What are the two muscles not found in the hypothenar or thenar compartments and are not lumbricals or interossei?

A

Palmaris brevis

Adductor pollicis

Both innnervated by ulnar nerve

2062
Q

Action of palmaris brevis

A

Wrinkles the skin of the hypothenar eminence and deepens the curvature of the hand

2063
Q

Actions of adductor pollicis

A

Adductor of the thumb

2064
Q

Causes of a limping child

A

DDH

Transient synovitis

Septic arthritis

Perthes

SUFE

JIA/Still’s Disease

2065
Q

Def: DDH

A

Congenital hip deformity in which the femoral head is or can be completely/partially displaced

2066
Q

Predisposing factors to DDH

A

Fhx

Breech presentation

Oligohydramnios

2067
Q

Screening

Asymmetric skin folds

Limp/abnormal gait

A

DDH

2068
Q

Ix in DDH

A

USS

2069
Q

Mx of DDH

A

Maintain abduction

Double nappies

Pavlik harness

Plaster hip spica

Open reduction: derotation varus osteotomy

2070
Q

What is the commonest cause of acute hip pain in children?

A

Transient synovitis

2071
Q

2-12 years

Sudden onset hip pain/limp

Often following or with viral infection

Not systemically unwell

A

Transient Synovitis

2072
Q

Ix in transient synovitis

A

PMN and ESR/CRP normal

-ve blood cultures

May need joint aspiraiton and culture

2073
Q

Mx of transient synovitis

A

Rest and analgesia

Settles over 2-3d

2074
Q

Def: Perthes disease

A

Osteochondritis of the femoral head 2o to AVN

2075
Q

Insidiuous onset

Hip pain initially then painless

10-20% bilateral

A

Perthes’ disease

2076
Q

Ix in Perthes’ disease

A

XR normal intially increased density of femoral head

Femoral head becomes fragmented and irregular

Flattening and sclerosis is seen

2077
Q

Mx of Perthes disease

A

Detected early and < half femoral head affected- bed rest and traction

More severe: maintain hip in abduction with plaster

Femoral or pelvic osteotomy

2078
Q

Def: SUFE

A

Postero-inferior displacement of femoral head epiphysis

10-15 years

Fat and sexually undeveloped tall annd thin

2079
Q

Groin pain

Shortened, externally rotated leg

All movements painful

Can also be chronic or acute on chronic

Fat and sexually underdeveloped or tall and thin

20% bilateral

A

SUFE

2080
Q

Mx of acute SUFE

A

Reduce and pin ephiphysis

2081
Q

Mx of chronic SUFE

A

In situ pinning as epihpyseal reduction risks AVN

2082
Q

Complications of SUFE

A

Chondrolysis: breakdown of articular cartilage

Increased risk with surgery

2083
Q

Organisms in acute osteomyelitis

A

Staph

Strep

E Coli

Pseudomonas

Salmonella (SCD)

2084
Q

What bacteria associated with acute osteomyelitis in sickle cell diseaes?

A

Salmonella

2085
Q

RFs for acute osteomyelitis

A

Vascular disease

Trauma

SCD

Immunosuppression e.g. DM

Children: rich blood supply to growth plate therefore usually affects the metaphysis

2086
Q

Pain tenderness, erythema, warmth, reduced ROM

Effusion in neighbouring joints

Signs of systemic infection

A

Acute OM

2087
Q

Ix in osteomyelitis

A

Raised ESR/CRP

Raised WCC

+ve blood cultures in 60%

XR changes take 10-14d

MRI is sensitive and specific

2088
Q

XR changes in osteomyelitis

A

Changes take 10-14d

Haziness and reduecd bone density

Sub-periosteal reaction

Sequestrum and involucrum

2089
Q

Mx of osteomyelitis

A

IV Abx: vanc and cefotaxime until MCS known

Drain abscess and remove sequestra

Analgesia

2090
Q

What are the commonest bone tumours?

A

Bony mets

2091
Q

Features of bone mets

A

Usually radiolucent except prostate is sclerotic

Usually axial skeleton (contains red marrow)

Present with pain or pathological fracture

2092
Q

What malignancy metastasise to bone

LTBKP

A

Lung

Thyroid

Breast

Kidney

Prostate

2093
Q

Ground glass lytic lesion

Shepherds crook deformity of proximal femur

Usually long bones

Ribs skull

Mono-ostotic

A

Fibrous dysplasia

2094
Q

Polyostotic fibrous dysplasia

Precocious puberty (females)

Cafe au lait spots

A

McCune Albright’s

2095
Q

Clinical features of fibrous dysplasia

A

0-30

F>M

Increased fracture risk

Long bones, rib, skull

Mono-ostotic

2096
Q

Well defined lytic lesion in proximal metaphysis of humerus or femur

Cortex can flll into cyst- fallen fragment

<20y/o

Lump migrates down shaft from original bone plate site

A

Simple bone cyst

2097
Q
A

Fibrous dysplasia

Ground glass appearance

2098
Q
A

Simple bone cyst

Well-defined lytic lesion

2099
Q

What are the tumour like non-neoplastic conditions of bone?

A

Fibrous dysplasia

Simple bone cyst

Aneurysmal bone cyst

Fibrous cortical defect and non-ossifying fibroma

2100
Q

Bone

<30y

Painful

MRI showing multiple fluid levels

A

Aneurysmal bone cyst

2101
Q

Children, not painful, benign lesion

Spontaneously regresses

Seen in distal femur

Proximal tibia

Often multiple

A

Fibrous cortical defect

2102
Q
A

Fibrous cortical defect

2103
Q

What are the benign catilaginous neoplasms?

A

Osteochondroma

Enchondroma/Chondromas

Chondroblastoma

2104
Q

Cartilage capped bony outgrowth

Knee

Commonest benign bone tumour

May be related to previous bone trauma

A

Osteochondroma (exostosis)

2105
Q

Oval lucencies with radiodense rim

Endosteal scalloping

Hands and feet

A

Enchondromas/chondromas

2106
Q

Ollier disease

A

Multiple enchondromas

2107
Q

Maffuci syndrome

A

Enchondromatosis and multiple soft tissue haemangiomas

2108
Q

Radiolucent with sclerotic border

Epiphysis

Knee

A

Chondroblastoma

2109
Q
A

Osteochondroma

2110
Q
A

Enchondroma

2111
Q
A

Chondroblastoma

2112
Q

What is the significance of having mutliple enchondromas?

A

Have risk of malignant transformation

2113
Q
A

Ollier disease

2114
Q
A

Marfucci syndrome

2115
Q

>40

Pain and lump

Arising de novo or from chondromas

Located in pelvis or axial skeleton

Lytic lesion

Fluffy popcorn calcification

A

Chondrosarcoma

2116
Q
A

Chondrosarcoma

2117
Q

What are the malignant cartilaginous neoplasms?

A

Chondrosarcoma

2118
Q

What are the benign bone-forming neoplasms

A

Osteoma

Osteoid osteoma

Osteoblastoma

Giant Cell tumour/osteoclastoma

2119
Q

Lump, usually solitary

In skull and faical bones

A

Osteoma

2120
Q
A

Osteoma

2121
Q

Multiple osteomas

A

Gardner syndrome

2122
Q
A

Gardner syndrome

2123
Q

M>F 2:1

Teens and 20s

Severe nocturnal pain relieved by aspirin

Hot on bone scan

Lower limb/diaphyseal cortex

Lytic lesion with central nidus and sclerotic rim

A

Osteoid osteoma

2124
Q
A

Osteoid osteoma

2125
Q

Similar to osteoid osteoma but pain unresponsive to aspirin

Spine

A

Osteoblastoma

2126
Q

20-40

After fusion of growth plate

Knee

Abutting joint surface

Soap bubble appearance

Solitary, expansile, lytic lesion

A

Giant Cell Tumour/osteocalstoma

2127
Q
A

Osteoclastoma (Giant cell tumour)

2128
Q

What are the malignant bone-forming neoplasms?

A

Osteosarcoma

Ewing’s Sarcoma

2129
Q

Adolescents

M>F

Commonest 1o bone tumour

Pain, warm, bruit

May arise 2o to Paget’s

Knee

Metaphysis

Periosteal elevation:

Sunburst appearance

Codman’s triangle

A

Osteosarcoma

2130
Q
A

Osteosarcoma (Codman triangle)

2131
Q

Painful, warm, enlarging mass

Systemic: fever, raised ESR, anaemia, raised WCC

<20

Long bone diaphysis

Lytic tumour

Onion-skin periosteal reaction

A

Ewing’s sarcoma

2132
Q
A

Ewing’s Sarcoma

2133
Q

Where do the roots of the roots of the brachial pleuxus leave the vertebral column?

A

Between scalenus anterior and medius

2134
Q

Where do the divisions of the brachial plexus occur?

A

Under the clavicle, medial to the coracoid process

2135
Q

Causes of brachial plexus injury

A

Direct e.g. shoulder girdle #, penetrating or iatrogenic injury.

Indirect: e.g. avulsion or traction injuries

2136
Q

What can be used to classify brachial plexus injuries?

A

Leffert Classification

2137
Q

What are the components of the Leffert classificaiton

A
  1. Open
  2. Closed:

a supraclavicular

b infraclavicular

  1. Radiation induced
  2. Obstetric

a upper

b lower

c mixed

2138
Q

Abductors and external rotators paralysed

Waiter’s tip

Loss of sensation in C5/C6

A

Erb’s palsy (high brachial plexus injury)

2139
Q

Paralysis of small hand muscles

Claw hand

Loss of sensation in C8/T1 dermatomes

A

Klumpke’s paralysis

2140
Q

Features of low radial nerve injury

A

Posterior interosseus nerve affected

Site: #around elbow or forearm e.g. head of radius

Loss of extension of CMC (finger drop)

No sensory loss

2141
Q

Features of high radial nerve injury

A

Site: #shaft of humerus where N is in radial groove

Wrist drop

Loss of sensation to dorsum of thumb

Triceps functions nromally

2142
Q

Wrist drop

Loss of sensation to dorsum of thumb

Triceps functions normally

A

High radial nerve injury

2143
Q

Features of very high radial nerve lesions

A

Axilla e.g. crutches or saturday night palsy

Paralysis of triceps and wrist drop

2144
Q

Site of ulnar nerve lesions

A

Elbow: cubital tunnel

Wrist: Guyon’s canal

2145
Q

What is Guyon’s canal?

A

The ulnar canal or ulnar tunnel (also known as Guyon’s canal or tunnel) is a semi-rigid longitudinal canal in the wrist that allows passage of the ulnar artery and ulnar nerve into the hand

2146
Q

Intrinsic hand muscle paralysis-> claw hand

Lesion at elbow has less clawing as FDP is paralysed, decreasing flexion of 4th and 5th digits

Sensory loss over little finger

A

Ulnar nerve lesion

2147
Q

Tests for ulnar nerve palsy

A

Can’t cross fingers for luck

Froment’s sign

2148
Q

Features of injury to median nerve above the antecubital fossa

A

Can’t flex index finger IPJs

Can’t flex terminal thumb phalanx

Loss of sensation in median distribution

2149
Q

median nerve injury at the wrist

A

Typically affects abductor pollicis brevis

2150
Q

Nerve damage mechanism

Accessory

A

Posterior triangle LN biopsy

2151
Q

Nerve damage mechanism

Sciatic

A

Posterior approach to hip

2152
Q

Nerve damage mechanism

Common peroneal

A

Legs in Lloyd davies position

2153
Q

What is Lloyd davies position?

A

Lloyd-Davies position is a medical term referring to a common position for surgical procedures involving the pelvis and lower abdomen. The majority of colorectal and pelvic surgery is conducted with the patient in the Lloyd-Davies position. It is also known as the Trendelenburg position with legs apart.

2154
Q

Nerve damage mechanism

Long thoracic

A

Axillary node clearance

2155
Q

Nerve damage mechanism

Pelvic autonomic nerves

A

Pelvic cancer surgery

2156
Q

Nerve damage mechanism

Recurrent laryngeal nerves

A

During thyroid surgery

2157
Q

Nerve damage mechanism

Hypoglossal nerve

A

During carotid endaterectomy

2158
Q

Nerve damage mechanism

Ulnar and median nerves

A

During upper limb fracture repairs

2159
Q

Damage to structure mechanism

Thoracic duct

A

During thoracic surgery e.g. pneumonectomy, oesophagectomy

2160
Q

Damage to structure mechanism

Parathyroids

A

During difficult thyroid surgery

2161
Q

Damage to structure mechanism

Ureters

A

During colonic resections/gynaecological surgery

2162
Q

Damage to structure mechanism

Bowel perforation

A

Use of verres needle to establish pneumoperitoneum

2163
Q

What is verres needle?

A

A Veress needle is a spring-loaded needle used to create pneumoperitoneum for laparoscopic surgery. Of the three general approaches to laparoscopic access, the Veress needle technique is the oldest and most traditional.

2164
Q

Damage to structure mecahnism

Bile duct injury

A

Failure to delineate Calot’s triangle carefully and careless use of diathermy

2165
Q

What is Calot’s triangle?

A

Hepatobiliary triangle bordered by the cystic duct inferiorly, common hepatic duct medially and the inferior surface of the liver superiorly.

Cystic artery lies within the triangle which is used to locate it during a laparoscopic cholecystectomy

2166
Q

Damage to structure mecahnism

Facial nerve

A

Always at risk during parotidectomy

2167
Q

Damage to structure mecahnism

Tail of pancreas

A

When ligating splenic hilum

2168
Q

Damage to structure mecahnism

Testicular vessels

A

During re-do open hernia surgery

2169
Q

Damage to structure mecahnism

Hepatic veins

A

During liver mobilisation

2170
Q

Physiological disturbance mechanism

Arrythmias following cardiac surgery

A

Susceptibility to hypokalaemia (<4 in cardiac patients)

2171
Q

Physiological disturbance mechanism

Neurosurgical electrolyte disturbance

A

SIADH following cranial surgery causing hyponatraemia

2172
Q

Physiological disturbance mechanism

Ileus following GI sx

A

Fluid sequestration and loss of electrolytes

2173
Q

Physiological disturbance mechanism

Pulmonary oedema following pneumonectomy

A

Loss of lung volume makes patients very sensitive to fluid overload

2174
Q

Physiological disturbance mechanism

Anastomotic leak

A

Generalised sepsis causing mediastinitis or peritonitis depending on site of leak

2175
Q

Physiological disturbance mechanism

MI

A

May follow any type of sx and in addition to direct cardiac effects, the decreased CO may compromsie grafts etc.

2176
Q

Identifying surgical complications:

Intra-abdominal abscess/air/ anastomotic leak

A

CT scanning
+luminal contrast

2177
Q

Identifying surgical complications:

Rectal anastomotic leak

A

Gastrograffin enema

2178
Q

Identifying surgical complications:

Ureteric injury

Pancreatic injury

A

U+E

Amylase

2179
Q

Identifying surgical complications:

Pericardial effusion

A

Echocardiogram if no pleural window made

2180
Q

A 53-year-old gentleman is 1 week post right-hemicolectomy for colorectal cancer and formation of ileostomy. He complains of intermittent shortness of breath and an arterial blood gas sample was taken. The results showed the following;

Normal range

pH: 7.25(7.35 - 7.45)

pO2: 11.1(10 - 14)kPa

pCO2: 3.2(4.5 - 6.0)kPa

HCO3: 11(22 - 26)mmol/l

BE: -15(-2 to +2)mmol/l

Normal range

Na: 110135-145 mmol/l

K: 33.5-5 mmol/l

Given the above information, what should form part of the most likely differential diagnosis for this gentleman?

Loss from high output stoma post-operatively

Uraemia

Lactic acidosis

Hospital acquired pneumonia

Pulmonary embolism

A

When considering acid-base disorders in post-operative patients, it is important to consider the potential side effects related to the specific procedure. In this case, this gentleman has an ileostomy for a stoma bag to drain bowel contents post-operatively. These patients may develop significant volume depletion, electrolyte and acid-base disturbances (metabolic acidosis) if the ileostomy output increases or if dietary intake is disrupted or altered. Hence, it is important to monitor fluid balance including stoma output in these patients.

2181
Q

What are the considerations in selecting type of resection for rectal cancer?

A

Tumours located in the rectum can be surgically resected with either an anterior resection or an abdomino-perineal excision of rectum (APER). The technical aspects governing the choice between these two procedures can be complex to appreciate and the main point to appreciate for the exam is that involvement of the sphincter complex or very low tumours require APER. In the rectum a 2cm distal clearance margin is required and this may also impact on the procedure chosen. In addition to excision of the rectal tube an integral part of the procedure is a meticulous dissection of the mesorectal fat and lymph nodes (total mesorectal excision/ TME).

2182
Q

Why is radiotherapy an option in rectal carcinoma?

A

In rectal cancer surgery involvement of the cirumferential resection margin carries a high risk of disease recurrence. Because the rectum is an extraperitoneal structure (until you remove it that is!) it is possible to irradiate it, something which cannot be offered for colonic tumours. This has a major impact in rectal cancer treatment and many patients will be offered neoadjuvent radiotherapy (both long and short course) prior to resectional surgery. Patients with T1 and 2 /N0 disease on imaging do not require irradiation and should proceed straight to surgery. Patients with T4 disease will typically have long course chemo radiotherapy. Those with T3 , N0 tumours may be offered short course radiotherapy prior to surgery. Patients presenting with large bowel obstruction from rectal cancer should not undergo resectional surgery without staging as primary treatment (very different from colonic cancer). This is because rectal surgery is more technically demanding, the anastomotic leak rate is higher and the danger of a positive resection margin in an unstaged patient is high. Therefore patients with obstructing rectal cancer should have a defunctioning loop colostomy.

2183
Q

Site of cancer, type of resection, type of anastomosis

Caecal, ascending or proximal transverse colon

A

Right hemicolectomy

Ileo-colic

2184
Q

Site of cancer, type of resection, type of anastomosis

Distal transverse, descending colon

A

Left hemicolectomy

Colo-colon

2185
Q

Site of cancer, type of resection, type of anastomosis

Sigmoid colon

A

High anterior resection

Colo-rectal

2186
Q

Site of cancer, type of resection, type of anastomosis

Upper rectum

A

Anterior resection (TME)

Colo-rectal

2187
Q

Site of cancer, type of resection, type of anastomosis

Low rectum

A

Anterior resection (Low TME)

Colo-rectal +/- defunctioning stoma

2188
Q

Site of cancer, type of resection, type of anastomosis

Anal verge

A

APER

None

2189
Q

What is the choice of surgery in an emergency setting where the bowel has perforated

A

In the emergency setting where the bowel has perforated the risk of an anastomosis is much greater, particularly when the anastomosis is colon-colon. In this situation an end colostomy is often safer and can be reversed later. When resection of the sigmoid colon is performed and an end colostomy is fashioned the operation is referred to as a Hartmans procedure. Whilst left sided resections are more risky, ileo-colic anastomoses are relatively safe even in the emergency setting and do not need to be defunctioned.

2190
Q

Colorectal screening programme features

A

60-74 years

Every 2 years to all men and women

2191
Q

Impact of colorectal screening programme on mortality?

A

16% reduction

2192
Q

Colonoscopy following abnormal FOB, results

A

5/10 will have normal exam

4/10 will have polyps which may be removed due to premalignant potential

1/10 will have cancer

2193
Q

Theme: Abdominal stomas

A.End ileostomy

B.End colostomy

C.Loop ileostomy

D.Loop colostomy

E.End jejunostomy

F.Loop jejunostomy

G.Caecostomy

A 56-year-old man is undergoing a low anterior resection for carcinoma of the rectum. It is planned to restore intestinal continuity.

A 23-year-old man with uncontrolled ulcerative colitis is undergoing an emergency sub total colectomy.

A 63-year-old women presents with large bowel obstruction. On examination she has a carcinoma 10cm from the anal verge.

A

The correct answer is Loop ileostomy

Colonic resections with an anastomosis below the peritoneal reflection may have an anastomotic leak rate (both clinical and radiological) of up to 15%. Therefore most surgeons will defunction such an anastomosis to reduce the clinical severity of an anastomotic leak. A loop ileostomy will achieve this end point and is relatively easy to reverse.

End ileostomy

Following a sub total colectomy the immediate surgical options include an end ileostomy or ileorectal anastomosis. In the emergency setting an ileorectal anastomosis would be unsafe.

The correct answer is Loop colostomy

Large bowel obstruction resulting from carcinoma should be resected, stented or defunctioned. The first two options typically apply to tumours above the peritoneal reflection. Lower tumours should be defunctioned with a loop colostomy and then formal staging undertaken prior to definitive surgery. An emergency attempted rectal resection carries a high risk of involvement of the circumferential resection margin and is not recommended

2194
Q

Def: stoma

A

Bringing of lumen or visceral contents onto the skin

With bowel stomas, the method and site will be determined by the bowel affected.

In practice small bowel stomas should be spouted so that their irritant contents are not in contact with the skin

Colonic stomas do not need to be spouted

2195
Q

Types and features of stoma

Gastrotomy

A

Gastric decompression or fixation

Feeding

Commonly sited in the epigastrium

2196
Q

Types and features of stoma

Loop jejunostomy

A

Seldom used as very high output

May be used following emergency laparotomy with planned early closure

Any location according to need

2197
Q

Types and features of stoma

Percutaneous jejunostomy

A

Usually performed for feeding purposes and sited in the proximal bowel

Usually LUQ

2198
Q

Types and features of stoma

Loop ileostomy

A

Defunctioning of colon e.g. following rectal cancer surgery

Does not decompress colon if ileocaecal valve competent

Usually RIF

2199
Q

Types and features of stoma

End ileostomy

A

Usually folllowing complete excision of colon or where ileo-colic anastomosis is not planned

May be used to defunciton colon but reversal is more difficult

Usually RIF

2200
Q

Types and features of stoma

End colostomy

A

Where a colon is diverted or resected and anastomosis is not primarly achievable or desirable

Either L or RIF

2201
Q

Types and features of stoma

Loop colostomy

A

To defunction a distal segment of colon

Since both lumens are present, the distal lumen acts as a vent

May be located in any region of the abdomen depending on colonic segment used

2202
Q

Types and features of stoma

Caecostomy

A

Stoma of last resort where loop colostomy is not possible

RIF

2203
Q

Types and features of stoma

Mucous fistula

A

To decompress a distal segment of bowel following colonic diversion or resection

Where closure of a distal resection margin is not safe or achievable

May be located in any region of the abdomen according to clinical need

2204
Q

A 50-year-old man is admitted to hospital with a 5 day history of worsening abdominal pain. Over the past 24 hours the pain has become very severe. On examination his pulse is 110/min, blood pressure 110/62 mmHg and temperature 37.7ºC. Bloods show the following:

Hb10.5 g/l

Platelets452 * 109/l

WBC16.3 * 109/l

CRP263 mg/l

The abdominal film is shown below:

What is the most likely underlying disorder?

Perforated diverticulum

Sigmoid volvulus

Ulcerative colitis

Spontaneous bacterial peritonitis

Crohn’s disease

A

This patient had toxic megacolon secondary to underlying ulcerative colitis - note the dilated transverse colon. The abdomen demonstrates a markedly dilated transverse colon (9 cm) with impression of slight dilatation of the descending colon with some ‘thumb printing’ in the wall. No free subphrenic gas is seen.

They went on to have a subtotal colectomy

2205
Q

A 54 year old man is referred to clinic with change in bowel habit, blood in his stools, lethargy and weight-loss. A colonoscopy is ordered which shows a high rectosigmoid mass. Which operation would be most appropriate?

Pancolectomy

Abdominoperineal resection

Left hemicolectomy

Posterior resection

Anterior resection

A

This man’s tumour is in the rectum and sigmoid colon. Therefore, removing only the colon (left hemicolectomy or pancolectomy) is not a valid management option.

For tumours that are in the distal 8cm of the rectum, an abdominoperoneal resection is the option of choice. In this procedure, the anus, rectum and distal sigmoid are removed and the remaining sigmoid is brought out to the surface as a permanent colostomy.

For tumours in the proximal part of the rectum (as in this case), an anterior resection is performed and after removal of the tumour, the remaining sigmoid is anastomosed to the lower rectum. The approach is anterior (through the abdominal wall), giving the procedure its name. Once upon a time the procedure was done with a posterior approach (posterior resection), but this has fallen out of favour.

2206
Q

A 58-year-old gentlemen presents to your clinic complaining of 4 weeks of altered bowel habit, with some per rectal bleeding which is mixed in with his stool, he also complains of tenesmus following defecation and has lost 6 kilos of weight in the last 8 weeks. You decide to do a PR examination. You feel a mass on the posterior wall of the rectum around 10 cm from the anal verge, it is 9cm in diameter and feels irregular.

You are highly concerned that this may be a rectal cancer and order an urgent suspected cancer review and urgent colonoscopy. If your suspicions are correct what is the most likely diagnosis?

Squamous cell carcinoma

Gastrointestinal stromal tumour

Secondary lymphoma

Adenocarcinoma

Carcinoid tumour

A

More than 90% of colorectal cancers are adenocarcinomas

2207
Q

Sigmoid volvulus associations

A

older patients

chronic constipation

Chagas disease

neurological conditions e.g. Parkinson’s disease, Duchenne muscular dystrophy

psychiatric conditions e.g. schizophrenia

2208
Q

Caecal volvulus associations

A

all ages

adhesions

pregnancy

2209
Q

Mx of sigmoid volvulus

A

Rigid sigmoidoscopy with rectal tube insertion

2210
Q

Mx of caecal volvulus

A

Usually operative

R hemicolectomy often needed

2211
Q

Relative proportions of different types of volvulus

A

Simoid (80%)

Caecal (20%)

2212
Q

A 59-year-old woman is admitted to hospital after developing an infection of a non-functioning left kidney, Intravenous antibiotics are given and a nephrostomy tube is inserted. During her recovery she starts to develop a distended abdomen and complains of nausea and abdominal pain.

An abdominal film is requested:

© Image used on license from Radiopaedia

What is the cause of the abdominal distension?

Bowel perforation

Intussusception

Bacterial peritonitis

Caecal volvulus

Subcutaneous emphysema

A

Small bowel obstruction is clearly visible on this film (note the valvulae conniventes, mucosal folds, that cross the full width of the bowel) secondary to caecal volvulus. Note the left nephrostomy tube in-situ.

2213
Q

Theme: Colonic resections

A.End ileostomy

B.Loop ileostomy

C.Ileo anal pouch

D.Loop colostomy

E.Pan proctocelectomy

F.Left hemicolectomy

G.Right hemicolectomy

H.Hartmann’s procedurce

I.Anterior resection with covering loop ileostomy

A 75-year-old man requires resection of an obstructing carcinoma of the splenic flexure.

A patient presenting with a large bowel obstruction from a low rectal cancer.

A 45-year-old man presents with a carcinoma 10cm from the anal verge, he has completed a long course of chemoradiotherapy and has achieved downstaging with no evidence of threatened circumferential margin on MRI scanning.

A

Carcinoma of the splenic flexure requires a left hemicolectomy.

This patient should be defunctioned-definitive surgery should wait until staging is completed. A loop ileostomy will not satisfactorily decompress an acutely obstructed colon. Low rectal cancers that are obstructed should not usually be primarily resected. The obstructed colon that would be used for anastomosis would carry a high risk of anastomotic dehisence. In addition, as this is an emergency presentation, staging may not be completed, an attempted resection may therefore compromise the circumferential resection margin, with an associated risk of local recurrence

Low rectal cancer is usually treated with a low anterior resection. Contraindications to this include involvement of the sphincters (unlikely here) and poor sphincter function that would lead to unsatisfactory function post resection. Most colorectal surgeons defunction resections below the peritoneal reflection as they have an intrinsically high risk of anastomotic leak. A loop ileostomy provides a safe an satisfactory method of defunctioning these patients. A contrast enema should be performed prior to stoma reversal.

2214
Q

A 72-year-old male is recovering from a partial colectomy that he had 3 days ago. The patient complains of worsening pain at the wound site. On closer examination there is pink serous discharge, separation of the wound edges and bowel can be seen protruding. The patient has no other obvious symptoms. How should this patient immediately be managed?

Cover the area with dry gauze and apply pressure

Pack the wound and begin intravenous fluids

Start sepsis six protocol

Non-urgent senior review

Call for senior help urgently

A

Wound dehiscence is a post-operative complication in which a wound ruptures along the surgical incision site. In this case, deep dehiscence has occurred as bowel can be seen protruding. This is an emergency and senior help should be called for immediately.

Non-urgent senior review should be considered for superficial dehiscence.

Applying pressure with dry gauze is inappropriate immediate management for this patient. However, a large sterile swab soaked in 0.9% saline can be used while waiting for senior help to arrived.

Packing the wound can be considered for superficial dehiscence but is an inappropriate immediate management for this patient.

Sepsis six protocol is a possibility and the patient’s vital signs should be recorded after senior help has been called for.

2215
Q

What are the NICE criteria for urgent referral to CRC services

A

>40 with unexplained weight loss and abdo pain

>50 with unexplained rectal bleeding

>60 with IDA or change in bowel habit

Tests show occult blood in faeces

2216
Q

Who should FOBT be offered to?

A

>50 with unexplained abdo pain or weight loss

<60 with change in bowel habit or IDA

>60 who have anaemia even in absence of IDA

2217
Q

When should an urgent CRC referral be considered?

A

Rectal or abdo mass

UNexplained anal mass or ulceration

<50 with rectal bleeding and any of:

abdo pain

change in bowel habit

weight loss

IDA

2218
Q

The most common type of inherited colorectal cancer:

Familial adenomatous polyposis

Li-Fraumeni syndrome

Hereditary non-polyposis colorectal carcinoma

Fanconi syndrome

Peutz-Jeghers syndrome

A

HNPCC

2219
Q

What are the 3 types of colon cancer?

A

sporadic (95%)

hereditary non-polyposis colorectal carcinoma (HNPCC, 5%)

familial adenomatous polyposis (FAP, <1%)

2220
Q

HNPCC features

A

HNPCC, an autosomal dominant condition, is the most common form of inherited colon cancer. Around 90% of patients develop cancers, often of the proximal colon, which are usually poorly differentiated and highly aggressive. Currently seven mutations have been identified, which affect genes involved in DNA mismatch repair leading to microsatellite instability. The most common genes involved are:

MSH2 (60% of cases)

MLH1 (30%)

2221
Q

What are the Amsterdam criteria for HNPCC?

A

at least 3 family members with colon cancer

the cases span at least two generations

at least one case diagnosed before the age of 50 years

2222
Q

FAP mutation

A

delta FAP

2223
Q

A 65-year-old female is admitted to your surgical ward for an elective hemicolectomy tomorrow due to Duke’s B colonic cancer. You are carrying out her admission assessment and notice her full blood count (FBC) demonstrates a microcytic anaemia with haemoglobin of 58 g/L. Her previous FBC 3 months earlier showed Hb 88 g/L. Haematinic blood tests show that the cause of the microcytosis is iron deficiency.

What is the most appropriate management of her anaemia?

Iron transfusion (e.g. Ferinject)

Pre-operative blood transfusion

Oral iron supplementation (e.g. ferrous sulphate)

Post-operative blood transfusion

No need to correct anaemia as removal of tumour will resolve iron loss

A

A haemoglobin of 58 g/L will need to be corrected prior to surgery and this will only be achieved in such a short time frame by arranging a blood transfusion. Iron transfusions or oral iron supplements would be recommended over a longer time frame of weeks to months had this been detected earlier.

2224
Q

Special preparation for surgery

Thyroid

A

Vocal cord check

2225
Q

Special preparation for surgery

Parathyroid

A

Consider methylene blue to identiffy gland

2226
Q

Special preparation for surgery

Sentinal node biopsy

A

Radioactive marker/patent blue dye

2227
Q

Special preparation for surgery

Surgery involving thoracic duct

A

Several methods have been described that vary from a high fat diet the evening before that includes heavy cream or ice cream.8 and 9 Some have advocated for enteral fat to continue during the intraoperative period via a nasogastric tube, nasoduodenal feeding tube, gastrostomy tube, or feeding jejunostomy.8 and 9 This will create a high-volume flow of chyle and allow the surgeon to identify the thoracic duct clearly.

2228
Q

Special preparation for surgery:

Phaeo

A

Will require alpha and beta blockade

2229
Q

Specail preparation for surgery:

Carcinoid tumours

A

Will require covering with octreotide

2230
Q

Specail preparation for surgery:

Colorectal cases

A

Bowel preparation

2231
Q

Special preparation for surgery:

Thyrotoxicosis

A

Lugols iodine/ medical therapy

2232
Q

What are the average proportions of colorectal cancer location?

A

Location of cancer (averages)

rectal: 40%
sigmoid: 30%

descending colon: 5%

transverse colon: 10%

ascending colon and caecum: 15%

2233
Q

CRC in terms of other cancers?

A

3rd most common in UK

Second most common cause of cancer deaths

2234
Q

Duke’s classification:

A

A

Tumour confined to mucosa (90%)

2235
Q

Duke’s classification:

Duke B

A

Tumour invading bowel wall (70% 5y survival)

2236
Q

Duke’s classification:

C

A

LN metastases (45%)

2237
Q

Duke’s classification:

Dukes D

A

Distant mets (6%)

2238
Q

What is the most common extra-intestinal feature in both CD and UC?

A

Arthritis

2239
Q

In which IBD is episcelritis more common?

A

CD

2240
Q

In which IBD is uveitis more common?

A

UC

2241
Q

What are the extraintestinal manifestations common to both Crohn’s and UC that are related to disease activity?

A

Arthritis: pauciarticular, asymmetric

Erythema nodosum

Episcleritis

Osteoporosis

2242
Q

What are the extraintestinal manifestation of Crohn’s and UC common to both unrelated to disease activity

A

Polyarticular symmetric arthritis

Uveitis

Pyoderma gangrenosum

Clubbing

PSC

2243
Q

What is Frimann Dahl’s sign?

A

three dense lines converging towards the site of obstruction (Frimann Dahl’s sign) in keeping with sigmoid volvulus.

2244
Q

TNM classification in CRC

T

A

Tx Primary tumour cannot be assessed

T0 No evidence of primary tumour

Tis Carcinoma in situ: intraepithelial or invasion of lamina propria

T1 Tumour invades submucosa

T2 Tumour invades muscularis propria

T3 Tumour invades through the muscularis propria into the pericolorectal tissues

T4a Tumour penetrates to the surface of the visceral peritoneum

T4b Tumour directly invades or is adherent to other organs or structures

2245
Q

TNM classification in colon cancer

Regional lymph nodes (N)

A

NX Regional lymph nodes cannot be assessed

N0 No regional lymph node metastasis

N1 Metastasis in 1-3 regional lymph nodes

N1a Metastasis in 1 regional lymph node

N1b Metastasis in 2-3 regional lymph nodes

N1c Tumour deposit(s) in the subserosa, mesentery, or non-peritonealised pericolic or perirectal tissues without regional nodal metastasis

N2 Metastasis in 4 or more lymph nodes

N2a Metastasis in 4-6 regional lymph nodes

N2b Metastasis in 7 or more regional lymph nodes

2246
Q

TNM classification in colon cancer

Distant metastasis (M)

A

M0 No distant metastasis

M1 Distant metastasis

M1a Metastasis confined to 1 organ or site (eg, liver, lung, ovary, non-regional node)

M1b Metastases in more than 1 organ/site or the peritoneum

2247
Q

Features of Li Fraumeni?

A

Autosomal dominant

Consists of germline mutations to p53 tumour suppressor gene

High incidence of malignancies particularly sarcomas and leukaemias

Diagnosed when:

*Individual develops sarcoma under 45 years
*First degree relative diagnosed with any cancer below age 45 years and another family member develops malignancy under 45 years or sarcoma at any age

2248
Q

Features of BRCA1 and 2

Remember BRCA1 is counterintuitive

A

Carried on chromosome 17

Linked to developing breast cancer (60%) risk.

Associated risk of developing ovarian cancer (55% with BRCA 1 and 25% with BRCA 2).

2249
Q

Lynch Syndrome

A

Autosomal dominant

Develop colonic cancer and endometrial cancer at young age

80% of affected individuals with get colonic and or endometrial cancer

High risk individuals may be identified using the Amsterdam criteria

2250
Q

Gardners syndrome

A

Autosomal dominant familial colorectal polyposis

Multiple colonic polyps

Extra colonic diseases include: skull osteoma, thyroid cancer and epidermoid cysts

Desmoid tumours are seen in 15%

Mutation of APC gene located on chromosome 5

Due to colonic polyps most patients will undergo colectomy to reduce risk of colorectal cancer

Now considered a variant of familial adenomatous polyposis coli

2251
Q

A 64 year old woman presents to her GP with a 2 month history of abdominal pain and diarrhoea. She describes it as severe, central pain and says that it is worse after eating. Her past medical history consists of hypertension, hypercholesterolaemia and a previous myocardial infarction. Which investigation is most likely to be diagnostic?

Capsule endoscopy

MR angiogram

CT abdomen

US renal tract

Barium studies

A

This lady is an arteriopath as evidenced by her past medical history. The pain she describes - severe, central and worse after eating, with associated diarrhoea is classic of mesenteric ischaemia - worse when the gut is working to digest food. The most appropriate investigation to confirm the diagnosis would be a magnetic resonance angiogram of the mesenteric vasculature.

2252
Q

Where is mesenteric ischaemia more likely to occur?

A

Mesenteric ischaemia is primarily caused by arterial embolism resulting in infarction of the colon. It is more likely to occur in areas such as the splenic flexure that are located at the borders of the territory supplied by the superior and inferior mesenteric arteries.

2253
Q

abdominal pain

rectal bleeding

diarrhoea

fever

bloods typically show an elevated WBC associated with acidosis

In an arteriopath

A

Mesenteric ischaemia

2254
Q

What are the predisposing factors for mesenteric ischaemia?

A

Increasing age

AF

other causes of emboli e.g. endocarditis

CV disease risk factors

Cocaine

2255
Q

A 72-year-old male has been diagnosed with rectal carcinoma. He is due to undergo a lower anterior resection. The aim of the resection is to restore intestinal continuity. Which is the most appropriate type of stoma?

Loop colostomy

End ileostomy

Loop ileostomy

Caecostomy

Total colectomy

A

Loop ileostomy is a method to divert bowel contents away from a distal anastomosis. It is often indicated in rectal cancers. Reversal of the ileostomy restores bowel continuity and improves the patient’s overall quality of life.

2256
Q

A 36-year-old lady with a past history of Crohn’s disease presents with painful bleeding per rectum. She tells you that the bleeding occurs after defaecation and it is bright red. What is the most likely cause of the bleeding?

Fissure in ano

Haemorrhoid

Haemorrhagic perianal abscess

Rectal cancer

Recto-uterine fistula

A

Pain on passing faeces accompanied by bleeding post-defaecation is suggestive of a diagnosis of fissure in ano. Thrombosed haemorrhoids may also present with painful PR bleeding but in this scenario a fissure is more likely. This young lady has a background of Crohn’s disease and that patients with Crohn’s disease are more susceptible to fissure formation.

Rectal cancer can present with rectal bleeding but would be unlikely in a 36-year-old. Recto-uterine fistulas typically cause faecal incontinence rather than bleeding. A perianal abscess would cause perianal pain and may be accompanied by pyrexia.

2257
Q

What is the dentate line?

A

The pectinate line (dentate line) is a line which divides the upper two thirds and lower third of the anal canal. Developmentally, this line represents the hindgut-proctodeum junction.

2258
Q

A 76-year-old patient undergoes a right hemicolectomy for bowel cancer. 4 days post-operative you are on-call and are asked to review him by one of the nurses as she is concerned about the patient’s blood pressure. On examination the patient is clammy, confused, heart rate 115 bpm, blood pressure 80/50 mmHg, urine output is 10 mls per hour and temperature is 38.6ºC. His wound is clean and dry, and his abdomen is non-peritonitic. Chest examination is clear. Heart sounds normal. Peripheral lines appear intact with no phlebitis. The patient does not have a urinary catheter. You perform an urgent arterial blood gas which demonstrates a normal gas exchange, haemoglobin 12.5 g/L, lactate of 5.4 µmol/L, and creatinine of 331 µmol/L. You fluid resuscitate him and arrange immediate imaging. Which imaging modality would be best in this situation?

Plain abdominal x-ray

CT abdomen with contrast

CT angiogram

US abdomen

MRI abdomen

A

This is a clinical picture of severe sepsis (i.e. sepsis with one or more organ dysfunction) and clearly points to an intra-abdominal origin as other sources of infection are unremarkable. The ABG is consistent with sepsis but has also impaired renal function (creatinine 331 µmol/L) and therefore intravenous contrast would be unwise due to the risk of developing contrast nephropathy. As such a CT abdomen would not be advisable and a CT angiogram is not warranted as there isn’t any suspicion of haemorrhage. An anastomotic leak is unlikely as the patient’s abdomen would be peritonitic. Plain abdominal x-ray would not detect a source of infection and an MRI is not going to be possible 4 days post-operative due to the use of metal surgical clips.

The best modality would therefore be ultrasound abdomen as this would be able to detect an intra-abdominal collection.

2259
Q

Theme: Surgery for inflammatory bowel disease

A.Proctectomy

B.Anterior resection

C.Panproctocolectomy

D.Panproctocolectomy and ileoanal pouch

E.Sub total colectomy

F.Right hemicolectomy

Please select the most appropriate surgical modality for treating the inflammatory bowel disease scenarios described. Each option may be used once, more than once or not at all.

  1. A 22-year-old man presents with his first presentation of ulcerative colitis. Despite aggressive medical management with steroids, azathioprine and infliximab his symptoms remain unchanged and he has developed a megacolon.
  2. A 22-year-old lady has a long history of severe perianal Crohns disease with multiple fistulae. She is keen to avoid a stoma. However, she has progressive disease and multiple episodes of rectal bleeding. A colonoscopy shows rectal disease only and a small bowel study shows no involvement with Crohns.
  3. A 22-year-old man has a long history of ulcerative colitis. His symptoms are well controlled with steroids. However, attempts at steroid weaning and use of steroid sparing drugs have repeatedly failed. He wishes to avoid a permanent stoma.
A

In patients with fulminant UC a sub total colectomy is the safest treatment option. The rectum will be left in situ as resection of the rectum in these acutely unwell patients carries an extremely high risk of complications.

Severe rectal Crohns that has developed complications such as haemorrhage and multiple fistulae is usually best managed with proctectomy. Although a diverting stoma may reduce the risk of local sepsis it is unlikely to reduce the bleeding. She is keen to conserve a rectum, however, an ileoanal pouch in this setting is unwise.

In patients with UC where medical management is not successful, surgical resection may offer a chance of cure. Those patients wishing to avoid a permanent stoma may be considered for an ileoanal pouch. However, this procedure is only offered in the elective setting.

2260
Q

What is an absolute indication for proctocolectomy in UC?

A

Longstanding UC is associated with a risk of malignant transformation. Dysplastic transformation of the colonic epithelium with associated mass lesions is an absolute indication for a proctocolectomy.

2261
Q

What is the management of poorly controlled colitis that presents as an emergency and fails to respond to medical therapy?

A

Emergency presentations of poorly controlled colitis that fails to respond to medical therapy should usually be managed with a subtotal colectomy. Excision of the rectum is a procedure with a higher morbidity and is not generally performed in the emergency setting. An end ileostomy is usually created and the rectum either stapled off and left in situ, or, if the bowel is very oedematous, may be brought to the surface as a mucous fistula.

2262
Q

What are the complications of ileoanal pouch?

A

Anastomotic dehiscence

Pouchitis

Poor physiological function with seepage and soiling

2263
Q

What are the restorative options in UC post sx?

A

Restorative options in UC include an ileoanal pouch. This procedure can only be performed whilst the rectum is in situ and cannot usually be undertaken as a delayed procedure following proctectomy.

2264
Q

What is a consideration regarding surgical treatment of Crohn’s?

A

Surgical resection of Crohns disease does not equate with cure, but may produce substantial symptomatic improvement.

Indications for surgery include complications such as fistulae, abscess formation and strictures.

Extensive small bowel resections may result in short bowel syndrome and localised stricturoplasty may allow preservation of intestinal length.

Staging of Crohns will usually involve colonoscopy and a small bowel study (e.g. MRI enteroclysis).

2265
Q

Sx mx of severe perianal and or rectal crohn’s?

A

Severe perianal and / or rectal Crohns may require proctectomy. Ileoanal pouch reconstruction in Crohns carries a high risk of fistula formation and pouch failure and is not recommended.

2266
Q

Considerations regarding mx of crohn’s in its most common disease site?

A

Terminal ileal Crohns remains the commonest disease site and these patients may be treated with limited ileocaecal resections.

Terminal ileal Crohns may affect enterohepatic bile salt recycling and increase the risk of gallstones.

2267
Q

A 67-year-old man presents to the Emergency department with sudden-onset sharp abdominal pain radiating to his back. His observations on arrival include: heart rate 110 beats per minute, blood pressure 102/67 mmHg, respiratory rate 30 breaths per minute, oxygen saturations 94% on air and temperature 37.4ºC. What is the most important first step in the management of this patient?

Abdominal x-ray

Immediate contrast CT of the abdomen

Immediate fluid resuscitation to keep blood pressure >120/80mmHg

Patient should go straight to theatre

Urine dip

A

This man most likely has a ruptured abdominal aortic aneurysm and is showing signs of shock. This carries a very high mortality and he should therefore be taken immediately to theatre. Diagnosing a ruptured abdominal aortic aneurysm requires a high index of suspicion and if suspected, it is inappropriate to waste time on imaging.

Whilst it is important to gain intravenous access and treat shock, systolic blood pressure should be maintained <100mmHg to avoid rupturing any contained leak.

2268
Q

A 60-year-old man is investigated for intermittent claudication. He is referred to the local vascular unit and a diagnosis of peripheral arterial disease is made. His blood pressure is 128/78 mmHg and his fasting cholesterol 5.8 mmol/l. Following recent NICE guidelines which of the following medications should he be taking?

Antiplatelet + statin + ACE inhibitor

Statin

Antiplatelet + statin + long-acting nitrate

Antiplatelet + statin + beta-blocker

Antiplatelet + statin

A

As this patient has established cardiovascular disease he should be taking a statin, regardless of the baseline cholesterol. The 2010 NICE guidelines on clopidogrel changed the previous advice that all patients with established cardiovascular disease should be taking aspirin, unless there is a contraindication. NICE propose that clopidogrel is now used first-line following an ischaemic stroke and also in peripheral arterial disease.

2269
Q

Mx of PAD

A

Treat comorbidites

All patients with established CVD should be taking a statin

Clopidogrel should be used first line in patients with PAD in preference to aspirin

Exercise training has been shown to ahve significant benefits

Severe PAD or critical limb iscahemia may be treated by:

Angioplasty

Stenting

Bypass surgery

Other drugs that are licensed for PAD include

naftidrofuryl oxalate: vasodilator sometimes used for patients with poor QoL

Cilostazol: PDE III inhibitor with both antiplatelet and vasodilator effects (not recommended by NICE)

2270
Q

Oesophageal TNM staging

Tumour

A

T1- the tumour is confined to the submucosa
T2- the tumour has grown into (but not through) the muscularis propria
T3- the tumour has grown into (but not through) the serosa
T4- the tumour has penetrated through the serosa and the peritoneal surface. If extending directly into other nearby structures (such as other parts of the bowel or other organs/body structures) it is classified as T4a. If there is perforation of the bowel, it is classified as T4b.

2271
Q

Oesophageal TNM staging

Nodes

A

N0- no lymph nodes contain tumour cells
N1- there are tumour cells in up to 3 regional lymph nodes
N2- there are tumour cells in 4 or more regional lymph nodes

2272
Q

Oesophageal TNM staging

Mets

A

Metastases

M0- no metastasis to distant organs
M1- metastasis to distant organs

2273
Q

A 67-year-old man with a 10-year history of gastro-oesophageal reflux disease is investigated for dysphagia. An endoscopy shows an obstructive lesion highly suspicious of oesophageal cancer. What is the biopsy most likely to show?

Squamous cell carcinoma

Normal squamous epithelium

Adenocarcinoma

Leiomyoma

Metaplastic columnar epithelium

A

Oesophageal adenocarcinoma is associated with GORD or Barrett’s

2274
Q

Usually follows long term use and is usually painless with non obstructive features

Often due to hepatic dysfunction and fatty liver which may occur with long term TPN usage.

A

TPN associated jaundice

2275
Q

A 60-year-old male smoker underwent endoscopy for recurrent dyspepsia despite Helicobacter Pylori eradication. A mass was noted around the oesophagogastric junction. What is the most appropriate modality for staging the primary tumour?

MRI

Non-contrast CT

CT with oral contrast only

Positron emission tomography (PET)

Endoscopic ultrasound

A

Oesophageal cancer is the eighth most common cancer worldwide and the thirteenth most common in the UK. It is 20 times more common in China but its incidence is rising in the western world. Risk factors include smoking, alcohol and obesity. In addition, squamous cell carcinoma is linked to achalasia and coeliac disease, while adenocarcinoma occurs in the setting of Barrett’s oesophagus.

Flexible upper GI endoscopy is the diagnostic investigation of choice and a minimum of eight biopsies should be obtained. CT is an acceptable staging modality and should be used routinely to identify metastatic disease. However, it requires the use of intravenous and oral contrast (unlike the options given above). Endoscopic ultrasound provides superior axial resolution to CT and is the most accurate way of determining the depth of spread of a malignant tumour through the oesophageal wall. PET is not routinely used as a staging modality in oesophageal carcinoma.

2276
Q

A 39-year-old overweight female undergoes an elective laparoscopic cholecystectomy for gallstone disease. Day 1 post-operatively you are asked to review her by the nurse in charge. The patient is complaining of severe right upper quadrant pain. On examination she is tachycardic, but normotensive and apyrexial. Her right upper quadrant is tender to palpation but there is no evidence of jaundice. The intra-abdominal drain in-situ has a small volume of green liquid draining from it.

What post-operative complication is most likely?

Intra-abdominal collection

Biliary leak

Intra-abdominal haemorrhage

Perforated viscus

Ileus

A

Right upper quadrant tenderness and bilious fluid in the intra-abdominal drain would suggest a bile leak following the cholecystectomy. As the patient is apyrexial and normotensive an intra-abdominal collection or haemorrhage would be unlikely. A perforation is a recognised complication of a laparoscopic cholecystectomy however the patient would normally develop peritonitis, as oppose to localised right upper quadrant tenderness. Finally, an ileus would not causes right upper quadrant pain or bilious fluid in the drain.

2277
Q

What is the gingko leaf sign?

A

Subcutaneous (surgical) emphysema which is a known complication of laparoscopic surgery. If the anterior chest wall is affected air can outline the pectoralis major muscle, giving rise to the ‘ginkgo leaf’ sign.

2278
Q

You are asked to review a 65-year-old woman who has become breathless on the surgical ward. Earlier in the day she had a laparoscopic cholecystectomy for gallstone disease. A chest x-ray has already been obtained:

© Image used on license from Radiopaedia

What complication has developed?

Pneumothorax

Intestinal perforation resulting in pneumoperitneum

Subcutaneous emphysema

Pulmonary embolism

Acute respiratory distress syndrome

A

This radiograph demonstrates subcutaneous (surgical) emphysema which is a known complication of laparoscopic surgery. If the anterior chest wall is affected air can outline the pectoralis major muscle, giving rise to the ‘ginkgo leaf’ sign.

2279
Q

What are some complications of laparoscopy?

A

General risks of anaesthetic

Vasovagal reaction e.g. bradycardia in response to abdominal distension

Extra-peritoneal gas insufflation: surgical emphysema

Trauma to GIT

Injury to BVs: Common iliacs, deep IEA

2280
Q

A 65-year-old man with a history of dyspepsia is found to have a gastric MALT lymphoma on biopsy. What treatment should be offered?

Gastrectomy

Laser ablation

None

CHOP chemotherapy

H. pylori eradication

A

Overview

associated with H. pylori infection in 95% of cases

good prognosis

if low grade then 80% respond to H. pylori eradication

Features

paraproteinaemia may be present

2281
Q

A.Lymphoma

B.Gastrinoma

C.Insulinoma

D.Glucagonoma

E.Phaeochromocytoma

F.Carcinoid syndrome

G.Vasoactive Intestinal Peptide secreting tumour

H.Pancreatic adenocarcinoma

Please select the most likely diagnosis for the scenario given. Each option may be used once, more than once or not at all.

A 65-year-old male attends surgical out patients with epigastric discomfort. He has recently been diagnosed with diabetes by the GP and is a heavy smoker. An OGD is normal.

A 50-year-old male presents with recurrent episodes of abdominal pain and diarrhoea. Blood tests reveal mild iron deficiency anaemia and an upper GI endoscopy demonstrates multiple ulcers in the first part of the duodenum.

An obese 40-year-old male presents with episodes of anxiety, confusion and one convulsive episode. CT brain is normal. An abdominal CT scan shows a small 1.5cm lesion in the head of the pancreas.

A

The dominant differential diagnosis should be of pancreatic adenocarcinoma in this setting. Glucagonomas are very rare and may be associated with a bullous rash.

Diarrhoea, abdominal pain and multiple ulcers should raise the suspicion of Zollinger Ellison syndrome cause by gastrinoma.

These episodes are due to hypoglycaemia. Insulinomas are normally solitary tumours and may not be seen by radiological imaging. Resection is the treatment of choice.

2282
Q

Ix in pancreatic adenocarcinoma?

A

Investigation

ultrasound has a sensitivity of around 60-90%

high resolution CT scanning is the investigation of choice if the diagnosis is suspected

2283
Q

What is purstcher retinopathy?

A

Purtscher retinopathy is a hemorrhagic and vasoocclusive vasculopathy, which, in 1912, was first described as a syndrome of sudden blindness associated with severe head trauma. These patients had findings of multiple white retinal patches and retinal hemorrhages that were associated with severe vision loss

Since its original description, Purtscher retinopathy has been associated with traumatic injury, primarily blunt thoracic trauma and head trauma, and numerous nontraumatic diseases.

Purtscher-like retinopathy is seen in diverse conditions, including acute pancreatitis; fat embolization; amniotic fluid embolization; preeclampsia; hemolysis, elevated liver enzymes, and low platelets (HELLP) syndrome; and vasculitic diseases, such as lupus.

2284
Q
A

Characteristic fundus findings of Purtscher retinopathy. Multiple cotton-wool spots surround the optic nerve after blunt thoracic trauma.

2285
Q

Tumour antigens:

S-100

A

Melanoma

Scwhwannomas

2286
Q

Tumour antigen:

Bombesin

A

SCLC

Gastric cancer

Neuroblastome

2287
Q

A 45-year-old male with a history of heavy alcohol intake presents with acute onset epigastric pain radiating to the right side. On examination his sclera are yellow and his abdomen is tender in the right upper quadrant with localised guarding.

Observations are: Heart rate 95/min, blood pressure 80/50 mmHg, saturation 99% on 2L, temperature 39.5ºC, Glasgow coma score 14/15 (confused speech).

Which of the following diagnoses would explain this set of signs and symptoms?

Cholecystitis

Pancreatitis

Gallbladder empyema

Ascending cholangitis

Perforated peptic ulcer

A

Reynolds Pentad (jaundice, right upper quadrant pain, fever/rigors, shock and altered mental status) is associated with ascending cholangitis. This patient would need adequate resuscitation before further investigation of biliary tree with USS/MRCP for common bile duct stones +/- ERCP.

2288
Q

A 65-year-old lady is undergoing an OGD for investigation of dysphagia. She is known to have achalasia. A mass is seen in the middle third of the oesophagus. There is no other pathology identified past this point.

What is the most likely type of cancer?

Adenocarcinoma of the oesophagus

Squamous cell carcinoma of the oesophagus

MALT tumour

Direct invasion from lung neoplasm

Benign tumour of the oesophagus

A

Whilst Barrett’s oesophagus increases the risk of oesophageal adenocarcinoma, achalasia increases the risk of squamous cell carcinoma of the oesophagus.

2289
Q

Why is sensation over the thenar area spared in carpal tunnel syndrome?

A

Palmar cutaneous branch of the median nerve travels superficially to the flexor retinaculum

2290
Q

Causes of carpal tunnel syndrome

WRIST

A

F>M

1o/ idiopathic

2o

Water: pregnancy, hypothyroidism

Radial: #

Inflamation: RA/gout

Soft tissue swelling: lipomas, acromegaly, amyloidosis

Toxic: DM, EtOH

2291
Q

Tingling/ pain in thumb, index and middle fingers

Pain worse at night or after repetitive actions

Relieved by shaking/ flicking

Clumsiness

A

Carpal tunnel syndrome

2292
Q

What are the signs of carpal tunnel syndrome

A

Reduced sensation over lateral 3.5 fingers

Reduced 2 point touch discrimination: early sign of irreversible damage

Wasting of thenar eminence: late sign of irreversible damage

Phalen’s Flexing

Tinnel’s Tapping

2293
Q

Phalen’s

A

The patient is asked to hold their wrist in complete and forced flexion (pushing the dorsal surfaces of both hands together) for 30–60 seconds. The lumbricals attach in part to the flexor digitorum profundus tendons. As the wrist flexes, the flexor digitorum profundus contracts in a proximal direction, drawing the lumbricals along with it. In some individuals, the lumbricals can be “dragged” into the carpal tunnel with flexor digitorum profundus contraction. As such, Phalen’s maneuver can moderately increase the pressure in the carpal tunnel via this mass effect, pinching the median nerve between the proximal edge of the transverse carpal ligament and the anterior border of the distal end of the radius. By compressing the median nerve within the carpal tunnel, characteristic symptoms (such as burning, tingling or numb sensation over the thumb, index, middle and ring fingers) conveys a positive test result and suggests carpal tunnel syndrome. Because not all individuals will draw the lumbricals into the carpal tunnel with this maneuver, this test cannot be perfectly sensitive or specific for carpal tunnel syndrome

2294
Q

Reverse Phalen’s

A

This test is performed by having the patient maintain full wrist and finger extension for two minutes. The reverse Phalen’s test significantly increases pressure in the carpal tunnel within 10 seconds of the change in wrist posture and the carpal tunnel pressure has the tendency to increase throughout the test’s duration. In contrast, the change in carpal tunnel pressure noted in the standard Phalen’s test is modest and plateaus after 20 to 30 seconds.

2295
Q

Tinel’s

A

Tinel’s sign is a way to detect irritated nerves. It is performed by lightly tapping (percussing) over the nerve to elicit a sensation of tingling or “pins and needles” in the distribution of the nerve. It takes its name from French neurologist Jules Tinel (1879–1952).[1][2][3]

For example, in carpal tunnel syndrome where the median nerve is compressed at the wrist, Tinel’s sign is often “positive” causing tingling in the thumb, index, middle finger and the radial half of the fourth digit. Tinel’s sign is sometimes referred to as “distal tingling on percussion” or DTP. This distal sign of regeneration can be expected during different stage of somatosensory recovery.[4]

Although most frequently associated with carpal tunnel syndrome, Tinel’s sign is a generalized term, and can also be positive in tarsal tunnel syndrome, or in ulnar nerve impingement at the wrist (Guyon’s canal syndrome), where it affects the other (ulnar) half of the fourth digit and the fifth digit.

2296
Q

Ix in carpal tunnel syndrome

A

Not usually performed

Nerve conduction studies

USS

2297
Q

Mx of carpal tunnel

A

Conservative:

Mx of underlying cause

Wrist splints: neutral position, especially at night

Medical:

Local steroid injections

Surgical:

Carpal tunnel decompression by division of the flexor retinacuum

2298
Q

Cx of carpal tunnel Sx

A

Scar formation: high risk for hypertrophied or keloid

Scar tenderness: up to 40%

Nerve injury: palmar cutaneous branch of the median nerve; motor branch to the thenar muscles

Failure to relieve symptoms

2299
Q

What are some other locations of median nerve entrapment?

A

Pronator syndrome

Anterior interossesous syndrome

2300
Q

What is pronator syndrome

A

Entrapment of the median nerve between two heads of pronator teres

2301
Q

What is anterior interosseus syndrome?

A

Compression of the anterior interosseous branch of the median nerve by deep head of pronator teres

Muscle weakness only: pronator quadratus, FPL, radial half of FDP

2302
Q

Def: Dupuytren’s contracture

A

Progressive, painless fibrotic thickening of palmar fascia

2303
Q

Patient in Dupuytren’s

A

M>F

Middle age/elderly

Skin puckering and tethering

Fixed flexion contracture of ring and little fingers.

Often bilateral and symmetrical

MCP and IP joint flexion

2304
Q
A

Dupuytren’s conracture

2305
Q

Associations of Dupuytren’s

BAD FIBERS

A

Bent penis: Peyronies

AIDS

DM

FH: D

Idiopathic: commonest

Booze: ALD

Epilepsy (+ epilepsy meds- phenytoin)

Reidel’s thyroiditis and other fibromatoses e.g. Ledderhose disease, Retroperitoneal fibrosis

Smoking

2306
Q

Peyronie’s

A

Peyronie’s disease or Peyronie disease (/peɪroʊˈniː/), also known as induratio penis plastica (IPP)[1] or chronic inflammation of the tunica albuginea (CITA), is a connective tissue disorder involving the growth of fibrous plaques[2] in the soft tissue of the penis affecting an estimated 5% of men.[3] Specifically, scar tissue forms in the tunica albuginea, the thick sheath of tissue surrounding the corpora cavernosa causing pain, abnormal curvature, erectile dysfunction, indentation, loss of girth and shortening.[4][5][6][7] A variety of treatments have been used, but none have been especially effective.

2307
Q

Ledderhose disease

A

Plantar fascial fibromatosis, also known as Ledderhose’s disease, Morbus Ledderhose, and plantar fibromatosis, is a relatively uncommon[1] non-malignant thickening of the feet’s deep connective tissue, or fascia. In the beginning, where nodules or cords start growing along tendons of the foot[citation needed], the disease is minor. Eventually, however, the cords thicken, the toes stiffen and bend, and walking becomes painful. The disease is named after Dr. Georg Ledderhose, a German surgeon who described the condition for the first time in 1894.[2][3] A similar disease is Dupuytren’s disease, which affects the hand and causes bent hand or fingers.

2308
Q

Mx of Dupuytren’s

A

Conservative:

PT/ exercise

Fasciectomy:

e.g. when hand can’t be placed flat on table

Z-shaped scars prevent contracture

However can damage ulnar nerve, usually recurs

2309
Q

DDx for Dupuytren’s

A

Skin contracture: old laceration or burn

Tendon fibrosis, trigger finger

Ulnar N palsy

2310
Q

Trigger finger features

A

Tendon nodule which catches on proximal side of tendon sheath-> triggering on forced extension-> fixed flexion deformity

Usually ring and middle fingers

Associated with RA

Rx: steroid injection or sx

2311
Q

Def: ganglion

A

Smooth multilocular cystic swellings

Mucoid degeneration of joint capsule or tendon sheath

May be in communication with joint capsules/tendons

2312
Q

90% located on dorsum of wrist

Subdermal, fixed to deeper structures

Limits plane of movement

May cause pain or nerve pressure symptoms

A

Ganglion

2313
Q

Mx of ganglion

A

50% disappear spontaneously

Aspiration +/- steroid and hyaluronidase injection

Sx excision

2314
Q

DDx for ganglion

A

Lipoma

Fibroma

Sebaceous cyst

2315
Q
A

Ganglion

2316
Q

Entrapment of lat cutaneous nerve of thigh beetween ASIS and inguinal ligament

Pain +/- paraesthesia on lateral thigh

No motor deficit

Increased risk with obesity

Can occasionally be damaged during lap hernia repair

A

Meralgia paraesthetica

2317
Q

Predominantly young women

Patellar aching after prolonged sitting or climbing stairs

Pain on patellofemoral compression

Ix: no abnormality on X ray

Rx: vastus medialis strengthening

A

Chondromalacia patellae

2318
Q

Popliteal swelling arising between the medial head of gastrocnemius and semimebranosus muscle

Herniation from joint synovium

Usually 2o to OA

Rupture: acute calf pain and swelling (NB ?DVT)

A

Baker’s cyst

2319
Q

Clarke’s test

A

In medicine, Clarke’s test is a component of knee examination which may be used to test for patellofemoral pain syndrome, or anterior knee pain. It is not a standard part of the knee examination but is used to diagnose anterior knee pain where the history indicates this as the likely pathology. The patient is asked to actively contract the quadriceps muscle while the examiner’s hand exerts pressure on the superior pole of the patella, so trying to prevent the proximal movement of the patella. While it can produce some discomfort even in normal people, the reproduction of the symptoms suggest pain of patello-femoral origin.[1]

2320
Q
A

Baker’s cyst

2321
Q

Great toes deviates laterally at MTP joint

Pressure of MTP against shoe-> bunion

Increased weight bearing at 2nd metatarsal head: transfer metatarsalgia, hammer toe

Aetiology:

pointed shoes

Wearing high heels

Mx:

Conservative:

Bunion pads, plast wedge between great and second toes

Metatarsal osteotomy

A

Hallux valgus

2322
Q
A
2323
Q
A

Hallux valgus

2324
Q

Pain from pressure on an interdigital neuroma between the metatarsals

Pain radiates to medial side of one toe and lateral side of another

A

Morton’s metatarsalgia/Neuroma

2325
Q

A runner presents with heel pain. On examination, there is diffuse tenderness which is worse 2cm distal to the medial calcaneal tuberosity. Although the patient has stopped running for the past week, pain is worse when they walk on their toes and when they are on their feet for a long period at work. What is the most likely diagnosis?

Achilles tendonitis

Plantar fasciitis

S1 radiculopathy

Morton’s neuroma

Subcalcaneal bursitis

A

This is typical of plantar fasciitis which is the most common cause of heel pain in adults. Pain in Achilles tendonitis is at the calcaneal insertion of the tendon or further up the tendon depending on the area affected. Thompson’s test excludes rupture of the tendon. S1 radiculopathy would cause sensory loss along the lateral aspect of the foot and may lead to reduced dorsiflexion of the foot. Morton’s neuroma is a thickening of the tissue around the nerve usually between the 3rd and 4th toes. Pain tends to be on the ball of the foot. Subcalcaneal bursitis presents with pain at the point of Achilles tendon insertion, and on the back and bottom of the heal. Plantar fasciitis is more common and would give the specific area of tenderness mentioned.

2326
Q

Thompson test (aka Simmonds)

A

The Thompson test is diagnostic for a complete tear of the Achilles tendon.

The Thompson Test: When the tendon is intact, and the calf is squeezed, the ankle will plantar flex .
The test is positive when the right ankle doesn’t plantar flex when the calf is squeezed,

2327
Q

What is the most common cause of heel pain in adults?

A

Plantar fasciitis

Pain usually worse around the medial calcaneal tuberosity

Mx:

Rest feet where possible

Wear shoes with good arch support and cushioned heels

Insoles and heel pads may be helpful

2328
Q

A 28-year-old male presents to the emergency room with severe pain in the right knee following an injury during a football match. He states that he was tackled from behind, and then felt a ‘pop’ and severe pain which was followed by rapid swelling of the joint. On examination there is a right sided knee effusion and a positive Lachman test.

What is the most likely diagnosis?

Medial collateral ligament (MCL) rupture

Lateral collateral ligament (LCL) rupture

Anterior cruciate ligament (ACL) rupture

Posterior cruciate ligament (PCL) rupture

Meniscal tear

A

Rapid joint swelling is suggestive of haemoarthrosis which can occur due to ACL or PCL rupture. The mechanism of injury suggests rupture of the ACL. A positive Lachman tests is also very suggestive of an ACL injury.

An LCL injury most commonly occurs due to direct blows to the medial aspect of the leg which puts strain on the LCL ligament. The classical symptoms would be a slow developing joint effusion and lateral joint line tenderness.

Similarly, an MCL injury most commonly occurs due to direct blows to the lateral aspect of the leg which results in strain on the MCL ligament. The classical symptoms would be a slow developing joint effusion and medial joint line tenderness.

Meniscal tears often occur due to twisting injuries. They are associated with delayed knee swelling and joint locking.

2329
Q

A 33-year-old woman presents with back pain which radiates down her right leg. This came on suddenly when she was bending down to pick up her child. On examination straight leg raising is limited to 30 degrees on the right hand side due to shooting pains down her leg. Sensation is reduced on the dorsum of the right foot, particularly around the big toe and foot dorsiflexion is also weak. The ankle and knee reflexes appear intact. A diagnosis of disc prolapse is suspected. Which nerve root is most likely to be affected?

L2

L3

L4

L5

S1

A

L5 lesion features = loss of foot dorsiflexion + sensory loss dorsum of the foot

2330
Q

A 75-year-old lady presents to the emergency room after falling onto her left elbow. She has marked bruising and tenderness of the left upper arm. On examination, you note a left wrist drop.

What is the most likely injury?

Fracture of the proximal humerus

Supracondylar fracture of humerus

Colle’s fracture

Smith’s fracture

Fracture of the shaft of the humerus

A

This example describes an injury of the humerus with associated radial nerve damage. The radial nerve is most susceptible to damage from a fracture of the shaft of the humerus.

Supracondylar fracture of humerus is most commonly associated with ulnar nerve damage. Fracture of the proximal humerus is most commonly associated with axillary nerve damage.

Colle’s fracture and Smith’s fracture are fractures of the distal radius. These fractures are not in keeping with the upper limb examination findings.

2331
Q

Mx of talipes equinovarus

A

ponseti method: manipulation and progressive casting starting sooon after birth (6-10w).

Achilles tenotomy required in around 85% of cases

Night-time braces applied until child is 4 years old

2332
Q

Sensory loss over anterior thigh
Weak quadriceps
Reduced knee reflex
Positive femoral stretch test

A

L3 nerve root compression

2333
Q

Sensory loss anterior aspect of knee
Weak quadriceps
Reduced knee reflex
Positive femoral stretch test

A

L4 nerve root compression

2334
Q

Sensory loss dorsum of foot
Weakness in foot and big toe dorsiflexion
Reflexes intact
Positive sciatic nerve stretch test

A

L5 nerve root compression

2335
Q

Sensory loss posterolateral aspect of leg and lateral aspect of foot
Weakness in plantar flexion of foot
Reduced ankle reflex
Positive sciatic nerve stretch test

A

S1 nerve root compression

2336
Q

Femoral nerve stretch tset

A

Femoral nerve stretch test (Mackiewicz sign[1]) - for the FNST the patient lies prone, the knee is passively flexed to the thigh and the hip is passively extended; the test is positive if the patient experiences anterior thigh pain. This test is usually strongly positive in patients with protrusions at L2–L3 and L3–L4, slightly positive or negative in L4–L5 disc protrusions and negative in cases with a lumbosacral protrusion.

2337
Q

Sciatic nerve stretch test

A

The therapist raises the leg, keeping the knee straight. The therapist moves the ankle into a dorsiflexed position which lengthens the nerve. If there are no neural symptoms (tingling, shooting pain or tight, restricted feeling throughout the back of the leg), then the therapist may ask the patient to raise the head to further increase the stretch.

2338
Q

A 47-year-old female presents as she is concerned about elbow pain. She has just spent the weekend painting the house. On examination there is localised pain around the lateral epicondyle and a diagnosis of lateral epicondylitis is suspected. Which one of the following movements would characteristically worsen the pain?

Resisted thumb flexion

Thumb extension

Flexion of the elbow

Pronation of the forearm with the elbow flexed

Resisted wrist extension with the elbow extended

A

Lateral epicondylitis: worse on resisted wrist extension/supination whilst elbow extended

2339
Q

A 5-year-old boy is seen in the Minor Injury Unit after falling in the playground. His mother observed the fall and describes him falling on an outstretched hand.

The x-ray is shown below

© Image used on license from Radiopaedia

What which one of the following best describes this injury?

Colles’ fracture

Salter-Harris type 1 fracture

Salter-Harris type 2 fracture

Buckle fracture

Greenstick fracture

A

Buckle, or torus, fractures are incomplete fractures of the shaft of a long bone that is characterised by bulging of the cortex. They typically occur in children aged 5-10 years.

As they are typically self-limiting they do not usually require operative intervention and can sometimes be managed with splinting and immobilisation rather than a cast.

2340
Q

Def: talipes equinovarus

A

Inverted and plantar flexed foot

Usually diagosed on newborn exam

M > F 2:1

1:1000 births

Idiopathic most commonly

Associated with

spina bifida, cerebal palsy, Edward’s, oligohydramnios, arthrogryposis

2341
Q

Hip pain in adults

A.Inflammatory arthritis

B.Trochanteric bursitis

C.Pubic symphysis dysfunction

D.Osteoarthritis

E.Meralgia paraesthetica

F.Avascular necrosis

G.Transient idiopathic osteoporosis

H.Referred lumbar spine pain

I.Perthes’ disease

J.Slipped upper femoral epiphysis

For each one of the following scenarios please select the most likely diagnosis:

A 43-year-old woman complains of right hip pain. During the examination the patient lies on her left side and the right hip is extended with a straight leg. Flexing the knee then recreates the pain

A

This is a femoral nerve stretch test

2342
Q

A 50-year-old woman presents with pain in the right forefoot for the past three months. The pain is worse around the third inter-metatarsophalangeal space and is described as a burning which is brought on by walking. There is no history of trauma and the patient does not do any regular exercise. Her alcohol intake is 28 units per week. On examination she complains of tenderness in the middle of the forefoot and her symptoms are recreated by squeezing the metatarsals together. What is the most likely diagnosis?

Metatarsal stress fracture

Gout

Alcohol-related peripheral neuropathy

Plantar fasciitis

Morton’s neuroma

A

Morton’s neuroma

2343
Q

forefoot pain, most commonly in the third inter-metatarsophalangeal space

worse on walking. May be described as a shooting or burning pain. Patients may feel they have a pebble in their shoe

Mulder’s click: one hand tries to hold the neuroma between the finger and thumb. The other hand squeezes the metatarsals together. A click may be heard as the neuroma moves between the metatarsal heads

there may be loss of sensation distally in the toes

A

Morton’s neuroma

2344
Q

Body composition of 70kg man

Proprotion intracellular

Proprotion extracellular

Components and volumes of extracellular?

A

Total water: 60% of 70kg= 42l

2/3rds intracellular: 28l

1/3rd extracellular: 14l

Plasma 3l

Interstitial 10l

Transcellular 1l

2345
Q

What are Starling’s forces?

A

Osmotic pressure

Hydrostatic pressure

2346
Q

Def: osmotic pressure?

A

Pressure which needs to be applied to prevent the inflow of water across a semipermeable membrane i.e. the ability of a solute to attract water

Oncotic pressure: form of osmotic pressure exerted by proteins

2347
Q

Def: Hydrostatic pressure

A

Pressure exerted by a fluid at equilibrium due to the force of gravity

2348
Q

What drives the distribution of fluid between the ECF and ICF?

A

Driven by differences in osmotic pressure only

2349
Q

What determines the distribution of fluid within the ECF?

A

Determined by Starling’s forces:

capillary and interstitial oncotic pressure

capillary and interstitial hydrostatic pressure

Filtration coefficient (capillary permeability)

2350
Q

Starling equation for fluid

A

The four Starling’s forces are:

hydrostatic pressure in the capillary (Pc)

hydrostatic pressure in the interstitium (Pi)

oncotic pressure in the capillary (pc )

oncotic pressure in the interstitium (pi )

The balance of these forces allows calculation of the net driving pressure for filtration.

Net Driving Pressure = [( Pc - Pi ) - ( pc - pi )]

2351
Q

3rd space losses lead to?

A

Reduction in ECF

e.g. bowel obstruction-> reduced fluid reabsorption-> 3rd space loss

Peritonitis-> ascites-> 3rd space loss

2352
Q

Daily fluid input/output?

A

40ml/kg/d

2353
Q

What is minimum urine output?

A

0.5ml/kg/h ~30ml/h

2354
Q

Na requirement?

A

1.5-2 mmol/kg/d= 100mmol/d

2355
Q

K requirement

A

1mmol/kg/d= 60mmol/d

2356
Q

Daily requirement- fluid regimens?

Normal

A

3L dex-saline with 20mM K in each bag (i.e. Hartmann’s)

or

1l NS + 2l dex with 20mM K in each bag

Each bag over 8h= 125ml/hr

2357
Q

What other losses need to be replaced in fluid homeostasis?

A

Vomiting and diarrhoea

NGT

Drains

Fever (+500ml for each oC)

Tachypnoea

High-output stomas

2358
Q

What does CVP indicate?

A

RV preload and depends on venous return and CO

2359
Q

Raised CVP seen in?

A

Increased circualting volume

Reduced CO i.e. pump failure

2360
Q

Reduced CVP seen in?

A

Reduced circulating volume

2361
Q

Normal CVP

A

5-10 cm H2O

2362
Q

Considerations regarding CVP readings

A

Single reading is not as useful as serial measurements before and after fluid challenge

Unchanged- hypovolaemic

Increase that reverses after 30 mins: euvolaemic

Sustained raise >5cm: overload/failure

Passive leg raising may be more useful than fluid challenge in determining the response to fluids.

Sustained increase in CVP suggests heart failure

2363
Q

Types of Crystalloid

A

NS

5% dextrose

Dextrose-Saline

Hartmann’s

2364
Q

Features of NS

A

0.9% NaCL= 9g/L

15mM NaCL

pH: 5-6

Use normal daily fluid requirements + replace losses

2365
Q

Features of 5% dextrose

A

Contains 50g dextrose/l

Used for normal daily fluid requirements

2366
Q

Features of dextrose-saline

A

Contains 4% dextrose= 4-g/l

0.18% NaCl= 31mM

Used for normal daily fluid requirements

2367
Q

Contents of Hartmann’s/Ringer’s lactate

A

Na 131mM

Cl: 111mM

K: 5mM

Ca 2.2mM

Lactate/HCO3: 29mM

2368
Q

Uses of Hartmann’s / Ringer’s lactate

A

Resuscitation in trauma patients

Maintenance

2369
Q

What is the Parkland Formula

A

The Parkland formula is a burn formula used to estimate the amount of replacement fluid required for the first 24 hours in a burn patient so as to ensure they remain hemodynamically stable.[1] The milliliter amount of fluid required for the first 24 hours -usually Ringer’s lactate- is four times the product of the body weight and the burn percentage (i.e. body surface area affected by burns).[2] The first half of the fluid is given within 8 hours from the burn incident, and the remaining over next 16 hours. Only area covered by second-degree burns or greater is taken into consideration, as first-degree burns do not cause hemodynamically significant fluid shift to warrant fluid replacement.[3]

The Parkland formula is mathematically expressed as:[2]

V = 4 ⋅ m ⋅ ( A ⋅ 100 ) {\displaystyle V=4\cdot m\cdot (A\cdot 100)}

where mass is in kilograms (kg), area as a percentage of total body surface area, and volume is in milliliters (mL). For example, a person weighing 75 kg with burns to 20% of his or her body surface area would require 4 x 75 x 20 = 6,000 mL of fluid replacement within 24 hours. The first half of this amount is delivered within 8 hours from the burn incident, and the remaining fluid is delivered in the next 16 hours.[4]

2370
Q

pH of Hartmann’s

A

pH=6.5 but Harmann’s is an alkalinising solution
Lactate is not an acid in itself it’s a conjugate base

Lactate metabolised in liver -> HCO3 production

2371
Q

Problems with crystalloid fluids

A

Give 1l NS-> 210ml remaining IV

Give 1L D5W-> 70ml remaining IV

Acidosis or electrolyte disturbances

Fluid overload

2372
Q

What are colloids?

A

Contain large molecular weight molecules e.g. gelatin, dextrans

Idea is to preserve oncotic pressure and thus remain intravascular

2373
Q

Give examples of colloid and categories

A

Synthetic:

Gelofusin

Volpex

Haemaccel

Voluven

Natural:

Albumin

Blood

2374
Q

Problems with colloid

A

Can interfere with cross-matching therefore take blood for cross matching before using

Anaphylaxis

Volume overload

2375
Q

How to assess fluid status

A

Hx: balance chart, surgery, other losses, thirsty

Impression: drowsy, alert

Inspect: drips, drains, stomas, catheter

Examination

2376
Q

What components of hydration can be assessed through examination?

A

IV volume

Tissue perfusion

End-organ perfusion

2377
Q

Indicators of IV volume

A

CRT

HR

BP lying and standing

JVP

2378
Q

Indicators of tissue perfusion

A

Skin turgor

Oedema: ankle, pulmonary, ascites

Mucus membranes

2379
Q

Indicators of end-organ perfusion

A

UO, raised U + Cr

Consciousness

Lactate

2380
Q

Other tests to assess fluid status

A

PCWP

CVP

2381
Q

What is PCWP

A

The pulmonary wedge pressure or PWP, or cross-sectional pressure (also called the pulmonary arterial wedge pressure or PAWP, pulmonary capillary wedge pressure or PCWP, pulmonary venous wedge pressure or PVWP, or pulmonary artery occlusion pressure or PAOP), is the pressure measured by wedging a pulmonary catheter with an inflated balloon into a small pulmonary arterial branch.[1]

Physiologically, distinctions can be drawn among pulmonary artery pressure, pulmonary capillary wedge pressure, pulmonary venous pressure and left atrial pressure, but not all of these can be measured in a clinical context.[2]

Noninvasive estimation techniques have been proposed.

Essentially an indirect measure of left atrial pressure

2382
Q

What are the issues with post-operative fluids

A

Raised ADH, raised aldosterone, raised cortisol-> Na and H2O conservation

Increased K: tissue damage, transfusion, stress hormones

2383
Q

Solutions to issues with post-operative fluids?

A

Use UO (aim >30ml/h) to guide fluid replacement, may need to reduce maintenance fluids to 2l for first 24h post op

Avoid K supplementation for first 24h post op

2384
Q

What is the problem with fluids in cardiac/renal failure?

And the solution

A

RAS activation-> Na and H2O retention

Avoid fluids with Na-> 5% dextrose

2385
Q

Issue with fluids in bowel obstruction

A

Patients have significant third space losses of both water and electrolytes

Likely to need significantly more than standard daily requirments

2386
Q

Fluid regimen in bowel obstruction

A

0.9% NS with 20-40mm KCL added to each bag

Titrate rate of fluid therapy to clinical findings on serial examination

Serial U+Es guide electrolyte replacement

2387
Q

Fluids in pancreatitis

A

Inflammation- significant fluid shift into the abdomen

Patients require aggressive fluid resuscitiation and maintenance

Inser urinary catheter and consider CVP monitoring

0.9% NS with 20-40mm KCL

Keep UO >30

Serial U+Es guide electrolyte replacement

2388
Q

Ileal fluid composition

A

Na: 130mM

Cl: 110mM

K: 10mM

HCO3: 30mM

Normal output: 10-15ml/Kg/d= 700ml/d

High output= 1000ml/d

Ileum will adapt to limit fluid and electrolyte losses

2389
Q

Fluids in ileostomy

A

0.9% saline and KCl

Daily requirements + replace losses titrated to UO

Serial U+Es guide electrolyte replacement

2390
Q

High output ileostomy

A

>1000ml/d

2391
Q

Rx treatment of high output ileostomy

A

Antimotility: loperamide, codeine phosphate

Antisecretory: omeprazole

2392
Q

Causes of reduced UO post-op

A

Post-renal:

Commonest cause, blocked/malsited catheter or acute urinary retention

Pre-renal: hypovolaemia

Renal: NSAIDs, gentamicin

Anuria normally= blocked or malsited catheter

Oliguria: inadeqauate fluid replacement

2393
Q

Mx of reduced UO post op

A

Collect info: Op Hx, Obs chart, drug chart

Examine patient: fluid status, palpable bladder, inspect drips, drains etc.

Action: flush with 50mL NS, fluid challenge

2394
Q

Def: refeeding syndrome

A

Life-threatening metabolic complication of refeeding via any route after prolonged period of starvation

2395
Q

Pathophysiology of refeeding syndrome

A

Reduced carbs-> catabolic state, reduced insulin, fat and protein catabolism and depletion of intracellular PO4

Refeeding: increased insuin in response to carbs and increased cellular PO4 uptake

Result is hypophosphataemia which can lead to:

rhabdo

respiratory insufficiency

arrhythmias

shock

seizures

2396
Q

Ions deranged in Refeeding

Consultancy company

A

KPMG (all reduced)

2397
Q

Patients at risk of refeeding syndrome

A

Malignancy

AN
Alcoholism

GI surgery

Starvation

2398
Q

Prevention of refeeding syndrome

A

Identify and monitor at-risk patients

Liaise with dietician

2399
Q

Rx of refeeding syndrome

A

Identify at risk patients in advance

Parenteral and oral PO4 supplementation

Rx complications

2400
Q

How to assess Nutrition

A

Clinically:

Hx: weight loss, diet

Examination: skin fat, dry hair, pressure sores, chelitis, Weight and BMI (<20)

Anthropometric

Skin-fold thickness

Arm circumference

Ix

Albumin

Transthyretin (prealbumin)

Phosphate

2401
Q

Daily nutritional requirements

A

per kg:

20-40kCal

Carbs: 2g

Fat: 3g

Protein 0.5-1g

Nitrogen: 0.2-0.4g

2402
Q

Delivery of enteral nutrition

A

PO is best, consider semi-solid diet if risk of aspiration

Other options include NGT

PEG

Jejunostomy

Build up feeds gradually to prevent diarrhoea

2403
Q
A

PEG tube

2404
Q
A

jejunostomy feeding

2405
Q

Feed options for enteral nutrition

A

Oral supplements

Polymeric e.g. osmolite, jevity. Intact proteins, starches and long chain FAs

Disease specific: e.g. redcued branch chain AAs in hepatic encephalopathy

Elemental e.g. simple AAs and oligo/monosaccharides, require minimal digestion and used if abormal GIT e.g. in Crohn’s

2406
Q

indications for enteral nutrition

A

Catabolic: sepsis, burns, major Sx

Coma/ITU

Malnutrition

Dysphagia: stricture, stroke

2407
Q

Cx of NGT

A

Nasal trauma

Malposition or tube blockage

2408
Q

Cx of feeding

A

Feed intolerance-> diarrhoea

Electrolyte imbalance

Aspiration

Refeeding syndrome

2409
Q

Parenteral nutrition

A

May be total or used to supplement enteral feeding

Combined with H2O to deliver total daily requirements

2410
Q

Indications for(T)PN

A

Prolonged obstruction or ileus (>7d)

High output fistula

Short bowel syndrome

Severe Crohn’s

Severe malnutrition

Severe pancreatitis

Unable to swallow e.g. oesophageal Ca

2411
Q

Delivery of TPN

A

Delivered centrally as high osmolality is toxic to veins

ST: CV catheter

LT: Hickmann or PICC line

Sterility is essential, use line only for PN

2412
Q

Monitoring of parenteral nutrition

A

Standard: weight, fluid balance and urine glucose daily. Zn, MG weekly

Initially: blood glucose, FBC, U+E, PO4 daily. LFTs 3/w

Once stable: blood glucose, FBC, U+E +PO4, 3/w

LFTs weekly

2413
Q

Contents of TPN

A

2000kCal: 50% fat, 50% carbs

10-14g nitrogen

Vitamins, minerals and trace elements

2414
Q

Cx of TPN

A

Line related:

Pneumothorax/haemothorax

Cardiac arrythmia

Line sepsis

Central venous thrombosis: PE or SVCO

Feed-related:

Villous atrophy of GIT

Electrolyte disturbance: refeeding syndrome, hypercapnea from excessive CO2 production

Hyperglycaemia and reactive hypoglycaemia

Line sepsis: increased risk with TPN

Vitamin and mineral deficiencies

Liver disease:

Fatty liver is usually a more long term complication of TPN, though over a long enough course it is fairly common. The pathogenesis is due to using linoleic acid (an omega-6 fatty acid component of soybean oil) as a major source of calories.[10][11] TPN-associated liver disease strikes up to 50% of patients within 5–7 years, correlated with a mortality rate of 2–50%. Onset of this liver disease is the major complication that leads TPN patients to requiring an intestinal transplant

2415
Q
A
2416
Q
A
2417
Q
A
2418
Q

Where is the facet for attachment of PCL?

A

Facet for attachment of the posterior cruciate ligament – Found on the medial wall of the intercondylar fossa, it is a large rounded flat face, where the posterior cruciate ligament of the knee attaches

2419
Q

Facet for attachment of anterior cruciate ligament

A

Found on the lateral wall of the intercondylar fossa, it is smaller than the facet on the medial wall, and is where the anterior cruciate ligament of the knee attaches.

2420
Q
A
2421
Q
A
2422
Q
A
2423
Q

Where is the common peroneal nerve found on the fibula?

A

Proximal end

On the posterolateral surface

2424
Q

What are the proximal tarsal bones?

A

Talus and calcaneus

2425
Q

What is the main function of the talus?

A

Transmit forces from the tibia to calcaneus

2426
Q

What are the intermediate tarsal bones?

A

Navicular: articulates with the talus posteriorly, the cuneiform bones anteriorly and the cuboid bone laterally.

Attachment on plantar surface for the attachment of tibialis posterior

2427
Q

What are the distal group of tarsal bones?

A

Cuboid and three cuneiforms

2428
Q
A
2429
Q

What are the stabilising factors at the hip joint?

A

Acetabulum

Fibrocartilaginous collar

Iliofemoral, pubofemoral and ischiofemoral ligaments also provide a degree of stability

2430
Q

What are the movements that can be carried out at the hip joint?

A

Flexion

Extension

Abduction

Adduction

Lateral rotation

Medial rotation

2431
Q

What determines the degree of flexion at the hip joint?

A

Knee flexion, knee flexion relaxes the hamstring muscles and increases the range of flexion

2432
Q

Hilton’s law

A

Hilton’s law, espoused by John Hilton in a series of medical lectures given in 1860–1862,[1] is the observation that in the study of anatomy, the nerve supplying the muscles extending directly across and acting at a given joint also innervate the joint.[2]

For example, the musculocutaneous nerve supplies the elbow joint of humans with pain and proprioception fibres. It also supplies coracobrachialis, biceps brachii, brachialis, and the forearm skin close to the insertion of each of those muscles.

2433
Q

What are the movements that can take place at the knee joint?

A

Extension

Flexion

Lateral rotation

Medial rotation

2434
Q

Why is the lateral ligament more likely to be sprained at the ankle?

A

Lateral ligament is weaker than medial ligament

Lateral ligament resists inversion

2435
Q

What is a Pott’s fracture?

A

Term used to describe a bimalleolar or trimalleolar fracture

Forced eversion pulls on the medial ligaments, producing an avulsion fracture of the medial malleolus.

The talus moves laterally, breaking off the lateral malleolus.

The tibia is then forced anteriorly, shearing off the distal and posterior part against the talus.

2436
Q
A

Pott’s (trimalleolar fracture)

2437
Q

Primary survery in trauma

A

Address problems in 1o survey in ABCDE order

2438
Q

1o survey Airway

A

Check for airway compromsie:

Ask pt question

Stridor

Orofacial inury or burns

Visualise airway and use suction if necessar

Manoeuvres to open airway:

Jaw thrust

Adjuncts if compromsie/potential compromised

NPA: gag reflex present

OPA: no gag refelex (stop tongue swallowing)

Emergency airways:

Needle circothyroidotomy or surgical cric

Definitive airways: no risk of aspiration

ETT

Tracheostomy

2439
Q

1o survery: C spine

A

Maintain in-line cervical support to keep neck stable

Place pt in hard collar and sandbags with tape

2440
Q

1o survery Breathing

A

Start 15l O2 via non-rebreathe mask (Hudson)

Inspection of chest

RR and chest expansion

Position of trachea

Breath sounds, vocal resonance

Perucssion

ABG

2441
Q

Respiratory distress

Raised JVP and reduced BP

Tracheal deviation and displaced apex

Reduced air entry and reduced vocal resonance

Hyper-resonant to percussion

A

Tension pneumothorax

2442
Q

Rx: tension pneumothorax

A

Insert large bore venflon into 2nd ICS MCL

Insert ICD later

2443
Q

Mx of open sucking chest wounds

A

Convert to closed wounds by covering with damp occlusive dressing stuck down on both sides

2444
Q

1o survery: Circulation

A

Two large bore cannulae: 14G/16G in each ACF

FBC, U+E, X-match (6U), clotting, VBG

Assess:

Inspection: pale, sweaty, active bleeding

Vascular status: BP, HR, JVP, heart sounds, cardiac monitoring

End-orgna: consciousness, UO

Sites of haemorrhage:

CAP: use pelvic binder

Floor

Mx: if haemodynamic compromsie give 2l warm Hartmann’s stat

Consider further colloid/blood

Inesrt CVP and catheter (after PR) to guide resus

2445
Q

How to assess response to fluids

A

Use UO, lactate, BP

2446
Q

Rapid respsonse to fluid resus?

A

Usually <20% loss

Slow fluid response to maintenance if haemodynamically stable

2447
Q

Transient response to fluid resus

A

20-40% loss

On-going losses or inadequate resus

2448
Q

No response to fluid resus

A

Exsanguinating haemorrhage-> theatre

Consider non-haemorrhagic shock: tamponade, pnuemothorax

2449
Q

Assessing disability

A

AVPU or GCS

PEARL

2450
Q

Exposure Assessment

A

Completely undress patient

Perform leg roll and PR: feel for high riding prostate (urethral rupture)

Look for bleeding

Prevent hypothermia

2451
Q

At end of primary survery?

A

Repate primary survery again

2452
Q

2o survery: Hx

A

Allergies

Medications

PMHx

Last ate/ drunk

Events

2453
Q

Examination in 2o survery

A

Head to toe

Examine every system

2454
Q

Ix in 2o survery

A

Trauma series:

C spine: lat and peg

CXR

Pelvis

FAST (focusseed assessment with sonogrpahy in trauma)

CT: when patient is stable

2455
Q

Assessing C-spine radiographs: views

A

Lateral

AP

Open-mouth Peg view

2456
Q

For adequacy of C-spine assessment

A

Need C7-T1 junction visualised

May need swimmer’s view with abducted arm

2457
Q

What are the 4 lines of alignment to assess in C-spine

A

Anterior vertebral bodies

Anterior vertebral canal

Posterior vertebral canal

Tips of spinous processes

2458
Q

Assessment of bones in C-spine radiograph

A

Shapes of bodies

Laminae

Processes

2459
Q

Assessment of cartilage in c-spine

A

IV discs should be equal height

2460
Q

Assesment of C-spine radiograph: soft tissue

A

Width of soft tissue shadow anterior to upper vertebrae should be 50% of vertebral width

2461
Q

C-spine assessment- systematic: lateral

A

C-spine systematic approach - Normal Lateral 1

Coverage - All vertebrae are visible from the skull base to the top of T2 (T1 is considered adequate)

  • If T1 is not visible then a repeat image with the patient’s shoulders lowered or a ‘swimmer’s’ view may be necessary

Alignment - Check the Anterior line (the line of the anterior longitudinal ligament), the Posterior line (the line of the posterior longitudinal ligament), and the Spinolaminar line (the line formed by the anterior edge of the spinous processes - extends from inner edge of skull)

  • GREEN = Anterior line
  • ORANGE = Posterior line
  • RED = Spinolaminar line

Bone - Trace the cortical outline of all the bones to check for fractures

Note: The spinal cord (not visible) lies between the posterior and spinolaminar lines

2462
Q
A

-spine systematic approach - Normal Lateral 2

Disc spaces - The vertebral bodies are spaced apart by the intervertebral discs - not directly visible with X-rays. These spaces should be approximately equal in height

Pre-vertebral soft tissue - Some fractures cause widening of the pre-vertebral soft tissue due to pre-vertebral haematoma

  • Normal pre-vertebral soft tissue (asterisks) - narrow down to C4 and wider below
  • Above C4 ≤ 1/3rd vertebral body width
  • Below C4 ≤ 100% vertebral body width

Note: Not all C-spine fractures are accompanied by pre-vertebral haematoma - lack of pre-vertebral soft tissue thickening should NOT be taken as reassuring

Edge of image - Check other visible structures

2463
Q

C spine systmatic assessment AP

A

C-spine systematic approach - Normal AP

Coverage - The AP view should cover the whole C-spine and the upper thoracic spine

Alignment - The lateral edges of the C-spine are aligned (red lines )

Bone - Fractures are often less clearly visible on this view than on the lateral

Spacing - The spinous processes (orange) are in a straight line and spaced approximately evenly

Soft tissues - Check for surgical emphysema

Edges of image - Check for injury to the upper ribs and the lung apices for pneumothorax

2464
Q

C-spine assessment odontoid peg

A

C-spine normal anatomy - Open mouth view

This view is considered adequate if it shows the alignment of the lateral processes of C1 and C2 (red circles)

The distance between the peg and the lateral masses of C1 (asterisks) should be equal on each side

Note: In this image the odontoid peg is fully visible which is not often achievable in the context of trauma due to difficulty in patient positioning

2465
Q
A

C2 odontoid peg fracture - Lateral view

The C2 bone ‘ring’ is incomplete due to a fracture

The odontoid peg is displaced posteriorly

2466
Q
A

C2 ‘hangman’ fracture - Lateral view

Loss of alignment at C2/C3 with anterior displacement of C2 (large arrow)

Following the cortical outline of C2 (white line) reveals discontinuity due to a fracture

2467
Q
A

Bilateral perched facets - Lateral view

(Same patient as image below)

Loss of alignment of all three lines at C5/C6 with ‘perching’ of the C5 facet on the C6 facet (ring)

No fracture is visible

The pre-vertebral soft tissue is widened due to a haematoma

Bilateral perched facets - AP view

(Same patient as image above)

There is widening of space between the C5 and C6 spinous processes (SP) with loss of normal alignment

Again no fracture is demonstrated

Note

The spinal canal lies between the posterior (Orange) and spinolaminar (Red) lines

Derangement of the spinal canal due to this injury results in a high incidence of spinal cord injury

2468
Q

Clinical clearance for C-spine injury crieria

A

NEXUS criteria

2469
Q

NEXUS Criteria

A

Focal neurologic deficit

Midline spinal tenderness

Altered level of consciousness

Intoxication present

Distracting injury

If no to all of the above, C-spine can be cleared clinically

2470
Q

Method for clinical clearance of c-spine

A

Examine for brusing or deformity

Palpate for deformity and tenderness

Ensure pain-free active movement

2471
Q

Indications for radiological C-spine clearance

A

Patient doesn’t meet criteria for clinical clearance

2472
Q

Radiographical C-spine clearance

A

Radiograph initially: clear if normal radiograph and clinical exam

CT C-spine if abnormal radiograph or clinical exam

2473
Q

How can haemorrhagic shock be classified?

A

By degree of blood loss

There is a correlation clinically between volume loss and clinical manifestation

2474
Q

What proportion of body mass is circulating blood volume?

A

7%

2475
Q

Classes of haemorrhagic shock

A

1: 0-15% loss: 750ml
2: 15-30% loss 750-1500ml
3: 30-40% loss 1500-2000,l
4: >40% loss >2000ml

2476
Q

Haemorrhagic shock, what grade?

RR normal

HR normal

BP normal

UO normal

Mental Normal

A

1: 0-15%

2477
Q

Haemorrhagic shock, what grade?

RR >20

HR >100

BP normal

UO <30

Anxious ++

A

2 15-30%

2478
Q

Haemorrhagic shock, what grade?

RR >30

HR >20

BP reduced

UO 5-20

Confused

A

3

30-40%

2479
Q

Haemorrhagic shock, what grade?

RR >35

HR >140

BP reduced ++

UO <5

Lethargic

A

4

>40%

2480
Q

Def: neurogenic shock

A

Disruption of symapthetic nervous system

2481
Q

Causes of neurogenic shock

A

Spinal anaesthesia

Hypoglycaemia

Cord inury above T5

Closed head injuries

2482
Q

Presentation of neurogenic shock

A

Hypotension

Bradycardia

Warm extremities

2483
Q

Mx of neurogenic shock

A

Vasopressors: vasopressors and noradrenaline

Atropine: reverse bradycardia

2484
Q

Def: spinal shock

A

Acute spinal cord transection

Loss of all voluntary and reflex activity below the level of injury

2485
Q

Trauma

Hypotonic paralysis

Loss of sensation

Bladder retention

A

?Spinal shock

2486
Q

DDx for life-threatening Chest injuries

ATOM FC

A

Airway obstruction

Tension pneumothorax

Open pneumothorax (sucking)

Massive haemothorax

Flail chest

Cardiac tamponade

2487
Q

Def: massive haemothorax

A

Accumulation of >1.5l of blood in chest cavity

Usually caused by disruption of hilar vessels

2488
Q

Signs of chest wall trauma

Reduced BP

Reduced expansion

Reduced breath sounds and vocal resonance

Stony dull percussion

A

Massive haemothorax

2489
Q

Mx of massive haemothorax?

A

X-match 6u

Large-bore chest drain with heparinised saline for autotransfusion

Thoracotamy if >1.5ml or >200ml/h

2490
Q
A

Massive haemothorax

2491
Q

Def: flail chest

A

Anterior or lateral # of >2 adjacent ribs in 2 places

Flail segment moves paradoxically with respiration

Reduced oxygenation: due to underlying pulmonary contusion and reduced ventilation of affected segment

2492
Q

Ix in flail chest

A

CXR/CT chest: pulmonary contusion (white)

Serial ABGs: Reduced PaO2:FiO2 ratio

2493
Q

Rx flail chest

A

O2

Good analgesia: PCA, epidural

Persistant respiratory failure: PPV

2494
Q
A

Flail chest

2495
Q

Def: cardiac tamponade

A

Disruption of myocardium or great vessles-> blood in pericardium-> reduced filing and contraction-> shock

Usually results from penetrating trauma

2496
Q

Raised JVP/distended neck veins

Reduced BP

Muffled heart sounds

SBP fall of >10mmHg on inspiration

Kussmaul’s sign: raised JVP on inspiration

Intensely restless patient

A

Cardiac tamponade

2497
Q

What is Beck’s triad

A

Seen in Cardiac tamponade:

Raised JVP/distended neck veins

Reduced BP

Muffled heart sounds

2498
Q

What is pulsus paradoxus

A

Seen in cardiac tamponade

SBP fall of >10mmHg on inspiration

Normally during inspiration, systolic blood pressure decreases ≤10 mmHg.,[1] and pulse rate goes up slightly. This is because inspiration makes intra-thoracic pressure more negative relative to atmospheric pressure. The negative pressure in the thorax increases venous return, so more blood flows into the right side of the heart. However, the decrease in intra-thoracic pressure also expands the compliant pulmonary vasculature. This increase in pulmonary blood capacity pools the blood in the lungs, and decreases pulmonary venous return, so flow is reduced to the left side of the heart. Also, the increased systemic venous return to the right side of the heart expands the right heart and directly compromises filling of the left side of the heart. Reduced left-heart filling leads to a reduced stroke volume which manifests as a decrease in systolic blood pressure. The decrease in systolic blood pressure leads to a faster heart rate due to the baroreceptor reflex, which stimulates sympathetic outflow to the heart.

Although it might be tempting to expect during inspiration that the increased volume of the right ventricle causes the septum to bulge dramatically into the left ventricle, this is unlikely under normal physiologic conditions, as there is still a large pressure gradient between the right and left ventricles during inspiration. However, during cardiac tamponade, this is the case. Here, pressure equalizes between all of the chambers of the heart.[4] This means that there is a zero-sum game, and as the right ventricle gets more volume, it can push the septum into the left ventricle and therefore reduce the volume of the left ventricle. This additional loss of volume of the left ventricle that only occurs with equalization of the pressures (as in tamponade) allows for the further reduction in volume, so cardiac output is reduced, leading to a further decline in BP. However, in situations where the left ventricular pressure remains higher than the pericardial sac (most frequently from coexisting disease with an elevated left ventricular diastolic pressure), there is no pulsus paradoxus

2499
Q

Kussmaul’s sign

A

Raised JVP on inspiration

Seen in Cardiac tamponade amongst other conditiosn

Ordinarily the JVP falls with inspiration due to reduced pressure in the expanding thoracic cavity and the increased volume afforded to right ventricular expansion during diastole. Kussmaul sign suggests impaired filling of the right ventricle due to a poorly compliant myocardium or pericardium. This impaired filling causes the increased blood flow to back up into the venous system, causing the jugular vein distension (JVD) and is seen clinically in the internal jugular veins becoming more readily visible.

2500
Q

Ix in cardiac tamponade

A

US: FAST or TTE

CXR: enlarged pericardium

CVP >12mmHg

ECG: low voltage QRS with electrical alternans

2501
Q

low voltage QRS with electrical alternans, tachycardic

A

Cardiac tamponade

2502
Q
A

Massive pericardial effusion

Massive pericardial effusion produces a triad of…

Low voltage

Tachycardia

Electrical alternans

Electrical alternans is…

when consecutive, normally-conducted QRS complexes alternate in height.

produced by the heart swinging backwards and forwards within a large fluid-filled pericardium.

Patients with this ECG pattern need to be immediately assessed for clinical and echocardiographic evidence of tamponade.

2503
Q

Mx of cardiac tamponade

A

Pericardiocentesis: spinal needle in R subxiphoid space aiming at 45o towards R tip of left scapula

Thoracotamy may be needed

2504
Q

Which ribs are usually # in trauma

A

Usually 5th-9th ribs

of upper 4 ribs= high energy trauma

2505
Q

2o survery chest injuries

A

Rib#

Sternal#

Pulmonary contusion

Myocardial contusion

Contained aortic disruption

Diaphragmatic injury

Oesophageal disruption

Trachoebronchial disruption

2506
Q

Cx if rib #

A

Pneumothorax

Lacerate thoracic or abdominal viscera

2507
Q

Rx rib #

A

Good analgesia

NSAIDs + opioids

Intrapleural analgesia

Intercostal block

2508
Q

Features of sternal #

A

Usually MVA driver vs steering wheel

Risk of mediastinal injury

2509
Q

Rx sternal #

A

Analgesia, admit, observe

Cardiac monitor

Troponin: rule out myocardial contusion

2510
Q

Features of pulmonary contusion

A

Usually due to rapid deceleration injury or shock waves

May-> ARDS

2511
Q

Dyspnoea, haemopytsis, resp failure following Trauma

A

?Pulmonary contusion

2512
Q

Ix in ?pulmonary contusion

A

CXR: opacification
Serial ABGs: reduced PaO2: FiO2 ratio

2513
Q

Rx pulmonary contusion

A

O2, ventilate if necessary

2514
Q

Direct blunt trauma over precordium

ECG: abnormal arrythmias

Troponin raised

A

?Myocardial contusion

2515
Q

Rx in myocardial contusion

A

Bed rest, cardiac monitoring, Rx arrythmias

2516
Q

Rapid deceleration injury (80% immediately fatal)

Initially stable but-> hypertensive

CXR: wide mediatsinum, deviation of NGT

Requries cardiothroacic consult

A

?Contained aortic disruption

2517
Q

Penetrating injury below 5th rib or high energy compression

CXR: visceral herniation, CT

A

Diaphragmatic injury

2518
Q

Usually penetrating trauma-> mediastinitis

Ix: CXR: pneumoediastinum, surgical emphysema

A

Oesophageal disruption

2519
Q

Persistent pneumothorax

Pneumomediastinum

A

Tracheobronchial disruption

2520
Q

Mechanisms of abdominal trauma

A

Penetrating:

All require exploration as tract may be deeper than it appears

Blunt:

Have a high index of suspicion for taking to theatre

2521
Q

Specific Ix in abdominal trauma

A

Urine dip:

Heamturia suggests injury to renal tract

FAST scan

Diagnostic peritoneal lavage

2522
Q

Features of FAST scan

A

Replacing DPL in most centres

Check for fluid in abdomen, pelvis and pericardium

Can be extended to look for pneumothoraces

2523
Q

Areas examined in FAST scan

A

Perihepatic space (Morrison’s pouch)

Perisplenic space

Pericardium

Pelvis

90% sensitive

2524
Q

Features of diagnostic peritoneal lavage

A

Insert urinary catheter and NGT (decompression to minimise risk of injury)

Midline incision through skin and fascia @ 1/3rd distance from umbilicus to pubic symphysis (arcuate line)

Carefully dissect to peritoneum and insert a urinary catheter

Instil 10ml/kg warmed Hartman’s

Drain fluid back into bag and send sample to lab

+ve= >100,000 RBCs/mm, bile/intestinal contents

2525
Q

Advantages and disadvanatges of DPL

A

98% sensitive for intra-abdominal haemorrhage

Useful if FAST unavailable

May be better for identifying injury to hollow viscus

Unable to identify retroperitoneal injury

2526
Q

What are the indications for emergency laparotomy in abdo tauma

A

Unexplained shock

Peritonism: rigid or silent abdomen

Evisceration: bowel or omentum

Radiological evidence of intraperitoneal gas

Radiological evidence of ruptured diaphragm

Gunshot wounds

+VE DPL or CT

2527
Q

Aim of damage control surgery

A

Early Mx of abdominal trauma should focus on damge control to limit physiological stress

Control haemorrhage: ligation and packing

Control contamination: stabilise in ITU

2528
Q

Kehr’s Sign

A

Shoulder tip pain 2o to blood in peritoneal cavity

Left Kehr’s is classic symptom of ruptured spleen

2529
Q

Classification of splenic inury (AAST)

A

1: capsular tear
2: tear + parenchymal injury
3: tear up to hilum
4: complete fracture

2530
Q

Mx of splenic injury

A

Haemodynamically unstable: laparotomy

Stable 1-3: observation in HDU

Stable 4: consider laparotomy: suture lac or partial/complete splenectomy

2531
Q

Mx of liver injury

A

Conservative if capsule is intact

Suture laceration

Partial hepatectomy

Packing

2532
Q

Mx of bowel injury

A

Resection may be required

2533
Q

Mx of bladder injury

A

Often associated with pelvic injury

Intraperitoneal rupture requires laparoscopic repair with urethral and suprapubic drainage

Extraperitoneal rupture can be treated conservatively with urethral drainage

Give prophylactic Abx

2534
Q

Classification of urethral injury

A

Anterior

Posterior

2535
Q

Anterior urethral injury

A

Injury to spongy urethra (penile + bulbar)

Occur following straddling injuries or instrumentation

2536
Q

Posterior injury to urethra

A

Membranous urethra

Occur following pelvic #s

2537
Q

Often associated with pelvic fracture

Blood in urethral meatus or scrotum

Perineal brusing

High-riding prostate

Inability to micturate and palpable bladder

A

?Urethral injury

2538
Q

Ix in urethral injury

A

Retrograde urethrogram

2539
Q

Mx of urethral injury

A

Suprapubic catheter

Surgical repair

2540
Q
A

Contained aortic dissection

2541
Q
A

Pneumomediastinum

2542
Q

Epidemiology of head injury

A

Head injury alone or in combination with other injuries is the commonest cause of trauma death (50%)

2543
Q

Def and classification of 1o brain injury

A

Occcurs at time of injury and is a result of direct or indirect injury to brain tissue

Diffuse:

Concusison/ mild TBI: temporary reduction in brain function.

Headache, confusion, visual symptoms, amnesia

Diffuse axonal injury:

Shearing forces disrupt axons. May lead to coma and persistent vegetative state.

Autonomic dysfunction: fever, HTN, seating

Focal:

Contusion e.g. coup and contra-coup, may have focal neurological deficit

Intracranial haemorrhage: EDH, SDH, SAH, ICH and laceration

2544
Q

Def and causes of 2o brain injury

A

Occurs after 1o injury

Hypoxia

Hypercapnoea

Hypotension

Raised ICP

Infection

2545
Q

What is the Monroe-Kelly Doctrine

A

Cranium is a rigid box. Therfore total volume of intracranial contents must remain constant if ICP is not to change.

Increase in a volume of one constitutent leads to a compensatory reduciton in another e.g. CSF, bloods (especially venous)

These mechanisms can compensate for a volume change of around 100ml before ICP increases

As autoregulation feails, ICP raipidly-> herniation

2546
Q

Features of Cerebral Blood Flow and ICP

A

CBF proportion to CPP x radius of vessels

CPP= MABP-ICP

Raised ICP-> reduced CPP-> Reduced CBF

Reduced CBF-> autoregulatory-> vasodilation-> increased volume-> increased ICP

Need to prevent or attenuate this vicious cycle by

ventilating to normocapnoea

IV fluid to normovolaemia

Mannitol bolus acutely

2547
Q

How does ventilating to normocapnoea neuroprotect

A

Ventilate to low normocapnia [end

-

tidal CO2 of 30 mmHg, 4.0KPa]. This

equates to a

PaCO2 of approximately 4.5KPa in normal individuals. This

minimises the risk of cerebral vasodilation (high PaCO2) and cerebral

vasoconstriction (low PaCO2)

2548
Q

Cushing’s Triad

What does it suggest

A

HTN

Bradycardia

Irregular breathing

Imminent herniation

2549
Q

What are the typical clinical criteria for use of mannitol or hypertonic saline

A

Unilateral or bilateral pupil dilation in association with GCS <8

Progressive hypertension and bradycardia in association with GCS <8

2550
Q

Key points in head injury Hx

A

LOC

Amnesia: anterograde worse

N+V

Fits

Focal neurology

Mechanisms

Drugs e.g. antiplatelets, warfarin

2551
Q

RFs: for burns

A

Age: children and elderly

Co-morbidities: epilepsy, CVA, dementia, mental illness

Occupation

2552
Q

How can burns be classfiied?

A

Superficial

Partial thickness

Full thickness

2553
Q

Features of superficial burn

A

Erythema

Painful

e.g. sunburn

2554
Q

Features of partial thickness burns

A

Heal within 2-3w if not complicated

Superficial:

No loss of dermis

Painful

Blisters

Deep:

Loss of dermis but adnexae remain

Healing from adenexae e.g. follicles

V painful

2555
Q

Features of full thickness burns

A

Complete loss of dermis

Charred, waxy, white, skin

Anaesthetic

Heal from edges-> scar

2556
Q

Early Cx of burns

A

Infection: loss of barrier funciton, necrotic tissue, SIRS

Hypovolaemia: loss of fluid in skin and increased capillary permeability

Metabolic distrubance: raised K, raised myoglobin, Raised Hb-> AKI

Compartment syndrome: circumferential burns

Peptic ulcers: Curling’s ulcers

Pulmonary: laryngeal oedema, CO poisoning, ARDS

Renal and hepatic impariment

2557
Q

Intermediate Cx of burns

A

VTE

Pressure sores

2558
Q

Late Cx of burns

A

Scarring

Contractures

Psychological problems

2559
Q

Assessment of burns: Wallace Rule of 9s

A

Head and neck: 9%

Arms: 9%

Torso 18% front and back

Legs: 18% each

Perineum 1%

Palms 1%

May also use Lund and Browder charts

2560
Q

General principles of burn managements

A

Based on ATLS principles

Specific concerns with burns: secure airway, manage fluids, prevent infection

2561
Q

Airway in burns:

A

Examine for respiratory burns: soot in oral or nasal cavity, burnt nasal hairs, hoarse voice/stridor

Flexible laryngoscopy can be helpful

Conider early intubation and dexamethasone

2562
Q

Breathing in burns

A

100% O2

Exclude constricting burns

Look for signs of CO poisoning e.g. headache, N+V, confusion, cherry red appearance

ABG: COHb level, SpO2 unreliable if CO poisoning

2563
Q

Circulation in burns

A

Fluids losses may be huge

2x large bore cannulae in each ACF

Bloods: FBC, U+E, G+S, X match

Start 2l warmed Hartmann’s immediately.

Formula guide additional fluid requirements in burns patients

2564
Q

Parkland formula in burns

A

1st 24h

4x weight (kg) x % burn=ml Hartmann’s in 24h

Replace fluid from time of burn

Give half in 1st 8h

Best guide is UO: 30-50ml

2565
Q

Alternative burns fluid formula

A

Muir and Barclay, relates to albumin as fluid replacement

Parkland formula is preferred by British Burns Association

2566
Q

Mx of Burns

A

Analgesia: morphine

Dress partial thickness burns:

biological e.g. cadaveric skin

synthetic

cream e.g. flamazine *silver sulfadiazine + sterile film

Full thickness burns: tangential excision debridement, split thickness skin grafts

Circumferential burns may require escharotomy to prevent compartment syndrome

Anti-tetanus toxoid

Consider prophylactic Abx esp. pseudomonas

2567
Q

Def: hypothermia

A

Core (rectal) T <35

2568
Q

What are the 4 mechanisms of body heat loss

A

Radiation: 60%

Conduction: 15%

direct contatct, primary means is cold water immersion

Convection: 15%

Evaporation: 10%

2569
Q

Aetiological classification of hypothermia

A

1o: environental exposure
2o: change in temperature set point e.g. age-related, hypothyroidism, autonomic neuropathy

2570
Q

Features of mild hypothermia

A

32-35 deg

Shivering

Tachycardia

Vasoconstriction

Apathy

2571
Q

Features of moderate hypothermia

A

28-32 deg

Dysrhythmia, bradycardia, hypotension

J waves

Reduced reflexes, dilated pupils, reduced GCS

2572
Q

Features of severe hypothermia

A

<28 deg

VT-> VF-> cardiogenic shock

Apnoea

Non-reactive pupils

Coagulopathy

Oliguria

Pulmonary oedema

2573
Q
A
2574
Q

Ix in hypothermia

A

Rectal/ear temperature

FBC, U+E, glucose

TFTs, blood gas

ECG: J waves between QRS and T wave, arrythmias

2575
Q

Mx of hypothermia

A

Cardiac monitor

Warm IVI 0.9% saline

Urinary catheter

Consider Abx for prevention of pneumonia routine if temp <32 and >65

Slowly rewarm: reheating too qucikly-> peripheral vasodilation and shock

Aim for 0.5 deg /hr

Passive external: blankets, warm drinks

Active external: warm water or warmed air

Active internal: mediastinal lavage and CPB (severe hypothermia only)

2576
Q

Cx in hypothermia

A

Arrhythmias

Pneumonia

Coagulopathy

ARF

2577
Q

Features of oesophageal anatomy

A

25cm long mucscular tube (40cm from GOJ to lips)

Starts at level of cricoid cartilage (C6)

Lies in the visceral compartment of the neck

Runs in posterior mediastinum and passes through right crus of diaphragm at T10

Continues before entering the cardia

2578
Q

Level of cricoid cartilage?

A

C6

2579
Q

Level of oesophagus passing through diaphragm

A

Right crus

T10

2580
Q

3 locations of oesophageal narrowing?

A

Level of cricoid

Posterior to left main bronchus and aortic arch
LOS

2581
Q

How is the oesophagus divided?

A

Into 3rds reflecting change in musculature from striated to mixed to smooth

2582
Q

What is the level of bronchoaortic oesophageal constriction?

A

T4

2583
Q

What is the Z line?

A

Where the oesophagus changes from squamous epithelium to gastric columnar aka squamocolumnar junction

2584
Q

Def: dysphagia

A

Concious difficulty in swallowing

2585
Q

Def: aphagia

A

Complete inability to swallow

2586
Q

Def: odynophagia

A

Painful swallowing

2587
Q

How can dysphagia be classified?

A

Considered from the point of its chronciity to be acute, chronic, or progressive

2588
Q

How can the causes of dysphagia be classified?

A

Inflammatory

Neurological/motility disorders

Mechanical obstruction

2589
Q

Inflammatory causes of dysphagia

A

Tonsillitis, pharyngitis

Oesophagitis: GORD, candida

Oral candidiasis

Apthous ulcers

2590
Q

Neurological/motility causes of dysphagia

A

Local:

Achalasia

Diffuse oesophageal spasm

Nutcracker oesophagus

Bulbar/pseudobulbar palsy (CVA, MND)

Systemic:

Systemic sclerosis/CREST

MG

2591
Q

Mechanical causes of dysphagia

A

Luminal:

FB

Large food bolus

Mural:

Benign stricutre: Web, oesophagitis, Trauma (e.g. OGD)

Malignant stricture: pharynx, oesopgaus, gastric

Pharyngeal pouch

Extra mural:

Retrosternal goitre

Rolling hiatus hernia

Lung Ca

Mediastinal LNs e.g. lymphoma

Thoracic aortic aneurysm

2592
Q

Ix in dypshagia

A

Upper GI endoscopy

Ba swallow

Manometry

2593
Q

Pathophysiology of achalasis

A

Degeneration of myenteric plexus (Auerbach’s)-> decresed peristalsis with failure of LOS to relax

2594
Q

Causes of achalasia

A

1o/idiopathic: commonest

2o: Chagas (T. cruzi)

2595
Q

Dysphagia: liquids then solids

Regurgitation esp @ night

Substernal cramps

Weight loss

A

?Achalasia

2596
Q

Cx of achalasia

A

Oesophageal SCC in 3-5%

2597
Q

Ix in achalasia

A

Ba swallow: dialted tapering oesophagus, Brid’s beak

Manometry: failure of relaxation and reduced peristalsis

CXR: widened mediatsinum, double heart border

OGD: exclude malignancy

2598
Q
A

Oesophageal achalasia

Bird’s beak

2599
Q

Mx of achalasia

A

Medical: CCBs, nitrates

Interventional: botox injection, endoscopic balloon dilatation

Surgery: Heller’s cardiomyotomy eith peroral, endoscopic

2600
Q

Def: Zenker’s diverticulum

A

Pharyngeal pouch

Outpouching between crico and thyropharyngeal components of the inferior pharyngeal constrictor at an area of weakness

known as Killian’s dehiscence

Defect usually occurs posteriorly but swelling usually bulges to left side of neck

Food debris-> pouch expansion-> oesophageal compression-> dysphagia

2601
Q

Regurgitation

Halitosis

Gurgling sounds

A

Pharyngeal Pouch (Zenker’s diverticulum)

2602
Q

Rx of Pharyngeal pouch

A

Excision

Endoscopic stapling

2603
Q

Features of diffuse oesophageal spasm

A

Intermittent severe chest pain +/i dysphagia

Barium swallow shows corkscrew appearance

Diffuse esophageal spasm (DES) is a condition characterized by uncoordinated contractions of the esophagus, which may cause difficulty swallowing (dysphagia) or regurgitation. In some cases, it may causes symptoms such as chest pain, similar to heart disease. The cause of DES remains unknown.

Certain abnormalities on x-ray imaging are commonly observed in DES, such as a “corkscrew” or “rosary bead esophagus”, although these findings are not unique to this condition. Specialized testing called manometry can be performed to evaluate the motor function of the esophagus, which can help identify abnormal patterns of muscle contraction within the esophagus that are suggestive of DES. The treatment of DES consists primarily of medications, such as acid suppressing agents (like proton pump inhibitors), calcium channel blockers, hyoscine butylbromide, or nitrates. In only extremely rare cases, surgery may be considered. People with DES have higher incidences of gastroesophageal reflux disease (GERD) and anxiety.

2604
Q

“corkscrew” or “rosary bead esophagus”,

A

Diffuse oesophageal spasm

2605
Q
A

DOS

Pathology

Aetiology is unknown but may be related to loss of inhibitory neurones in the distal oesophagus.

Radiographic features

Fluoroscopy

only 60% of barium swallows will be abnormal

<5% will show “corkscrew oesophagus” or “rosary bead oesophagus” where normal peristalsis is interrupted by many tertiary (non-propulsive) contractions occurring in the distal oesophagus

nonperistaltic contractions, pushing contrast in two directions, can be seen (sometimes the only feature) 5

sacculations and pseudodivertucula may be seen 5

2606
Q

Features of nutcracker oesophagus

A

Intermittent dysphagai +/- chest pain

Increased contraction pressure with normal peristalsis

Nutcracker esophagus, or hypertensive peristalsis, is a disorder of the movement of the esophagus characterized by contractions in the smooth muscle of the esophagus in a normal sequence but at an excessive amplitude or duration. Nutcracker esophagus is one of several motility disorders of the esophagus, including achalasia and diffuse esophageal spasm. It causes difficulty swallowing, or dysphagia, to both solid and liquid foods, and can cause significant chest pain; it may also be asymptomatic. Nutcracker esophagus can affect people of any age, but is more common in the sixth and seventh decades of life. The diagnosis is made by an esophageal motility study (esophageal manometry), which evaluates the pressure of the esophagus at various points along its length. The term “nutcracker esophagus” comes from the finding of increased pressures during peristalsis, with a diagnosis made when pressures exceed 180 mmHg; this has been likened to the pressure of a mechanical nutcracker. The disorder does not progress, and is not associated with any complications; as a result, treatment of nutcracker esophagus targets control of symptoms only.[1][2]

2607
Q

Features of Plummer Vinson Syndrome

A

Severe IDA-> hyperkeratinsiation of upper 3rd of oesophagus-> web formation

Premalignant: 20% risk of SCC

2608
Q

Causes of oesophageal rupture

A

Iatrogenic (85-90%): endoscopy, biopsy, dilatation

Violent emesis: Boerhaave’s syndrome

Carcinoma

Caustic ingestion

Trauma: surgical emphysema +/- pneumothorax

2609
Q

Odynophagia

Mediastinitis: tachypnoea, dyspnoea, fever, shock

Surgical emphysema

A

?Oesophageal rupture

2610
Q

Mx of iatrogenic oesophageal rupture

A

PPI

NGT

Abx

2611
Q

Mx of other causes of oesophageal rupture

A

ABC

PPI

ABx

ANtifunglas

Debridement and formation of oesophagocutaneous fistula with T-tube

2612
Q

A 65-year-old man presents with severe epigastric pain radiating to his back. He states that the pain is 9/10 severity. He has associated nausea and vomiting. Serum amylase is raised. You suspect a diagnosis of acute pancreatitis.

How would you initially manage his acute pain?

IV morphine in 1-2mg boluses until comfortable

IM pethidine

IV morphine 10mg STAT

Regular paracetamol

Regular paracetamol and ibuprofen

A

This patient has severe pain which will likely need an opiate to settle. IV morphine titrated in 1-2mg boluses until comfortable would be a sensible first choice. Regular paracetamol would also be sensible as this would be opioid sparing, however this would not be the first choice for initially managing severe acute pain.

2613
Q

WHO Analgesic ladder structure

A

nitially peripherally acting drugs such as paracetamol or non-steroidal anti-inflammatory drugs (NSAIDs) are given.

If pain control is not achieved, the second part of the ladder is to introduce weak opioid drugs such as codeine or dextropropoxyphene together with appropriate agents to control and minimise side effects.

The final rung of the ladder is to introduce strong opioid drugs such as morphine. Analgesia from peripherally acting drugs may be additive to that from centrally-acting opioids and thus, the two are given together.

2614
Q

The World Federation of Societies of Anaesthesiologists (WFSA) Analgesic Ladder for management of acute pain

A

Severe pain and may need controlling with strong analgesics with local anaesthetic blocks and peripherally acting drugs

Second rung on postoperative pain ladder is restoration of oral route and strong opioids may no longer be required with adequate analgesia obtained using combinations of peripherally acting agents and weak opioids

Final step is when pain can be controlled with peripherally acting agents alone

2615
Q

Feautes of local anaesthetics

A

Infiltration of a wound with a long-acting local anaesthetic such as Bupivacaine

Analgesia for several hours

Further pain relief can be obtained with repeat injections or by infusions via a thin catheter

Blockade of plexuses or peripheral nerves will provide selective analgesia in those parts of the body supplied by the plexus or nerves

Can either be used to provide anaesthesia for the surgery or specifically for postoperative pain relief

Especially useful where a sympathetic block is needed to improve postoperative blood supply or where central blockade such as spinal or epidural blockade is contraindicated.

2616
Q

Features of spinal anaesthesia

A

Provides excellent analgesia for surgery in the lower half of the body and pain relief can last many hours after completion of the operation if long-acting drugs containing vasoconstrictors are used.

2617
Q

Side effects of spinal anaesthesia

A

Hypotension

Sensory and motor block

Nausea

Urinary retention

2618
Q

Features of epidural anaesthesia

A

An indwelling epidural catheter inserted. This can then be used to provide a continuous infusion of analgesic agents. It can provide excellent analgesia. They are still the preferred option following major open abdominal procedures and help prevent post operative respiratory compromise resulting from pain.

2619
Q

Disdavanatges of epidural

A
  • Disadvantages of epidurals is that they usually confine patients to bed, especially if a motor block is present. In addition an indwelling urinary catheter is required. Which may not only impair mobility but also serve as a conduit for infection. They are contraindicated in coagulopathies.
2620
Q

Features of transversus abdominal plane block

A

In this technique an ultrasound is used to identify the correct muscle plane and local anaesthetic (usually bupivicaine) is injected. The agent diffuses in the plane and blocks many of the spinal nerves. It is an attractive technique as it provides a wide field of blockade but does not require the placement of any indwelling devices. There is no post operative motor impairment. For this reason it is the preferred technique when extensive laparoscopic abdominal procedures are performed. They will then provide analgesia immediately following surgery but as they do not confine the patient to bed, the focus on enhanced recovery can begin sooner.

2621
Q

Disadvanatges of TAP

A

The main disadvantage is that their duration of action is limited to the half life of the local anaesthetic agent chosen. In addition some anaesthetists do not have the USS skills required to site the injections.

2622
Q

Give some examples of strong opioids

A

Morphine

Pethidine

Tramadol

Buprenorphine

Methadone

Diamorphine

Fentanyl

Hydromorphone

Oxycodone

Used for severe pain arising from deep or visceral structures

2623
Q

Features of morphine

A

Short half life and poor bioavailability.

Metabolised in the liver and clearance is reduced in patients with liver disease, in the elderly and the debilitated

Side effects include nausea, vomiting, constipation and respiratory depression.

Tolerance may occur with repeated dosages

2624
Q

Features of pethidine

A

Synthetic opioid which is structurally different from morphine but which has similar actions. Has 10% potency of morphine.

Short half life and similar bioavailability and clearance to morphine.

Short duration of action and may need to be given hourly.

Pethidine has a toxic metabolite (norpethidine) which is cleared by the kidney, but which accumulates in renal failure or following frequent and prolonged doses and may lead to muscle twitching and convulsions. Extreme caution is advised if pethidine is used over a prolonged period or in patients with renal failure.

2625
Q

Mild opioids

A

Codeine

Dihydrocodeine

2626
Q

Features of paracetamol

A

Inhibits prostaglandin synthesis.

Analgesic and antipyretic properties but little anti-inflammatory effect

It is well absorbed orally and is metabolised almost entirely in the liver

Side effects in normal dosage and is widely used for the treatment of minor pain. It causes hepatotoxicity in over dosage by overloading the normal metabolic pathways with the formation of a toxic metabolite.

2627
Q

Features of NSAIDs

A

Analgesic and anti-inflammatory actions

Inhibition of prostaglandin synthesis by the enzyme Cyclooxygenase which catalyses the conversion of arachidonic acid to the various prostaglandins that are the chief mediators of inflammation. All NSAIDs work in the same way and thus there is no point in giving more than one at a time. .

NSAIDs are, in general, more useful for superficial pain arising from the skin, buccal mucosa, joint surfaces and bone.

Relative contraindications: history of peptic ulceration, gastrointestinal bleeding or bleeding diathesis; operations associated with high blood loss, asthma, moderate to severe renal impairment, dehydration and any history of hypersensitivity to NSAIDs or aspirin.

2628
Q

Management of neuropathic pain

A

First line: Amitriptyline (Imipramine if cannot tolerate) or pregabalin

Second line: Amitriptyline AND pregabalin

Third line: refer to pain specialist. Give tramadol in the interim (avoid morphine)

If diabetic neuropathic pain: Duloxetine

2629
Q

Diabetic patients and preop care

A

Diabetic patients have greater risk of complications.

Poorly controlled diabetes carries high risk of wound infections.

Patients with diet or tablet controlled diabetes may be managed using a policy of omitting medication and checking blood glucose levels regularly. Diabetics who are poorly controlled or who take insulin will a require variable rate intravenous insulin infusion. Potassium supplementation should also be given.

Diabetic cases should be operated on first.

2630
Q

Contraindications to COCP

A

The decision of whether to start a women on the combined oral contraceptive pill is now guided by the UK Medical Eligibility Criteria (UKMEC). This scale categorises the potential cautions and contraindications according to a four point scale, as detailed below:

UKMEC 1: a condition for which there is no restriction for the use of the contraceptive method

UKMEC 2: advantages generally outweigh the disadvantages

UKMEC 3: disadvantages generally outweigh the advantages

UKMEC 4: represents an unacceptable health risk

2631
Q

UKMEC 3

A

more than 35 years old and smoking less than 15 cigarettes/day

BMI > 35 kg/m^2*

family history of thromboembolic disease in first degree relatives < 45 years

controlled hypertension

immobility e.g. wheel chair use

carrier of known gene mutations associated with breast cancer (e.g. BRCA1/BRCA2)

2632
Q

UKMEC 4

A

more than 35 years old and smoking more than 15 cigarettes/day

migraine with aura

history of thromboembolic disease or thrombogenic mutation

history of stroke or ischaemic heart disease

breast feeding < 6 weeks post-partum

uncontrolled hypertension

current breast cancer

major surgery with prolonged immobilisation

2633
Q

Medical patients at risk of VTE

A

if mobility significantly reduced for >= 3 days or

if expected to have ongoing reduced mobility relative to normal state plus any VTE risk factor (see below)

2634
Q

Surgical patients at risk of VTE

A

if total anaesthetic + surgical time > 90 minutes or

if surgery involves pelvis or lower limb and total anaesthetic + surgical time > 60 minutes or

if acute surgical admission with inflammatory or intra-abdominal condition or

if expected to have significant reduction in mobility or

if any VTE risk factor present (see below)

2635
Q

What are the options for post-procedure VTE prophylaxis

A

Dabigatran

Fondaparinux

LMWH

Rivaroxaban

Apixaban

2636
Q

When can the post-operative procedure VTE prophylaxis be started?

A

dabigatran, started 14 hours after surgery

fondaparinux, started 6 hours after surgery

LMWH, started 6-12 hours after surgery

rivaroxaban, started 6-10 hours after surgery.

apixaban

2637
Q

Length of VTE prophylaxis

Elective hip

A

28-35d

2638
Q

Length of VTE prophylaxis

Elective knee

A

10-14d

2639
Q

Length of VTE prophylaxis

Hip #

A

28-35d

2640
Q

A 43-year-old lady with a metallic heart valve has just undergone an elective paraumbilical hernia repair. In view of her metallic valve, she is given unfractionated heparin perioperatively. How should the therapeutic efficacy be monitored, assuming her renal function is normal?

Therapeutic monitoring is not required

Measurement of APTT

Measurement of INR

Measurement of Prothromin time

None of the above

A

Unlike low molecular weight heparins that do not require monitoring unfractionated heparin does require monitoring, this is done by measuring the APTT.

2641
Q

MOA standard heparin

A

Activates antithrombin III. Forms a complex that inhibits thrombin, factors Xa, IXa, Xia and XIIa

2642
Q

MOA LMWH

A

Activates antithrombin III. Forms a complex that inhibits factor Xa

2643
Q

Monitoring of unfractionated heparin

A

APTT

2644
Q

Monitoring of lMWH

A

Anti-factor Xa (although routine monitoring not required)

2645
Q

MOA HIT

A

immune mediated - antibodies form against complexes of platelet factor 4 (PF4) and heparin

these antibodies bind to the PF4-heparin complexes on the platelet surface and induce platelet activation by cross-linking FcγIIA receptors

usually does not develop until after 5-10 days of treatment

despite being associated with low platelets HIT is actually a prothrombotic condition

features include a greater than 50% reduction in platelets, thrombosis and skin allergy

treatment options include alternative anticoagulants such as lepirudin and danaparoid

2646
Q

What electrolyte disturbance can be caused by both unfractionated and LMWH?

A

Hyperkalaemia

?through inhibition of aldosterone secretion

2647
Q

A 78-year-old man is due to have an anterior resection for colorectal carcinoma. He is currently on clopidogrel. He has been on clopidogrel for the past 6 months due to the insertion of a drug eluting stent during primary percutaneous coronary intervention for a STEMI.

When should you advise him to stop taking his clopidogrel?

3 days before surgery

5 days before surgery

7 days before surgery

10 days before surgery

14 days before surgery

A

The BNF states that for elective procedures, where an antiplatelet effect is not needed, clopidogrel should be discontinued 7 days before surgery. The indication for clopidogrel is important, and if related to cardiovascular disease, discontinuation of clopidogrel therapy should only be considered after discussion with the patients cardiologist.

2648
Q

Features of clopidogrel

A

Clopidogrel is an antiplatelet agent used in the management of cardiovascular disease. It was previously used when aspirin was not tolerated or contraindicated but there are now a number of conditions for which clopidogrel is used in addition to aspirin, for example in patients with an acute coronary syndrome. Following the 2010 NICE technology appraisal clopidogrel is also now first-line in patients following an ischaemic stroke and in patients with peripheral arterial disease.

2649
Q

MOA clopidogrel

A

Antagonist of P2Y12 adenosine diphosphate R, inhibiting platelet activaton

2650
Q

What is clopidogrel’s drug class

A

Clopidogrel belongs to a class of drugs known as thienopyridines which have a similar mechanism of action. Other examples include:

prasugrel

ticagrelor

ticlopidine

2651
Q

Clopidogrel +PPI

A

May make clopidogrel less effecive

Generally this seems to be with omeprazole and esomeprazole

Other PPIs such as lansoprazole are ok

2652
Q

Non-immune mediated blood trasnfusion reactions

A

Hypocalaceamia

CCF

Infections

Hyperkalaemia

2653
Q

A 75-year-old man comes into the orthopaedic ward for an elective hip replacement. He has been assessed for venous thromboembolism (VTE) prophylaxis. Apart from the operation and his age he does not have any additional risk factors and he does not have any risk factors for bleeding. What is the recommended VTE prophylaxis measures for this gentleman?

TED stockings

TED stockings + dalteparin sodium started at least 6 hours post-operation

TED stockings + dalteparin sodium started the morning of surgery

Dalteparin sodium started at least 6 hours post-operation

Dalteparin sodium stared the morning of surgery

A

NICE recommends that patients who will undergo an elective hip replacement have both mechanical and pharmacological methods of venous thromboembolism (VTE) prophylaxis. TED stockings should be administered and the patient should wear them once they have been admitted. As long as there are not contraindications, such as bleeding risk, pharmacological VTE prophylaxis is administered after surgery. Dalteparin sodium, a low molecular weight heparin, is started 6 hours after surgery. Other pharmacological methods can also be used.

2654
Q

A 77-year-old male is day 1 post-op TURP for benign prostatic hyperplasia. You are bleeped by the nurses as the patient has spiked a temperature.

Observations are: Respiratory rate 16/min, saturations 95% on air, Blood pressure 120/70 mmHg, Heart rate 70 bpm, temperature 38.3ºC, Glasgow coma score 15/15.

On arrival the patient is lying in bed. He reports feeling well in himself. On examination there is reduced air entry in the lungs bilaterally.

Post op bloods show:

Hb119 g/l

Platelets245 * 109/l

WBC10.2 * 109/l

CRP89 mg/l

What is the most appropriate next step?

Commence IV antibiotics

Inform the on-call registrar

Organise a CXR and take blood cultures

Encourage the patient to mobilise and engage in deep breathing exercises

Commence high flow oxygen through a non-rebreathe mask

A

This is a case of post-operative pyrexia which is a very common finding for junior doctors on a surgical firm. This questions tests the ability to know the common causes of post-operative pyrexia and the time frames they are likely to occur within. From the given vignette it is testing the user to identify a clinically stable patient with no evidence of focal infection (CRP commonly raised post-op, chest likely collapsed bilaterally following a general anaesthesia) but who presents with an isolated temperature spike. These findings are in keeping with post-surgical inflammation/atelectasis given the stable picture.

It is importan to be able to clinically synthesise the history, examination and clinical picture (I.e. 24 hours post TURP - a relatively low risk procedure) and be guided by this in further management instead of performing blanket investigations (i.e. CXR, blood cultures) for an isolated temperature spike 24 hours post-op. Chest physiotherapy with mobilisation and breathing exercises would help manage post-op lung atelectasis which is a very common post-op complication; especially within the first 24 hours. If the patient were to become unwell (I.e. infective symptoms, further temperature spikes, development of SIRS) they would then need further review and investigations based on the clinical picture.

Benign temperature spikes in the first 24 hours after surgery due to atelectasis is a common presentation. Treatment involves mobilisation, chest physiotherapy and saline nebulisers for excessive secretions. Ongoing temperature spikes or infective symptoms would warrant further investigation.

2655
Q

A 49-year-old man is having an elective repair of a right-sided inguinal hernia under general anaesthetic. What is the most appropriate advice to give him about eating and drinking before the operation?

He should be nil-by-mouth from midnight before his operation

No food or clear fluids for 6 hours before his operation

No food for 6 hours and no clear fluids for 4 hours before his operation

No food for 6 hours and no clear fluids for 2 hours before his operation

No food for 6 hours and no clear fluids for 1 hour before his operation

A

The Royal College of Anaesthetists recommend that patients should have no food for 6 hours before the induction of general anaesthesia. Patients should be allowed to drink water or other clear fluids until 2 hours before the induction of general anaesthesia. This is to reduce the likelihood of pulmonary aspiration of gastric contents.

2656
Q

Adverse effects of halothane

A

Hepatotoxicity

Myocardial depression

Malignant hyperthermia

2657
Q

Adverse effects of thiopental

A

Laryngospasm

2658
Q

After a complicated revision of a total hip replacement, an 80-year-old lady receives two units of packed red cells. Which drug should be prescribed between the units?

Furosemide

Paracetamol

Cetirizine

Hydrocortisone

Platelets

A

Although packed red cells have a higher haematocrit than blood, transfusion of two units or more can result in fluid overload. Therefore, furosemide should be prescribed to be given between every other unit, if two or more units are given. Paracetamol can be used if there is a non-haemolytic febrile reaction to the transfusion. Cetirizine and hydrocortisone are used in cases of an anaphylactic reaction to the transfusion. Platelets may be given in cases of major haemorrhage, alongside packed red cells.

2659
Q

Packed red cells

A

Used for transfusion in chronic anaemia and cases where infusion of large volumes of fluid may result in cardiovascular compromise. Product obtained by centrifugation of whole blood.

2660
Q

Platelet rich plasma

A

Usually administered to patients who are thrombocytopaenic and are bleeding or require surgery. It is obtained by low speed centrifugation.

2661
Q

Platelet concentrate

A

Prepared by high speed centrifugation and administered to patients with thrombocytopaenia.

2662
Q

Fresh frozen plasma

A

Prepared from single units of blood.

Contains clotting factors, albumin and immunoglobulin.

Unit is usually 200 to 250ml.

Usually used in correcting clotting deficiencies in patients with hepatic synthetic failure who are due to undergo surgery.

Usual dose is 12-15ml/Kg-1.

It should not be used as first line therapy for hypovolaemia.

2663
Q

Cryoprecipitate

A

Formed from supernatant of FFP.

Rich source of Factor VIII and fibrinogen.

Allows large concentration of factor VIII to be administered in small volume.

2664
Q

SAG-Mannitol Blood

A

Removal of all plasma from a blood unit and substitution with:

Sodium chloride

Adenine

Anhydrous glucose

Mannitol

Up to 4 units of SAG M Blood may be administered. Thereafter whole blood is preferred. After 8 units, clotting factors and platelets should be considered.

2665
Q

Methods of Warfarin reversal

A

Stop Warfarin

  1. Vitamin K (reversal within 4-24h)

IV takes 3-6h to work (at least 5mg)

Oral can take 24h to be clinically effective

  1. FFP

Used less commonly now as first line

30ml/kg

need to give at least 1l fluid in 70kg person (therfore not appropriated in fluid overload

Need blood group

Only used if human prothrombin complex is not available

  1. Human prothrombin complex (reversal within 1 hour)

Bereplex 50u/kg

Rapid action but factor 6 short t1/2, therefore give with vit K

2666
Q

A 50-year-old diabetic woman is admitted to the day surgery unit for an elective incision and drainage of a groin abscess. Her diabetes is usually well controlled on metformin. What should be done with regard to her diabetic control?

Stop metformin, restart 48 hours after surgery

Continue her normal regimen

Admit the day before surgery and commence variable rate insulin infusion

Commence variable rate insulin infusion on the day of surgery

Stop metformin for 2 weeks and delay surgery

A

This patient is an orally controlled diabetic and therefore is unlikely to need a sliding scale regimen unless it is particularly major surgery, which an incision and drainage is not. If the patient was undergoing significant surgery, then they would usually be admitted the night before, and commenced on a variable rate infusion.

Delaying surgery is not normally advised unless there are significant contraindications.

This type of surgery is likely to be a day case and therefore she can continue on her normal metformin regimen.

2667
Q

normal haemoglobin level associated with a microcytosis. In patients not at risk of thalassaemia

A

this should raise the possibility of polycythaemia rubra vera which may cause an iron-deficiency secondary to bleeding.

2668
Q

A 22-year-old female is extubated following an uncomplicated laparoscopic appendicectomy. However, no respiratory effort is made and she is re-intubated and ventilated. She is monitored in the intensive care unit and all observations are normal. She is weaned from the ventilator 24 hours later successfully. What complication has occurred?

Misplacement of the endotracheal tube in intubation

Suxamethonium apnoea

Opioid toxicity

Malignant hyperpyrexia

Overdose of propofol

A

A small subset of the population has an autosomal dominant mutation, leading to a lack of the specific acetylcholinesterase in the plasma which acts to break down suxamethonium, terminating its muscle relaxant effect. Therefore, the effects of suxamethonium are prolonged and the patient needs to be mechanically ventilated and observed in ITU until the effects of suxamethonium wear off.

Opioid toxicity causes respiratory depression but is unlikely to be extreme enough to cause no respiratory effort in the monitored conditions of an anaesthetic. Misplacement of the endotracheal tube would cause hypoxia with a respiratory acidosis soon after intubation and potentially a pneumothorax ipsilaterally with collapse contralaterally. A propofol overdose would cause a fall in blood pressure. Malignant hyperpyrexia would manifest with a rise in temperature, rise in blood pressure, muscle spasm, type II respiratory failure and metabolic acidosis and arrhythmias.

2669
Q

Suxamethonium

A

Depolarising neuromuscular blocker

Inhibits action of acetylcholine at the neuromuscular junction

Degraded by plasma cholinesterase and acetylcholinesterase

Fastest onset and shortest duration of action of all muscle relaxants

Produces generalised muscular contraction prior to paralysis

Adverse effects include hyperkalaemia, malignant hyperthermia and lack of acetylcholinesterase

2670
Q

Atracurium

A

Non depolarising neuromuscular blocking drug

Duration of action usually 30-45 minutes

Generalised histamine release on administration may produce facial flushing, tachycardia and hypotension

Not excreted by liver or kidney, broken down in tissues by hydrolysis

Reversed by neostigmine

2671
Q

Vecuronium

A

Non depolarising neuromuscular blocking drug

Duration of action approximately 30 - 40 minutes

Degraded by liver and kidney and effects prolonged in organ dysfunction

Effects may be reversed by neostigmine

2672
Q

Pancuronium

A

Non depolarising neuromuscular blocker

Onset of action approximately 2-3 minutes

Duration of action up to 2 hours

Effects may be partially reversed with drugs such as neostigmine

2673
Q

A 72-year-old gentleman is about to undergo an elective hernia repair. He has mild asthma which is well-controlled using a salbutamol inhaler PRN, on average about once per week. His asthma causes no limitation to his daily activity. What is his ASA (American Society of Anesthesiologists) status?

ASA 3

ASA 1

ASA 5

ASA 2

ASA 4

A

An ASA (American Society of Anesthesiologists) score is an indicator of a patient’s fitness for surgery. A patient is classified as ASA 2 if they have a mild systemic disease without any functional limitations. Examples include (but not limited to): current smoker, social alcohol drinker, pregnancy, obesity (30 < BMI < 40), well-controlled diabetes mellitus or hypertension, and mild lung disease.

2674
Q

ASA classificaiton

Normal healthy patient

A

ASA I

2675
Q

ASA classification

A patient with mild systemic disease

e.g. current smoker, socal EtOH, pregnancy, obesity <30, well controlled DM, HTN, mild lung disease

A

ASA II

2676
Q

ASA classificaiton

A patient with severe systemic disease

Examples include (but not limited to): poorly controlled Diabetes Mellitus/Hypertension, COPD, morbid obesity (BMI 40), active hepatitis, alcohol dependence or abuse, implanted pacemaker, moderate reduction of ejection fraction, End Stage Renal Disease (ESRD) undergoing regularly scheduled dialysis, history (>3 months) of Myocardial infarction, Cerebrovascular accidents

A

ASA III

2677
Q

ASA classification

A patient with severe systemic disease that is a constant threat to life

A

ASA IV

recent (< 3 months) of Myocardial infarction, Cerebrovascular accidents, ongoing cardiac ischaemia or severe valve dysfunction, severe reduction of ejection fraction, sepsis, DIC, ARD or ESRD not undergoing regularly scheduled dialysis

2678
Q

ASA V

A moribund patient not expected to survive without the operation

A

ruptured abdominal/thoracic aneurysm, massive trauma, intra-cranial bleed with mass effect, ischaemic bowel in the face of significant cardiac pathology or multiple organ/system dysfunction

2679
Q

ASA VI

A

A declared brain-dead patient whose organs are being removed for donor purposes

2680
Q

Theme: Blood product ordering prior to surgery

A.No group and save or cross-match required

B.Group and save

C.Cross-match 2-6 units depending on local protocols

D.Cross-match 8-12 units depending on local protocols

E.Transfuse 2 units prior to operation regardless of haemoglobin

For each of the following procedures select the most appropriate preparation with respect to ordering blood products:

Laparoscopic cholecystectomy

Total gastrectomy

Elective lower segment caesarean section

A

G+S

X-match 2-6u dependant on local protocols

G+S

2681
Q

A 69-year-old man has just undergone laparoscopic abdominal surgery for appendicitis. There was minimal blood loss. He weighs 75 kg. He initially does not feel able to drink as he is nauseous from the anaesthetic. His pre-operative bloods were normal. What fluid should initially be prescribed to initiate a maintenance regime and how fast should it be given?

500 ml 0.9% sodium chloride at 100 ml/hr

500 ml 0.9% sodium chloride at 150 ml/hr

500 ml 0.9% sodium chloride STAT

500 ml Gelofusine at 72.5 ml/hr

1 L 0.9% sodium chloride at 50 ml/hr

A

This patient has just left theatre and will require fluids. Oral fluids should be initiated as soon as possible but if a patient is unable to drink then maintenance fluids should be prescribed through the IV route.

Sodium chloride is a recommended fluid to be used for maintenance. Maintenance fluids should be prescribed at a rate of 30 ml/kg/24hr.

Amount: 75 kg X 30 ml = 2250 mL in a 24 hour period

Rate: 2500 / 24 = 93.75 ml/hr.

Initially prescribe 500 ml and then reassess the patients fluid status and ability to drink. So prescribe 500 ml at a rate of 100 ml/hr.

2682
Q

What is the maximum safe dose of Lidocaine?

A

3mg/kg

Available pre-mixed with adrenaline which increases the duration of aciton and reduces blood loss secondary to vasoconstriction

Should never be used near extremities due to risk of ischaemia

2683
Q

Give some examples of non-absorbable suture material

A

Slik

Novafil

Prolene

Ethilon

2684
Q

Give some examples of absorbable sutures

A

Vicryl

Dexon

PDS

2685
Q

When should non-absorbable sutures be removed from face?

A

3-5d

2686
Q

When should non-absorbable sutures be removed from scalp, limbs, chest

A

7-10d

2687
Q

When should non-absorbable sutures be removed from hand, foot, back

A

10-14d

2688
Q

A 55-year-old gentleman has been admitted to the surgical assessment unit with acute abdominal pain, worse in right iliac fossa, and associated nausea and mild fever. Your surgical registrar suspects an appendicitis and arranges an urgent US scan of his abdomen. This is consistent with acute appendicitis and your surgical registrar asks you to prepare the patient for theatre. As you are reviewing his blood results you notice his platelet count is 87x10^9/L. On questioning the patient tells you he has idiopathic thrombocytopenic purpura (ITP). He is due for theatre in 4 hours. What is the most appropriate management of his pre-operative thrombocytopenia?

Fresh frozen plasma

Monitor for intra-operative bleeding

Cryoprecipitate

Start eltrombopag (thrombopoietin receptor agonist)

Intravenous immunoglobulin (IVIG)

A

Patients who are not actively bleeding and have a platelet count of > 80x10^9/L do not require any immediate management of their ITP pre-operatively and can be monitored intra-operatively. When treatment is required urgently this normally involves corticosteroids +/- IVIG. Eltrombopag is used in chronic ITP but requires 10-14 days to have an outcome on platelet count. Finally, cryoprecipitate and fresh frozen plasma are not indicated for the treatment of ITP.

2689
Q

A 68-year-old man is undergoing a total hip replacement. He has a past medical history of ischaemic heart disease. During surgery he becomes hypotensive (blood pressure of 65/35 mmHg) with a pulse rate of 85 beats per minute. He has peripheral IV access.

What is the most appropriate treatment?

Adrenaline

Dobutamine

Cardioversion

Noradrenaline

Metaraminol

A

Drugs used to increase blood pressure in intra-operative hypotension can be broadly divided into those which are predominantly vasopressors and those which are positive inotropes (although there is a significant overlap).

In this case, the patient has a normal heart rate and therefore a vasopressor such as metaraminol would be the most sensible choice. Metaraminol is primarily an alpha 1-adrenergic receptor agonist which causes systemic arteriole vasoconstriction, thereby increasing mean arterial blood pressure.

Phenylephrine and metaraminol have dominant actions on alpha 1 receptors. Therefore these drugs are most suitably classed as vasopressors. Dobutamine is a powerful beta 1 receptor agonist which acts on the myocardium to increase myocardial contractility and heart rate. Doputamine is therefore most suitably named a positive inotrope. It is important to remember that there is often a significant overlap in properties of these agents, and noradrenaline, dopamine, and adrenaline have mixed alpha 1 and beta 1 receptor agonist properties and are therefore both positive inotropes and vasopressors.

2690
Q

A 68-year-old lady is due to have a cholecystectomy for gallstone disease. She is on warfarin and her last INR two weeks ago was 2.7.

When should you advise her to stop taking her warfarin?

3 days before surgery

1 day before surgery

5 days before surgery

10 days before surgery

14 days before surgery

A

In general, warfarin is usually stopped 5 days before planned surgery, and once the person’s international normalized ration (INR) is less than 1.5 surgery can go ahead. Warfarin is usually resumed at the normal dose on the evening of surgery or the next day if haemostasis is adequate.

2691
Q

Causes of addisonian crisis

A

Sepsis or surgery causing an acute exacerbation of chronic insufficiency

Adrenal haemorrahge e.g. Waterhouse-Friedrichsen

Steroid withdrawal

2692
Q

Mx of Addisonian crisis

A

100mg IM or IV

1L NS infused over 30-5mins or with dextrose if hypogylcaemic

Continue hydrocortisone 6 hourly until patient is stable.

No fludrocortisone is necessary because high cortisol exerts weak MC action

Oral replacement may begin after 24h and reduced to maintenance over 3-4d

2693
Q

Epidemiology of oesophageal cancer

A

12/100000 (increassing due to increased Barret/s)

50-70y

M>F 5:1

Iran, Transkei, China

2694
Q

RFs for oesophageal cancer?

A

EtOH

Smoking

Achalasia

GORD-> Barrett;s

Plummer Vinson

Fatty Diet

Reduced Vit A and C

Nitrosamine exposure

2695
Q

Relative prevalences of the different kinds of oesophageal cancer

A

65% adenocarcinoma

35% SCC

2696
Q

Oesophageal cancer

Lower 3rd

GORD-> Barrett’s-> Dysplasia-> Ca

A

Adenocarcinoma

2697
Q

Oesophageal cancer

Upper and middle 3rd

Associated with EtOH and smoking

Commonest type worldwide

A

SCC

2698
Q

Progressive dysphagia: solids-> liquids often altering dietary habit

Weight loss

Retrosternal chest pain

Lymphadenopathy

A

Oesopageal carcinoma

2699
Q

Oesophageal cancer with hoarsenss due to recurrent laryngeal nerve invasion

Cough +/- aspiration pneumonia

Location

A

Likely Upper 3rd

2700
Q

Spread of oesophageal carcinoma

A

Direct extension, lymphatics and blood

75% of patients have mets at diagnosis

2701
Q

Ix in oesophageal Ca

A

Bloods:

FBC: anaemia

LFTs: hepatic mets, albumin

Dx:

Upper GI endsocopy: biopsy

Ba swallow, not often used, apple core stricture

Staging:

CT

EUS

Laparoscopy/mediastinoscopy: mets

2702
Q
A

Apple core appearance

Oesophageal carcinoma

2703
Q

TNM Staging Oesophagus

A

􀁸 Tis: carcinoma in situ
􀁸 T1: submucosa
􀁸 T2: muscularis propria (circ / long)
􀁸 T3 adventicia
􀁸 T4: adjacent structures
􀁸 N1: regional nodes
􀁸 M1: distant mets

2704
Q

MDT mx of oesophageal carcinoma

A

Upper GI surgeon and gastro

Radiologist

Pathologist

Oncologist

Specialist nurses

Macmillan nurses

Palliative care

2705
Q

Surgical approach to oesophageal carcinoma

A

Only 25-30% have resectable tumours

May be offered neo-adjuvant chemo before surgery to downstage tumour e.g. cisplatin + 5FU

Approaches@

Ivor-Lewis

McKeown

Hiatal

2706
Q

Ivor Lewis Oesophagectomy

A

2 stage approach

In the Ivor Lewis esphagectomy, the esophageal tumor is removed through an abdominal incision and a right thoracotomy (a surgical incision of the chest wall). The esophagogastric anastomosis (reconnection between the stomach and remaining esophagus) is located in the upper chest.

2707
Q

McKeown Oesophagectomy

A

Three stage

Abdominal + R thoractomoy

and left neck incision

Used for lesions higher in the oesophagus

Cervical anastomosis

2708
Q

Px of oesophageal resection

A

Stage dependant

15% 5y

2709
Q

Palliative approach to oesophageal carcinoma

LAASAR

A

Majority of patients

Laser coagulation

Alcohol injeciton and reduce ascites (spironolactone)

Stenting and secretion reduction e.g. hyoscine patch

Analgesia e.g. fentanyl patch

RTx: external or brachytherapy

Referral: palliative care team

Macmillan nurses

Px:

5y <5%

Median: 4mo

2710
Q

Benign tumours of oesophagus

A

Leiomyoma

Lipoma

Haemangioma

Benign polyps

2711
Q

Pathophysiology of GORD

A

LOS dysfunciton-> reflux of gastric contents-> oesophogaitis

2712
Q

RFs: for GORD

A

Hiatus hernia

Smoking

EtOH

Obesity

Pregnancy

Drugs: anti-AChM, nitrates, CCBs, TCAs

Iatrogenic: Heller’s myotomy

2713
Q

Symptoms of GORD

A

Oesophageal:

Retrosternal pain: heartburn

Related to meals

Worse lying down and stooping

Relieved by antacids

Belching

Regurgitation

Acid brash, water brash

Odynophagia

Extra-oesophageal

Nocturnal asthma

Chronic cough

Laryngitis, sinusitis

2714
Q

Cx of GORD

A

Oesophagitis

Ulceration: rarely-> haematemesis, melaena, reduced Fe

Benign stricture: dysphagia

Barrett’s oesophagus: intestinal metaplasia of squamous epithelium-> dysplasia-> adenocarcinoma

Oesophageal adenocarcinoma

2715
Q

Ix in GORD

A

Isolated symptoms don’t need Ix

Bloods: FBC

CXR: hiatus hernia may be seen

OGD if fulfil criteria

Ba swallow: hiatus hernia, dysmotility

24h pH testing + manometry

2716
Q
A

Hiatus hernia

retrocardiac opacity with air-fluid level

2717
Q

Indications for OGD in GORD

ALARMS

A

If >55yrs

Persistent symptoms despite Rx

Anaemia

Loss of weight

Anorexia

Recent onset progressive symptoms

Melaena

Swallowing difficulty

2718
Q

What is the grading system for oesophagitis

A

Los Angeles grade A to D

Grade A: one or more mucosal breaks no longer than 5 mm, none of which extends between the tops of the mucosal folds.

Grade B: one or more mucosal breaks more than 5 mm long, none of which extends between the tops of two mucosal folds.

Grade C: mucosal breaks that extend between the tops of two or more mucosal folds but which involve <75% of the mucosal circumference

Grade D: mucosal breaks which involve ≥75% of the mucosal circumference

2719
Q

Rx in GORD

A

Conservative:

Reduce weight

Raise head of bed

Small regular meals >3h before bed

Stop smoking and EtOH

Avoid hot drinks and spicy food

Stop drugs: NSAIDs, anti-AChM, nitrates, CCB, TCAs

Medical:

OTC antacids: Gaviscon, Mg Triscillate

  1. Full dose PPI for 102mo

Lansoprazole 30mg OD

  1. No response-> double dose PPI BD
  2. No response add H2RA:

Ranitidine 300mg

Control, low dose acid suppression PRN

Step down to lowest effective dose

Surgical:

Nissen fundoplication

2720
Q

Indications for surgical Mx of GORD

A

Severe symptoms

Refractory to medical therapy

Confirmed reflux

2721
Q

Nissen fundoplication

A

Aim: prevent reflux

Repair diaphragm

Can be open or lap

Mobilise gastric fundus and wrap around lower oesophagus

Close any diaphragamtic hiatus

2722
Q

Cx of Nissen fundo

A

Gas-bloat syndrome: inability to belch/vomit

Dysphagia if wrap too tight

2723
Q

Alternatives to Nissen

A

Lap insertion of magnetic bead band

With the patient under general anaesthesia, an implant is placed so that it encircles the distal oesophagus at the gastro-oesophageal junction.

The implant is then secured in place. The implant consists of a ring of interlinked titanium beads, each with a weak magnetic force which holds the beads together to keep the distal oesophagus closed.

When the patient swallows, the magnetic force is overcome, allowing the ring to open. After swallowing, magnetic attraction brings the beads together and the distal oesophagus is again closed.

MRI is contra-indicated after this procedure.

NICE currently recommends that the evidence on the safety and efficacy of laparoscopic insertion of a magnetic bead band for GORD is limited in quantity. Therefore, this procedure should only be used with special arrangements for clinical governance, consent and audit, or for research

2724
Q

What are the indications for continuous PPI therapy

A

Patients with documented NSAID induced ulcer who must consitinue with NSAIDs

Patients with severe reflux oesophagitis

Patients with complicated reflux disease

2725
Q

What are the different types of hiatus hernia

A

Sliding (80%)

Rolling (15)

Mixed (5%)

2726
Q

Features of Sliding hiatus hernia

A

80%

GOJ slides up into chest

Often associated with GORD

2727
Q

Thin little old lady

Confused

Distended

AF

A

Think Rolling/ paraoesophageal hiatus hernia

2728
Q

Features of rolling hiatus hernia

A

GOJ remains in abdomen but bulge of stomach rolls into chest alongside oesophagus

LOS remains intact so GORD uncommon

Can lead to strangulation

2729
Q

Ix in hiatus hernia

A

CXR: gas bubble and fluid level in chest

Ba swallow

OGD: assess for oesphagitis

24pH and manometry to exclude dysmotility or achalasia

2730
Q

Rx in hiatus hernia

A

Lose weight

Treat reflux

Surgery if intractable symptoms despite medical Rx

Should repair rolling hernia due to risk of strangulation (even if asymptomatic)

2731
Q
A

Hiatus hernia

2732
Q

Epigastric pain

Before meals and at night

Relieved by eating

A

Duodenal ulcer

2733
Q

Epigastric pain

Worse on eating (weight loss)

Relieved by antacids

A

Gastric ulcer

2734
Q

RFs for PUD

A

H. pylori

NSAIDs, steroids

Smoking

EtOH

Stress GU

Cushing’s ulcers: head injury

Curling’s ulcers: burns

2735
Q

Pathology of PUD

A

Punched out ulcers

Usualy background of chronic inflammation

DU: 4x commoner than GU, 1st part of duodenum

GU: lesser curvature of gastric antrum

2736
Q

Cx of PUD

A

Haemorrhage:

Haematemesis or melaeana

IDA

Perforation: peritonitis

Gastric outflow obstruction

Vomiting, colic, distension

Malignancy:

Incresed risk with H. pylori infection

Actual malignant transformation probably doesn’t occur

2737
Q

Ix in PUD

A

Bloods: FBC, urea (raised in UGI haemorrhage)

C13 breath test or stool antigen test (C13 as test of eradication)

OGD: Stop PPIs 2w before

Biopsy ulcers to check for malignancy (only necessary with gastric)

Gasrtin levels if Zollinger-Ellison suspected

2738
Q

Mx of PUD

A

Conservative:

Lose weight

Stop smoking and reduce EtOH

Avoid hot drinks and spicy food

Stop drugs: NSAIDs, steroids

OTC antacids

Medical:

OTC antacids: Gaviscon, Mgtriscillate

Aimed at H pylori eradication:

PAC500 or PMC250

If not H. pylroi induced:

PPI

2739
Q

PAC500

H pylori eradication triple therapy:

A

PPI: lansoprazole mg/BD

1g amoxicllin BD

Clarithromycin 500BD

2740
Q

PMC250

H prylori eradication

A

PPI

Metronidazole

Clarithromycin

2741
Q

PPI standard dose

A

Lansoprazole 30mg/d

2742
Q

H2RA standard dose

A

300mg nocte

2743
Q

Surgical concepts in treatment of PUD

A

No acid- no ulcer

Acid secretion is stimulated by gastrin from antral G cells and vagus nerve

2744
Q

What are the surgical options for PUD

A

Rarely used

Vagotomy

Antrectomy with vagotomy

Subtotal gastrectomy with Roux en Y

2745
Q

Features of truncal vagotomy

A

Reduce acid secretion directly and via reduced gastrin

Prevents pyloric sphincter relaxation

Must be combined with pyloroplasty (widening of pylorus) or gastroenterostomy

2746
Q

Features of selective vagotomy

A

Vagus nerve only denervated where it supplies lower oesophagus and stomach

Pylorus left intact

2747
Q

Features of antrectomy with vagotomy

A

Distal half of stomach removed

Anastomosis:

Billroth 1: directly to duodenum

Billroth 2: to small bowel loop with duodenal stump oversown

2748
Q

Physical cx with sx for PUD

A

Ca: increased risk of gastric Ca

Reflux or billious vomiting (improves with time)

Abdominal fullness

Stricture

Stump leakage

2749
Q

Metabolic Cx of PUD sx

A

Dumping syndrome:

Abdo distension, flushing, N/v, fainting, sweating

Early: osmotic hypovolaemia

Late: reactive hypoglycaemia

Blind loop syndrome:

malbasorption, diarrhoea

Overgrowth of bacteria in duodenal stump

Vitamin deficiency:

Reduced parietal cells-> B12 deficiency

Bypassing proximal SB-> Fe and folate deficiency

Osteoporosis

Weight loss: malabsroption of reduced calorie intake

2750
Q

pH testing GORD

A

pH <4 for >4hrs

2751
Q

Pathophysiology of perforated peptic ulcer

A

Perforated duodenal ulcer is commonest

1st part of duodenum: highest acid concentration

Anterior perforation-> air under diaphragm

Posterior perforation can erode into GDA-> bleed

3/4 of duodenum is retroperitoneal therefore no air under diaphragm if perforated

Perforated GU

Perforated Gastric Ca

2752
Q

Sudden onset severe pain beginning in the epigastrium and then becoming generalised

Peritonitis

A

Perforated peptic ucler

2753
Q

DDx perforated peptic ulcer

A

Pancreatitis

Acute cholecystitis

AAA
MI

2754
Q

Ix in perforated peptic ulcer

A

Bloods;

FBC, U+E, amylase, CRP, G+S, clotting

ABG: mesenteric ischaemia

Urine dipstick

Erect CXR:

must be erect for 15 minutes first

Air under diaphragm seen in 70%

False positive in Chilaiditi’s sign

AXR:

Rigler’s: aire on both sides of bowel wall

2755
Q

Chailadati syndrome

A

Chilaiditi syndrome is a rare condition when pain occurs due to transposition of a loop of large intestine (usually transverse colon) in between the diaphragm and the liver, visible on plain abdominal X-ray or chest X-ray.

2756
Q
A

Chest X-ray showing obvious Chilaiditi’s sign, or presence of gas in the right colic angle between the liver and right hemidiaphragm.

2757
Q
A

Subdiaphragmatic air

2758
Q
A
2759
Q

Mx of perforated peptic ulcer

A

ABC:

NBM

Aggressive fluid resusc: urinary catheter +/- CVP line

Analgesia: 5-10mg/2h max +/- cyclizine

Abx: cef and met

NGT

Conservative:

May be considered if patient is not peritonitic

Careful monitorig, fluid + Abx

Omentum may seal perforation preventing operation in 50%

Surgical: Laparotomy

DU: abdominal washout and omental patch repair

GU: excise ulcer and repair defect

Partial/gastrectomy may be required

Send specimen for histo: exclude Ca

Test and treat:

90% perforated PU associated with H. pylori

2760
Q

Cause of gastric outlet obstruction

A

Late complication of PUD-> fibrotic stricturing

Gastric Ca

2761
Q

Hx of bloating, early satiety and nausea

Copious projectile non bilious vomiting a few hours after meals

Contains stale food

Epigastric distension and succusion splash

Adult

A

Gastric outlet obstruction

2762
Q

Ix in gastric outlet obstruction

A

ABG: hypochloraemic, hypokalaemic metabolic alkalosis

AXR: dilated gastric air bubble, air flud level, collapsed distal bowel

OGD

Contrast meal

2763
Q

Rx in gastric outlet obstruction

A

Correct metabolic abnormality: 0.9% NS and KCl

Benign:

Endoscopic balloon dilatation

Pyloroplasty or gastroenterostomy

Malignant:

Stenting

Resection

2764
Q

6-8w old

Projectile vomiting minutes after feed

RUQ mass: olive

Visible peristalsis

A

Py Sten

2765
Q

Dx in Py sten?

A

Test feed: palpate mass and see peristalsis

Hypochloraemic hypokalaemic metabolic alkalosis

USS

2766
Q

Mx of py sten

A

Resus and correct metabolic abnormality

NGT

Ramstedt pyloromyotomy: divide muscularis propria

2767
Q

Epidemiology of gastric cancer

A

23/100000

50y/o

M>F

Increased in Japan, Eastern Europe, China, SAm

2768
Q

RF for gastric cancer

A

Atrophic gastritis (intestinal metaplasia): pernicious anaemia/AI gatritis. H. pylori

Diet: high in nitrates: smoked pickled salted. Metabolised to carcinogenic nitrosamines in GIT

Smoking

Blood group A

Low SEC

Familial: cadherin abnormality

Partial gastrectomy

2769
Q

Pathology of gastric cancer

A

Mainly adenocarcinomas

Usually located on gastric antrum

H. pylroi may -> MALTOMA

2770
Q

How can gastric cancer be classified?

A

By depth of invasion:

Early gastric Ca: mucosa or submucosa

Late gastric cancer: muscularis propria breached

Microscopic appearanace:

Intestinal: bulky glandular tumours, heaped ulceration

Diffuse: infiltravive with signet ring morphology

2771
Q

Anatomy of alimentary canal

A

4 basic tissue layers:

Mucosa

Submucosa

Muscularis

Serosa

2772
Q

Where is Aurbach’s plexus?

A

Myenteric plexus

2773
Q

Where is Meissner’s plexus

A

Submucosal plexus

2774
Q
A
2775
Q

What can be used to classify late gastric carcinoma?

A

Bormann Classification

2776
Q

Outline Bormann classificaiton

A

Polypoid/fungating

Excavating

Ulcerating and raised

Linitis plastica: leather bottle like thickening with flat rugae

2777
Q

Usually presents late

Weight loss and anorexia

Dyspepsia: epigastric or retrosternal pain/discomfort

Dysphagia

N/V

A

Gastric Ca

2778
Q

Signs of gastric Ca

A

Anaemia

Epigastric mass

Jaundice

Ascites

Hepatomegaly

Virchow’s node (Troisier’s sign)

Acanthosis nigricans

2779
Q

Cx of gastric carcinoma

A

Perforation

Upper GI bleed

Gastric outlet obstruction: succession splash

2780
Q

Succession splash

A

Succussion splash is a sloshing sound heard through the stethoscope during sudden movement of the patient on abdominal auscultation. It reflects the presence of gas and fluid in an obstructed organ, as in gastric outlet obstruction.

Physical examination can show an abdominal succussion splash, which is elicited by placing the stethoscope over the upper abdomen and rocking the patient back and forth at the hips. Retained gastric material greater than three hours after a meal will generate a splash sound and indicate the presence of a hollow viscus filled with both fluid and gas. An example would be a gastric outflow obstruction as pyloric stenosis, with abdominal succussion splash

2781
Q

Spread of gastric carcinoma

A

Within stomach: linitis plastica

Direct invasion: pancreas

Lymphatic: Virchow’s node

Blood: liver and lung

Transcoelomic:

Ovaries: Krukengerg tumour
Sister Mary Joseph nodule: umbilical mets

2782
Q

Krukenburg tumour

A

Transcoelomic spread to ovaries associated with gastric Ca and signet ring morphology

2783
Q

Sister Mary Joseph Nodule

A

Palpable node bulging in the umbilicus

2784
Q
A

Sister Mary Joseph Nodule

2785
Q

Trosier’s Sign

A

Virchow’s node (or signal node) is a lymph node in the left supraclavicular fossa (the area above the left clavicle). It takes its supply from lymph vessels in the abdominal cavity. The finding of an enlarged, hard node (also referred to as Troisier’s sign) has long been regarded as strongly indicative of the presence of cancer in the abdomen, specifically gastric cancer, that has spread through the lymph vessels. It is sometimes called the signal node or sentinel node for the same reason. Despite this, the concept is not directly related to the sentinel node procedure sometimes used in cancer surgery, and it is also unrelated to the “sentinel gland” of the greater omentum.[citation needed]

It is named after Rudolf Virchow (1821–1902), the German pathologist who first described the gland and its association with gastric cancer in 1848.[1] The French pathologist Charles Emile Troisier noted in 1889 that other abdominal cancers, too, could spread to the node

2786
Q

Ix in gastric carcinoma

A

Bloods:

FBC: anaemia

LFTs and clotting

Imaging:

CXR: mets

USS: liver mets

Gastroscopy and biopsy

Ba meal

Staging:

Endoluminal US

CT/MRI

Diagnostic laparoscopy

2787
Q

Mx of gastric carcinoma

A

Medical palliation:

Analgesia e.g. fentanyl patch

PPI

Secretion control

CTx: epirubicin, 5FU, cisplatin

Palliative care team package

Surgical palliation

Pyloric stenting

Bypass procedures

Curative surgery

Can be resected endoscopically

Partial or total gastrectomy with roux en Y to prevent bile reflux

Spleen and part of pancreas may be removed

2788
Q

Roux en Y

A

In general surgery, a Roux-en-Y anastomosis, or Roux-en-Y, is a surgically created end-to-side anastomosis, most commonly performed for weight loss or to remove a malignancy. Typically, it is between stomach and small bowel that is distal (or further down the gastrointestinal tract) from the cut end.

The name is derived from the surgeon who first described it (César Roux)[1] and the stick-figure representation. Diagrammatically, the Roux-en-Y anastomosis looks a little like the letter Y.

Typically, the two upper limbs of the Y represent (1) the proximal segment of stomach and the distal small bowel it joins with and (2) the blind end that is surgically divided off, and the lower part of the Y is formed by the distal small bowel beyond the anastomosis.

Roux-en-Ys are used in several operations and collectively called Roux operations.[1]

When describing the surgery, the Roux limb is the efferent or antegrade limb that serves as the primary recipient of food after the surgery, while the hepatobiliary or afferent limb that anatamoses with the biliary system serves as the recipient for biliary secretions, which then travel through the residual stomach from the liver and into the Roux limb to aid digestion. The altered anatomy can contribute to indigestion following surgery

2789
Q

Px in gatric carcinoma

A

5y 10%

Sx: 20-50%

2790
Q

Benign gastric neoplasms

A

Benign polyps: adenomas

Leiomyoma

Lipoma

Haemangioma

Schwannoma

2791
Q

Malignant additional gastric neoplasms

A

Lymphoid

Carcinoid

GIST

2792
Q

GIST

A

Gastrointestinal stromal tumour

Commonest mesenchymal tumour of the GIT

>50% occur in the stomach

2793
Q

Epidemiology of GIST

A

M=F

60y

Increased with NF1

2794
Q

Pathology of GIST

A

Airse from intestinal cells of cajal located in muscularis propria

Pacemaker cells

On OGD: well demarcated spherical ams with central punctum

2795
Q

Presentation of GIST

A

Mass effects: abdo pain, obstruction

Ulceration: bleeding

2796
Q

Poor Px in GIST

A

Size

Extra-gastric location

Raised mitotic index

2797
Q

Mx of GIST

A

Medical:

Unresectable, recurrent or metastatic disese

Imatinib: seletive RtkI

Surgical:

Resection

2798
Q

Def: carcinoid tumours

A

Diverse group of neuroendocrine tumours of enterochromaffin cell origin

May secrete multiple horomones

10% occur in stomach

2799
Q

Features of gastric carcinoid tumours

A

Atrophic gastritis-> reduced acid produciton-> raised gastrin-> ECL hyperplasia-> carcinoid tumour

Gastrinomas may also-> carcinoid

2800
Q

What is the most common site of extranodal lymphoma?

A

Gastric lymphoma

2801
Q

Features of gastric lymphoma

A

Most commonly MALToma due to chronic H. pylori gastritis

H. pylroi eradication can be curative

2802
Q

Pathophysiology of ZE syndrome

A

Gastrin-secreting tumour (gastrinoma) most commonly found in the small intestine or pancreas

Increased gastrin-> raised HCL-> PUD + chronic diarrhoea

Diahrroea is a consequence of pancreatic enzymes

ECL proliferation can -> carcinoid tumours

60-90% of gastrinomas are malignant

25% associated with MEN1

2803
Q

Abdominal pain and dyspepsia

Chronic diarrhoea/steatorrhoea

Refractory PUD

A

ZE syndrome

2804
Q

Ix in ZE syndrome

A

Raised gastrin with raised ++HCl (pH<2)

MRI/CT

Somatostatin R scintigraphy

2805
Q

Rx in ZE syndrome

A

High dose PPI

Sx:

Tumour resection

May do subtotal gastrectomy wtih Roux en Y

2806
Q

Benefits of bariatric surgery?

A

Sustained weight loss

Symptom improvement:

sleep apnoea

Mobility

HTN

DM

2807
Q

Indications for bariatric surgery

A

All criteria must be met:

BMI >40 or >35 with significant co-morbidities that could improve with weight

Failure of non-surgical mx to achieve and maintain clinically beneficail weight loss for 6m

Fit for Sx and anaesthesia

Integrated programm providing guidance on diet, physical activity, psychosocial concerns and lifelong medical monitoring

Well informed and motivated patient

If BMI >50 sx is first line Rx

2808
Q

Surgical approaches: bariatric

A

Laparoscopic gastric banding

Roux-en-Y Gastric Bypass

2809
Q

Laparoscopic Gastric Bypass

A

Inflatable silicone band around proximal stomach-> small pre-stomach pouch

Limits food intake

Slows digestions

At 1y 46% mean XS weight loss

2810
Q

Roux en Y Gastric Bypass

A

Oesophagojejunostomy allows bypass of stomach, duodenum and proximal jejunum

Alters secretion of hormones influencing glucose regulation and perception of hunger/satiety

Greater weight loss and lower reoperation rates

2811
Q

Cx of Roux-en-Y gastric bypass

A

Dumping syndrome

Wound infection

Hernias

Malabsorption

Diarrhoea

Mortality: 0.5%

2812
Q

a large vacuole of mucin which displaces the nucleus to one side

A

Signet ring cells

2813
Q

Patients with inter current illness (e.g. diabetes, organ failure)
Patient of systemically unwell
Gallbladder inflammation in absence of stones
High fever

A

Acalculous cholecystitis

2814
Q

A 34-year-old male is admitted with central abdominal pain radiating through to the back and vomiting. The following results are obtained:

Amylase1,245 u/dl

Which one of the following medications is most likely to be responsible?

Phenytoin

Sodium valproate

Metoclopramide

Sumatriptan

Pizotifen

A

Sodium valproate induced pancreatitis is more common in young adults and tends to occur within the first few months of treatment. Asymptomatic elevation of the amylase level is seen in up to 10% of patients

2815
Q

Drugs causing acute pancreatitis

A

Azathioprine

Mesalazine (*pancreatitis is 7 times more common in patients taking mesalazine than sulfasalazine)

Didanosine

Bendroflumethiazide

Furosemide

Pentamidine

Steroids

VPA

2816
Q

Which of the following is the most sensitive blood test for diagnosis of acute pancreatitis?

Amylase

Lipase

C-peptide

Trypsin

Trysinogen

A

Lipase
The serum amylase may rise and fall quite quickly and lead to a false negative result. Should the clinical picture not be concordant with the amylase level then serum lipase or a CT Scan should be performed.

2817
Q

An 83-year-old female, with a history of osteoporosis, presented to the emergency department with a suspected hip fracture.

She is generally frail and has a background of dementia, ischaemic heart disease and chronic obstructive pulmonary disease. There is no history of hip pain due to osteoarthritis but she does walk with the aid of a walking stick.

She received an x-ray, which shows a displaced subcapital fracture of the femur. What is the most likely surgical treatment option for this type of fracture?

Dynamic hip screw

Hemiarthroplasty

Kirschner wires

Intramedullary nail

Total hip replacement

A

A subcapital fracture is the commonest type of intracapsular fracture of the proximal femur.

The intertrochanteric line is the line connecting the greater and lesser trochanters. Any fracture proximal to that line is classed as intracapsular, while any fracture distal is classed as extracapsular.

Since the blood supply is threatened in intracapsular fractures, as a general rule:

Intracapsular femoral fracture - hemiarthroplasty

extracapsular femoral fracture - dynamic hip screw

A hemiarthroplasty is favoured over a total hip replacement in this situation as the patient is:

frail, with a background of chronic health problems

no history of hip osteoarthritis

2818
Q

Def: hernia

A

Protrusion of a viscus or part of a viscus through the walls of its containing cavity into an abnromal position

2819
Q

def: reducible hernia

A

Sac can be returned to the abdominal cavity either spontaneously or with manipulation

2820
Q

def: irreducible hernia

A

Sac cannot be returned despite pressure or maniupulation

2821
Q

def: strangulated hernia

A

Blood supply of contents is compromised due to pressure at the neck of the hernia

2822
Q

def: sliding hernia

A

Part of the sac is formed by bowel e.g. caecum or sigmoid, take care when excising sac

2823
Q

def: Maydl’s hernia

A

Herniating double loop of bowel. Strangulated portion may reside as a single loop inside the abdomen

2824
Q
A

Maydl’s hernia

2825
Q

Def: Maydl’s hernia

A

Herniating double loop of bowel. Strangulated portion may reside as a single loop inside the abdomen

2826
Q

def: Littre’s hernia

A

Hernia sac containing strangulated Meckel’s diverticulum

2827
Q

Def: Richter’s hernia

A

Only part of circumference of bowel is within sac. Most commonly seen with femoral hernias.

Can strangulate without obstructing

2828
Q

Def: Amyand’s hernia

A

Inguinal hernia containing strangulated appendix

2829
Q

def: Pantaloon

A

Simultaneous direct and indirect hernia

2830
Q

Herniotomy

A

Excivision of hernial sac

2831
Q

Herniorrhaphy

A

Suture repair of hernia defect

2832
Q

Hernioplasty

A

Mesh repair of hernial defect

2833
Q

Epidemiology of inguinal hernia

A

3% of adults will require hernioplasty

4% of male neonates will have hernia

M>F 9:1 (due to testes descent)

Majority present in 50s

2834
Q

Pathology of inguinal hernia

A

Commoner on R (?damage to ilioinguinal N and appendicectomy-> muscle weakness)

5% are bilateral

8-15% present as emergency with strangulation/obstruction

2835
Q

Aetiology of inguinal hernias

A

Congenital

Acquired

2836
Q

Congenital inguinal hernias

A

Patent processus vaginalis

Should obliterate following descent of the testes

If it stays patent, it may fill with Fluid-> hydrocele

Bowel/omentum-> indirect hernia

2837
Q

Acquired causes of inguinal hernia

A

Mainly things which raise IAP

Chronic cough: COPD, asthma

Prostatism

Constipation

Severe muscular effort e.g. heavy lifting

Previous incision/repair

Ascites/ obesity

Appendicectomy

2838
Q

Classification of inguinal hernias

A

Indirect

Direct

2839
Q

What is the most common type of inguinal hernia?

A

Indirect (80%), commoner in young

2840
Q

Features of indirect inguinal hernia

A

Congenital patent processus vaginalis

Emerge through deep ring

Same 3 coverings as cord and descend into the scrotum

Can strangulate

2841
Q

Features of direct hernia

A

20%: commoner in elderly

Acquired

Emerge through Hesselbach’s triangle

Can acquire internal and external spermatic fascia

Rarely descend into the scrotum

Rarely strangulate

2842
Q

Borders of Hesselbach’s trianagle

A

Inguinal ligament as base

Inferior epigastric vessels as the lateral border

Lateral border of rectus sheath as medial border

FC Hesselbach: German anatomist and surgeon

2843
Q

Direct inguinal hernia occur through which structure?

A

Hesselbach’s triangle

2844
Q

Layers of the abdominal wall

A

Skin

Camper’s fascia: fatty

Scarpa’s fascia: membranous

External olbique

Internal oblique

Nerves and vessels

Transversus abdominis

Transversalis fascia

Pr-peritoneal fat

Parietal peritoneum

2845
Q

Location of deep inguinal ring

A

1.5cm above femoral pulse or mid-point of inguinal ligament (ASIS-> PT)

2846
Q

Contents of the inguinal canal

A

Ilioinguinal N and spermatic cord in males

Ilioinguinal nerve, genital branch of genitofemoral nerve and round ligament of uterus in females

2847
Q

Course of the ilioinguinal nerve

A

Enters canal directly through the anterior wall, does not pass through the deep ring

Exits through the superficial ring

2848
Q

Function of the ilioinguinal nerve

A

Supplies skin at the root of the penis and the scrotum and small area of skin on the upper thigh

2849
Q

Contents of the spermatic cord

Rule of 3s

A

3 layers of fascia:

External spermatic: from external oblique

Cremasteric: from internal oblique

Internal spermatic: from transversalis fascia

3 arteries:

Testicular: from aorta

Cremasteric: from inferior epigastric

Artery of vas: from inferiror vesciular artery

3 veins:

Pampiniform plexus-> R drains into IVC, left to left renal

Cremasteric vein

Vein of vas

2 nerves:

Nerve to cremaster from genito-femoral nerve

Sympathetic fibres from T10-T11

ilioinguinal N (is on the cord)

3 other structures:

Vas deferens

Lymphatics of the testes-> para aortic nodes

Obliterated processus vaginalis

2850
Q

Layers of fascia in spermatic cord

A

External speramtic fascia: from external oblique

Cremasteric fascia: from internal oblique

Internal spermatic fascia: from transversalis fascia

2851
Q

Arteries of the spermatic cord

A

Testicular: from aorta

Cremasteric: from inferior epigastric

Artery of vas: from the inferior vesicular arteries

2852
Q

Veins of the spermatic cord

A

Pampiniform plexus-> R-> IVC, L-> LRV

Cremasteric vein

Vein of vas

2853
Q

Nerves of spermatic cord

A

Ilioinguinal on the cord

Nerve to cremaster: from genitofemoral nerve

Sympathetic fibres from T10-T11

2854
Q

Additional structures in the spermatic cord

A

Vas deferens

Lymphatics of the testes (drain to para-aortic nodes)

Obliterated processus vaginalis

2855
Q

Features of inferior epigastric vessels

A

Arise from EIA immediately superior to inguinal ligament

Can be seen passing deep to the posterior wall

Sac arising laterally= indirect

Sac arising medially= direct

2856
Q

Structure of the inguinal canal

A

4cm long

Floor: inguinal ligament

Roof: arching fibres of the internal olbique and transversus abdominis

Anterior: external oblique aponeurosis and internal oblique for lateral 3rd

Posterior: transveralis fascia and conjoint tendon for medial 3rd

Laterally: deep ring

Medially: pubic tubercle

2857
Q

Conjoint tendon=

A

Combined insertion of internal oblique and trasnversus abdominis into pubic crest and pectineal line

2858
Q

Where does the rectus sheath end?

A

Below arcuate line

2859
Q

Ix in inguinal hernias

A

US if Dx in doubt

2860
Q

Lump in groin which may descend into scrotum

Exacerbated by crying

Commonly obstruct

Child

A

Indirect inguinal hernia

2861
Q

Lump in groin exacerbated by straining/cough

May be clear precipitating event

Dragging pain radiating to groin

May present with obstruction/strangulation

Adult

A

Inguinal hernia ?direct

2862
Q

Key questions in hernia hx

A

Reducible?

Ever episodes of obstruction/strangulation

Predisposing factors: cough, straining, lifting

Occupation and social circumstances

2863
Q

Mx of hernia

A

Non-surgical:

Rx RFs: cough, constipation

Lose weight

Truss

Surgical:

Tension-free mesh (e.g. Lichenstein repair) better cf. suture repair (<2% vs 10% recurrence)

Open approach can be done under LA or GA

Lap approach allows bilateral repair and improved cosmesis. Also preferred for recurrent hernias

1o unilateral repairs should be open

Children only require sac excision (herniotomy)

2864
Q

Early Cx of hernia repair

A

Haematoma/seroma formation (10%)

Intra-abdominal injury (lap)

Infection: 1%

Urinary retention

2865
Q

Late cx of hernia repair

A

Recurrence (<2%)

Iscaehmic orchitis: 0.5%

Chronic groin pain/paraesthesia: 5%

2866
Q

Def: femoral hernia

A

Protrusion of viscus through the femoral canal

2867
Q

Epidemiology of femoral hernia

A

F>M

Inguinal hernias are still more comon in F

Middle aged and elderly

2868
Q

Aetiology of femoral canal

A

Acquired: raised intra-abdominal pressure

Femoral canal larger in females due to shape of pelvis and changes in its configuration due to childbirth

2869
Q

Neck inferior and lateral to pubic tubercle

Cough impulse

Often irreducible (due to tight borders)

Commonly present with obstruciton or strangulation

Tender, red and hot

Abdo pain, distension, vomiting, constipation

A

?Femoral hernia

2870
Q

Mx of femoral hernia

A

50% risk of strangulation within 1m

Require urgent surgery

Elective or Emergency

2871
Q

Elective femoral hernia repair

A

Lockwood Approach

Low incision over hernia with herniotomy and herniorrhaphy: suture inguinal ligament to pectineal ligament

2872
Q
A
2873
Q

Emergency femoral hernia repair

A

High approach in inguinal region to allow inspection and resection of non-viable bowel

Then herniotomy and herniorrhaphy

Vertical incision

2874
Q

Anatomy of femoral sheath

A

In the femoral triangle, femoral artery vein and lymphatics are enclosed within femoral sheath

Sheath is contiuous superiorly with transversalis fascia

Each structure has its own compartment

Most medial compartment is the femoral canal

2875
Q

Borders of the femoral canal

A

Anterior: inguinal ligaemnt

Posterior: pectineal ligament (ligament of Cooper and pectineus)

Medially: lacunar ligament and pubic bone

Laterally: femoral vein

Contents: fat and Cloquet’s node

2876
Q

Cloquet’s node

A

Cloquet’s node (or gland) is a lymph node[1][2] found in the inguinal region.

It is named for French surgeon Jules Germain Cloquet.[3][4]

It can be considered the superiormost deep inguinal lymph nodes or inferiormost external iliac lymph nodes.[

2877
Q

Def: incisional hernia

A

Hernia arising through a previously acquired defect

2878
Q

Epidemiology of incisional hernia

A

6% of surgical incisions

2879
Q

Categorisation of RFs for incisional hernia

A

Pre-operative

Intra-operative

Post-operative

2880
Q

Pre-operative RFs for incisional hernia

A

Increased age

Obesity or malnutirtion

Comorbidities: DM, renal failure, malignancy

Drugs: steroids, chemo, RTx

2881
Q

Intra-operative RFs for incisional hernia

A

Surgical technique/skill (major factor)

Too small suture bites

Inappropriate suture material

Incision type e.g. midline

Placing drains through wounds

2882
Q

Post-operative RFs for incisional hernia

A

Raised IAP: chronic cough, post-op ileus, straining

Infection

Haematoma

2883
Q

Mx of incisional hernia

A

Sx not appropriate for all patients

Must balance risk of operation and recurrence with risk of obstruction/strangulation and patient choice

Usually broad-necked, therefore low risk of strangulation

2884
Q

Mx of incisional hernia

A

Conservative:

Manage RFs e.g. constipation, cough

Weight loss

Elasticated corset or truss

Surgical:

Pre-op:

Optimise cardiorespiratory funciton

Encourage weight loss

Nylon mesh repair: open or lap

2885
Q

Features of umbilical hernias

A

Congenital

3% of LBs

Defect in umbilical scar

2886
Q

RFs for umbilical hernia

A

Afro-caribbean

Trisomy 21

Congenital hypothyroidism

2887
Q

Mx of umbilical hernia

A

Usually resolves by 2-3y

Mesh repair if no closure

Can recur in adults: pregnancy or gross ascites

2888
Q

Features of paraumbilical hernia

A

Acquired: middle aged obese men

Defect through linea alba just above or below umbilicus

Small defect-> omentum, often strangulates

2889
Q

RFs for paraumbilical hernia

A

Chronic cough

Straining

2890
Q

Mx of paraumbilcal repair

A

Mayo (double breast linea alba with sutures)/ mesh repair

2891
Q

Feature of epigastric hernia

A

Young M>F

Pea-sized swelling caused by defect in linea alba above umbilicus

Usually contains omentum: can strangulae

2892
Q

Mx of epigastric hernia

A

Mesh repair

2893
Q

Features of Spigelian hernia

A

Hernia through linea semilunaris

Hernia lies between layers of abdo wall

Palpable mass more likely to be colon Ca

2894
Q

Features of obturator hernia

A

Old aged F>M

Sac protrudes through obturator foramen

Pain on inner aspect of thigh or knee

Frequently present obstructed/strangulated

2895
Q

Features of lumbar hernia

A

Middle-aged M>F

Typically following loin incisions

Hernia through superior/inferior lumbar triangles

2896
Q

Features of sciatic hernia

A

Hernia through LSF

Usually presents as SBO + gluteal mass

2897
Q

Gluteal hernia

A

Hernia through GSF

Usually presents as SBO + gluteal mass

2898
Q

Anatomy of the rectum

A

12cm

Sacral promontory to levator ani muscle

3 tenia coli fuse around the rectum to form a continuous muscle layer

2899
Q

Anatomy of the anal canal

A

4cm

Levator ani muscle to anal verge

Upper 2/3rd of the canal:

lined by columnar epithelium

Insensate

Superior rectal artery and vein

Internal iliac nodes

Lower 1/rd of the canal:

Lined by squamous epithelium

Sensate

Middle and inferior rectal arteries and veins

Superficial inguinal nodes

2900
Q

What is the dentate line?

A

Squamomucosal junciton between columnar epithelium of upper 2/3rd of anal canal and lower 1/3rd which is lined by squamous epihtelium

2901
Q

What is the white line

A

Where anal canal becomes true skin

2902
Q

What are the anal sphincters?

A

Internal

External

2903
Q

Features of internal anal sphincter

A

Thickening of rectal smooth muscle

Involuntary control

2904
Q

Features of external anal sphincter

A

Three rings of skeltal muscle: deep, superficial, subcutaenous

Voluntary control

2905
Q

What is the anorectal ring

A

Deep segment of external sphincter which is continuous with puborectalis muscle (part of levator ani)

Palpable on PR: 5cm from anus

Demarcates junction between the anal canal and rectum

Must be preserved to maintain continence

2906
Q

Def: perianal haematoma

A

Subcutaneous bleeding from a burst venule caused by straining or passage of hard stool

Also called an external pile: misnomer

2907
Q

Tender blue lump at the anal margin

Pain worsened by defacation or movement

A

Perianal haematoma

2908
Q

Rx of perianal haematoma

A

Analgesia and spontaenous resolution

Evacuation under LA

2909
Q

Young anxious men

Crampy anorectal pain, worse at night

Unrelated to defecation

Associated with trigeminal neuralgia

A

Proctalgia fugax

2910
Q

Rx in proctalgia fugax

A

Reassurance

GTN cream

2911
Q

Features of perineal warts

A

Commonly seen in MSM

HPV

Syphillis

2912
Q

Condylomata accuminata

A

HPV

Rx: podophyllin paint, cryo, surgical excision

2913
Q

Condylomata lata

A

Syphillis

Rx: penicllin

2914
Q

Causes of pruritus ani

A

50% idiopathic

Poor hygiene

Haemorrhoids

Anal fissure

Anal fistula

Fungi, worms

Crohn’s

Neoplasia

2915
Q

Def: haemorrhoids

A

Disrupted and dilated anal cushions

2916
Q

Pathophysiology of haemorrhoids

A

Anal cushion: mass of spongy vascular tissue

Positioned at 3, 7, and 11 o’ clock where the three major arteries that feed the vascular plexuses feed the anal canal

Gravity, straining-> engorgement and enlargement of anal cushions

Hard stool disrupts connective tissue around cushions

Cushions protrude and can be damaged by hard stool-> bright red capillary bleeding

Haemorrhoids arise above dentate line therefore NOT PAINFUL

May be gripped by anal sphincter-> thrombosis, strangulated piles are acutely painful. May ulcerate or infarct

2917
Q

Causes of haemorrhoids

A

Constipation with prolonged straining

Venous congestion may contribute

Pregnancy, abdominal tumour, portal HTN

2918
Q

Internal haeomorrhoid classification

A

Grade 1: never prolapse

2nd: prolapse on defecation but spontaneously reduce
3rd: prolapse on defecation but require digital reduction
4th: remain permanently prolapsed

2919
Q

Fresh painless PR bleeding: bright red, on paper, on stool, may drip into pan

Pruritus ani

Lump in perianal area

Severe pain= thrombosis

A

Haemorrhoids

2920
Q

Examination in haemorrhoids

A

Full abdo exam palpating for masses

Inspect perianal area: masses, recent bleeding

DRE: can’t palpate piles unless thrombosed

Rigid sig to ideitify higher rectal pathology

Proctoscopy: also allows Rx

2921
Q

DDx in haemorrhoids

A

Perianal haematoma

Fissure

Abscess

Tumour (must exclude in all cases)

2922
Q

Mx of haemorrhoids

A

Conservatve:

Increase fibre and fluid intake

Stop straining at stool

Medical:

Topical preparations: anusol (hydrocortisone), topical analgesics

Laxatives: lactulose, fybogel

Interventional:

Injection scleropathy

Barron’s banding

Cryotherapy

Infrared coagulation

Bipolar diathermy

Surgical:

Haemorrhoidectomy

2923
Q

Injection scleropathy

A

Injection sclerosant (5% phenol in Almond oil)

Injection above dentate line

SE: impotence, prostatisi

2924
Q

Barron’s banding

A

Haemorrhoid intervention: thrombosis and separation

SE: bleeding, infection

2925
Q

SE cryotherpay for haemorrhoids

A

Watery discharge3 post procedure

2926
Q

What is most effective outpatient haemorrhoid rx

A

Band ligation

2927
Q

Surgical options for haemorrids

A

Haemorrhoidectomy:

Excision of piles and ligation of vascular perdicles

Lactulose and metronidazole 1w pre-op

Discharge with laxatives post-op

Haemorrhoid artery ligation can also be used

2928
Q

SE of haemorrhoidectomy

A

Bleeding

Stenosis

2929
Q

Mx of thrombosed piles

A

Analgesia

Ice-packs

Stool softeners

Bed rest with elevate foot of bed

Pain usually resolves in 2-3w

Hamoerrhoidectomy is not usually necessary

2930
Q

Def: anal fissure

A

Tear of squamous epithelial lining in lower anal canal

2931
Q

Causes of anal fissure

A

Mostly trauma 2o to the passage of hard stool

Associated with constipation

Spasm of internal anal sphincter contributes to pain and ischaemia-:> poor healing

Commoner in women

Rarer causes often lead to multiple +/- lateral fissures: crohn’s, herpes, anal ca

2932
Q

Intense anal pain, especially on defecation, may prevent patients from passing stools

Fresh rectal bleeding, mostly on paper

A

Anal fissure

2933
Q

Examination findings in anal fissure

A

PR often impossible

Midline ulcer seen, usually posteriorly at 6 o’clock

May be anterior

May be a mucosal tag: sentinal pile, usually at 6 o’clock

Groin LNs suggest complicating factor e.g. HIV

2934
Q
A

Anal fissure with sentinal skin pile

2935
Q

Mx of recurrent, chronic or difficult to Rx anal fissures

A

EUA

2936
Q

Mx of fissure in ano

A

Conservative:

Soaks in warm bath

Toileting and dietary advice

Medicals:

Lactulose + fybogel

Topical: resolution in 75: lignocaine, GTN, diltiazem

Botulinum injeciton

Surgical:

Lateral partial sphincterectomy

2937
Q

Surgical Mx of anal fissure

A

Lateral partial sphincterectomy

Division of internal anal sphincter at 3 o’clock

Pre-op anorectal US and mannometry

Lord’s operation is no longer used due to high rates of incontinence (anal dilatation under anaesthesia)

2938
Q

Cx of lateral partial sphincterectomy

A

Minor feacal/flatus incontinence

Perianal abscess

2939
Q

Def: anal fistula

A

Abnromal connection between ano-rectal canal and the skin

2940
Q

Pathogenesis of anofistula

A

Usually occurs 2o to perianal sepsis

Blockage of intramuscular glands-> abscess

Abscess discharges to form a fistula

2941
Q

Associations of fistula in ano

A

Crohn’s

Diverticular disease

Rectal Ca

Immunosuppression

2942
Q

Classficiation of anal fistula

A

Parks Classificaiton

inter-sphincteric (~70%): fistula crosses the intersphincteric space and does not cross the external sphincter

trans-sphincteric (25%): fistula crosses from the intersphincteric space, through the external sphincter and into the ischiorectal fossa

supra-sphincteric (5%): fistula passes superiorly into the intersphincteric space, and over the top of the puborectalis muscle then descending through the iliococcygeus muscle into the ischiorectal fossa and then skin

extra-sphincteric (1%): fistula crosses from perineal skin through the ischiorectal fossa and levator ani muscle complex into the rectum (i.e. is outside the external anal sphincter)

2943
Q

What is Goodsall’s rule

A

Determines the path of an anal fistula tract

Internal opening of the fistula is dependant on where the fistula is located relative to the anal clock and the transverse anal line

If the internal opening is anterior to the transverse anal line, there will be a usually simple direct radial fistulous tract

If the internal opening is posterior to the transverse anal line, there will be a tortusous and often more complex fistulous tract that enters posteriorly in the midline (6 o’clock)

Exception: anterior fistulas lying more than 3cm from the anus which may have a curved track that opens into the posterior midline of the anal canal

2944
Q
A
2945
Q
A
2946
Q

Persitent anal discharge

Perianal pain or discomfort

A

?anal fistula

2947
Q

Examination may visualise external opening: may be pus

Induration around fistula on DRE

Proctoscopy may reveal internal opening

A

anal fistula

2948
Q

Ix in anal fistula

A

MRI

Endoanal US

2949
Q

Rx of anal fistula

A

Extent of fistula must be delineated by probing the fistula @ EUA

Approach varies dependant on whether the fistula is high or low

2950
Q

Mx of low anal fistula

A

Fistulotomy and excision

Laid open to heal by 2o intention

2951
Q

Mx of high anal fistula

A

Fistulotomy could damage the anorectal ring

Suture: a seton: passed through fistula and gradually tightened over months. Stimulates fibrosis of tract. Scar tissue holds sphincter together

2952
Q

Pathogenesis of peri-anal sepsis/abscess

A

Anal gland blockage-> infection-> abscess

e.g. E. Coli, bacteroides

May develop from skin infections e.g. sebaceous gland or hair follcile

Staphs

2953
Q

Associations of peri-anal sepsis/abscess

A

Crohn’s

DM

Malignancy

2954
Q

Classficaiton of peri-anal abscesses

A

Perianal: 45%

discrete local red swelling close to the anal verge

Ischiorectal <30%:

systemic upset, extremely painful on DRE

Intersphincteric/intermuscular: >20%

Pelvirectal/supralevator: 5%

Systemic upset bladder irritaiton

2955
Q
A
2956
Q

Throbbing perianal pain, worse on sitting

Occasionally a purulent anal discharge

Perianal mass or cellulitic area

Fluctuant mass on PR

septic signs

A

?Peri-anal abscess

2957
Q

Rx in perianal abscess

A

Abx may suffice if instigated early

Most cases require EUA with I&D

Wound packed

Heals by second intention

Daily dressing for 7-10d

Look for anal fistula which complicates 30% of abscesses

2958
Q

Different types of wound healing

A

Primary intention: healing of clean wound without tissue loss, i.e. edges brought together

Secondary intention: wound allowed to granulate, resulting in broader scar

Teritary intention: intially, cleaned debrided, purposely left open.

2959
Q

Def: pilonoidal sinus

A

Pilonoidal: nest of hair

Sinus: blind ending tract, lined either by epithelial or granulation tissue which opens onto an epithelial surface

2960
Q

Pathophysiology of pilnoidal sinus

A

Hair works way beneath skin-> foreign body reaction-> abscess formation

usually occurs in the natal cleft

2961
Q

RFs for pilnoidal sinus

A

M>F 4:1

Mediterranean, Middle east, Asians

Often overweight with poor personal hygiene

Occupation with lots of sitting

2962
Q

Persistent discharge or purulent or clear fluid

Recurrent pain

Abscesses

Natal cleft

A

Pilonidal sinus

2963
Q

Rx in pilonidal sinus

A

Conservative:

Hygiene advice

Shave/remove hair from affected area

Surgical:

I&D of abscesses

Elective sinus excision: methylene blue to outline tract. Recurrence in 4-15%

2964
Q

Epidemiology of anal carcinoma

A

Relatively uncommon 250-300 caes per year in UK

2965
Q

Pathology of anal carcinoma

A

80% are SCCs, others: melanomas, adenocarcinomas

Anal margin tumours

Anal canal tumours

2966
Q

Features of anal margin tumours

A

Well differentatied keratinising lesions

Common in men

Good Px

2967
Q

Features of anal canal tumours

A

Arise above dentate line

Poorly differentiated, non-keratinising

Commoner in women

Worse prognosis

2968
Q

Spread of anal carcinoma

A

Above dentate line-> internal iliac nodes

Below dentate line-> inguinal nodes

2969
Q

Aetiology of anal carcinoma

A

HPV 16, 18, 31, 33 is important factor

Increased incidence in MSM

Increased incidence if perianal warts

2970
Q

Perianal pain and bleeding

Pruritus ani

Faecal incontinence

May-> rectovaginal fistula

A

Anal carcinoma

2971
Q

Faecal incontinence in anal carcinoma

A

70% have sphincter involvement at presentation

Important RED FLAG

2972
Q

Examination findings in anal carcinoma

A

Palpable lesion in only 25%, +/- palpable inguinal nodes

2973
Q

Ix in anal carcinoma

A

FBC: ACD

Endoanal US

Rectal EUA and biopsy

CT/MRI to assess pelvic spread

2974
Q

Rx of anal carcinoma

A

ChemoRTx: most patients. 50% 5y

Sx: reserved for

Tumours that fail to respond

GI obstruction

Small anal margin tumours without sphincter involvement

2975
Q

Def: rectal prolapse

A

Protrusion of rectal tissue through the anal canal

2976
Q

Classification of rectal prolapse

A

Type 1: mucosal prolapse

Partial prolapse of redundant mucosa

Commoner in children, especially in CF

Essentially large piles, same Rx

Type 2: Full thickness prolapse

Commoner cf. type 1

Usually elderly females with poor O+G hx

2977
Q

Mass extruding from rectum on defecation

May reduce spontaneously or require manual reduction

May become oedmeatous and ulcerated, pain and bleeding

Faecal soiling

Associated with vaginal prolapse and urinary incontinennce

A

Rectal prolapse

2978
Q

Examination findings in rectal prolapse

A

Visibile prolapse: brought out on straining

+/- excoriation and ulceration

Reduced sphincter tone on PR

Associated with uterovaginal prolapse

2979
Q

Ix in rectal prolapse

A

Sigmoidoscopy to exclude proximal lesions

Anal manometry

Endoanal US

MRI

2980
Q

Rx partial rectal prolapse

A

Phenol injection

Rubber band ligation

Delorme’s procedure

2981
Q

Delorme’s procedure

A

A Delorme’s procedure aims to repair the prolapse. This operation involves the surgeon remov- ing some of the prolapsed lining of the rectum (mucosa) and reinforcing the muscle of the rectum by placating stitches. This is done via the anus. No external incision is needed.

2982
Q

Mx of complete rectal prolapse

A

Conservative:

Pelvic floor exercises

Stool softeners

Sx:

Abdominal appraoach:

Rectopexy, lap or open. Mobilised rectum fixed to sacrum with mesh

Perineal appraoch: Delorme’s procedure

2983
Q

Relative proportion of small bowel neoplasms

A

35% benign

65% malignant (2% of GI malignancies)

2984
Q

Benign small bowel neoplasms

A

Lipoma

Leiomyoma

Neurofibroma

Haemangioma

Adenomatous polyps (FAP, Peutz-Jehgers)

2985
Q

Malignant small bowel neoplasms

A

Adenocarcinoma (40% of malignant tumours)

Carcinoid (40%)

Lymphoma (especially with coeliac disease: EATL)

Gastrointestinal stromal tumours

2986
Q

Presentation of small bowel neoplasms

A

Often non-specific symptoms so present late

N/V, obstruction

Weight loss and abdominal pain

Bleeding

Jaundice from biliary obstruction or liver mets

2987
Q

Imaging in small bowel neoplasms

A

AXR: SBO

Ba follow through

CT

Endocscopy:

Push enteroscopy

Capsule enteroscopy

2988
Q

Def: carcinoid tumours

A

Diverse group of neuroendocrine tumours of enterochromaffin cell origin capable of producing 5HT

May be derived from

foregut: resp tract

Midgut: stomach, ileum, appendix

Hindgut: colorectum

2989
Q

What can carcinoid tumours secrete

A

5HT

Vasoactive intestinal peptide

Gastrin

Glucagon

Insulin

ACTH

Hindgut tumours rarely secrete 5HT

2990
Q

What does carcinoid syndrome suggest?

A

Bypass of FPM and is strongly associated with metastatic disease

2991
Q

With what condition are carcinoid tumours associated?

A

10% with MEN1

2992
Q

Sites of caricnoid tumours

A

Appendix: 45%

Ileum: 30%

Colorectum: 20%

Stomach: 10%

Elsewhere in GIT

Bronchus: 10%

Cosider all as malignant

2993
Q

What is the most common site of carcinoid tumours?

A

Appendix

2994
Q

Features of carcinoid syndrome

FIVE HT

A

Flushing: paroxysmal, upper body +/- wheals

Intestinal: diarrhoea

Valve fibrosis: TR and PS

whEEze: bronchoconstriction

Hepatic involvement: bypassed FPM

Tryptophan deficiency: pellagra (3Ds)

2995
Q

Local presentation of carcinoid tumours

A

Appendicits

Intussuception or obstruction

Abdominal pain

2996
Q

Ix in carcinoid tumours

A

Increased urine 5-hydroxyindoleacetic acid: 5HT metabolite

Raised plasma chromogranin A

CT/MRI: find primary

2997
Q

Rx in carcinoid tumour

A

Symptoms: octreotide or loperamide

Curative:

tumours are very yellow

Give octerotide to avoid carcinoid crisis

2998
Q

Pathophysiology of carcinoid crisis?

A

Tumour outgrows blood supply or is handled too much-> massive mediator release

Leads to vasodilatoin, hypotension, bronchoconstriction, hyperglycaemia

2999
Q

Rx in carcinoid crisis?

A

High-dose octreotide

3000
Q

Px in carcinoid tumours

A

Median survival is 5-8y

~3y if mets present

3001
Q

Def: acute appendicits

A

Inflammation of the vermiform appendix ranging from oedema to ischaemic necrosis and perforation

3002
Q

Epidemiology of appendicitis?

A

Incidence: 6% lifetime incidence, commonest surgical emergency

Rare <2y. Maximal peak during childhood, decreases thereafter

3003
Q

Pathogenesis of appendicitis

A

Obstruction of the appendix:

faceolith most commonly

lymphoid hyperplasia post-infection

Tumour e.g. caecal Ca, carcinoid

Worms e.g. Ascaris lumbicoides, Schisto

Gut organisms-> infection behind obstruction

Leads to oedema, ischaemia, necrosis, perforation

Peritonitis

Abscess

Appendiceal mass

3004
Q

Pattern of abdominal pain in appendicits?

A

Early inflammation-> appendiceal irritation:

Visceral pain is not well localised cf. somatic pain

Nociceptive info travels in the sympathetic afferent fibres that supply the viscus

Pain is refrred to the dermatome corresponding toth e spinal cord level of these sympathetic fibres

Append= midgut= lesser splanch (T10/11)= umbilical pain

Late inflammation-> parietal peritoneum irritation

Pain localised to RIF

3005
Q

Symptoms of acute appendicits

A

Colicky abdominal pain. Central-> localised in RIF. Worse with movement

Anorexia

Nausea (vomiting is rarely prominent)

Constipation/diarrhoea

3006
Q

Colicky abdominal pain. Central-> localised in RIF. Worse with movement

Anorexia

Nausea (vomiting is rarely prominent)

Constipation/diarrhoea

A

Appendicits

3007
Q

Signs of appendicits

A

Low grade pyrexia: 37.5-38,5

Increased HR, shallow breathing

Foetor oris

Guarding and tenderness @ McBurney’s point: +ve cough/percussion tenderness

Appendix may be palpable in RIF

Pain PR suggests pelvic appendix

3008
Q

Low grade pyrexia: 37.5-38,5

Increased HR, shallow breathing

Foetor oris

Guarding and tenderness @ McBurney’s point: +ve cough/percussion tenderness

Appendix may be palpable in RIF

Pain PR suggests pelvic appendix

A

Acute appendicits

3009
Q

McBurney’s point

A

1/3rd from ASIS to the umbilicus

3010
Q

Blumberg’s sign

A

Blumberg’s sign, also referred to as rebound tenderness, is a clinical sign that is elicited during physical examination of a patient’s abdomen by a doctor or other health care provider. It is indicative of peritonitis. It refers to pain upon removal of pressure rather than application of pressure to the abdomen.

3011
Q

Rovsing’s sign

A

Pressure in LIF-> more pain RIF

Appendicits

3012
Q

Psoas sign

A

Pain on extending the hip: retrocaecal appenidix

3013
Q

Cope Sign

A

Flexion + internal rotation of R hip-> pain

Appendix lying close to obturator internus

3014
Q

DDx for appendicits

A

Surgical:

Cholecystitis

Diverticulitis

Meckel’s diverticulitis

Gynae:

Cyst accident: torsion, rupture, haemorrhage

Salpingitis/PID

Medical:

Mesenteric adenitis

UTI

Crohn’s

3015
Q

Ix in appendicitis

A

Dx is primarily clinical

Bloods: FBC, CRP, amylase, G+S. clotting

Urile:

Sterile pyruia may indicate bladder irritation

Ketones: anorexia

Exclude UTI

Beta HCG

Imaging:

USS: exclude gynae path. Visualise inflamed appendix

CT: can be used

Diagnostic lap

3016
Q

Mx of appendicits

A

Fluids

Abx: cef 1.5mg + met 500g IV TDS

Analgesia: paracetamol, NSAIDs, codeine phosphate

Certain dx-> appendicectomy

Uncertain dx-> active observation

3017
Q

Cef + met doseages

A

cef 1.5g met 500g IV TDS

3018
Q

Cx of appendicitis

A

Appendix mass

Apppendix abscess

Perforation

3019
Q

Appendiceal mass

A

Inflammed appendix with adherent covering of omentum and small bowel

3020
Q

Dx of appendix mass

A

US or CT

3021
Q

Mx of appendix mass

A

Initially: Abx and NBM

Resolution of mass-> interval appendicectomy

Exclude a colonic tumour: colonoscopy

3022
Q

Features of appendix abscess

A

Results if appendix mass doesn’t resolve

Mass enlarges, patient deterioirates

3023
Q

Mx of appendix abscess

A

Abx + NBM

CT guided percutaneous drainage

If no resolution, sx may involve right hemicolectomy

3024
Q

Features of appendicits-> perforation

A

Commoner if faecolith present and in young children (as dx is often delayed)

Deteriorating patient with peritonitis

3025
Q

Epidemiology of UC

Prev

Age

Sex

A

100-200/100,000

30s

F>M

3026
Q

Aetiology of UC

A

Concordance= 10%

Smoking protective

TH2 mediated

3027
Q

Epdiemiology of Crohn’s

Prev

Age

Sex

A

50-100,000

20s

F>M

3028
Q

Aetiology of Crohns

A

Concordance= 70%

Smoking increases risk

TH1/Th17 mediated

3029
Q

Macroscopic appearance of UC

A

Found in rectum and colon +/- backwash ileitis

Contiguous

No strictures

3030
Q

IBD

Found in rectum and colon +/- backwash ileitis

Contiguous

No strictures

A

UC

3031
Q

Macroscopic appearance of Crohn’s

A

Found mouth to anus, especially terminal ileum

Skip lesions

Strictures present

3032
Q

IBD

Found mouth to anus, especially terminal ileum

Skip lesions

Strictures present

A

Crohn’s

3033
Q

Microscopic apperance of UC

A

Mucosal inflammation

Shallow and broad ulceration

No fibrosis

No granulomas

Marked pseudopolyps

No fistulae

3034
Q

IBD

Mucosal inflammation

Shallow and broad ulceration

No fibrosis

No granulomas

Marked pseudopolyps

No fistulae

A

UC

3035
Q

Microscopic appearance of Crohn’s

A

Transmural inflammation

Deep, thin, serpiginious ulceration-> cobblestone mucosa

Marked fibrosis

Granulomas present

Minimal pseudopolyps

Fistulae present

3036
Q

IBD

Transmural inflammation

Deep, thin, serpiginious ulceration-> cobblestone mucosa

Marked fibrosis

Granulomas present

Minimal pseudopolyps

Fistulae present

A

Crohn’s

3037
Q

Systemic features of UC and crohn’s

A

Fever, malaise, anorexia, weight loss in active disease

3038
Q

Diarrhoea

Blood +/- mucus PR

Abdominal discomfort

Tenesmus, faecal urgency

IBD

A

UC

3039
Q

Diarrhoea (not usually bloody)

Abdominal pain

Weight loss

IBD

A

Crohn’s

3040
Q

Abbdominal signs of UC

A

Fever

Tender, distended abdomen

3041
Q

Abdominal signs of Crohn’s

A

Apthous ulcers

Glossitis

Abdominal tenderness

RIF mass

Perianal abscesses, fistulae, tags

Anal/rectal strictures

3042
Q

How can the extraintestinal signs of IBD be classified?

A

Skin

Eyes

Joints

HPB

Other

3043
Q

Skin manifestations of IBD

A

Clubbing

Erythema nodosum

Pyoderma gangrenosum (esp. UC)

3044
Q

Eye manifestations of IBD

A

Iritis

Conjunctivitis

Episcleritis

Scleritis

3045
Q

Joint manifestations of IBD

A

Arthritis (non-deforming, asymmetrical)

Sacroilitis

Ank spond

3046
Q

HPB manifestations of IBD

A

PSC and cholangiocarcinoma (esp. UC)

Gallstones (esp. Crohn’s)

Fatty liver

3047
Q

Other extra-intestinal manifestations of IBD

A

Amyloidosis

Oxalate renal stones (esp. Crohn’s)

3048
Q

Cx of UC

A

Toxic megacolon

Bleeding

Malignancy

Cholangiocarcinoma

Strictures-> obstruction

Venous thrombosis

3049
Q

Features of toxic megacolon

A

Diameter >6cm

Risk of perforation

3050
Q

Malignancy in UC

A

CRC in 15% with pancolitis for 20y

Cholangiocarcinoma also a risk

3051
Q

Cx of CD

A

Fistulae

Strictures-> obstruction

Abscesses

Malabsorption

Toxic megacolon and Ca may occur although this is less common than in UC

3052
Q

Fistulae in Crohn’s

A

Entero-enteric/colonic-> diarrhoea

Enterovescial-> frequency, UTI

Enterovaginal

Perianal-> pepperpot anus

3053
Q

Abscess in crohn’s

A

Abdominal

Anorectal

3054
Q

Malabsorption in Crohn’s

A

Fat-> steatorrhoea, gallstones

B12-> megaoloblastic anaemia

Vit D-> osteomlacia

Protein-> oedema

3055
Q

Ix in UC

A

Bloods

Stool

Imaging

Ileocolonoscopy and regional biopsy

3056
Q

Bloods in UC

A

FBC: reduced Hb, raised WCC

LFT: reduced albumin

Raised CRP/ESR

Blood cultures

3057
Q

Stool in UC

A

MCS: exclude campy, shigella, salmonella

CDT: C d.iff may complicate or mimic

3058
Q

Imaging in UC

A

AXR: megacolon (>6cm)

CXR: perforation

CT

Ba/gastrograffin enema:

Lead pipe: no haustra

Thumbprinting: mucosal thickening

Pseudopolyps: regenerating mucosal island

3059
Q

Ileocolonoscopy and regional biopsy in UC:

A

Baron Score

3060
Q

Ba

A

Lead pipe colon

UC

3061
Q
A

Dilated transverse colon noted. The abdomen demonstrates markedly dilated transverse colon (9 cm) with impression of slight dilatation of the descending colon with some” thumb printing” in the wall (consistent with mucosal inflammation in the clinical circumstances). No free subphrenic gas is seen.

UC

3062
Q

Baron score

A

n endoscopic scoring system devised for ulcerative colitis, which has been used in clinical practice as the modified Baron score:
• Normal—normal mucosa; inactive disease.
• Mild—erythema; decreased vascular pattern.
• Moderate—marked erythema; loss of vascular pattern; mucosal friability.
• Severe—spontaneous bleeding; ulceration.

3063
Q

What can be used to grade the severity of UC?

A

Truelove and Witts criteria

3064
Q

Components of Truelove and Witts criteria

A

Bowel movements

PR bleeding

Pyrexia (>37.8)

HR (>90)

Anaemia

ESR

3065
Q

Truelove and Witts criteria:

Mild UC

A

<4 bm

Small PR bleeding

No pyrexia

HR <90

Anaemia not present

ESR <30

3066
Q

Truelove and Witts criteria

Moderate UC

A

4-6bm

Between mild and severe PR bleed

No pyrexia

No HR >90

No anaemia

ESR <30

3067
Q

Truelove and Witts criteria

Severe UC

A

?6 bm

Large PR bleed

Pyrexic >37.8

HR >90bpm

Hb <10.5

ESR >30

3068
Q

Mx of acute severe UC

A

Resus: admit, IV hydration, NBM

IV hydrocortisone 100mg QDS + PR (can consider adding IV ciclosporin if not improvement)

Transfuse if required

Thromboprophlyaxis

Monitoring

Abx not routinely recommended may be used in megacolon, peforation, uncertain Dx

3069
Q

Monitoring in acute severe UC

A

Bloods: FBC, ESR, CRP, U+E

Vitals and stool chart

Twice daily examination

+/- AXR

3070
Q

Acute complications of acute severe UC

A

Perforation

Bleeding

Toxic megacolon

VTE

3071
Q

Mx of acute severe UC

If improvement

A

Switch to oral prednisolone and a 5-ASA (e.g. mesalazine)

Taper pred after full remission

3072
Q

Mx of acute severe UC, no improvement

A

Rescue therapy

On day 3: stool freq >8, or CRP >45

Predicts 85% chance of needing a colectomy during admission

Medical: ciclosporin, infliximab or visilzumab (anti T cell)

Surgical

3073
Q

Oral therapy to induce remission in mild/mod UC

A

1st line: 5-asas e.g. mesalazine

2nd line: oral pred

3074
Q

Topical therapies to induce remission in mild/moderate UC

A

Mainly used for left-sided disease

Proctitis: suppositories

More proximal disese: enemas or foams

5 ASA +/- steroids (pred or budenoside)

3075
Q

Additional therpapy for mild-moderate UC disease

A

Can add pred to 5-ASA

Oral tacrolimus can be added

Steroid sparing agents include: azathioproine or mercaptopurine

Infliximab: steroid-dependant patients

3076
Q

Maintaining remission in UC

A

1st line: 5-ASA PO: sulfasalzine or mesalazine

Topical can be used in proctitis

2nd line: Azathioprine or mercaptopurine

Relapsed on ASA or are steroid dependent. Give 6-mercaptopurine if azathioprine intolerant

3rd line:

Infliximab/ adalimumab

3077
Q

Indications for emergency surgery in UC

A

Toxic megacolon

Perforation

Massive haemorrhage

Failure to respond to medical Rx

3078
Q

Surgery in UC

A

20% require surgery at some stage

30% with colitis require surgery within 5y

3079
Q

Emergency surgical procedures in UC

A

Total/subtotal colectomy with end ileostomy +/- mucus fistula

Followed after 3m by either completion proctectomy and ileal pouch anal anastomosis or end ileostomy. Or ileorectal anastomosis

Panproctocolectomy and permanent end ileostomy

Acute colitis op mortality: 7% (30% if perforated)

3080
Q

Indications for elective surgery in UC

A

Chronic symptoms despite medical therapy

Carcinoma or high grade dysplasia

3081
Q

Elective surgical procedures in UC

A

Panproctocolectomy with end ileostomy or ileal pouch anal anastomosis

Total colectomy with IRA (ileorectal anastomosis)

3082
Q

Cx of UC sx

A

Abdominal:

SBO

Anastomotic stricture

Pelvic abscess

Stoma:

Retraction

Stenosis

Prolapse

Dermatitis

Pouch:

Pouchitis

Reduced female fertility

Faecal leakage

3083
Q

Rx in pouchitits

A

Metronidazole and cipro

3084
Q

Ix in Crohn’s disease

A

Bloods

Stool

Imaging

Endoscopy

3085
Q

Bloods in Crohns’

A

FBC: anaemia, raised WCC

LFT: hypoalbuminaemia

Raised CRP/ESR

Haematinics: Fe B12 folate

Blood cultures

3086
Q

Stool in CD

A

MCS: exclude campy, shigella, salmonella

CDT

3087
Q

Imaging in Crohn’s

A

AXR: obstruction, sacroileitis

CXR: perforation

MRI: assess pelvic disease and fistula. Assess disease severity

Small bowel follow thourgh or enterocylsis:

Skip lesions

Rose-thorn ulcers

Cobllestoning: ulceration and mural oedema

String sign of Kantor: narrow terminal ileum

Endoscopy:

Ileocolonsocopy and regional biopsy

Wireless capsule endoscopy

Small bowel enteroscopy

3088
Q

Severity markers of acute Crohn’s attack: bloods

A

FBC

LFT

CRP/ESR

3089
Q

Assessment of severe attack in CD

A

Temperature (increased)

HR (increased)

ESR (raised)

CRP (raised)

WCC (raised

Albumin (reduced)

3090
Q

String sign of Kantor

A

Narrow terminal ileus

3091
Q
A

String sign of Kantor

3092
Q

Mx of severe Crohn’s attack

A

ABC, NBM, IV hydration

Hydrocortisone IV + PR if rectal disease

Abx: metronidazole PO or IV

Thromboprophylaxis: LMWH

Dietician review:

Elemental diet

Consider TPN

Monitoring: vitals and stool chart, daily examination

3093
Q

Mx of severe crohn’s attack, improvement

A

Switch to oral pred 40mg/d

3094
Q

Mx of severe crohns, no improvement

A

Rescue therapy

Methotrexate +/- infliximab

Surgical

3095
Q

Inducing remisison in mild/moderate CD

Supportive

A

High fibre diet

Vitamin supplements

3096
Q

Oral therapy in CD, remission induction

A

1st line: prednisolone

2nd line: budenoside or sulfasalazine

Add on treatment

Consider azathioprine or mercaptopurine addition to pred or budenoside

Consider adding methotrexate in those unable to tolerate azathioprine or in whom TPMT activity is deficient

Biologic:

Infliximab/ adalimumab

3097
Q

Perianal disease in CD

A

Occurs in 50%

Ix: MRI and EUA

Rx: oral abx: metronidazole

immunosuppression +/- infliximab

Local surgery +/- seton insertion

3098
Q

Maintaining remission in CD

A

1st line: azathioprine or mercaptopurine

2nd line: methotrexate

3rd line: infliximab/adalimumab

3099
Q

Sx in CD

A

50-80% need >1 operation in their life

Never curative

Should be as conservative as possible

3100
Q

Emergency sx indications in CD

A

Failure to respond to medical Rx

Intestinal obstruction or perforation

Massive haemorrhage

3101
Q

Elective sx indications in CD

A

Abscess or fistula

Perianal disease

Chronic ill health

Carcinoma

3102
Q

Surgical procedures in CD

A

Limited resection e.g. ileocaecal

Stricturoplasty

Defunction distal disease with temproray loop ileostomy

3103
Q

Cx of CD sx

A

Stoma complications

Enterocutaneous fistulae

Anastamotic leak or stricture

3104
Q

Def: short gut

A

<1-2m small bowel

3105
Q

Features of short gut syndrome

A

Steatorrhoea

ADEK and B12 malabsorption

Bile acid depletion-> gall stones

Hyperoxaluria-> renal stones

3106
Q

Rx in short gut syndrome

A

Dietician

Supplements or TPN

Loperamide

3107
Q

General medical mx of bowel obstruction:

Resuscitate

A

ABC

Drip and suck

IV fluids: aggressive as patient may be v dehydrated

NGT: decompress upper GIT, stops vomiting, prevents aspiration

Catheterise: moinotr UO

3108
Q

General medical Rx of bowel obstruction

Therapy

A

Analgesia: may require strong opioid

Abx: cef + met if strangulation or peforation

Gastrograffin study: oral or via NGT

Consider need for parenteral nutrition

3109
Q

General medical Rx of bowel obstruction

Monitoring

A

Regular clinical examination is necessary to ensure that the patient is not deteriorating

Change in distension, pain or tenderness, HR or RR

Repeat imaging and bloods

Non-operative mx succesful in 80% of patients with SBO without peritonitis

Pts with LBO are more likely to need sx

3110
Q

Indications for Sx in bowel obstruction

A

Closed loop obstruction

Obstructing neoplasm

Strangulation/perforation-> sepsis/peritonitis

Failure of conservative mx (up to 72h)

3111
Q

Principles of surgical management of bowel obstruction

A

Aim to treat cause

Typically involves resection of obstructing lesion

Colon has not been cleaned, therefore most surgeons utilise a proximal ostomy post-resection

Patients with substantial comorbidity or unresectable tumours may be offered bypass procedures

Endoscopically placed expanding metal stents offer palliation or a bridge to surgery allowing optimisation

3112
Q

Surgical procedures in bowel obstruction

A

Must consent patient for possible resection +/- stoma

SBO: adhesiolysis

LBO:

Hartman’s

Colectomy and 1o anastomosis and on table lavage

Palliative bypass procedure

Transverse loop colostomy or loop ileostomy

Caecostomy

3113
Q

What proportion of volvulus is sigmoid

A

80%

3114
Q

Pathophysiology of sigmoid volvulus

A

Long mesentry with narrow base predisposes to torsion

Usally due to sigmoid elongation 2o to chronic constipation

Increased risk in neuropsych patients: MS, PD, psychiatric as disease Rx interferes with intestinal motility

Leads to closed loop obstruction

3115
Q

Commoner in males

Often eldelry, constipated, co-morbid patients

Massive distension with tympanic abdomen

A

Sigmoid volvulus

3116
Q
A

Sigmoid volvulus

Coffee bean sign

3117
Q

What can differentiate radiographically between sigmoid volvulus and caecal volvulus

A

Ahaustral wall and the lower end pointing to the pelvis

3118
Q

Mx of sigmoid volvulus

A

Often relieved by sigmoidoscopy and flatus tube insertion

Monitor for signs of bowel ischaemia

Sigmoid colectomy occassionally required:

failed endoscopic decompression

bowel necrosis

Often recurs: elective sigmoidectomy may be needed

3119
Q

Features of caecal volvulus

A

Associated with congenital malformation where caecum is not fixed in the RIF

Only 10% of patients can be detorsed with colonoscopy therefore typically requires sx

Right hemi with 1o ileocolic anastomosis

Caecostomy

3120
Q
A

Caecal volvulus

Despite the varying positions of the distended cecum, the plain radiographic features of a caecal volvulus are characteristic, and the caput caecum can typically be identified. The colonic haustral pattern is generally maintained in contradistinction to a sigmoid volvulus although some effacement may be present if ischemia develops.

3121
Q

Triad in gastro-oesophageal obstruction

A

Vomiting-> retching with regurgitation of saliva

Pain

Failed attempts to pass an NGT

3122
Q

RFs for gastric volvulus

A

Congenital:

Bands

Rolling/paraoesophageal hernia

Py Sten

Acquired

Gastric/oesophageal sx

Adhesions

3123
Q

Ix in gastric volvulus

A

Gastric dilatation

Double fluid level on erect films

3124
Q

Mx of gastric volvulus

A

Endoscopic manipulation

Emergency laparotomy

3125
Q

Adynamic bowel 2o to the absence of normal peristalsis

Usually SBO

Reduced or absent bowel sounds

Mild abdominal pain: not colicky

A

Paralytic ileus

3126
Q

Causes of paralytic ileus

A

Post-op

Peritonitis

Pancreatitis or any localised inflammation

Poisons/Drugs: AntiAChM e.g. TCAs

Pseudo-obstruction

Metabolic: hypokalaemia, hyponatraemia, hypomagnesaemia, uraemia

Mesenteric ischaemia

3127
Q

Prevention of paralytic ileus

A

Reduced bowel handling

Laparoscopic approach

Peritoneal lavage after peritonitis

Unstarched gloces

3128
Q

Mx of paralytic ileus

A

Conservative drip and suck Mx

Correct any underlying causes e.g. drugs, metabolic abnormalities

Consider need for parenteral nutrition

Exclude mechanical cause if protracted

3129
Q

Clinical signs of mechanical obstruction without obstructing lesion

Usually distension only: no colic

A

Colonic pseudo-obstruction/ Ogilvie’s syndrome

3130
Q

Cause of pseudo-obstruction

A

Aetiology unknown

Associated with elderly, cardiorespiratory disorders, pelvic sx e.g. hip arthroplasty, trauma

3131
Q

Ix in colonic pseudo-obstruction

A

Gastrograffin enema to exclude mechanical cause

3132
Q

Mx of Ogilvie’s syndrome

A

Neostigmine: anti-cholinesterase

Colonoscopic decompression: 80% successful

3133
Q

Epidemiology of CRC

A

3rd commonest cancer

2nd commonest cause of cancer death

Age: peak in 60s

Sex: rectal Ca commoner in men

Geo: Western disease

3134
Q

Pathophysiology of CRC

Colonic adenomas

A

Benign precursors to CRC

Characterised by dysplastic epithelium

Malignant potential increases with size, dysplasia and villous component

3135
Q

Classificaiton of colonic adenomas

A

Tubular: small, pedunculated, tubular glands

Villous: large, sessile, covered by villi

Tubulovillous: mixture

3136
Q

Presentation of colonic polyps

A

Typically asymptomatic

Large polyps can bleed-> IDA

Villous adenomas can lead to hypokalaemia and hypoproteinaemia

3137
Q

APC role in CRC

A

Negative regulator of beta-catenin (component of WNT pathway)

APC binds to and promotes beta catenin degradation

mutAPC leads to increased beta catenin-> incrased tx of genes that promote cell proliferation

Proliferation-> mutation of other genes which promote growth and prevent apoptosis

e.g. KRAS (proto-oncogen)

p53 (TSG)

3138
Q

Adenoma-> carcinoma sequence

A

First hit: mutation of one APC copy

Second hit: mutation of second APC copy-> adenoma formation

Additional mutations in adenoma-> malignant transformation

3139
Q

Other aetiological factors in CRC

A

Diet: reduced fibre, increased refined carbs

IBD: CRC in 15% with pancolitis for 20y

Familial: FAP, HNPCC, Peutz-Jehgers

Smoking

Genetics: one 1st degree relaticve= 1/10 risk (no relative 1/50)

NSAIDs/Aspirin: protective

3140
Q

Pathology of CRC

A

95% adenocarcinoma

Others: lymphoma, GIST, carcinoid

3141
Q

Locations of CRC

A

Rectum: 35%

Sigmoid: 25%

Caecum and ascending colong: 20%

Transverse: 10%

Descending: 5%

3142
Q

Most common location of CRC

A

Rectosigmoidal

3143
Q

Spread of CRC

A

Local

Lymphatic

Blood (liver, lungs)

Transcoelomic

3144
Q

Altered bowel habit

PR mass (60%)

Obstruction (25%)
Bleeding/mucus PR

Tenesmus

old man

A

Left CRC

3145
Q

Symptoms of left sided CRC

A

Altered bowel habit

PR mass (60%)

Obstruction (25%)
Bleeding/mucus PR

Tenesmus

old man

3146
Q

Anaemia

Weight loss

Abdominal pain

Old man

A

Right sided CRC

3147
Q

Symptoms of right sided CRC

A

Anaemia

Weight loss

Abdominal pain

Old man

3148
Q

Symptoms common to CRC on both sides of colon

A

Abdominal mass

perforation

haemorrhage

fistula

3149
Q

Palpable mass: per abdomen or PR
Perianal fistulae

Hepatomegaly

Anaemia

Signs of obstruction

A

?CRC

3150
Q

Examination findings in CRC

A

Palpable mass: per abdomen or PR
Perianal fistulae

Hepatomegaly

Anaemia

Signs of obstruction

3151
Q

A 75-year-old man attends the surgical assessment unit prior to an elective Hartmann’s procedure in 7 days due to bowel cancer. He has a past medical history of atrial fibrillation, hypertension and previous cerebrovascular accident. Your registrar asks you to review him prior to his procedure next week. You notice that he is currently taking warfarin and his INR today is 2.6. His remaining blood tests are normal. What is the most appropriate management for his anticoagulation peri-operatively?

Stop his warfarin

Continue his warfarin at the current dose as his INR is within therapeutic range

Stop his warfarin and commence treatment dose low molecular weight heparin

Initiate an unfractionated heparin continuous infusion

Stop his warfarin and commence aspirin

A

Managing anticoagulation peri-operatively is challenging and depends on the reasons for and agent used to anti-coagulate. Each patient should have a venous thromboembolic risk assessment undertaken to consider risk factors for thromboembolism versus bleeding. In this scenario the patient is at high risk of thromboembolic disease (previous CVA, known AF) but also significant bleeding due to major abdominal surgery. Therefore the best option is a shorter acting anticoagulant (e.g. low molecular weight heparin) given at treatment dose whilst withholding warfarin. This would then be withheld the evening before surgery, and mechanical prophylaxis used.

3152
Q

What is the maximum safe volume of lidocaine 1% that may be used during minor surgery on an average sized adult?

10 ml

30 ml

50 ml

20 ml

5 ml

A

20ml

3153
Q

You are the on-call doctor and are called to see a 45 year old lady who is 1 day post-appendicectomy because she has become acutely short of breath. She has just been given a dose of co-amoxiclav for a chest infection infection. She is now breathless with a respiratory rate of 30 breaths per minute and has a heart rate of 125 beats per minute. Her blood pressure is 87/52mmHg and saturations on air are 87%. The nurse is applying oxygen as you make your assessment. What should be the first step in your management?

Give oral prednisolone

Perform an arterial blood gas

Give intramuscular adrenaline

Give intravenous fluids

Give nebulised salbutamol

A

The history here (sudden onset respiratory distress after a penicillin-containing antibiotic) is of anaphylaxis. Giving 0.5mg IM adrenaline 1:1000 as Resuscitation Council UK guidelines for anaphylaxis is the most sensible option.

3154
Q

Features of propofol

A

Rapid onset of anaesthesia

Pain on IV injection

Rapidly metabolised with little accumulation of metabolites

Proven anti emetic properties

Moderate myocardial depression

Widely used especially for maintaining sedation on ITU, total IV anaesthesia and for daycase surgery

3155
Q

Features of sodium thiopentone

A

Extremely rapid onset of action making it the agent of choice for rapid sequence of induction

Marked myocardial depression may occur

Metabolites build up quickly

Unsuitable for maintenance infusion

Little analgesic effects

3156
Q

Features of ketamine

A

May be used for induction of anaesthesia

Has moderate to strong analgesic properties

Produces little myocardial depression making it a suitable agent for anaesthesia in those who are haemodynamically unstable

May induce state of dissociative anaesthesia resulting in nightmares

3157
Q

Features of etomidate

A

Has favorable cardiac safety profile with very little haemodynamic instability

No analgesic properties

Unsuitable for maintaining sedation as prolonged (and even brief) use may result in adrenal suppression

Post operative vomiting is common

3158
Q

A 65-year-old female is admitted for an elective total hip replacement of the right hip. On admission she is given thigh-length anti-embolism stockings to wear before surgery and until she regains mobility. It is hospital policy to also use a low molecular weight heparin for postoperative thromboprophylaxis. According to NICE guidelines, when should this be initiated?

6-12 hours before surgery

1-4 hours after surgery

Immediately after surgery

6-12 hours after surgery

30 minutes - 1 hour after surgery

A

For elective total hip replacement surgery NICE recommend commencing a low molecular weight heparin 6-12 hours after surgery.

3159
Q

What are some procedures in which prophylactic antibiotics are recommended

A
  • Cataract surgery (reduces the risk of endophthalmitis)
  • Appendicectomy
  • Colorectal surgery
  • Caesarean section
  • Abdominal/vaginal hysterectomy
  • Transurethral resection of the prostate
  • Arthroplasty
  • Hip fracture
  • 3rd/4th degree perineal tear repair
3160
Q

What are some procedures where prophylactic antibiotics are not normally recommended

A
  • Tonsillectomy
  • Inguinal hernia repair
  • Laparoscopic removal of ectopic pregnancy
  • Assisted delivery (e.g. forceps)
  • Evacuation of incomplete miscarriage
3161
Q

A 45-year-old lady, with a past medical history of rheumatoid arthritis, is scheduled to have a laparoscopic cholecystectomy. What imaging should be performed pre-operatively?

Anteroposterior and lateral cervical spine radiographs

CT cervical spine

Hand radiographs

Anteroposterior and lateral cervical spine, plus hand, radiographs

Anteroposterior cervical spine radiographs

A

Atlantoaxial subluxation is a rare complication of rheumatoid arthritis, but important as it can lead to cervical cord compression. Anteroposterior and lateral cervical spine radiographs preoperatively screen for this complication, ensuring the patient goes to surgery in a C-spine collar and the neck is not hyperextended on intubation.

Radiographs of the hands are useful in diagnosis, but not necessary pre-operatively. CT scanning of the cervical spine is not necessary for screening pre-operatively, but may be useful is any abnormalities are picked up.

3162
Q

You are the foundation doctor covering surgery. You are asked to review a 77 year old patient (75kg) who is post right hemicolectomy for bowel cancer. The patient is hypotensive (87/40 mmHg), tachycardic (128 bpm) and has a urine output of 25 mls per hour . His only past medical history is hypertension. You conduct a fluid assessment. He appears dry with sunken eye balls and reduced skin turgor. You want to conduct a fluid challenge to assess his response. What is the most appropriate fluid to px?

1 Litre of gelofusin

1 Litres 0.9% normal saline over 8 hours

500 mls 0.9% normal saline STAT

500 mls 0.9% normal saline over 8 hours

250 mls 0.9% normal saline STAT

A

In patients with no clinical signs or documentation of heart failure a 500 ml prescription of normal saline delivered STAT is the recommended fluid challenge. You must remember to reassess the patient to decide whether to prescribe another 500 mls.

250 mlx of 0.9% Normal Saline would be appropriate in patients with evidence of heart failure. This does not put as much strain on their physiology and risk the patient devoting worsening cardiac failure.

3163
Q

Why is Hartmanns/Ringers lactate preferred as postoperative fluids?

A

In the UK the GIFTASUP guidelines (see reference below) were devised to try and provide some consensus guidance as to how intravenous fluids should be administered. A decade ago it was a commonly held belief that little harm would occur as a result of excessive administration of normal saline and many oliguric post operative patients received enormous quantities of IV fluids. As a result they developed hyperchloraemic acidosis. With greater understanding of this potential complication the use of electrolyte balanced solutions (Ringers lactate/ Hartmans) is now favored over normal saline. In addition to this solutions of 5% dextrose and dextrose/saline combinations are now generally not recommended for surgical patients.

3164
Q

An 85-year-old female with multiple comorbidities is scheduled to receive a bowel resection in her local hospital. She attends a pre-operative assessment clinic with the senior anaesthetist to discuss her suitability for surgery and arrange any pre-operative investigations required. In whom do NICE recommend should receive a chest X-ray as part of their pre-operative assessment?

Patients over the age of 65

Patients with a degree of renal impairment

Patients with hypertension

Not routinely recommended

Patients with diabetes

A

Chest x-rays are now not routinely recommended before surgery.

Patients over the age of 65 may need an ECG before major surgery.

Patients with renal disease may need a full blood count and an ECG depending on their ASA grade even before intermediate surgery.

Patients with hypertension do not need any specific investigations pre-operation.

Patients with diabetes may need an ECG before intermediate surgery.

3165
Q

Steroid cover: minor surgery

A

25mg hydrocortisone at induction, then resume normal medication post-operatively

3166
Q

Steroid cover: moderate surgery

A

Usual dose pre-operatively

Then 25mg hydrocortisone at induction followed by 25mg IV every 8 hours for 24h

Continue normal pre-operative dose afterwards

3167
Q

Steroid cover: major surgery

A

Usual dose of steroids preoperatively

50mg IV hydrocortisone at induction

Followed by 50mg IV evey 8h for 48-72h.

Continue infusion until patient has started light eating, then restart normal pre-operative dose

3168
Q

Causes of addisonian crisis

A

sepsis or surgery causing an acute exacerbation of chronic insufficiency (Addison’s, Hypopituitarism)

adrenal haemorrhage eg Waterhouse-Friderichsen syndrome (fulminant meningococcemia)

steroid withdrawal

3169
Q

Mx of Addisonian crisis

A

Hydrocortisone 100mg IM or IV

1L NS over 30-60 mins + dextrose if hypoglycaemic

Cotninue hydrocortisone 6hly until patient is stable

Oral replacement may begin after 24h and be reduced to maintenance over 3-4d

3170
Q

What is the most common ADR in PRC infusion

A

Pyrexia

3171
Q

What is the most common ATR following FFP infusion

A

Urticaria

3172
Q

Features of NG feeding

A

Usually administered via fine bore naso gastric feeding tube

Complications relate to aspiration of feed or misplaced tube

May be safe to use in patients with impaired swallow

Often contra indicated following head injury due to risks associated with tube insertion

3173
Q

Features of naso-jejunal feeding

A

Avoids problems of feed pooling in stomach (and risk of aspiration)

Insertion of feeding tube more technically complicated (easiest if done intra operatively)

Safe to use following oesophagogastric surgery

3174
Q

Features of feeding jejunostomy

A

Surgically sited feeding tube

May be used for long term feeding

Low risk of aspiration and thus safe for long term feeding following upper GI surgery

Main risks are those of tube displacement and peritubal leakage immediately following insertion, which carries a risk of peritonitis

3175
Q

Features of PEG

A

Combined endoscopic and percutaneous tube insertion

May not be technically possible in those patients who cannot undergo successful endoscopy

Risks include aspiration and leakage at the insertion site

3176
Q

Features of TPN

A

The definitive option in those patients in whom enteral feeding is contra indicated

Individualised prescribing and monitoring needed

Should be administered via a central vein as it is strongly phlebitic

Long term use is associated with fatty liver and deranged LFT’s

3177
Q

A 32-year-old male is receiving a blood transfusion after being involved in a road traffic accident. A few minutes after the transfusion he complains of loin pain. His observations show temperature 39 oC, HR 130bpm and blood pressure is 95/40mmHg. What is the best test to confirm his diagnosis?

USS abdomen

Direct Coomb’s test

Blood cultures

Blood film

Sickle cell test

A

The diagnosis is of an acute haemolytic transfusion reaction, normally due to ABO incompatibility. Haemolysis of the transfused cells occurs causing the combination of shock, haemoglobinaemia and loin pain. This may subsequently lead to disseminated intravascular coagulation. A Coomb’s test should confirm haemolysis. Other tests for haemolysis include: unconjugated bilirubin, haptoglobin, serum and urine free haemoglobin.

Note that delayed haemolytic reactions are normally associated with antibodies to the Rh system and occur 5-10 days after transfusio

3178
Q

What are the contraindications to day surgery?

A

Day surgery is suitable theoretically for any procedure in which GA time is <1hr

severe dementia

severe learning disabilities

ASA III and above (unless conditions can be optimised and stabilised pre-operatively)

BMI (body mass index) 32 kg/m^2 and above

diabetes type I (if sliding scale commenced)

infection at the site of surgery

social factors

uncontrolled pain or nausea expected post-operatively

3179
Q

Theme: Anaesthetic agents

A.Halothane

B.Propofol

C.Ketamine

D.Etomidate

E.Sodium thiopentone

F.Flumazenil

G.Naloxone

H.Sevoflurane

Please select the drug which most closely matches the description given. Each option may be used once, more than once or not at all.

An agent which reverses the action of midazolam

An agent which is associated with hepatotoxicity
An anaesthetic agent which has anti emetic properties

A

Flumazenil

Flumazenil antagonises the effects of benzodiazepines by competition at GABA binding sites. Since may benzodiazepines have longer half lives than flumazenil patients still require close monitoring after receiving the drug.

An agent which is associated with hepatotoxicity

Halothane

Halothane is hepatotoxic. Despite this it remains in mainstream use. It should be avoided in patients with hepatic dysfunction, and scavengers should be used in theatres as accumulation of the drug may be injurious to theatre staff.

The correct answer is Propofol

Propofol is rapidly metabolised and has mild/ moderate anti emetic properties. It is the agent of choice in most day case operations for this reason.

3180
Q

High T with RIF pain

A

More typical of conditiosn such as mesenteric adenitis rather than appendicitis

3181
Q

SBO features on AXR

A

Maximum normal diameter= 35mm

Valvulae conniventes extend all the way across

3182
Q

LBO features on AXR

A

Maximum normal diamter= 55mm

Haustra extend about a third of the way acrosS

3183
Q

Sudden onset loin to groin pain

Colicky in nature

Associated with macro or microscopic haematuria

A

Renal colic

3184
Q

Dx of renal colic

A

CT KUB is most sensitive and specific diagnostic test

3185
Q

Mx of renal colic

A

Stones <5mm will pass within 4w of symptom onset

More intensive and urgent treatment required in presence of ureteric obstruciton, renal developmental abnormality or previous renal transplant.

Ureteric obstruction due to stones + infection is a surgical emergency.

3186
Q

Stone burden of less than 2cm in aggregate

A

Lithotripsy

3187
Q

Stone burden of less than 2cm in pregnant females

A

Ureteroscopy

3188
Q

Complex renal calculi and staghorn calculi

A

Percutaneous nephrolithotomy

3189
Q

Ureteric calculi less than 5mm

A

Manage expectantly

3190
Q

Features of shockwave lithotripsy

A

A shock wave is generated external to the patient, internally cavitation bubbles and mechanical stress lead to stone fragmentation. The passage of shock waves can result in the development of solid organ injury. Fragmentation of larger stones may result in the development of ureteric obstruction. The procedure is uncomfortable for patients and analgesia is required during the procedure and afterwards.

3191
Q

Features of ureteroscopy

A

A ureteroscope is passed retrograde through the ureter and into the renal pelvis. It is indicated in individuals (e.g. pregnant females) where lithotripsy is contraindicated and in complex stone disease. In most cases a stent is left in situ for 4 weeks after the procedure.

3192
Q

Features of percutaneous nephrolithotomy

A

In this procedure access is gained to the renal collecting system. Once access is achieved, intra corporeal lithotripsy or stone fragmentation is performed and stone fragments removed.

3193
Q

A 72-year-old man presents to the Emergency Department. Whilst walking back from a friends house he slipped on some ice and fell backwards, landing on his right arm and banging his head on the kerb in the process. His past medical history includes atrial fibrillation for which he takes bisoprolol and warfarin. A routine INR taken four days ago was 2.2. There are no signs of any external injury to his right arm or scalp. What is the most appropriate course of action with relation to his head injury?

Arrange a CT head scan to be performed within 8 hours

Discharge with standard head injury advice

Admit for 24 hours of observation

Admit for 8 hours of observation

Discharge with standard head injury advice + advise he stops warfarin for 5 days

A

For patients (adults and children) who have sustained a head injury with no other indications for a CT head scan and who are having warfarin treatment, perform a CT head scan within 8 hours of the injury. A provisional written radiology report should be made available within 1 hour of the scan being performed.

3194
Q

Def: anal fissure

A

Anal fissures are longitudinal or elliptical tears of the squamous lining of the distal anal canal. If present for less than 6 weeks they are defined as acute, and chronic if present for more than 6 weeks. Around 90% of anal fissures occur on the posterior midline

3195
Q

A 24-year-old man presents due to severe pain when defecating for the past 2 weeks. He has occasionally noted some blood on the toilet paper when wiping himself. On examination a tear is seen on the posterior midline of the anal verge. Which one of the following should not be recommended as a treatment option?

Bulk-forming laxatives

Application of lubricant prior to defecation

Topical steroids

Dietary advice

Paracetamol

A

Topical steroids have been shown in studies to be of little benefit in treating anal fissures

3196
Q

Abdominal swelling:

History of malignancy/previous operations
Vomiting
Not opened bowels recently
‘Tinkling’ bowel sounds

A

Intestinal obstruction

3197
Q

Abdominal swelling

History of alcohol excess, cardiac failure

A

Ascites

3198
Q

Abdominal swelling

History of prostate problems

Dullness to percussion around suprapubic area

A

Urinary retention

3199
Q

Abdominal swelling:

Older female

Pelvic pain

Urinary symptoms

Raised CA125

Early satiety/bloating

A

Ovarian cancer

3200
Q

A 22-year-old man has undergone an inguinal hernia repair. Seven days later he presents with an erythematous and tender wound that is discharging a purulent material. What is the most likely cause?

Infection with Staphylococcus aureus

Discharging haematoma

Infection with Pseudomonas

Infection with Streptococcus pyogenes

Infection with Bacteroides

A

In this setting Staphylococcus aureus Infection is the most likely cause. In the UK between 2010 and 2011 the commonest cause of wound infection was enterobacter infections (usually following cardiac or colonic surgery). 23% of infections were due to Staph aureus, which fits the scenario above. Infection with the other organisms including strep pyogenes was much rarer.

3201
Q

You are the foundation doctor on call for the surgical ward. A 65 year old male is six hours post thyroidectomy. You are bleeped and asked to review this gentlemen because of worsening stridor. As you arrive at the bedside the nurse hands you the patients notes. When reviewing the notes, it is apparent the operation was uneventful. The surgeons notes describe adequate intra-operative haemostasis and closure using sutures. What is the most appropriate management for this patient?

Intramuscular 1:10,000 Adrenaline

Call anaesthetist and request immediate intubation

Urgent removal of sutures and call for senior help

Reassure patient

Nebulised hypertonic saline

A

Post operative stridor in patients who have undergone neck surgery is a life threatening emergency. Using an ABCDE approach, this patient has potentially compromised airway and breathing. Each patient undergoing head and neck surgery is returned to the ward with a suture blade. In the event of a post operative bleed. If a bleed occurs, the pressure behind the suture line increases and the trachea becomes compressed resulting in stridor. Therefore 3 is the answer becuase immediate removal of the pressure will relieve the stridor. Senior assistance will be required as this patient will require further surgery for haemostasis.

Intubation will improve this patients airway and breathing, however taken in the clinical context removal of the compression is the correct answer.

For those individuals who chose 1, this is not anaphylaxis. There are no predisposing factors in the question. Moreover to treat anaphylaxis you prescribe 1:1000 intramuscular adrenaline not 1:10,000.

Nebulised hypertonic saline will not be of any use in this patient.

3202
Q

Head injury:

Immediate CT head if

A

GCS < 13 on admission

GCS < 15 2h after admission

Suspected open or depressed skull fracture

Suspected skull base fracture (panda eyes, Battle’s sign, CSF from nose/ear, bleeding ear)

Focal neurology

Vomiting > 1 episode

Post traumatic seizure

Coagulopathy

3203
Q

Contact neurosurgeon following head injury if

A

Persistent GCS < 8 or = 8

Unexplained confusion > 4h

Reduced GCS after admission

Progressive neurological signs

Incomplete recovery post seizure

Penetrating injury

Cerebrospinal leak

3204
Q

If C-spine injury suspected?

When is CT c-spine preferred?

A

If a c-spine injury is suspected a 3 view c-spine x-ray is indicated. CT c-spine is preferred if:

  • Intubated
  • GCS <13
  • Normal x-ray but continued concerns regarding c-spine injury
3205
Q

A 76-year-old woman with a history of atrial fibrillation presents with abdominal pain and bloody diarrhoea. On examination her temperature is 37.8ºC, pulse 102 / min and respiratory rate 30 / min. Her abdomen is tender with generalised guarding. Blood tests reveal the following:

Hb10.9 g/dl

MCV76 fl

Plt348 * 109/l

WBC23.4 * 109/l

Na+141 mmol/l

K+5.0 mmol/l

Bicarbonate14 mmol/l

Urea8.0 mmol/l

Creatinine118 µmol/l

What is the most likely diagnosis?

Diverticulitis

Mesenteric ischaemia

Campylobacter infection

Ruptured abdominal aortic aneurysm

Ulcerative colitis

A

The low bicarbonate points to a metabolic acidosis - highly suggestive of mesenteric ischaemia.

3206
Q

A 78-year-old gentleman presents to the emergency department with a 3 hour history of lower back pain. It is achey in nature and a 6/10 on the pain scale. On examination he has some tenderness on palpation. His blood pressure is 100/70 mmHg despite 500ml fluid bolus and his heart rate is 110/min. What’s the most likely diagnosis?

Kidney stone

Spinal cancer

Ischaemic colitis

Osteoporotic fracture

Abdominal Aortic Aneurysm (AAA)

A

Any elderly gentleman that presents with back pain needs a ultrasound scan to exclude a AAA before any other diagnosis is considered, especially if they have haemodynamic instability.

3207
Q

A 48 year old man presents to the Emergency Department with severe abdominal pain which has developed over the previous four hours. He opened his bowels earlier today and has had no problems passing water. He has been unwell recently with coryzal symptoms and a cough. He has been seen by his GP and not commenced on any antibiotics as it is a likely viral upper respiratory tract infection.

His past medical history includes idiopathic atrial fibrillation for which is is currently taking warfarin. He has never had any abdominal surgery.

His observations are: heart rate 110/min, respiratory rate 14/min, blood pressure 98/75 mmHg, temperature 37.4º, Sats 99% on air.

On examination he is in obvious pain. Inspection reveals a discoloured purple non-demarcated area to the left of the umbilicus. Palpation of the abdomen demonstrates tenderness localised to a firm non-pulsatile mass just to the left of the midline at the level of the umbilicus. There is no guarding or rigidity. Bowel sounds are present.

What is the most likely cause of this mans symptoms?

Aortic aneurysm

Strangulated hernia

Abdominal wall haematoma

Constipation

Mesenteric ischaemia

A

Abdominal wall (rectus sheath) haematoma can occur following trauma, either directly to the abdominal wall or iatrogenic trauma from surgery, or can be spontaneous following excessive straining of the rectus muscle. This can occur during prolonged valsalva manoeuvres experienced with strenuous excesses or, in this case, coughing.

3208
Q

Difficulty swallowing, dysphagia to both liquids and solids and sometimes chest pain

Usually caused by failure of distal oesphageal inhibitory neurones

Diagnosis is by pH and manometry studies together with contrast swallow and endoscopy

Treatment is with either botulinum toxin, pneumatic dilatation or cardiomyotomy

A

Achalasia

3209
Q

Spectrum of oesophageal motility disorders

Caused by uncoordinated contractions of oesphageal muscles

May show “nutcracker oesophagus” on barium swallow

Symptoms include dysphagia, retrosternal discomfort and dyspepsia

A

Diffuse oseophagea spasm

3210
Q

Commonest approach to the abdomen

Structures divided: linea alba, transversalis fascia, extraperitoneal fat, peritoneum (avoid falciform ligament above the umbilicus)

Bladder can be accessed via an extraperitoneal approach through the space of Retzius

A

Midline incision

3211
Q

Parallel to the midline (about 3-4cm)

Structures divided/retracted: anterior rectus sheath, rectus (retracted), posterior rectus sheath, transversalis fascia, extraperitoneal fat, peritoneum

Incision is closed in layers

A

Paramedian incision

3212
Q

Similar location to paramedian but rectus displaced medially (and thus denervated)

Now seldom used

A

Battle insicsion

3213
Q

Incision under right subcostal margin e.g. Cholecystectomy (open)

A

Kocher’s

3214
Q

Incision in right iliac fossa e.g. Appendicectomy

A

Lanz

3215
Q

Oblique incision centered over McBurneys point- usually appendicectomy (less cosmetically acceptable than Lanz

A

Gridiron incision

3216
Q

Rooftop incision

A

Gable

3217
Q

Transverse supra pubic, primarily used to access pelvic organs

A

Pfannenstiel’s

3218
Q

Groin incision e.g. Emergency repair strangulated femoral hernia

A

McEvedy’s

3219
Q

Extraperitoneal approach to left or right lower quadrants. Gives excellent access to iliac vessels and is the approach of choice for first time renal transplantation.

A

Rutherford Morrison

3220
Q
A
3221
Q

Disseminated infection with Chlamydia.
Usually seen in females.
Consists of evidence of pelvic inflammatory disease together with peri-hepatic inflammation and subsequent adhesion formation.

A

Fitz-Hugh Curtis syndrome

3222
Q

Boas sign

A

Boas’ or Boas’s sign is hyperaesthesia (increased or altered sensitivity) below the right scapula can be a symptom in acute cholecystitis (inflammation of the gallbladder).[1] It is one of many signs a medical provider may look for during an abdominal examination.[2]

Originally this sign referred to point tenderness in the region to the right of the 10th to 12th thoracic vertebrae.[3] It is less than 7% sensitive.[4][5]

3223
Q

Murphy’s sign

A

Classically, Murphy’s sign is tested for during an abdominal examination; it is performed by asking the patient to breathe out and then gently placing the hand below the costal margin on the right side at the mid-clavicular line (the approximate location of the gallbladder). The patient is then instructed to inspire (breathe in). Normally, during inspiration, the abdominal contents are pushed downward as the diaphragm moves down (and lungs expand). If the patient stops breathing in (as the gallbladder is tender and, in moving downward, comes in contact with the examiner’s fingers) and winces with a ‘catch’ in breath, the test is considered positive. In order for the test to be considered positive, the same maneuver must not elicit pain when performed on the left side.

3224
Q

A.Multiple endocrine neoplasia type I

B.Multiple endocrine neoplasia type II

C.Gardner’s syndrome

D.Lynch Syndrome

E.Kartagener’s syndrome

F.Von Recklinghausen’s disease

Please select the most likely condition for the disease process described. Each option may be used once, more than once or not at all

A 5-year-old boy presents with recurrent episodes of sinusitis. The casualty staff are surprised to find his liver lying in the left upper quadrant of the abdomen
A 22-year-old man presents with carcinoma of the caecum. His brother died from colorectal cancer aged 25 and his father died from the disease aged 30.
A tall 32-year-old lady presents with a diffuse neck swelling a carcinoma of the thyroid medullary type is diagnosed.

A

This is a case of Kartagener’s syndrome. The primary problem is of immotile cilia syndrome. When associated with situs inversus Kartagener’s syndrome is diagnosed.

This is a case of Lynch syndrome HNPCC. It is transmitted in an autosomal dominant fashion.

This is a case MEN type IIb. It is associated with phaeochromocytomas and is transmitted in an autosomal dominant pattern if inherited. All MEN II tend to have medullary carcinoma of the thyroid as a presenting feature

3225
Q

A 25-year-old female presents to surgery with a 2 week history of painless rectal bleeding. Inspection of perineum and rectal examination is unremarkable. Proctoscopy reveals haemorrhoidal cushions at the left lateral and right anterior position. What is the most important component of management?

Sitz baths

Topical nitrate

Fibre supplementation

Improving anal hygiene

Application of lubricant prior to defecation

A

Fibre supplementation has been shown to be as effective as injection sclerotherapy in some studies

3226
Q

A 60-year-old retired man who lives alone and was previously a publican for 40 years is due to have a low anterior resection for a rectal adenocarcinoma.

His preoperative blood tests including full blood count, clotting screen and liver function tests were markedly normal other than a moderately raised gamma-glutamyl transpeptidase and mean cell volume.

During the operation the surgeon encounters significant blood loss when going through the abdominal wall. The bleeding has no obvious source, making ligation and coagulation very difficult. The operation is abandoned before entering the abdominal cavity. Blood tests taken during the operation show a normal activated partial thromboplastin time (APTT), prothrombin time (PT) and international normalised ratio (INR). What is the most likely cause of the excessive bleeding from the options below?

Bud chiari syndrome

Thrombocytopaenia

Venous portal hypertension

Disseminated intravascular coagulation

Haemophilia A

A

The patients social history suggests they have worked in an environment serving alcohol for many years, though this does not mean they were a heavy drinker the raised gamma-glutamyl transpeptidase alludes to this.

As a result of heavy drinking the patient has a degree of liver cirrhosis and therefore portal hypertension. This has caused raised venous pressure in the veins of the abdomen. This explains why a source of the bleeding is difficult to identify during the operation, as is typical of extensive venous bleeds. Arterial bleeds are pulsatile in nature and tend to clearly spout from their source.

Thrombocytopenia - Would be shown by a low platelet count in the pre-operative full blood count.

Disseminated intravascular coagulation - This would cause a prolonged PT and APTT

Haemophilia A - This would cause a prolonged APTT

Bud Chiari Syndrome - This could also be a cause of portal hypertension, though it is far rarer than cirrhosis.

Von Willebrand Disease would be important to rule out in a patient such as this. As it is relatively common, and PT and APTT can both be norm

3227
Q

Dx of mesenteric infarction

A

Arterial pH and lactate
Arterial phase CT scanning is the most sensitive test

3228
Q

Mx of mesenteric infarction

A

Immediate laparotomy and resection of affected segments, in acute embolic events SMA embolectomy may be needed.

3229
Q

A 62-year-old man with no significant past medical history presents with a right sided groin lump which he noticed whilst having a shower. It has been present for 2 weeks and disappears when he lies down. It never causes him any discomfort and there are no other gastrointestinal symptoms of note. Examination reveals an small reducible swelling in the right groin consistent with an inguinal hernia. What is the most appropriate management?

Refer for fitting of a truss

Refer to vascular surgeon

Routine referral for surgical repair

Advise no action as will probably improve with time

Fast-track referral to colorectal service

A

This patient has an asymptomatic inguinal hernia. Studies looking at conservative management tend to find that many patients become symptomatic and eventually have surgery anyway. As this patient is medically fit most clinicians would refer for surgical repair.

3230
Q

A 62-year-old man has fallen over climbing some rocks on a beach and struck his head. On initial presentation, he has a GCS of 15, no signs of basal skull fracture, no neurological symptoms and no vomiting. He has a history of atrial fibrillation and diabetes, for which he takes verapamil, warfarin and metformin. Which of the following is the most appropriate next step?

CT head scan within 1 hour

CT head scan within 8 hours

MRI head scan within 1 hour

MRI head scan within 8 hours

Discharge the patient and tell him to return if further symptoms develop

A

CT imaging of the head is currently the primary investigation of choice for detecting clinical important brain injuries in the acute setting.

For safety, logistic and resource reasons MRI is not the investigation of choice in this setting.

NICE recommends that patients who are taking warfarin who suffer a head injury should receive a CT head scan within 8 hours, even without further indications. If the patient has any indications for a CT head scan within 1 hour, this would obviously take precedence.

3231
Q

A 71-year-old man has fallen off his bike and hit his head. According to a friend at the scene, he was unconscious for less than a minute after falling. At the time he was cycling slowly on a path beside a canal. On initial assessment, he has some bruising on his upper and lower limbs, has a Glasgow coma score (GCS) of 15, and no neurological deficit. He has not vomited or had a seizure since the accident, which he was able to describe to you. He is taking antihypertensives but otherwise has no notable medical history. Which of the following would be the most appropriate next step?

Perform a CT head scan within 1 hour

Perform a CT head scan within 8 hours

Perform a MRI head scan within 1 hour

Perform a MRI head scan within 8 hours

Discharge the patient and tell him to return if further symptoms develop

A

NICE recommends that patients who suffer a head injury with loss of consciousness or amnesia, and who are aged over 65 years old, should have a CT head scan within 8 hours. If the patient had an indication for a CT head scan within 1 hour, obviously this would take precedence. The indications for CT head scan within 1 hour and within 8 hours, are found below.

3232
Q

A 53-year-old man presents with an ulcerated mass at the anal verge. A biopsy is taken and the histology demonstrates as squamous cell carcinoma. Infection with which of the viruses below is most likely to have contributed to the development of the condition?

Human papillomavirus 7

Human immunodeficiency virus 1

Human immunodeficiency virus 2

Human papillomavirus 16

Human T-lymphotropic virus 1

A

Infection with human papilloma virus 16 is a risk factor for the development of intra epithelial dysplasia of the anal skin with subsequent increased risk of invasive malignancy.

3233
Q

A.E-coli and bacteroides

B.Staphylococcus aureus

C.Streptococcus viridians

D.Staphylococcus epidermidis

E.Klebsiella

F.Clostridium tetani

G.Clostridium difficile

H.None of the above

A 32-year-old women undergoes mastectomy and latissimus dorsi flap reconstruction for breast cancer, to provide optimal cosmesis a McGhan implant is placed under the myocutaneous flap. Three weeks post operatively the patient continues to suffer from recurrent wound infections that have proved resistant to multiple courses of antibiotics.

A 68-year-old man with diabetes presents with an area of necrosis of the perineum at the base of the scrotum, there is some surrounding erythema. He is systemically unwell and hypotensive.

A 68-year-old women with previous rheumatic fever is admitted with pyrexia of unknown origin. Her blood cultures are unhelpful but transoesophageal echocardiography reveals vegetations on the mitral valve.

A

The correct answer is Staphylococcus epidermidis

This tends to colonise plastic devices and forms a biofilm which allows colonisation with other bacterial agents. It is notoriously difficult to eradicate once established and the usual treatment is removal of the device.

-coli and bacteroides

This is likely to be Fournier’s Gangrene. A number of agents are implicated. E-coli and bacteroides are the most commonly isolated organisms. The key point is that both aerobic and anaerobic organisms must be present and only A has this option.

Streptococcus viridians

This is the most common organism affecting previously abnormal heart valves

3234
Q

During a surgical ward round you are asked to request a nurse cleans a patient’s surgical wound when the dressing is changed. The patient is 36 hours post surgery. According to NICE guidelines, what is the most appropriate substance to use to clean the wound?

Iodine solution

Sterile saline

Patient may shower

Alcohol swabs

Sterile water

A

NICE recommend the following regarding postoperative wound cleansing:

Use sterile saline for wound cleansing up to 48 hours after surgery.

Advise patients that they may shower safely 48 hours after surgery.

Use tap water for wound cleansing after 48 hours if the surgical wound has separated or has been surgically opened to drain pus.

3235
Q

A.Anastamotic leak

B.Chyle leak

C.Air leak

D.Biliary leak

E.Deep vein thrombosis

F.Portal vein thrombosis

G.Biliary obstruction

Please select the most likely complication for the scenario given. Each option may be used once, more than once or not at all.

A 67-year-old female undergoes an oesophagogastrectomy for carcinoma of the distal oesophagus. She complains of chest pain. The following day there is brisk bubbling into the chest drain when suction is applied.

A 20-year-old man has a protracted stay on ITU following a difficult appendicectomy for perforated appendicitis with pelvic and sub phrenic abscesses. He has now deteriorated further and developed deranged liver function tests.

A 63-year-old man undergoes an Ivor - Lewis oesophagogastrectomy for carcinoma of the distal oesophagus. The following day a pale opalescent liquid is noted to be draining from the right chest drain.

A

Air leak

Damage to the lung substance may produce an air leak. Air leaks will manifest themselves as a persistent pneumothorax that fails to settle despite chest drainage. When suction is applied to the chest drainage system, active and persistent bubbling may be seen. Although an anastomotic leak may produce a small pneumothorax, a large volume air leak is more indicative of lung injury.

Portal vein thrombosis

Such marked intra-abdominal sepsis may well produce coagulopathy and the risk of portal vein thrombosis.

Chyle leak

Damage to the lymphatic duct may occur during this procedure and some surgeons administer a lipid rich material immediately prior to surgery to facilitate its identification in the event of iatrogenic damage.

3236
Q

Difference between mesenteric ischaemia and ischaemic colitits

A

you can also see the Hx which risk factors do the patient present with. CAD risks are more likely for mes ischemia. This particular pt has afib, a strong risk for emboli.

dec in blood flow (eg decomp CHF, prolonged shock, dehydration) accompanied w abd pain and bloody diarrhea, will point to ischemic colitis.

3237
Q

Theme: Tumour markers

A.Invasive ductal carcinoma of the breast

B.Prostate cancer

C.Gastric cancer

D.Ovarian cancer

E.Colorectal cancer

F.Pancreatic adenocarcinoma

G.Seminoma testicular cancer

H.Non-seminomatous testicular cancer

I.Hepatocellular carcinoma

For each tumour marker please select the most likely underlying malignancy. Each option may be used once, more than once or not at all.

Raised beta-human chorionic gonadotropin with a raised alpha-feto protein level

A

The correct answer is Non-seminomatous testicular cancer

A raised alpha-feto protein level excludes a seminoma

3238
Q

A 65-year-old woman presents to the Emergency Department with lower abdominal pain and vomiting. On further questioning she has not opened her bowels for the past 2 days and feels bloated.

A CT of her abdomen is requested:

© Image used on license from Radiopaedia

What is the most likely diagnosis?

Diverticulitis

Small bowel obstruction secondary to a strangulated inguinal hernia

Linitis plastica of the stomach

Large bowel obstruction secondary to volvulus

Ovarian cancer with associated ovarian torsion

A

The CT shows multiple dilated loops of small bowel. CT is more sensitive than radiographs and will also demonstrate the cause in around 80% of cases. There are variable criteria for maximal small bowel obstruction, but 3.5 cm is a conservative estimate of dilated bowel.

Note the protrusion of small bowel through the inguinal ring

3239
Q

Medical indications for circumcision

A

phimosis

recurrent balanitis

balanitis xerotica obliterans

paraphimosis

3240
Q

How can the symptoms of BPH be categorised

A

Voiding symptoms:

weak or intermittenet flow, straining, hesitancy, terminal dribbling, incomplete emptying

Storage symptoms:

urgency, frequency, urgency incontinence and nocturia

Post micturition:

Dribbling

Cx:

UTI, retention, obstructive uropathy

3241
Q

Mx of BPH

A

Watchful waiting

Rx:

Alpha 1 antagonists

5 alpha reductase inhibitor

Medical therapy of prostatic symptoms trial: (MTOPS) advocated combination therapy

Sx:

TURP

3242
Q

E.g. of alpha-1 antagonists

A

Tamsulosin

Afluzosin

3243
Q

MOA alpha 1 antagonists

A

Decrease smooth muscle tone (prostate and bladder)

3244
Q

Efficacy of alpha1 antagonists in BPH

A

Considered first line

Improve symptoms in 40% of men

3245
Q

Adverse effects of alpha1` anatgonists

A

Dizziness

Postural hypotension

Dry mouth

Depression

3246
Q

E.g. 5 alpha-reductase inhibtors

A

Block the conversion of testosterone to DHT (testosterone known to induce BPH)

3247
Q

Efficacy of 5-alpha RIs

A

unlike alpha-1 antagonists causes a reduction in prostate volume and hence may slow disease progression. This however takes time and symptoms may not improve for 6 months. They may also decrease PSA concentrations by up to 50%

3248
Q

Adverse effects of 5aRIs

A

ED

Reduced libido

Ejaculation problems

Gynaecomastia

3249
Q

Gallbladder location

A

Tip of the 9th costal cartilage

3250
Q

Location of arcuate line

A

Half way between the umbilicus and pubic crest

3251
Q

Transpyloric plane of Addison, location

A

L1 vertebra

9th Costal cartilage

3252
Q

What structures are found on the transpyloric plane of Addison?

A

Pylorus

Fundus of the gallbladder

Origin of SMA

Duodojejunal junction

Neck of pancreas

Hila of kidneys

Formation of portal vein

3253
Q

Location of the supracristal plane

A

L4/L5

Also location of bifurcation of the aorta

3254
Q

Causes of hepatomegaly

Smooth without jaundice

A

CCF

Cirrhosis

Lymphoreticular disease

Budd Chiari

Amyloid

3255
Q

Causes of hepatomegaly

Smooth with jaundice

A

Hepatitis

Biliary tract obstruction

Cholangitis

Portal pyaemia

3256
Q

Causes of hepatomegaly

Irregular without jaundice

A

2o mets

Macronodular cirrhosis

Polycystic disease

Primary HCC

3257
Q

Causes of hepatomegaly

Irregular with jaundice

A

Extensive 2o mets

Cirrhosis

Localised swellings:

Reidel’s lobe

Hydatid cyst

Liver abscess

3258
Q

Why is it the spleen?

A

LUQ to RIF mass

Notch

Moves with respiration

Dull to precussion

Cannot get above it

Not ballotable

3259
Q

Massive kidneys in PACES?

A

Polycystic kidneys

3260
Q

Stoma examination

A

Look- location, parastomal hernia

Stoma: stoma itself, colour, surface ect. Bag + contents and surroundings

Abdomen

Ask to inspect perineum

Palpate around and inside the stoma: need gloves and KY jelly

Site

Calibre

Number of lumens

Funcitoning

Healthy

3261
Q

How to tell the difference between periumbilical and umbilical hernia

A

Paraumbilical are crescenteric

Umbilical are hemispherical

3262
Q

Key clinical defect between hernia and diverification of the recti

A

There will be no palpable defect in diverification whereas with hernia, can palpate defect

3263
Q

What is the rule of PR bleeding

A

Bright fresh rectal bleeding- visible with proctoscope: haemorrhoids

Dark mixed in bleeding: upper GI/right colon

3264
Q

If pathology stays in what type of stoma?

Hartmann’s=

A

= defunctioning

Pathology taken out

3265
Q

Low colorectal anastomosis type of stoma

A

Loop ileostomy

3266
Q

What is the benefit of loop ileostomy vs loop colostomy

A

Loop ileostomy:

  • High output stoma

+ easy to make, easy to reverse

Loop colostomy:

  • Tend to prolapse
  • Harder to reverse

+ Not high output

3267
Q

Difference between feeding and NG tube

A

Feeding tube: into jejunum, can be guided under radiology

Used in high output stomas

Longer term than NGT

3268
Q

Cx of NGT

A

LT use can lead to oesophageal stenosis

3269
Q

Graded TEDs mecahnisms

A

Decreasing pressure up to knee

Compress blood upwards

Used in combination with calf exercises

3270
Q

Female with urinary retention?

A

?Pelvic malignancy

3271
Q

Male or female cause of postrenal obstruction

A

Megarectum i.e. constipation

3272
Q

Why are tracheostomies used over ET tubes in ITU

A

Larynx is barrier to weaning from ventilation.

Easier to wean from LT ITU ventilation or patients with elective maxfax sx

3273
Q

What can central line be used to monitor in addition to presssure

A

Central venous O2: if >75% patient adequately filled

3274
Q

Suction drains shouldn’t be used in?

A

Abdomen due to risk of fistula

Can be used following breast sx etc

3275
Q

Midline laparotomy and bilateral vertical groin incisisions in vascular

A

Aortobifemoral bypass graft

3276
Q

Oblique LIF scar (Rutherford morrison scar) in vascular exam

A

Access to retroperitoneal iliac artery

3277
Q

Midline laparotomy scar with externsion into 5th intercostal space=

A

Thoracolaparotomy

Used for approaches to aort above the renal artery

3278
Q

Oblique incicision anteriror to sternocleidomastoid

A

Access to carotid for carotid endarterectomy

3279
Q

Bilateral groin incisions with rigid tube across pubic symphysis

A

Femorofemoral crossover graft

3280
Q

Axillo bifemoral graft

A

Access axillary artery with cross over to both femorals

3281
Q

Aortobifemoral bypass graft

A

Blockage in aorta or iliacs

3282
Q

Scar on the inside of the leg running from 2cm lateral and inferior to pubic tubercle, one handsbreath below the knee and 2 centres anterior to medial malleolus

A

= long saphenous vein harvest scar.

3283
Q

Definition of critical icshaemia (3 things)

A

Greater than two weeks duration (otherwise acute)

Rest pain or tissue loss

Ankle pressure of less than 40mmHg or <0.6ABPI

3284
Q

Relation between aneurysms

A

1 popliteal aneurysm 50% chance of having another

1 popliteal aneurysm 10% chance of infrarenal aneurysm

3285
Q

How did the 5.5cm figure come around for AAA threhold

A

Small aneurysm trial showed that sureillance is safe for patients under 5.5cm

3286
Q

Thoracic outlet syndrome

A

Can be due to an additional cervical rib, can be due to overgrowth of scalene, can be due to prolonged abudction e.g. violinists, cellists

Can be classified as:

Venous: upper limb DVT and long term swelling: heparinise

Arterial: raynauds, claudication, embolisation

Neurological: pain, radiculopathy

3287
Q

How to remember stages of shock

Game of tennis

A

0-15

15-30

30-40

Game

3288
Q

Best arterial bypass prosthesis of choice in a young patient for fem-pop bypass

A

Autlogous saphenous vein

3289
Q

Most common perforation site of duodenum

A

First due to exposure to acid

3290
Q

Clinical signs of #

A

Pain

Swelling

Crepitus

Deformity

Adjacent structural injury: nerves/vessels/ligament/tendons

3291
Q

Two types of displacement in#?

A

Translation (lateral): proximal/distal. ant/post. medial/lateral

Angulation: internal/external rotation. dorsal/volar/ varus/valgus

3292
Q

Different types of external fixation

A

Monoplanar (i.e. in one plane)

Multiplaner e.g. circular ext fixation

3293
Q

When is external fixation used?

A

When there is extensive soft tissue injury or complex periarticular fracture.

3294
Q

External fixation and infection

A

100% of external fixators are infected.

Pins get infected but controllable, prevents infection in the bone.

Allows control of infection

3295
Q

Classification of surgical complications

A

Immediate

Early

Late

under these, local and general

3296
Q

Cx of fractures

A

General (early or late):

Fat embolus

DVT

Infection

Prolonged immobility: UTI

Specific:

Neurovascular injuery

Muscle/tissue injury

Non/malunion

Compartment synrome

Local infection

Degenerative change

Reflex sympathetic dystrophy

AVN

Growth disturbance

3297
Q

XR in orthopaedics

A

Weight bearing views

3298
Q

OA how to phrase weight loss

A

Coming down the stairs is like 5x your body weight. If you lose 5kg- like reducing 25kg of strain on knee

3299
Q

Def: frozen shoulder

A

Pain in all ranges of movement with a normal shoulder xray.

(mostly external rotation is limited)

3300
Q

Causes of post THR pain

A

Adductor weakness/rupture

Leg length discrepancy

Prosthesis failure

Infection

3301
Q

Constant dysuria with negative MSU

A

Red flag, require cystoscopy

3302
Q

What is the main cause of bladder cancer?

A

Smoking

3303
Q

Anticoagulation and haematuria

A

Doesn’t exclude underlying cause

May mean presentation is earlier

3304
Q

Cytotoxic associated with bladder cancer

A

Cyclophosphamide

3305
Q

How to classify haematuria

A

Micro and macroscopic

3306
Q

How to classify haematuria anatomically

A

Upper UT:

Kidney stones

TCC of renal pelvis

RCC

Inury to kidney: tear

Medical causes e.g. nephritis… TB

Ureter:

luminal: stone
mural: tumour e.g. malignant stricture or urothelial

Lower urinary tract

Bladder:

stone, tumour, injury to bladder

Prostate: distended veins over large prostate, prostatitis, prostate cancer

Urethra

Testes

3307
Q

beta naphthylamine dye

A

Bladder Ca

3308
Q

Cytotoxic drug causing haematuria

A

Cyclophosphamide

3309
Q

TWR indications for haematuria

A

All frank haematuria

Persistent haematuria with dysuria in the absence of UTI

Micro/macrohaematuria with LUTS

Female retention with pain and haematuria

3310
Q

Microscopic haematuria ix in TWR

A

Renal bladder/ USS

3311
Q

Frank haematuria TWR Ix

A

CT urogram

3312
Q

USS in urology

A

Noninvasive

Differentiates between cyst and solid (tumour)

Any evidence of hydronephrosis

3313
Q

What is a CT urogram

A

Examination of KUB before and after IV contrast material.

Cotrol CT, 5 and 15 min scans.

Picks up urothelial filling defects.

Instead of IV pyelogram

3314
Q

What is a CT KUB?

A

Non contrast CT scan of the kidneys, ureturs and bladder

Dx of renal stones which show up as white within the urinary trract

Taken at 2.5mm cuts

3315
Q

What is a triple phase CT scan?

A

Scan used to further evaluate renal tumours

Non contrast: look for fat in tumour (to ensure not angiomyolipoma)

Arterial: contrast enhancement

Venous phase

Examines location of tumour, size, renal vein involvement, any LNs or distant mets.

Always comment on presence/absence/state of contralateral kidney

3316
Q

Who needs a cystoscopy

A

Smokers

Occupational exposure to chemicals/dyes

History of persistent dysuria

History of phenacitin abuse

Hx of pelvic Rtx

Cyclophosphamide exposure

3317
Q

What is the false negative rate on renal biopsy?

A

11%

3318
Q

Initial Mx of TCC bladder

A

TURBT

R/V stage and grade

3319
Q

What is HexFix

A

Allows examination of bladdder under blue light

Red patch shows TCC in situ

Enhances view

3320
Q

=Staging of TCC bladder

A

CIS

Ta: confined to mucosa

T1: in the lamina propria

T2: in the muscularis propria

T3: into perivesical fat

3321
Q

Layers of bladder

A

Mucosa

Submucosa

Muscularis propria

Pervivesical fat

3322
Q

Mx of T1G3 bladder tumour

A

Intravescial immunotherapy: BCG

Close cystoscopic surveillance

Ultimately radical cystectomy

3323
Q

Features of bladder CIS

A

Present with bladder pain and dysruia

If untreated for 2y: 50% develop invasive TCC

Treated with intravesical BCG with radical cystectomy for those who fail to respond

3324
Q

TCC: velvet in bladder

A

CIS

3325
Q

Treatment of TCC of upper tract

A

Nephroureterectomy (kidney, ureteur and cuff of bladder)

Ureteroscopy and laser for low grade tumours in unfit patients/solitary kidneys

3326
Q

Why are patients with upper tract TCC at increased risk of bladder recurrence?

A

Urine carries malignant cells down to bladder

20-40% incidence of bladder TCC

3327
Q

Presentation of renal cancer

A

Usually an incidental finding on abdo USS

3328
Q

Classical triad in RCC

A

Haematuria

Loin pain

Palpable mass

Late presentaiton

3329
Q

Stauffer Syndrome

A

Stauffer syndrome is a constellation of signs and symptoms of liver dysfunction that arise due to presence of renal cell carcinoma, and, more rarely, in connection with other malignant neoplasms, though the specific pathogenesis is currently unknown. It is named for Dr. Maurice Stauffer, a gastroenterologist at the Mayo Clinic in Rochester, MN. The hepatic abnormalities are not due to tumor infiltration of the liver or intrinsic liver disease; they instead reflect the presence of a paraneoplastic syndrome.[1]

Stauffer syndrome causes abnormal liver function tests, especially those that reflect the presence of cholestasis, i.e. abnormal bile flow. Hepatosplenomegaly may also be observed. The symptoms and signs resolve if the renal cell carcinoma (or another associated tumor) is successfully ablated

3330
Q

Paraneoplastic in RCC

A

High clacium

LFTs (stauffer syndrome)

3331
Q

RCC staging

A

Stage 1 <7cm

Stage 2: >7cm

Stage 3: Invading into renal vein or into Gerota’s fascia

Stage IV: to other organs/LNs

3332
Q

What is Gerota’s fascia

A

Fascia covering kidney

3333
Q

Renal mass <3cm

A

Treated conservatively: active surveillance, total/partial nephrectomy, radiofrequency ablation or cryotherapy

AKA Small renal mass (SRM)

3334
Q

Mx of renal cancer: organ confined

A

Radical nephrectomy: lap or open

Lap unless massive

3335
Q

Absolute indications for partial nephrectomy in RCC

A

Solitary

Bilateral renal mass

Renal impairment

3336
Q

Renal stones different types

A

85% calcium

15% combination or uric acid (radiolucent) cystine and struvite

Struvite stones: combination of Mg, NH3 and P: occur in infected urine

3337
Q

Bacterial causes of staghorn calculus

A

Urease producing bacteria:

Staghorn calculi are composed of struvite (MAP, magnesium ammonium phosphate) and are usually seen in the setting of recurrent urinary tract infection with urease producing bacteria (e.g. Proteus, Klebsiella, Pseudomonas and Enterobacter).

3338
Q

Renal colic + T

A

Indication for admission

3339
Q

Renal colic NSAID

A

Rectal diclofenac

3340
Q

Renal stone threshold for treatment

A

<5mm have an 80% chance of passing spontaneously

3341
Q

How to divide lower UT symptoms

A

Storage symptoms:

Weak stream

Straining

Hesitancy

Terminal dribbling

Incomplete emptying

Voiding:

Urgency

Frequency

Nocturia

Incontinence

Post micturition dribbling

3342
Q

Mx of bladder outflow obstruction

A

Treatment:

alpha blockers: relax neck of bladder e.g. alfusoin, tamsulosin

5 alpha reductase inhibitor e.g. finasteride (takes 6m)

Can be used in combination: combidart

Surgical:

TURP

Enulceation of prostate

If very large: open prostatectomy (Millin’s prostatectomy)

Freyer’s prostatectomy (transvesicular) used if patient has a stone with the prostate

3343
Q

Assessment of prostate

A

DRE

PSA

Flow rate

US prostate (MRI is used more and more)

Bone scan if ?mets

3344
Q

Time limit on TURP

A

<1 hour

To prevent turp syndrome

3345
Q

Normal PSA

A

<4

3346
Q

Staging of prostate cancer

A

Can be biopsied transrectally or transperineal.

Stage 1: confined to periphery

Stage 2: in both lobes

May not be palpable

Stage 3: multiple and palpable LN

Stage 4: in contiguous organs

Palpable

3347
Q

Mx of prostate Ca

A

Active surveillance

Hormone treatment

RTx/Brachytherapy

Sx: radical prostatectomy

Cryotherapy/HIFU

3348
Q

Doubling time for testicular cancer

A

10d

Seen same day

Painless enlarging lump in body of testes

3349
Q

Approach to orchidectomy

A

Inguinal:

The inguinal orchiectomy is a necessary procedure if testicular cancer is suspected. While it is possible to remove a testicle through an incision in the scrotum, this is not done when cancer is suspected because it disrupts the natural lymphatic drainage patterns. Testicular cancer usually spreads into the lymph nodes inside the abdomen in a predictable manner. Cutting the skin in the scrotum may disrupt this and cancer may spread to the inguinal lymph nodes, making surveillance and subsequent operations more difficult.

3350
Q

Mx of nonseminoma post orchidectomy

A

Chemo BEP/CHOP

may need retroperitoneal LN dissection

One of the problems is retrograde ejaculation so need to consent

3351
Q

2 way catheter used for

A

Drainage

3352
Q

3 way catheter

A

Used for irrigation/haematuria/post TURP/TURBT

Coude tipped, allow irrigation to prevent clot causing outflow obstruction

3353
Q

Colours on urinary catheters

A

Used to size them

Green is the smallest

3354
Q

Purpose of JJ stent

A

Hollow tube: urine drains through and around

Ureter stops peristalsing and dilates- can allow the uretur to become suitable for ureteroscopy

3355
Q

Nephrostomy insertion

A

Antegrade: from above

Retrograde: from below

3356
Q

Flexible cystoscopy

A

Inspection or urethra and bladder under LA

3357
Q

Rigid cystoscopy

A

Inspection of urethra and bladder under GA

3358
Q

Cystoscopy and retrograde pyelogram

A

Contrast examination of ureter

3359
Q

Cystoscopy and insertion of JJ stent

A

Under GA: obstructed ureter

3360
Q

PCNL

A

Percutaenous nephrolithotomy (insertion of a scope directly into the kidney to laser stones in the kdiney): GA

3361
Q

Simple nephrectomy

A

Removal of non functioning kidney: lap or open

3362
Q

Radical nephrectomy

A

Removal of kidney for cancer: kidney/perinephric fat and adrenal gland

3363
Q

Ix in CRC

A

Bloods:

FBC: Hb

LFTs: mets

Tumour marker: CEA

Imaging:

CXR: lung mets

USS liver: mets

CT and MRI: staging, MRI best for rectal Ca and liver mets

Endoanal US: staging rectal tumours

Ba/gastrograffin enema: apple-core lesion

Endoscopy + biopsy:

Flexi sig: 65% of tumors accessible

Colonoscopy

3364
Q
A

Apple-core sign

CRC

3365
Q

What system is used to stage CRC

A

Dukes (Sir Cuthbert Dukes: St Mary’s Pathologist)

3366
Q

Duke’s CRC staging

A

A: confined to bowel wall :90%

B: through bowel wall but no LNs: 60%

C: Regional LNs: 30%

D: Distant mets: <10% 5ys

3367
Q

TNM staging of CRC

A

Tis: carcinoma in situ

T1: submucosa

T2: muscularis propria

T3: subserosa

T4: through serosa to adjacent organs

N1: 1-3 nodes

N2: >4 nodes

3368
Q

Grading of CRC

A

Grading from low to high based on cell morphology

Dysplasia, mitotic index, hyperchromatism

3369
Q

Mx of CRC principles

A

MDT

Confirm Dx

Stage with CT or MRI

60% amenable to radical Sx

3370
Q

Sx for CRC

A

Use ERAS pathway

Pre-operative bowel prep (except R sided lesions)

e.g. Kleen Prep (macrogol: osmotic laxative) the day before and phosphate enema in the AM

Consent for stoma

Stoma nurse consult for siting

3371
Q

Principles of CRC

A

Excision depends on lymphatic drainage which follows the arterial supply

Mobility of bowel and blood supply at cut ends is also important

Hartmann’s often used if obstruction

Laparoscopic approach is the standard of care

3372
Q

TWR for CRC

A

>40y reporting rectal bleeding with change in bowel habit persisting for >6w

>60 with retal bleeding for >6w without change in bowel habit

>60 with change in bowel habit without rectal bleeding

Of any age with right lower abdominal mass consistent with involvement of large bowel

Of any age with palpable rectal mass (intraluminal and not pelvic, if pelvic requires referral to uro or gynae)

Men of any age with unexplained IDA and Hb <11

Non-menstruating women with unexplained IDA and Hb <10

3373
Q

Sx mx of Rectal Ca

A

Neoadjuvant RTx may be used to reduced local recurrence and increase 5y survival

Anterior resection

AP resection

+ total mesorectal excision

3374
Q

Anterior resection in rectal Ca

A

Tumour 4-5cm from anal verge

Defunction with loop ileostomy

3375
Q

AP resection in rectal Ca

A

<4cm from anal verge

3376
Q

Total mesorectal excision in rectal Ca

A

For tumours of the middle and lower third

Aims to reduce recurrence

Increased anastomotic leak and feacal incontinence

3377
Q

Sigmoid Ca Sx

A

High anterior resection or sigmoid colectomy

3378
Q

Left CRC Sx

A

Left hemicolectomy

3379
Q

Transverse CRC Sx

A

Extended right hemicolectomy

3380
Q

Caecal/right CRC

A

Right hemicolectomy

3381
Q
A
3382
Q
A
3383
Q

Other Rx in CRC

A

Local exision: e.g. transanal endoscopic microsurgery

Bypass surgery: palliation

Hepatic resection: if single lobe mets

Stenting: palliation or bridge to surgery in obstruction

CTx

3384
Q

CTx in CRC

A

Adjuvant 5-FU for Duke’s C reduces mortality by 25% i.e. LN +ve patients

High grade tumour

Palliation of metastatic disease

3385
Q

NHS Screening for CRC

FOB

A

Introduced in 2006

60-75y

Home FOB every 2y: 1/50 have +ve FOB

Colonoscopy if +ve: 1/10 have Ca

Lindholm et al BJS: screening reduced risk of dying from CRC by 25%

3386
Q

CRC screening:

Flexi sig

A

Introduced inn 2011/12

One off screen at 55

Atkin et al Lancet:

Reduced CRC incidence by 33%

Reduced CRC mortality by 43%

3387
Q

FAP cause

A

Autosomal dominant

APC gene on 5q21

3388
Q

100-1000s adenomas by 16y

Mainly in large bowel

Also stomach and duodenum (near ampulla)

100% develop CRC, often by 40y

May be associated with congenital hypertrophy of the retinal pigment epithelium

A

Familial Adenomatous Polyposis

3389
Q

<100 adenomas

Later CRC (>50y)

A

Attenuated FAP

3390
Q

Gardner’s Syndrome

TODE

A

FAP variant

Thyroid tumours

Osteomas of the mandible, skull and long bones

Dental abnormalities: supernumerary teeth

Epidermal cysts

3391
Q

Turcots

A

Variant of FAP

In addition: CNS tumours medullo and glioblastomas

3392
Q

Mx of FAP

A

Prophylactic colectomy before 20y

Total colectomy + IRA: requires life long stump surveillance

Proctocolectomy: IPAA

Remain at risk of Ca in stomach and duodenum, require endoscopic screening

3393
Q

What is the commonest cause of hereditary CRC?

A

HNPCC

3394
Q

Pathology of HNPCC

A

AD

Mutation of MMR enzymes e.g. MSH2 on Chr 2p

Commonest cause of hereditary CRC: 3% of all CRC

3395
Q

Lynch 1 presentation

A

HNPCC

right sided CRC

3396
Q

Lynch 2 presentation

A

CRC + gastric, endometrial, prostate, breast

3397
Q

Lynch 1 vs 2

A

1 is familial colon cancer

2 is associated with other Ca of the GIT and reproductive tissue

3398
Q

Dx of HNPCC

3, 2, 1

A

Amsterdam II criteria

3 or more relatives with an associated cancer (CRC, endometrial, small intestine, uretur or renal pelvis)

2 or more successive generations involved

At least 1 diagnosed <50y/o

1 should be a first degree relative of the other 2

FAP should be exluded

Tumours should be verified via pathologic examination

3399
Q

Pathology of Peutz-Jehgers

A

AD

STK11 mutation

3400
Q

10-15y/o

Mucocutaneous hyperpigmentation: macules on palms, buccal mucosa

Multiple GI hamartomous polyps: intussuception, haemorrhage

CRC, pancreas, breast, lung, ovaries, uterus

A

Peutz-Jehgers

3401
Q
A

Peutz Jehgers

3402
Q

What are the different types of GI poylps

A

Inflammatory pseudopolyps

Hyperplastic polyps

Hamartomatous

Neoplastic

3403
Q

Inflammatory pseudopolyps=

A

Regnerating islands of mucosa in UC

3404
Q

Hyperplastic polyps=

A

Piling up of goblet cells and absorptive cells

Serrated surface architecture

No malignant potential

3405
Q

Piling up of goblet cells and absorptive cells

Serrated surface architecture

No malignant potential

A

Hyperplastic polyps

3406
Q

Tumour like growths composed of tissues present at site where they develop

Sporadic or part of familial syndromes

A

Hamartomous polyps

3407
Q

Solitary hamartoma in children

Cherry on a stalk appearance

A

Juvenile polyp

3408
Q

Tubular or villous adenomas

Usually asymptomatic

May have blood/mucus PR, tenesmus

A

Neoplastic polyps

3409
Q

Autosomal dominant

>10 hamartomatous polyps

Increased CRC risk: need surveilance and polypectomy

A

Juvenile polyposis

3410
Q

Autosomal dominant

Macrocephaly and skin stigmata

Intestinal hamartomas

Increased risk of extra-intestinal Ca

A

Cowden syndrome

3411
Q

How can the causes of acute mesenteric ischaemia be classified?

A

Arterial

Non occlusive

Venous

Other

3412
Q

Arterial mesenteric ischaemia

A

Thrombotic (35%), embolic (35%)

3413
Q

Non-occlusive mesenteric acute mesenteric ischaemia

A

Splanchnic vasoconstriction e.g. 2o to shock

3414
Q

Other causes of mesenteric ischaemia

A

Trauma, vasculitis, strangulation

3415
Q

What is the most common cause of acute mesenteric ischaemia?

A

Arterial (70%)

(non-occlusive-20%, VT- 5%)

3416
Q

What is the triad of acute mesenteric iscahemia

A

Acute severe abdominal pain +/- PR bleed

Rapid hypovolaemia-> shock

No abdominal signs

3417
Q

Nearly always small bowel

Acute severe abdominal pain +/- PR bleed

Rapid hypovolaemia-> shock

No abdominal signs

Degree of illness >>> clinical signs

May by in AF

A

?acute mesenteric ischaemia

3418
Q

Ix in acute mesenteric iscahemia

A

Bloods:

Raised Hb: plasma loss

Raised WCC

Raised amylase

Persistent metabolic acidosis: raised lactate

Imaging:

AXR- gasless abdomen

Arteriography/CT/MRI angio

3419
Q

Cx of acute mesenteric ischaemia

A

Septic peritonitis

SIRS-> MODS

3420
Q

Mx of acute mesenteric ischaemia

A

Fluids

Abx: gent + met

LMWH

Laparotomy: resect necrotic bowel

3421
Q

Cause of chronic small bowel ischaemia

A

Atheroma + low flow state e.g. LVF

3422
Q

Severe colicky post-prandial abdominal pain (gut claudication)

PR bleeding

Malabsorption

Weight loss

A

Chronic small bowel ischaemia

3423
Q

Mx of chronic small bowel ischaemia

A

Mx: angioplasty

3424
Q

Cause of chronic large bowel ischaemia

A

Follows low flow in IMA territory

3425
Q

Lower, left sided abdominal pain

Bloody diarrhoea

Pyrexia

Tachycardia

A

?Chronic large bowel ischaemia

3426
Q

Ix in chronic large bowel ischaemia

A

Raised WCC

Ba enema: thumb-printing

MR angiography

3427
Q

Cx of chronic large bowel ischaemia

A

May-> peritonitis and septic shock

Strictures in the LT

3428
Q

Mx of chronic large bowel ischaemia

A

Usually conservative: fluids and Abx

Angioplasty and endovascular stenting

3429
Q

What are the common/important causes of lower GI bleed

A

Rectal: haemorrhoids, fissure

Diverticulitis

Neoplasm

3430
Q

What are the causes of lower GI bleed

A

Important:

Rectal: haemorrhoids, fissure

Diverticulitis

Neoplasm

Other:

IBD

Infection: shigella, campylobacter, C diff

Polyps

Large upper GI bleed (15% of lower GI bleeds)

Angio: dysplasic, ischaemic colitis, HHT

3431
Q

Ischaemic colitis=

A

Colonic ischaemia

3432
Q

Ix in lower GI bleed

A

FBC, U+E, LFT, x-match, clotting, amylase

Stool: MCS

Imaging:

AXR erect CXR

Angiography: necessary if no source on endoscopy

Endoscopy

3433
Q

Endoscopy in lower GI bleed

A

1st: Rigid proctoscopy/sigmoidoscopy
2nd: OGD
3rd: colonoscopy: difficult in major bleeding

3434
Q

Mx of Lower GI bleed

A

ABC

Urinary catheter

Abx if evidence of sepsis or perf

PPI if upper GI bleed

Keep bed bound: need to pass stool may be large bleed-> collapse

Stool chart

Diet: keep on clear fluids (allows colonoscopy)

Surgery: only if unremitting, massive bleed

3435
Q

Features of colonic angiodysplasia

A

Submucosal AV malformation

70-90% occur in right colon

Can affect anywhere in GIT

3436
Q

Presentation of angiodysplasia

A

Elderly

Fresh PR bleeding

3437
Q

Mx of angiodysplasia

A

Exlcude other dx: PR, Ba enema, colonoscopy

Mesenteric angiorgraphy or CT angiography

Tc-labelled RBC scan: indentify active bleeding

3438
Q

Rx of angiodysplasia

A

Embolisation

Endoscopic laser electrocoagulation

Resection

3439
Q

Benign tumour of mature adipocytes

Sarcomatous change probably doesn’t occur

Liposarcomas arise de novo

A

Lipoma

3440
Q

Occur anywhere fat can expand i.e. not scalp or palms. Include spermatic cord, submucosa

Soft

Subcutaneous

Imprecise margin

Fluctuant

A

Lipoma

3441
Q
A

Soft

Subcutaneous

Imprecise margin

Fluctuant

3442
Q

Mx of lipomas

A

Non-surgical

Surgical excision

3443
Q

Diseases causing lipomas

A

Dercum’s disease/Adiposis dolorosa

Familial Mutliple Lipomatosis

Madelung’s Disease

Bannayan-Zonana Syndrome

3444
Q

Dercum’s disease

A

Multiple painful lipomas

Associated with peripheral neuropathy

Obese, postmenopausal women

Dercum’s disease was first described at Jefferson Medical College by neurologist Francis Xavier Dercum in 1892

3445
Q

Familial Multiple Lipomatosis

A

is a rare condition that is characterized by multiple lipomas on the trunk and extremities. As the name suggests, FML is diagnosed when multiple lipomatosis occurs in more than one family member, often over several generations. The lipomas associated with FML are usually painless, but may impact quality of life as they can be numerous and large.[1][2] Although the condition appears to be passed down through families in an autosomal dominant manner, the underlying genetic cause is currently unknown.[3]Treatment is based on the signs and symptoms present in each person. Surgical excision may be necessary if the tumors interfere with movement and/or daily life.[1][2

3446
Q

Madelung’s disease

A

Benign symmetric lipomatosis (also known as Benign symmetric lipomatosis of Launois–Bensaude, Madelung’s disease, multiple symmetric lipomatosis, and cephalothoracic lipodystrophy) is a cutaneous condition characterized by extensive symmetric fat deposits in the head, neck, and shoulder girdle area.[1] The German surgeon Otto Wilhelm Madelung was the first to give a detailed description of the disorder. This condition is very rare, with an estimated incidence rate of 1 in 25,000, and affects males up to 30 times more frequently than females.[2]

The cause of the disease remains unknown, but its incidence strongly correlates with alcohol abuse; abstinence from alcohol prevents disease progression. Defects in the adrenergic-stimulated lipolysis and accumulation of embryological brown fat have also been reported. Cosmetic disfigurement due to the fat deposition in the cervicothoracic region results in a “pseudoathletic appearance”, resembling the Italian statue Warrior of Capestrano and carvings of Queen of Punt (Egypt).[3] Traditionally the treatment is mainly surgical, consisting of the removal of the lipomas, although recent study has proposed liposuction and phosphatidylcholine injection as possible alternatives.[4]

3447
Q
A

Madelung’s disease

3448
Q

Bannayan-Zonana Syndrome

A

Multiple lipomas

Macrocephaly

Haemangiomas

AD

3449
Q

Epithelial lined cyst containing keratin

Occur at sites of hair growth: scalp, face, neck chest and back

Not soles or palms

Central punctum

Firm

Smooth

Intradermal

A

Sebaceous cyst

3450
Q

What are the two subtypes of sebaceous cyst

A

Epidermal cyst:

arise from hair follicle infundibulum

Trichilemmal cyst/Wen

Arise from hair follicle epithelium

Often multiple

May be AD

3451
Q

Central punctum

Firm

Smooth

Intradermal

A

Sebaceous cyst

3452
Q

Cx of sebaceous cysts?

A

Infection: pus discharge

Ulceration

Calcification

3453
Q

Large ulcerating trichilemmal cyst on the scalp

Resembles an SCC

A

Cock’s Peculiar Tumour

3454
Q

Mx of sebaceous cyst

A

Non-surgical or with excision

3455
Q
A

Cock’s peculiar tumour

3456
Q

Cystic swelling related to a synovial lined structure: joint, tendon

Myxoid degeneration of fibrous tissue

Contain thick, gelatinous material

A

Ganglion

3457
Q

Can be found anywhere

90% on dorsum of hand or wrist

Dorsum of ankle

May be a scar from recurrence

Weakly transilluminable

Soft

Subcutaneous

May be tethered to a tendon

A

Ganglion

3458
Q

DDx for ganglion

A

Bursae

Cystic protrusion from synovial cavity of arthritic joint

3459
Q

Mx of ganglion

A

Non-surgical:

Aspiration followed by 3w of immobilisation

Surgical excision:

Recurrence can be 50%

Risk of neurovascular damage

3460
Q
A

Ganglion

3461
Q

Benign hyperplasia of basal epithelial layer

Hyperkeratosis: keratin layer thickening

Acanthosis: prickle layer thickening

A

Seborrhoeic keratosis

3462
Q

Stuck on appearance

Dark brown

Greasy

A

Seborrheic keratosis

3463
Q

Mx of seborrheic keratosis

A

Nonsurgical

3464
Q
A

Seborrheic keratosis

3465
Q

Benign nerve sheath tumour arising from schwann cells

A

Neurofibroma

3466
Q

Solitary or multiple

Pedunculated nodules

Fleshy consistency

Pressure can-> paraesthesia

Examin the eyes (?Lisch), the axilla(freckling) and the CNs (esp. 8)

BP

A

Neurofibroma

3467
Q

Mx of neurofibroma

A

Surgical excision only indicated if malignant growth suspected

Local regrowth is common

3468
Q

Overgrowth of all the layers of the skin with a central vascular core

Pedunculated

Flesh coloured

Fibroepithelial polyp

A

Papilloma

3469
Q

Mx of papilloma

A

Excision and diathermy to control bleeding

3470
Q

Rapidly growing capillary haemangioma

Most commonly on hands face, gums and lips

Bright red hemispherical nodule

May have serous/purulent discharge

Soft on palpation

Bleed very easily

A

Pyogenic granuloma

NB neither pyogenic nor a granuloma

3471
Q

Additional features of pyogenic granuloma

A

Possibly associated with previous trauma

More common in pregnancy

3472
Q

Mx of pyogenic granuloma

A

Curettage with diathermy at the bases

3473
Q
A

Pyogenic granuloma

3474
Q

Epidermal lined cyst deep to the skin

Smooth spherical swelling

Sites of embryological fusion

Scar from recurrence

Soft

Non-tender

Subcutaneous

A

Dermoid cyst

3475
Q
A

Dermoid cyst

3476
Q

Classification of dermoid cysts

A

Congenital/inclusion:

Developmental inclusion of epidermis along lines of skin fusion

Midline of neck and nose

Medial and lateral ends of eyebrows

Acquired/implantation:

Implantation of epidermis in dermis

Often 2o to trauma e.g. piercing

3477
Q

Mx of congenital dermoid cyst

A

CT to establish extent

Surgical excision

3478
Q

Acquired dermoid cyst

A

Surgical excision

3479
Q

What to ask an adult with a dermoid cyst

A

Have you always had it

Trauma

3480
Q

Benign neoplasm of dermal fibroblasts

Can occur anywhere

Mostly on the lower limbs of young to middle-aged women

Small, brown, pigmented nodule

Firm, woody feel characteristic

Intradermal: mobile over deep tissue

A

Dermatofibroma

3481
Q
A

Dermatofibroma

3482
Q

DDx for dermatofibroma

A

Melanoma, BCC

Requires excision and histology

3483
Q

Benign overgrowth of hair follicle cells

Cytologically similar to well-differentiated SCCs

Fast-growing

Dome-shaped with a keratin plug

Intradermal

A

Keratoacanthoma

3484
Q

Keratoacnathoma

A

Typically regress

As it cannot be clinically differentaited from SCC with reliability they require surgical treatment

3485
Q

Epdiemiology of malignant melanoma

A

F>M: 1.5:1

10000/yr and 2000 deaths/yr

Increased 80% over past 20y

3486
Q

ABCDE of melanoma

A

Asymmetry

Border: irregular

Colour: >1

Diameter: >6m

Evolving/elevation

3487
Q

RFs for MM

A

Sunlight especially intesnse exposure in early years

Low Fitzpatrick skin type

Increased no of common moles

+ve FH

Increased age

Immunosuppression

3488
Q

How can MM be classified?

A

Superficial spreading

Lentigo maligna melanoma

Acral lentiginous

Nodular melanoma

Amelanotic

3489
Q

Most common type of MM

A

Superficail spreading

3490
Q

MM

Irregular borders, colour variation

Commonest in caucasians

Grow slowly, metastasise late= better Px

A

Superficial spreading melanoma

3491
Q
A

Superficial spreading melanoma

3492
Q

MM

Often elderly patients

Face or scalp

A

Lentigo maligna melanoma

3493
Q
A

Lentigo Maligna melanoma

3494
Q

MM

Asians/blakcs

Palms, soles, subungual (with Hutchinson’s sign)

A

Acral lentiginous MM

3495
Q

Hutchinson’s sign in hands

A

Melanonychia: important sign of subungual melanoma

3496
Q
A

Acral lentigionous melanoma

3497
Q

MM

All sites

Younger age, new lesion

Invade deeply and metastasis early=poor Px

A

Nodular melanoma

3498
Q
A

Nodular Melanoma

3499
Q

MM

Atypical appearance-> delayed dx

A

Amelanotic melanoma

3500
Q
A

Amelanotic melanoma

3501
Q

Staging of MM

A

Breslow depth

Clark’s staging

3502
Q

Breslow depth

A

Thickness of tumour to deepest point of dermal invasion

<1mm= >75% 5ys

4mm= 50% 5ys

3503
Q

Clark’s staging

A

MM

Stratifies depth by 5 anatomical levles

Stage 1: epidermis

Stage 5: SC fat

3504
Q

MM mets

A

Liver

Eye

3505
Q

Mx of MM

A

Excision and 2o margin excision depending on Breslow depth +/- lymphadenectomy

+/- adjuvant chemo: may use isolated limb perfusion

3506
Q

Poor pxic indicators in MM

A

Male sex (more tumours on trunk cf. females)

Increased mitosis

Satellite lesions

3507
Q

Ulcerated lesion with hard, raised everted edges found on sun exposed areas

A

Squamous cell carcinoma

3508
Q

Causes of SCC skin

A

Sun exposure: scalp, face, ears, lower leg

May arise in chronic ulcers: Marjolin’s ulcer

Xeroderma pigmentosa

3509
Q
A

Xeroderma pigmentosa

Increased risk of SCC skin

3510
Q

Evolution of SCC skin

A

Solar/actinic keratosis-> Bowen’s-> SCC
LN spread is rare

3511
Q
A

SCC

3512
Q

Irregular crusty warty lesions

With pre-malignant potential

Found on sun exposed areas

A

Actinic keratosis

3513
Q

Pre-malignant potential of AK

A

1% /y

3514
Q
A

Actinic keratoses

3515
Q

Mx for Actinic keratoses

A

Cautery

Cryotherapy

5-FU

Imiquimod

Photodynamic phototherapy

3516
Q

Red/brown scaly plaques

Typically on the legs of older women

A

Bowen’s disease= SCC in situ

3517
Q
A

Bowen’s disease

3518
Q

Rx of Bowen’s disease

A

As for AK

3519
Q

Commonest cancer

Pearly nodule with rolled telangiectactic edge

May ulcerate

Typically on face in sun-exposed area above line from tragus-> angle of mouth

A

BCC

3520
Q

Behaviour of BCC

A

Low-grade malignancy-> very rarely metastasies

Locally invasvie

3521
Q

Rx BCC

A

Ecision: Mohs- complete circumferential margin assessment using frozen section histology

Cryo/radio may be used

3522
Q
A

BCC

3523
Q

Dx of neck lumps: general approach

A

85% of neck lumps are LNs: especially if present <3w

Infection: EBV, tonsilitis, HIV

Ca: lymphoma or mets

8% are goitres

7% other e.g. sebaceous cyst or lipoma

3524
Q

Ix in neck lumps

A

Clinical assessment

Imaging: USS

Cyto/histology: aspiration or biopsy

3525
Q

Borders of the anterior triangle

A

Anterior margin of SCM

Midline

Ramus of the mandible

Roof: investing fascia

Floor: prevertebral fascia

3526
Q

Pulsatile causes of anterior triangle neck lump

A

Carotid artery aneurysm

Tortuous carotid artery

Carotid body tumour

3527
Q

Non pulsatile causes of anterior triangle neck lump

A

Branchial cysts

Laryngocele

Goitre

Parotid tumour: lump in postero superior area

3528
Q

Borders of the submandibular tirangle

A

Mental process

Ramus of the mandible

Line between the two angles of the mandible

3529
Q

Causes of submandibular triangle lumps

A

Salivary stone

Sialadenitis

Salivary tumour

3530
Q

Margins of the posterior triangle of the neck

A

Posterior margin of SCM

Anterior margin of trapezius

Middle 1/3rd of clavicle

3531
Q

Causes of posterior triangle neck lumps

A

LNs

Cervical ribs

Pharyngeal pouch

Cystic hygromas

Pancoast’s tumours

3532
Q

Midline lump <20y

A

Thyroglossal cyst

Dermoid cyst

3533
Q

Midline lump >20y

A

Thyroid isthmus mass

Ectopic thyroid tissue

3534
Q

Def: branchial cyst

A

Embryological remnant 2nd branchial cleft

3535
Q

Age <20y

Anterior margin of SCM at junction of upper and middle 3rd

May become infected-> abscess

May be associated with a fistula

Lined by squamous epithelium

Contain “glary fluid” with cholesterol crystals

A

Branchial cysts

3536
Q
A

Branchial cyst

3537
Q

Rx of branchial cyst

A

Med:

Abx for infection

Sclerotherapy with OK-432 can be used

Sx:

Definitive Rx

May be difficult due to proximity of carotids

3538
Q

Small opening in lower 3rd of neck on anterior margin of SCM

Between tonsillar fossa and anterior border of SCM

May discharge mucus

A

Branchial sinus or fistula

3539
Q

Features of chemodectoma

A

Carotid body tumour

Very rare

Located @ carotid bifurcation

Detect pO2, pCO2 and H+

3540
Q

Just anterior to upper 3rd of SCM

Pulsatile

Move laterally but not vertically

May be bilateral

Pressure may-> dizziness and syncope

Mostly bengin (5% malignant)

A

Chemodectoma

3541
Q

Ix in chemodectoma

A

Doppler or angio: splaying of bifurcation

3542
Q

Rx in chemodectoma

A

Extirpation by vascular surgeon

3543
Q

Cystic dilatation of the laryngeal saccule

Congenital or acquired

Exacerbated by blowing

A

Laryngocele

3544
Q

Common <20y

Found at junctions of emryological fusion

Neck midline, lateral angles of eyebrow, under tongue

Contains ectodermal elements: hair follciles, sebaceous glands

Rx: excision

A

Dermoid Cyst

3545
Q

Cyst formed from persistent thyroglossal duct

Path of thyroid descent from base of tongue

A

Thyroglossal cyst

3546
Q

Can be located anywhere between foramen caecum and the thyroid: usually just inferior to the hyoid (subhyoid or just above, suprahyoid)

Fluctuant lump that moves up with tongue protrusion

Can become infected-> fistula

A

Thyroglossal cyst

3547
Q
A

Infected thyroglossal cyst

3548
Q

Def: cervical ribs

A

Overdevelopment of transverse process of C7

Occur in 1:150

3549
Q

Mostly asymptomatic

Hard swelling in posterior triangle of the neck

Reduced radial pulse on abduction and external rotation of the arm

Can-> vascular symptoms

Compresses subclavian A

Raynaud’s

Subclavian steal

Reduced venous outflow-> oedema

Can-> neurological symptoms

Compresses lower trunk of brachial plexus, T1 nerve root or stellate ganglion

Wasting of intrinsic hand muscles

Paraesthesia along medial border of arm

A

Cervical rib

3550
Q
A

Cervical rib

3551
Q

Herniation of pharyngeal mucosa through its muscular coat at its weakest point: Killian’s dehiscence: between thyroid and cricopharyngeal muscles that form inferior constrictor

A

Zenker’s diverticulum

3552
Q

Killian’s dehiscence

A

Killian’s dehiscence (also known as Killian’s triangle, Laimer triangle, Laimer-Killian triangle, or Laimer-Haeckermann area) is a triangular area in the wall of the pharynx between the thyropharyngeal and cricopharyngeus of the inferior constrictor of the pharynx

3553
Q

Why is Zenker’s diverticulum a misnomer

A

It is a pseudodiverticulum as it doesn’t involve the muscle

3554
Q

Swelling on LHS of neck in posterior triangle

Regurgitation and aspiration

Halitosis

Gurgling sounds

Food debris-> expansion-> oesophageal compression-> dysphagia

A

Zenker’s diverticulum/Pharyngeal pouch

3555
Q

Ix in pharyngeal pouch

A

Barium swallow

3556
Q

Rx in Pharyngeal pouch

A

Excision and cricopharyngeal myotomy

Endoscopic stapling

3557
Q

Congenital multiloculated lymphangioma arising from jugular lymph sac

A

Cystic hygroma

3558
Q

Infants

Lower part of posterior triangle but may extend to axilla

Increases in size when child coughs/cries

Transilluminates

A

Cystic hygroma

3559
Q
A

Cystic hygroma

3560
Q

Rx of cystic hygroma

A

Excision or hypertonic saline sclerosant

May recur

3561
Q

How to differentiate between nipple eczema and Paget’s disease of the breast

A

Pagets disease differs from eczema of the nipple in that it involves the nipple primarily and only latterly spreads to the areolar (the opposite occurs in eczema).

3562
Q

A 25-year-old man from the far east presents with a fever and right upper quadrant pain. As part of his investigations a CT scan shows an ill defined lesion in the right lobe of the liver.

A.Haemangioma

B.Hepatocellular carcinoma

C.Hepatic metastasis

D.Polycystic liver disease

E.Simple liver cyst

F.Hyatid cyst

G.Amoebic abscess

H.Mesenchymal hamartoma

A

Amoebic abscesses will tend to present in a similar fashion to other pyogenic liver abscesses. They should be considered in any individual presenting from a region where Entamoeba histiolytica is endemic. Treatment with metronidazole usually produces a marked clinical response.

3563
Q

A 42-year-old lady presents with right upper quadrant pain and a sensation of abdominal fullness. An ultrasound scan demonstrates a 6.5 cm hyperechoic lesion in the right lobe of the liver. Serum AFP is normal.

A.Haemangioma

B.Hepatocellular carcinoma

C.Hepatic metastasis

D.Polycystic liver disease

E.Simple liver cyst

F.Hyatid cyst

G.Amoebic abscess

H.Mesenchymal hamartoma

A

A large hyperechoic lesion in the presence of normal AFP is likely to be a haemangioma. An HCC of equivalent size will almost always result in rise in AFP.

3564
Q

90% develop in women in their third to fifth decade

Linked to use of oral contraceptive pill

Lesions are usually solitary

They are usually sharply demarcated from normal liver although they usually lack a fibrous capsule

On ultrasound the appearances are of mixed echoity and heterogeneous texture. On CT most lesions are hypodense when imaged prior to administration of IV contrast agents

In patients with haemorrhage or symptoms removal of the adenoma may be required

A

Liver cell adenoma

3565
Q

Most common benign tumours of mesenchymal origin

Incidence in autopsy series is 8%

Cavernous haemangiomas may be enormous

Clinically they are reddish purple hypervascular lesions

Lesions are normally separated from normal liver by ring of fibrous tissue

On ultrasound they are typically hyperechoic

A

Haemangioma

3566
Q

Congential and benign, usually present in infants. May compress normal liver

A

Mecenchymal hamartomas

3567
Q

Liver abscess is the most common extra intestinal manifestation of amoebiasis

Between 75 and 90% lesions occur in the right lobe

Presenting complaints typically include fever and right upper quadrant pain

Ultrasonography will usually show a fluid filled structure with poorly defined boundaries

Aspiration yield sterile odourless fluid which has an anchovy paste consistency

Treatment is with metronidazole

A

Amoebic abscess

3568
Q

Biliary sepsis is a major predisposing factor

Structures drained by the portal venous system form the second largest source

Common symptoms include fever, right upper quadrant pain. Jaundice may be seen in 50%

Ultrasound will usually show a fluid filled cavity, hyperechoic walls may be seen in chronic abscesses

A

Liver abscess

3569
Q

Usually occurs in association with polycystic kidney disease

Autosomal dominant disorder

Symptoms may occur as a result of capsular stretch

A

Polycystic liver disease

3570
Q

Rare lesions with malignant potential

Usually solitary multiloculated lesions

Liver function tests usually normal

Ultrasonography typically shows a large anechoic, fluid filled area with irregular margins. Internal echos may result from septa

Surgical resection is indicated in all cases

A

Cystadenoma

3571
Q

Seen in cases of Echinococcus infection

Typically an intense fibrotic reaction occurs around sites of infection

The cyst has no epithelial lining

Cysts are commonly unilocular and may grow to 20cm in size. The cyst wall is thick and has an external laminated hilar membrane and an internal enucleated germinal layer

Typically presents with malaise and right upper quadrant pain. Secondary bacterial infection occurs in 10%.

Liver function tests are usually abnormal and eosinophilia is present in 33% cases

Ultrasound may show septa and hyatid sand or daughter cysts.

Percutaneous aspiration is contra indicated

Treatment is by sterilisation of the cyst with mebendazole and may be followed by surgical resection. Hypertonic swabs are packed around the cysts during surgery

A

Hyatid cysts

3572
Q

A 34-year-old lady is admitted with recurrent episodes of non-specific abdominal pain. On each admission all blood investigations are normal, as are her observations. On this admission a CT scan was performed. This demonstrates a 1.5cm nodule in the right adrenal gland. This is associated with a lipid rich core. Urinary VMA is within normal limits. Other hormonal studies are normal.

A

This is typical for a benign adenoma.Benign adenomas often have a lipid rich core that is readily identifiable on CT scanning. In addition the nodules are often well circumscribed.

3573
Q

Neuroendocrine tumour of the chromaffin cells of the adrenal medulla

A

Phaeochromocytoma

3574
Q

A 70-year-old patient with prostate cancer is commenced on goserelin therapy. A week after starting treatment, he attends a local emergency department complaining of worsened lower urinary tract symptoms and new onset back pain. Which of the following treatment options may have helped avoid this deterioration?

Higher dose goserelin

Pretreatment with flutamide

Lower dose goserelin

Low dose-rate brachytherapy

Joint therapy with corticosteroids

A

During the first stages of treatment, goserelin may cause a transient increase in symptoms of prostatic cancer. This is known as the ‘flare effect’ and is caused by an initial increase in luteinizing hormone production prior to receptor down-regulation.

Flutamide, a synthetic antiandrogen, can be used preemptively to attenuate the tumour flare through its antagonistic effects at androgen receptors.

The new onset back pain in this patient is significant and demands further investigation of spinal metastasis.

3575
Q

A 28-year-old man with Crohn’s disease has undergone a number of resections. His BMI is currently 18 and his albumin is 18. He feels well but does have a small localised perforation of his small bowel. The gastroenterologists are giving azathioprine. What is the most appropriate advice regarding feeding?

Nil by mouth

Nil by mouth and continuous intra venous fluids until surgery

Enteral feeding

Parenteral feeding

Nutritional supplements

A

This man is malnourished, although he is likely to require surgery it is best for him to be nutritionally optimised first. As he may have reduced surface area for absorption and has a localised perforation TPN is likely to be the best feeding modality.

3576
Q

A 72-year-old man has just undergone an emergency repair for a ruptured abdominal aortic aneurysm. Pre operatively he was taking aspirin, clopidogrel and warfarin. Intra operatively he received 5000 units of unfractionated heparin prior to application of the aortic cross clamp. His blood results on admission to the critical care unit are as follows:
Full blood count

Hb8 g/dl

Platelets40 * 109/l

WBC7.1 * 109/l

His fibrin degradation products are measured and found to be markedly elevated. Which of the following accounts for these results?

Anastomotic leak

Disseminated intravascular coagulation

Heparin induced thrombocytopenia

Adverse effect of warfarin

Adverse effects of antiplatelet agents

A

The combination of low platelet counts and raised FDP in this setting make DIC the most likely diagnosis.

3577
Q

Prolonged PT

Prolonged APTT

Prolonged Bleeding time

Plt: low

A

DIC

3578
Q

Stone burden of less than 2cm in aggregate

Renal colic

A

Lithotripsy

3579
Q

Stone burden of less than 2cm in pregnant females

Renal colic

A

Ureteroscopy

3580
Q

Complex renal calculi and staghorn calculi

A

Percutaneous nephrolithotomy

3581
Q

Ureteric calculi less than 5mm

A

Manage expectantly

3582
Q

A 19-year-old female presents with severe anal pain and bleeding which typically occurs post defecation. On examination she has a large posteriorly sited fissure in ano.

A.Excision and primary closure

B.Incision and drainage

C.Topical steroids

D.Topical diltiazem

E.Steroid injections

F.Haemorroidectomy

G.Manual anal dilation

H.Injection with 88% aqueous phenol

I.Sphincterotomy

A

Topical diltiazem

Theme from January 2013 Exam
Initial therapy should be with pharmacological agents to relax the sphincter and facilitate healing. This is particularly true in females presenting for the first time

3583
Q

A 27-year-old man is involved in a road traffic accident. He is seen in the emergency department with chest pain. Clinical examination is essentially unremarkable and he is discharged. He subsequently is found dead at home. What is the most likely underlying injury?

Tracheobronchial tree injury

Traumatic aortic disruption

Cardiac laceration

Diaphragmatic rupture

Rupture of the oesophagus

A

Aortic injuries that do not die at the scene may have a contained haematoma. Clinical signs are subtle and the diagnosis may not be apparent on clinical examination. Without prompt treatment the haematoma usually bursts and the patient dies.

3584
Q

A 24-year-old lady from Western India presents with symptoms of lethargy and dizziness, worse on turning her head. On examination her systolic blood pressure is 176/128. Her pulses are impalpable at all peripheral sites. Auscultation of her chest reveals a systolic heart murmur.

A

Takayasu’s arteritis most commonly affects young Asian females. Pulseless peripheries are a classical finding. The CNS symptoms may be variable.

3585
Q

Large vessel granulomatous vasculitis

Results in intimal narrowing

Most commonly affects young asian females

Patients present with features of mild systemic illness, followed by pulseless phase with symptoms of vascular insufficiency

Treatment is with systemic steroids

A

Takayasu’s arteritis

3586
Q

What is used to grade prostate cancer?

A

Gleason score

3587
Q

A 34-year-old man presents to an emergency surgery with abdominal pain. This started earlier on in the day and is getting progressively worse. The pain is located on his left flank and radiates down into his groin. He has had no similar pain previously and is normally fit and well. Examination reveals a man who is flushed and sweaty but is otherwise unremarkable. What is the most suitable initial management?

Oral ciprofloxacin

IM diclofenac 75 mg

Oral co-amoxiclav and metronidazole

IM morphine 5 mg

IM diclofenac 75 mg + start bendroflumethiazide to prevent further episodes

A

Diclofenac

3588
Q

What is recommended as analgesia of choise in renal colic

A

Diclofenac

3589
Q

How to prevent Ca renal stones?

A

high fluid intake

low animal protein, low salt diet (a low calcium diet has not been shown to be superior to a normocalcaemic diet)

thiazides diuretics (increase distal tubular calcium resorption)

3590
Q

How to prevent oxalate stones

A

cholestyramine reduces urinary oxalate secretion

pyridoxine reduces urinary oxalate secretion

3591
Q

How to prevent uric acid stones

A

allopurinol

urinary alkalinization e.g. oral bicarbonate

3592
Q

What is an important consideration re: diclofenac

A

Increased CV risk, likely to be substituted for other NSADIs e.g. naproxen

3593
Q

A.Interstitial nephritis

B.Membranous glomerulonephritis

C.Endometriosis

D.Placenta percreta

E.Adult polycystic kidney disease

F.Renal vein thrombosis

G.Urinary tract infection

A 22 year female who is 24 weeks pregnant presents with frank haematuria. She is sexually active. She has had a previous pregnancy resulting in caesarean section.

A

Pregnancy and frank haematuria, especially if there is a history of placenta previa or prior caesarean section, should indicate this diagnosis. There is invasive placental implantation into the myometrium, which can rarely extend into the bladder causing severe bleeding.

3594
Q

A 22-year-old woman presents with macroscopic haematuria. She is sexually active. She is known to have renal calculi and had a berry aneurysm clipped.

A

APKD is associated with liver cysts (70%), berry aneurysms (25%) and pancreatic cysts (10%). Patients may have a renal mass, hypertension, renal calculi and macroscopic haematuria.

3595
Q

A 45-year-old woman presents with haematuria and loin pain. She has a temperature of 37 oC and is found to have a Hb 180 g/l and a creatinine of 156 umol/l. Her urine dipstick shows 3+ blood. Blood and urine cultures are negative.

A

Renal vein thrombosis is a common feature of renal cell carcinoma as it invades the renal vein. Other features include PUO, left varicocele and paraneoplastic endocrine effects due to erythropoietin factor, renin, ACTH and PTH like substance.

3596
Q

A 56-year-old man presents with episodic facial pain and discomfort whilst eating. He has suffered from halitosis recently and he frequently complains of a dry mouth. He has a smooth swelling underneath his right mandible. What is the most likely underlying diagnosis?

Stone impacted in Whartons duct

Stone impacted in Stensens duct

Benign adenoma of the submandibular gland

Adenocarcinoma of the submandibular gland

Squamous cell carcinoma of the submandibular gland

A

The symptoms are typical for sialolithiasis. The stones most commonly form in the submandibular gland and therefore may occlude Whartons duct. Stensens duct drains the parotid gland.

3597
Q

Submandibular tumours

A

Only 8% of salivary gland tumours affect the sub mandibular gland

Of these 50% are malignant (usually adenoid cystic carcinoma)

Diagnosis usually involves fine needle aspiration cytology

Imaging is with CT and MRI

In view of the high prevalence of malignancy, all masses of the submandibular glands should generally be excised.

3598
Q

Mass of submandibular gland, action

A

Excision 50% are malignant

3599
Q

A 55-year-old lady has undergone a wide local excision and sentinel lymph node biopsy for breast cancer. The histology report shows a completely excised 1.3cm grade 1 invasive ductal carcinoma. The sentinel node contained no evidence of metastatic disease. The tumour is oestrogen receptor negative.

A

The correct answer is Radiotherapy

Radiotherapy is routine following breast conserving surgery. Without irradiation the local recurrence rates are approximately 40%.

3600
Q

An 88-year-old lady presents with a large mass in the upper inner quadrant of her right breast. Investigations confirm an oestrogen receptor positive, invasive ductal carcinoma. She has declined operative treatment.

A

he correct answer is Endocrine therapy using letrozole

Elderly patients may be managed using endocrine therapy alone. Eventually most will escape hormonal control. In post menopausal women oestrogens are produced by the peripheral aromatization of androgens and aromatase inhibitors are therefore the most popular agent in this age group.

3601
Q

A 38-year-old lady has undergone a mastectomy and axillary node clearance for invasive ductal carcinoma. The histology report shows a completely excised 3.5cm lesion which is grade 3. Two of the axillary lymph nodes contain metastatic disease. The tumour is oestrogen receptor negative.

A

The combination of a grade 3 tumour and axillary nodal metastasis in a young female would attract a recommendation for chemotherapy. Some may also add herceptin (if they are HER 2 positive).

3602
Q

Indications for mastectomy

A

Multifocal tumour

Central tumour

Large lesion in small breast

DCIS >4cm

3603
Q

Indications for Wide local excision

A

Solitary lesion

Peripheral tumour

Small lesion in large breast

DCIS <4cm

3604
Q

RTx in breast ca

A

Whole breast RTx recommended in all those with WLE

Offered for T3-T4 tumours in women who’ve had a mastecomty and those with four or more +ve axillary nodes

3605
Q

ER breast cancer therapy in pre and peri-menopasual women

A

Tamoxifen

3606
Q

ER breast bancer, post menopausal women

A

Aromatase inhibotrs e.g. anastrozaloe

This is because the process of aromatisation accounts for most oestrogen production

3607
Q

CI to trastuzumab in HER2 positive breast Ca

A

Heart disorders

3608
Q

Issues with PSA

A

Poor specificity and sensitivity

around 33% of men with PSA 4-10 will have prostate cancer. Increases to 0% in PSA10-20

Around 20% with prostate cancer have normal PSA

PSA levels increase with age

3609
Q

Causes of raised PSA

A

BPH

Prostatitis and UTI (NICE recommend that PSA test should be postponed for at least 1m)

Ejaculation

Vigorous exercise

Urinary retention

Insturmentaiton of the urinary tract

(DRE)

3610
Q

A 47-year-old woman presents with loin pain and haematuria. Urine dipstick demonstrates:

Blood++++

NitritesPOS

Leucocytes+++

Protein++

Urine culture shows a Proteus infection. An x-ray demonstrates a stag-horn calculus in the left renal pelvis. What is the most likely composition of the renal stone?

Xanthine

Calcium oxalate

Struvite

Cystine

Urate

A

Stag-horn calculi are composed of struvite and form in alkaline urine (ammonia producing bacteria therefore predispose)

3611
Q

A 67-year-old woman is reviewed 6 months after she had a mastectomy following a diagnosis of breast cancer. Which one of the following tumour markers is most useful in monitoring her disease?

CA 125

CD 34

CA 15-3

CA 19-9

CD 117

A

CA-15-3

3612
Q

A 4-year-old boy is brought to the clinic by his mother who has noticed a small lesion at the external angle of his eye. On examination there is a small cystic structure which has obviously been recently infected. On removal of the scab, there is hair visible within the lesion. What is the most likely diagnosis?

Dermoid cyst

Desmoid cyst

Sebaceous cyst

Epidermoid cyst

Keratoacanthoma

A

The lesion is unlikely to be a desmoid cyst as these are seldom located either at this site or in this age group. In addition they do not contain hair. Sebaceous cysts will usually have a punctum and contain a cheesy material. Epidermoid cysts contain keratin plugs.

3613
Q

Progressive dysphagia, may have previous symptoms of GORD or Barretts oesophagus.

A

Adenocarcinoma of the oesophagus

3614
Q

Longer history of dysphagia, often not progressive. Usually symptoms of GORD. Often lack systemic features seen with malignancy

A

Peptic stricture

3615
Q

A 42-year-old man presents with a painless lump in the left testicle that he noticed on self examination. Clinically there is a firm nodule in the left testicle, ultrasound appearances show an irregular mass lesion. His serum AFP and HCG levels are both within normal limits. What is the most likely diagnosis?

Yolk sack tumour

Seminoma

Testicular teratoma

Epididymo-orchitis

Adenomatoid tumour

A

This mans age, presenting symptoms and normal tumour markers make a seminoma the most likely diagnosis. Epididymo-orchitis does not produce irregular mass lesions which are painless.

3616
Q

What is the commonest type of testicular malgiancny

A

Seminoma

3617
Q

Testicular tumour

>40

AFP normal

HCG elevated in 10%

LDH elevated in 10-20%

A

?Seminoma

3618
Q

20-30y/o at presentation

AFP elevated in 70% of cases

HCG elevated in up to 40$

A

Teratoma or other nonseminomatous germ cell tumours

3619
Q

Testicular tumour

Sheet like lobular patterns of cells with substantial fibrous component. Fibrous septa contain lymphocytic inclusions and granulomas may be seen.

A

Seminoma

3620
Q

Testicular tumour

Heterogenous texture with occasional ectopic tissue such as hair

A

Teratoma

3621
Q

RFs for testciular cancer

A

Cryptorchidism

Infertility

FHx

Klinfelter’s

Mumps orchtitis

3622
Q

Key features to identify in cervical lymphadenopathy

A

Consistency

Number

Fixation

Symmetry

Tenderness

Face and scalp for infection or neoplasm

Chest exam: infection or neoplasm

Breast examinatoin

Formal full ENT examination

Rest of RES

3623
Q

Hx in cervical lymphadenoapthy

A

Symptoms form the lumps

e.g. EtOH induced pain

General symptoms

e.g. fever, malaise, weight loss

Systemic disease:

PMH, previous operations

Social Hx:

Ethnic origin, HIV RFs

3624
Q

Causes of cervical lymphadenopathy

LIST

A

Lymphoma and leukaemia

infection

Sarcoid

Tumours

3625
Q

Infective causes of cervical lymphadenopathy

A

Bacterial

Tonsilitis, dental abscess

TB

Bartonella henselae

Viral:

EBV

HIV

Protozoal

Toxoplasmosis

3626
Q

Ix in cervical lymphadenopathy

A

FBC, ESR, film (atypical lymhpocytes0
TFTs, serum ACE

Monospot test, HIV test

Radiological:

US

CT scan

Path

FNAC

Excision biopsy

3627
Q

Features of hypertrophic scars

A

Scar confined to wound margins

Across flexor surfaces and skin creases

Appear soon after injury and regress spontaneously

Ang age: commonly 8-20y

M=F

All races

3628
Q
A

Hypertrophic scar

3629
Q

Features of keloid scar

A

Scar extends beyond wound margins

Earlobes, chin, neck, shoulder, chest

Appear months after injury and continue to grow

Puberty to 30y

F>M

Black and hispanic

3630
Q
A

Keloid scar

3631
Q

Wound associations of hypertrophic and keloid scars

A

Infection

Trauma

Burns

Tension

Certain body areas

3632
Q

Mx of hypertrophic and keloid scars

A

Mechanical-pressure therapy

Topical silicone gel sheets

Intralesional steroid and LA injections

Revision of scar with closure by direct suturing

3633
Q

Embryology of the thyroid

A

Migrates from its origin at the foramen caecum at the base of the tonuge

Passes behind the hyoid bone

Lies anterior to 3rd and 4th tracheal rings in the pretracheal fascia

Leaves behind the thyroglossal cyst which atrophies

Ectopic thyroid tissue can be found anywhere aloing this descent

3634
Q

Congenital thyroid lesions

A

Lingual thyroid

Thyroglossal cyst

Ectopic thyroid tissue

3635
Q

DDx Diffuse goitre

A

Simple colloid goitre: endemic iodine deficiency, sporadic: autoimmune, hereditary, goitrogens e.g. sulphonylureas

Grave’s

Thyroditis: Hashimoto’s De Quervain’s, Subacute lymphocytic

3636
Q

Ddx multinodular goitre

A

Multinodular colloid goitre (commonest)

Multiple cysts

Multiple adenomas

3637
Q

Ddx in solitary thyroid nodule

A

Dominant nodule in multinodular goitre

Adenoma: hot or cold

Cyst

Malignancy

3638
Q

Ix in thyroid lumps

A

TFTs

FBC, Ca, LFTs, eSR

Antibodies: anti-TPO, TSH

Ix:

CXR

High resolution US

Radionucleotide Tc or I scan

Histology or cytology (FNAC can’t distinguish adneoma vs follicular Ca). Biopsy

Laryngoscopy:

Important pre-op to assess vocal cords

3639
Q

Indications for thyroid sx: 5Ms

A

Mechanical obstruction

Malignancy

Marred beauty

Medical Rx failure

Mediastinal extension

3640
Q

History in salivary gland lesions

A

Swellling/pain related to food: calculi

Malaise, fever, mumps

Dry eyes/mouth: Sjogrens (Sicca, Mickulicz)

3641
Q

Causes of whole salivary gland enlargement

A

Parotitis

Sjogrens

Sarcoid

Amyloid

ALL

CLD

Anorexia or bulimia

3642
Q

Causes of localised salivary gland enlargement

A

Tumours

Stones

3643
Q

Viral causes of acute parotitis

A

Mumps, Coxsackie A, HIV

3644
Q

Bacterial causes of acute parotitis

A

Staph aureus

Associated with calculi and poor oral hygiene

3645
Q

Features of salivary gland calculi

A

Recurrent unilateral swelling and pain

Worse on eating

Red, tender, swollen gland (80%) submandibular

3646
Q

Ix in ? salivary gland calculi

A

Plain XR or sialography

3647
Q

Rx in salivary gland calculi

A

Gland excision

3648
Q

What is the classic sign of salivary gland neoplasm

A

Deflection of ear outwards

3649
Q

Salivary gland neoplasm with CNVII palsy=

A

Malignancy

3650
Q

Demographics of salivary gland lesions

A

80% in parotid (80% superficial)

80% are pleomorphic adneomas

3651
Q

How can salivary gland neoplasms be classified

A

Benign

1st: pleomorphic adneoma
2nd: adenolymphoma (Warthin’s)

Malignant (CN7 palsy and fast growing)

1st: Mucoepidermoid
2nd: Adenoid cystic

3652
Q

Ix in salivary gland neoplasms

A

ENT exam

US +/- CT

FNAC

3653
Q

Features of pleiomorphic adenoma

A

Commonest salivary gland neoplasm (80%)

Presentation:

90% in parotid

Occur in middle age

F>M

Benign and slow growing

Histo: different tissue types

Rx: superficial parotidectomy

3654
Q

Warthin’s tumour

A

Benign soft cystic tumour

Older men

Rx: enucleation

3655
Q

Features of adenoid cystic carcinoma

A

One of the commonest malignant salivary tumours

Highly malignant and often incurable

Rapid growth

Hard, fixed mass

Pain

Facial palsy

3656
Q

Sx in salivary gland tumours

A

Superificial or radical parotidectomy

Facia lata face lift for facial palsy

3657
Q

Cx of salivary gland sx

A

Facial palsy

Salivary fistula

Frey’s syndrome

3658
Q

Frey’s syndrome

A

Gustatory sweating

Redness and sweating skin over parotid area

Occurs in relation to food (inc. thinking)

Auriculotemporal branch of CNV3 carrries sympathetic fibres to sweat glands over parotid area and parasympathetic fibres to the parotid

Reinervation of divided sympathetic nerves by fibres from the secretomotory branch or auriculotemporal branch of CN V3

3659
Q

Epidemiology of breast cancer

A

Affects 1/10 women

20,000 cases per year in the UK

Commonest cause of cancer death in females 15-54

Second commonest caues of cancer deaths overall

3660
Q

How can the aetiology/risk factors in breast Ca be categorised?

A

Familial

Oestrogen exposure

Other factors

3661
Q

FHx in breast Ca

A

10% Ca breast is familial

One first degree relative = 2x risk

5% assodiated with BRCA mutations

BRCA1-> 80% breast, 40% ovarian

BRCA2- 80% bresat

3662
Q

Oestrogen exposure in breast ca

A

Early menarche, late menopause

HRT, OCP

First child >35y/o

Obesity

3663
Q

Other factors in breast Ca risk

A

Proliferative breast disease with atypia

Previous Ca breast

Increased age (v. rare <30)

Breast feeding is protective

3664
Q

What is commonest breast malignancy?

A

Invasive ductal carcinoma

3665
Q

Features of DCIS/LCIS

A

Non-inavsive pre-malignant condition

Microcalcificaiton on mammography

10x increased risk of invasive Ca

3666
Q

Non-invasive pre-malignant condition

Microcalcification on mammography

10x increased risk of invasive Ca

A

DCIS/LCIS

3667
Q

Features of invasive ductal carcinoma, NST/NOS

A

Commonest (70%) of cancers

Feels hard (scirrhous)

3668
Q

20% of breast cancers are this histological subtype

A

Invasive lobular

3669
Q

This type of breast malignancy affects younger patients and may feel soft

A

Medullary

3670
Q

This type of breast cancer occurs in the elderly

A

Colloid/mucinous

3671
Q

Breast cancer

Pain

Erythema

Swelling

Peau d’orange

A

Inflammatory breast ca

3672
Q

This type of breast cancer is a stromal tumour presenting as a large, non-tender mobile lump

A

Phyllodes tumour

3673
Q

Spread of breast cancer

A

Direct extension-> muscle and or skin

Lymph-> p’eau d’oange and arm oedema

Blood:

Bones- bone pain, #, raised Ca

Lungs: dyspnoea, pleural effusion

Liver- abdo pain, hepatic impairment

Brain: headache, seizures

3674
Q
A

Inflammatory breast cancer with peau d’orange

3675
Q

Features of the breast cancer screening programme

A

50-70y/o women.

Every 3 years.

Being extended as a trial to some women aged 47-73

3676
Q

Impact of the breast cancer screening programme

A

Reduce breast Ca deaths by 25%

10% false negative rate

3677
Q

Presentation of breast Ca

A

Lump: commonest presentation

Usually painless

50% in UOQ

+/- axillary nodes

Skin changes:

Paget’s disease of skin: persistent eczema

Peau d’orange

Nipple:

Discharge

Inversion

Mets:

Pathological #s

SOB

Abdo pain

Seizures

May present through screening

3678
Q

Triple assessment of any breast lump

A

Hx and clinical examination

Radiology

<35y: US

>35y: US + mammography

Pathology

Solid lump- core biopsy

Cystic lump: FNAC (18g needle)

Reassure if clear fluid

Send cytology if bloody fluid

Core biopsy residual mass

Core biospy if +ve cytology

3679
Q

Additional Ix in ?breast ca

A

Bloods:

FBC, LFTs, ESR, bone profile

Imaging:

help staging

CXR

Liver USS

CT scan

Breast MRI

Bone scan and PET/CT

May need wire-guided excision biopsy

3680
Q

Clinical staging of breast Ca

A

Stage 1:

Confined to breast, mobile, no LNs

Stage 2:

Stage 1 and nodes in ipsilateral axilla

Stage 3:

Stage 2 + fixation to muscle but not chest wall. LNs matted and fixed and large skin involvement

Stage 4:

Complete fixation to chest wall + mets

3681
Q

TNM staging in breast cancer

A

Tis (no palpable tumour): CIS

T1 <2cm: no skin fixation

T2: 2-5cm, skin fixation

T3: 5-10cm, ulceration + pectoral fixation

T4: >10cm, chest wall extension, skin involved

N1: mobile nodes

N2: fixed nodes

3682
Q

Principles of Breast Ca management

A

Manage in MDT with individual approach

Oncologist

Breast sx

Breastcare nures

Radiologist

Histopathologist

Try to enroll in a trial

Factors: age, fitness, wishes, clinical stage

1-2: surgical

3-4: palliation

3683
Q

Aims of sx in breast cancer

A

To gain local control

3684
Q

What are the two surgical options in breast cancer

A

WLE- conservation surgery

Mastectomy- removal of whole breast

3685
Q
A

WLE + LN biopsy

3686
Q
A

Mastectomy

3687
Q

WLE vs mastectomy

A

Same survival but WLE has increased recurrence rates

3688
Q

Rationale for sentinal node biopsy

A

SN= first node that a section of breast drains to

If clinically -ve axillary LNs, no need for further dissection if SN is clear

3689
Q

Procedure in SN biopsy

A

Blue dye/ radiocolloid injected into tumour

Visual inspection/gamma probe at surgery to ID SN

SN removed and sent for frozen section

If node +ve-> axillary clearance or RTx

3690
Q

Evidence of SNB vs axillary clearance

A

2 RCTs compared

No differences in overall or disease-free surivival

Reduced morbidity with SNB alone:

lymphoedema

Pain

numbness

3691
Q

What are the other axillary options for clinically negative axilla

A

Axillary sampling:

Removal of lower nodes. Clearance or DXT if +ve

Axillary clearance:

Can be done to various levels

3692
Q
A

?Axillary clearance

3693
Q

Sx complications of breast cancer treatment

A

Haematoma, seroma

Frozen shoulder

LTN palsy

Lymphoedema

3694
Q

What can be used for prognosis in breast cancer?

A

Nottingham Prognostic Index

3695
Q

Features of Nottingham prognostic index

A

Predicts survival and risk of relapse

Guides appropriate adjuvant systemic therapy

0.2 x tumour size + histo grade + nodal status

3696
Q

What is used to grade breast tumours histologically?

A

Bloom-Richardson system

3697
Q

What are the systemic Rx used in breast cancer management?

A

RTx

CTx

Endocrine therapy

3698
Q

Indications for RTx in breast ca

A

Post-WLE: reduces local recurrence

Post-mastecomty: only if high risk of local recurrence. Large, poorly differentiated, node +ve

Axillary: node +ve disease

Palliation: bone pain

3699
Q

Indications for CTx in breast cancer

A

Pre-menopasual

Node

High grade

Recurrent tumours

3700
Q

CTx in breast Ca

6 x FEC

A

5-FU

Epirubicin

Cyclophosphamide

3701
Q

Trastuzumab

A

Anti-Her2

3702
Q

Trastuzumab SE

A

Cardiac toxicity

3703
Q

|ndications for endocrine therapy in breast Ca

A

Used in ER or PR +ve disease: reduces recurrence, increases survival

5y of adjuvant therapy

3704
Q

Tamoxifen MOA

A

SERM antagonist @ breast

Agonist at uterus

3705
Q

Se of tamoxifen

A

Menopausal symptoms

Endometrial Ca

3706
Q

Anastrazole MOA

A

Aromatse inhibitor-> reduced oestrogen

Better cf. tamoxifen if post-men (ATAC trial)

3707
Q

Se of anastrazole

A

Menopausal symptoms

3708
Q

Alternative endocrine rx to tamoxifen and anastrazole

A

If pre-menopausal and ER +ve may consider ovarian ablation or GnRH anlogues

3709
Q

Rx in treating advanced breast Ca

A

Tamoxifen if ER +ve

Chemo for relapse

Her 2 +ve tumours may respond to trastuzumab

Supportive

Bone pain: DXT, bisphosphonates, analgeisa

Brain: occassional surgery, DXT, steroids, AEDs

Lymphoedema: decongestion, compression

3710
Q

Features of breast reconstruction following mastectomy

A

Offered either at 1o sx or delayed procedure

Implants: silastic or saline inflatable

3711
Q

What are the myocutaneous flaps used in breast reconstruction

A

Latissimus dorsi myocutaneous flap (LDMF)

Transverse recturs abdominis myocutaneous flap (TRAM)

3712
Q

Features of lat dorsi myocutaneous flap

A

Pedicled flap: skin, fat, muscle and blood supply

Supplied by thoracodorsal A via subscapular A

Usually used with an implant

3713
Q

Features of transverse rectus abdominis myocutaneous flap

A

Gold standard

Pedicled (inf. epigastric A) or free: attached to internal thoracic artery

No implant necessary and combined tummy tuck

Risk of abdominal hernia

3714
Q

CI to transverse rectus abdominis myocutaneous flap

A

Poor circulation: smokers, obese, PVD, DM

3715
Q
A

Lat dorsi myocutaneous flap

Usually used with implant

3716
Q
A

Transverse rectus abdominis myocutaneous flap

3717
Q

Congenital breast disease

A

Amastia: complete absence of breast and nipple

Hypoplasia more common, some asymmetry normal

Accessory nipples: can occur anywhere along the milk line. Present in 1%

3718
Q

Gynaecomastia

A

Occurs in 30% of boys at puberty

Hormone secreting tumours e.g. sex-cord testicular

CLD: hypogonadism and reduced E2 metabolism

Drugs: sprionolactone, digoxin, cimetidine

3719
Q

<35

Breat pain

Pre-menstrual

Relieved by menstruation

Commonly in UOQ bilaterally

A

Cyclical mastalgia

3720
Q

Breast pain

>45y

Severe lancing breast pain (often left)

May be associated with back pain

A

Non-cyclical mastalgia

3721
Q

Mx of mastalgia

A

Conservative:

Reassurance, better fitting bra

Medical:

Topical diclofenac

Evening primrose oil

2nd line:

Danazol (anti-gonadotrophin) is licesnsed for severe pain

3722
Q

What are the inflammatory breast lesions

A

Acute mastitis

Fat necrosis

Duct ectasia (dilatation)

Periductal mastitis

3723
Q

What are the benign epithelial breast lesions

A

Benign mammary dysplasia

Cystic disease

Ductal papilloma

3724
Q

What are the stromal breast diseases

A

Fibroadenoma

Phyllodes tumour

3725
Q

Lactating

Painful, red breast

May-> abscess (lump near nipple)

A

Acute mastitis

3726
Q

Rx in acute mastitis

A

Fluclox alone in early stages

Fluclox and I+D if abscess

3727
Q

Breast lesion

Associated with previous trauma

Painless, palpable, non-mobile mass

May calcify simulating Cancer

A

Fat necrosis

3728
Q

Rx in breast fat necrosis

A

Analgesia

No F/U necessary

3729
Q

Post-menopausal (50-80y)

Slit-like nipple

Often bilateral

+/- peri-areolar mass

Thick white/green discahrge

May be calcified on mammography

A

Duct ectasia

3730
Q

Smoker

30y/o

Painful, erythematous, sub-areolar mass

Associated with inverted nipple +/- purulent discharge

May-> abscess or discharging fistula

A

Periductal mastitis

3731
Q

Rx in duct ectasia

A

Need to distinguish from Ca

Surgical duct excision if mass present or discharge troublesome

Close f/u

3732
Q

Rx in peri-ductal mastitis

A

Broad-spectrum abx

3733
Q

30-50y/o

Pre-menstural breast nodularity and pain often in upper outer quadrant

Tender, lumpy bumpy breasts

ANDI

A

Benign mammary dysplasia

Aberration of normal development and involution:

Fibroadenosis

Cyst formation

Epitheliosis

Papillomatosis

3734
Q

Peri-menopausal

Distinct, fluctuant round breast mass

Often painful

A

Cystic disease

3735
Q

40-50y/o

Boody breast discharge (common cause)

not often palpable

A

Duct papilloma

3736
Q

Rx in benign mammary dysplasia

A

Triple assessment

Reassurance

Analgesia

Good bra

+/- Evening primrose oil

Danazol

3737
Q

Rx in cystic breast disease

A

Aspiration: green-brown fluid

Persistence or blood-> triple assessment

3738
Q

Rx in duct papilloma

A

Triple assessment

Excise due to incresaed risk of Ca

3739
Q

<35y/o

Rare post-menopause

Commonest benign breast tumour

A

Fibroadenoma

3740
Q

Painless, mobile, rubbery breast mass.

Often multiple and bilateral

Popcorn calcification

A

Fibroadenoma

3741
Q

>50y/o

Large, fast growing breast mass

Mobile, non-tender

Epithelial and connective tissue elements

A

Pnyllodes tumour

3742
Q

Rx in phyllodes tumour

A

WLE

3743
Q

Presents as microcalcification on mammogram

Rarely associated with symptoms e.g.

lump, discharge, eczematous change

A

DCIS

3744
Q

Ca risk in DCIS

A

1%/y (10x risk) in ipsilateral breast

3745
Q

Rx in DCIS

A

WLE + RTx

Extensive or multifocal: mastectomy + reconstruction + SNB

3746
Q

Unilatreral scaly, erythematous, itchy breast skin

Persistent

+/- palpable mass

A

Paget’s

3747
Q

Incidental biopsy finding (no calcification)

Often bilateral (20-40%)

Young women

breast lesion

A

LCIS

3748
Q

Ca risk in LCIS

A

x10 in both breasts

3749
Q

Rx in LCIS

A

Bilateral prophylactic mastectomy or close watching with mammographic screening

3750
Q

Rx in paget’s disease of breast

A

usually underlying invasive or DCIS breast cancer

Mastectomy + radio +/- CTx

3751
Q

Level 1 evidence

A

Evidence obtained from systematic review of all relevant randomised controlled trials

3752
Q

Level II evidence

A

Evidence derived from at least one properly designed randomised controlled trial

3753
Q

Level III evidence

A

Evidence derived from well designed pseudo-randomised controlled trials (e.g. alternate allocation) or historical controls

3754
Q

Level IV evidence

A

Evidence derived from case series or case reports

3755
Q

Level V evidence

A

Panel or expert opinion

3756
Q

A 45-year-old lady recovering from a mastectomy and axillary node clearance notices that sensation in her armpit is impaired

A.Intercostobrachial

B.Median

C.Axillary

D.Radial

E.Ulnar

F.Musculocutaneous

G.Brachial plexus upper cord

H.Brachial plexus lower cord

A

The intercostobrachial nerves are frequently injured during axillary dissection. These nerves traverse the axilla and supply cutaneous sensation.

3757
Q

An 8-year-old boy falls onto an outstretched hand and sustains a supracondylar fracture. In addition to a weak radial pulse the child is noted to have loss of pronation of the affected hand.

A.Intercostobrachial

B.Median

C.Axillary

D.Radial

E.Ulnar

F.Musculocutaneous

G.Brachial plexus upper cord

H.Brachial plexus lower cord

A

This is a common injury in children. In this case the angulation and displacement have resulted in median nerve injury.

3758
Q

A 24-year-old man falls sustaining an inversion injury to his ankle. On examination he is tender over the lateral malleolus only. On x-ray there is a fibular fracture that is distal to the syndesmosis.

A.Surgical fixation

B.Below knee amputation

C.Aircast boot

D.Application of full leg plaster cast to include midfoot

E.Application of below knee plaster cast to include the midfoot

F.Application of external fixation device

G.Application of compression bandage and physiotherapy.

A

Application of below knee plaster cast to include the midfoot

Theme from 2008 Exam
These distal injuries are generally managed conservatively. Conservative management will involve a below knee cast, this will need to extend to the midfoot. It can be substituted for an aircast boot once radiological union is achieved.

3759
Q

An 86-year-old lady stumbles and falls whilst opening her front door. On examination her ankle is swollen with both medial and lateral tenderness. X rays demonstrate a fibular fracture at the level of the syndesmosis.

A.Surgical fixation

B.Below knee amputation

C.Aircast boot

D.Application of full leg plaster cast to include midfoot

E.Application of below knee plaster cast to include the midfoot

F.Application of external fixation device

G.Application of compression bandage and physiotherapy.

A

Although, this is a potentially unstable injury operative fixation in this age group generally gives poor results owing to poor quality bone. A below knee cast should be applied in the first instance. If this fails to provide adequate control it can be extended above the knee.

3760
Q

Maisonneuve fracture

A

Weber classification
Related to the level of the fibular fracture.

Type A is below the syndesmosis

Type B fractures start at the level of the tibial plafond and may extend proximally to involve the syndesmosis

Type C is above the syndesmosis which may itself be damaged

A subtype known as a Maisonneuve fracture may occur with spiral fibular fracture that leads to disruption of the syndesmosis with widening of the ankle joint, surgery is required.

3761
Q

A 56-year-old man presents with symptoms of nasal pain, anosmia and rhinorrhea. He has been well until recently and has worked as a wood carver for many years.

A.Ethmoid sinus cancer

B.Maxillary sinus cancer

C.Ethmoid adenoma

D.Maxillary adenoma

E.Ethmoidal fracture

F.Nasal polyps

G.Sphenoid osteoma

H.Ethmoidal sinusitis

I.Maxillary sinusitis

A

Theme from September 2012 Exam
Theme from September 2013 Exam
Nasopharyngeal cancer is strongly associated with wood work. Most cases require an occupational exposure of greater than 10 years and are adenocarcinomas on histology.
Most cases are ethmoidal in origin

3762
Q

With what products is the risk of TRALI greatest?

A

Plasma components

3763
Q

Def: massive haemorrhage

A

Loss of one blood volume in 24h or loss of 50% of circulating blood volume in 3 hours. Bloods loss of 150ml is also included

3764
Q

Normal adult blood volume

A

7% of total adult bodweight

5 litres

3765
Q

Cx of massive haemorrhage

A

Hypothermia

Hypocalcaemia

Hyperkalaemia

Delayed TRs

TRALI

Coaglopathy

3766
Q

Hypothermia in massive blood transfusion

A

Blood is refrigerated
Hypothermic blood impairs homeostasis
Shifts Bohr curve to the left

3767
Q

Hypocalcaemia in massive blood transfusion

A

Both FFP and plt contain citrate anticoagulant, this may chelate calcium

3768
Q

Hyperkalaemia following massive blood transfusion

A

Plasma of red cells stored for 4-5w contains 5-10mmol K+

3769
Q

What is the leading cause of transfusion related death?

A

TRALI

3770
Q

What causes lung injury in TRALI

A

Occurs as a result of leucocyte antibodies in transfused plasma

Aggregation and degranulation of leucocytes in lung tissue accounts for lung injury

3771
Q

Coagulopathy following massive blood transfusion

A

Anticipate once circulating blood transfused

1 blood volume usually drops plt to <100

1 bv will both dilute and not replace clotting factors

Fibrinogen concentration halves per 0.75 blood volume tranfused

3772
Q

A 70-year-old female is admitted with a history of passing brown coloured urine and abdominal distension. Clinically she has features of large bowel obstruction with central abdominal tenderness. She is maximally tender in the left iliac fossa. There is no evidence of haemodynamic instability. What is the most appropriate investigation?

Cystogram

Abdominal X-ray of the kidney, ureters and bladder

Computerised tomogram of the abdomen and pelvis

Flexible sigmoidoscopy

Barium enema

A

This lady is most likely to have a colovesical fistula complicating diverticular disease of the sigmoid colon. In addition she may also have developed a diverticular stricture resulting in large bowel obstruction. A locally advanced tumour of the sigmoid colon may produce a similar clinical picture. The best investigation of this acute surgical patient is an abdominal CT scan, this will demonstrate the site of the disease and also supply regional information such as organ involvement and other local complications such as a pericolic abscess. A barium enema would require formal bowel preparation and this is contra indicated where large bowel obstruction is suspected. A flexible sigmoidoscopy is unlikely to be helpful and the air insufflated at the time of endoscopy may make the colonic distension worse. A cystogram would provide only very limited information.

3773
Q

What can be used to describe perforations of the colon due to diverticulitis?

A

Hinchey classificaiton

3774
Q

Post-splenectomy blood film features

A

Howell- Jolly bodies
Pappenheimer bodies
Target cells
Irregular contracted erythrocytes

3775
Q

A 20-year-old lady is referred to the vascular clinic. She has been feeling generally unwell for the past six weeks. She works as a typist and has noticed increasing pain in her forearms whilst working. On examination she has absent upper limb pulses. Her ESR is measured and mildly elevated.

A.Wegeners granulomatosis

B.Polyarteritis nodosa

C.Giant cell arteritis

D.Takayasu’s arteritis

E.Buergers disease

A

Takayasus arteritis may be divided into acute systemic phases and the chronic pulseless phase. In the latter part of the disease process the patient may complain of symptoms such as upper limb claudication. In the later stages of the condition the vessels will typically show changes of intimal proliferation, together with band fibrosis of the intima and media.

3776
Q

Inflammatory, obliterative arteritis affecting aorta and branches

Females> Males

Symptoms may include upper limb claudication

Clinical findings include diminished or absent pulses

ESR often affected during the acute phase

A

Takyasu’s arteritis

3777
Q

Segmental thrombotic occlusions of the small and medium sized lower limb vessels

Commonest in young male smokers

Proximal pulses usually present, but pedal pulses are lost

An acuter hypercellular occlusive thrombus is often present

Tortuous corkscrew shaped collateral vessels may be seen on angiography

A

Buergers disease

3778
Q

Systemic granulomatous arteritis that usually affects large and medium sized vessels

Females > Males

Temporal arteritis is commonest type

Granulomatous lesions may be seen on biopsy (although up to 50% are normal)

A

Giant cell arteritis

3779
Q

Systemic necrotising vasculitis affecting small and medium sized muscular arteries

Most common in populations with high prevalence of hepatitis B

Renal disease is seen in 70% cases

Angiography may show saccular or fusiform aneurysms and arterial stenoses

A

Polyarteritis nodosa

3780
Q

Predominantly affects small and medium sized arteries

Systemic necrotising granulomatous vasculitis

Cutaneous vascular lesions may be seen (ulceration, nodules and purpura)

Sinus imaging may show mucosal thickening and air fluid levels

A

Wegeners granulomatosis

3781
Q

What are the classical features of a Colles #?

A

FOOSH

  1. Transverse fracture of the radius
  2. 1 inch proximal to the radio-carpal joint
  3. Dorsal displacement and angulation

Colles’ fracture (dinner fork deformity)

3782
Q

Volar angulation of distal radius fragment (Garden spade deformity)

Caused by falling backwards onto the palm of an outstretched hand or falling with wrists flexed

A

Smith’s fracture (reverse Colles’ fracture)

3783
Q

Intra-articular fracture of the first carpometacarpal joint

Impact on flexed metacarpal, caused by fist fights

X-ray: triangular fragment at ulnar base of metacarpal

A

Bennet’s fracture

3784
Q
A

Bennet’s Fracture

3785
Q

Dislocation of the proximal radioulnar joint in association with an ulna fracture

Fall on outstretched hand with forced pronation

Needs prompt diagnosis to avoid disability

A

Monteggia’s fracture

3786
Q

Radial shaft fracture with associated dislocation of the distal radioulnar joint

Direct blow

A

Galeazzi fracture

3787
Q

Bimalleolar ankle fracture

Forced foot eversion

A

Pott’s fracture

3788
Q

Distal radius fracture (Colles’/Smith’s) with associated radiocarpal dislocation

Fall onto extended and pronated wrist

Submit answer

A

Barton’s fracture

3789
Q

What BP is required to generate a palpable femoral pulse?

A

>65mmHg

3790
Q

Mg homeostasis

A

Magnesium is required for both PTH secretion and its action on target tissues. Hypomagnesaemia may both cause hypocalcaemia and render patients unresponsive to treatment with calcium and vitamin D supplementation.

Magnesium is the fourth most abundant cation in the body. The body contains 1000mmol, with half contained in bone and the remainder in muscle, soft tissues and extracellular fluid. There is no one specific hormonal control of magnesium and various hormones including PTH and aldosterone affect the renal handling of magnesium.

Magnesium and calcium interact at a cellular level also and as a result decreased magnesium will tend to affect the permeability of cellular membranes to calcium, resulting in hyperexcitability.

Submit answer

3791
Q

A.Fissure in ano

B.Intersphincteric abscess

C.Haemorroidal disease

D.Proctitis

E.Solitary rectal ulcer syndrome

F.Rectal cancer

G.Anal cancer

An 18-year-old man with a previous history of constipation presents with bright red rectal bleeding and diarrhoea. He has suffered episodes of faecal incontinence, which have occurred randomly throughout the day and night.

A

Nocturnal diarrhoea and incontinence are typical of inflammatory bowel disease.

3792
Q

A.Fissure in ano

B.Intersphincteric abscess

C.Haemorroidal disease

D.Proctitis

E.Solitary rectal ulcer syndrome

F.Rectal cancer

G.Anal cancer

A 56-year-old man presents with episodes of pruritus ani and bright red rectal bleeding. On examination there is a mass in the ano rectal region and biopsies confirm squamous cell cancer.

A

These are features of anal cancer. Anal cancers arise from the cutaneous epithelium and are therefore typically squamous cell. They are usually sensitive to chemoradiotherapy.

3793
Q

A 19-year-old man presents with bright red rectal bleeding. He has a longstanding history of irritable bowel syndrome. At flexible sigmoidoscopy a lesion is biopsied and reported as showing ‘fibromuscular obliteration’.

A.Fissure in ano

B.Intersphincteric abscess

C.Haemorroidal disease

D.Proctitis

E.Solitary rectal ulcer syndrome

F.Rectal cancer

G.Anal cancer

A

This is the typical presentation of SRUS. These patients require careful diagnostic work up to elicit the underlying cause of their altered bowel habit. The histological appearances of solitary rectal ulcers are characteristic and extensive collagenous deposits are often seen. This is usually termed fibromuscular obliteration.

3794
Q

A 42-year-old teacher is admitted with a fall. An x-ray confirms a fracture of the surgical neck of the humerus. Which nerve is at risk?

A.Median nerve

B.Ulnar nerve

C.Radial nerve

D.Posterior interosseous nerve

E.Anterior interosseous nerve

F.Musculocutaneous nerve

G.Axillary nerve

H.Brachial Trunks C5-6

I.Brachial trunks C6-7

J.Brachial Trunks C8-T1

A

Axillary nerve

The Axillary nerve winds around the bone at the neck of the humerus. The axillary nerve is also at risk during shoulder dislocation.

3795
Q

A 32-year-old window cleaner is admitted after falling off the roof. He reports that he had slipped off the top of the roof and was able to cling onto the gutter for a few seconds. The patient has Horner’s syndrome.

A.Median nerve

B.Ulnar nerve

C.Radial nerve

D.Posterior interosseous nerve

E.Anterior interosseous nerve

F.Musculocutaneous nerve

G.Axillary nerve

H.Brachial Trunks C5-6

I.Brachial trunks C6-7

J.Brachial Trunks C8-T1

A

The patient has a Klumpke’s paralysis involving brachial trunks C8-T1. Classically there is weakness of the hand intrinsic muscles. Involvement of T1 may cause a Horner’s syndrome. It occurs as a result of traction injuries or during delivery.

3796
Q

A 32-year-old rugby player is hit hard on the shoulder during a rough tackle. Clinically his arm is hanging loose on the side. It is pronated and medially rotated.

A.Median nerve

B.Ulnar nerve

C.Radial nerve

D.Posterior interosseous nerve

E.Anterior interosseous nerve

F.Musculocutaneous nerve

G.Axillary nerve

H.Brachial Trunks C5-6

I.Brachial trunks C6-7

J.Brachial Trunks C8-T1

A

The patient has an Erb’s palsy involving brachial trunks C5-6.

3797
Q

A 30-year-old women is involved in a road traffic accident. She is a passenger in a car involved in a head on collision with another vehicle. Her car is travelling at 60mph. She has been haemodynamically stable throughout with only minimal tachycardia. On examination she has marked abdominal tenderness and a large amount of intra abdominal fluid on CT scan

A.Tension pneumothorax

B.Haemopericardium

C.Haemothorax

D.Aortic transection

E.Ruptured spleen

F.Duodeno-jejunal flexure disruption

G.Aorto iliac disruption

H.Recto-sigmoid junction disruption

A

This is another site of sudden deceleration injury. Given the large amount of free fluid, if it were blood, then a greater degree of haemodynamic instability would be expected.

3798
Q

A 30-year-old women is involved in a road traffic accident. She is a passenger in a car involved in a head on collision with another vehicle. Her car is travelling at 60mph. She has been haemodynamically stable throughout with only minimal tachycardia. On examination she has marked abdominal tenderness and a large amount of intra abdominal fluid on CT scan

A.Tension pneumothorax

B.Haemopericardium

C.Haemothorax

D.Aortic transection

E.Ruptured spleen

F.Duodeno-jejunal flexure disruption

G.Aorto iliac disruption

H.Recto-sigmoid junction disruption

A

Duodeno-jejunal flexure disruption.

This is another site of sudden deceleration injury. Given the large amount of free fluid, if it were blood, then a greater degree of haemodynamic instability would be expected.

3799
Q

Acanthosis nigricans involving the mucous membranes

A

Brown to black, poorly defined, velvety hyperpigmentation of the skin.

Usually found in body folds such as the posterior and lateral folds of the neck, the axilla, groin, umbilicus, forehead, and other areas.

The most common cause of acanthosis nigricans is insulin resistance, which leads to increased circulating insulin levels. Insulin spillover into the skin results in its abnormal increase in growth (hyperplasia of the skin).

In the context of a malignant disease, acanthosis nigricans is a paraneoplastic syndrome and is then commonly referred to as acanthosis nigricans maligna. Involvement of mucous membranes is rare and suggests a coexisting malignant condition

3800
Q

A.Giardia Infection

B.Cryptosporidium infection

C.Clonorchis sinensis infection

D.Ancyclostoma duodenale infection

E.Ascaris lumbricoides infection

F.Echinococcus granulosus infection

G.Enterobius vermicularis infection

A 6-year-old boy presents with symptoms of recurrent pruritus ani. On examination there is evidence of a small worm like structure protruding from the anus.

A

nfection with enterobius is extremely common. Pruritus is the main symptom, as there is a lack of tissue invasion it is rare for individuals to have any signs of systemic sepsis.

3801
Q

A 25-year-old man returns from a backpacking holiday in India. He presents with symptoms of coughing and also of episodic abdominal discomfort. Peri anal examination is normal. Stool microscopy demonstrates both worms and eggs within the faeces.

A.Giardia Infection

B.Cryptosporidium infection

C.Clonorchis sinensis infection

D.Ancyclostoma duodenale infection

E.Ascaris lumbricoides infection

F.Echinococcus granulosus infection

G.Enterobius vermicularis infection

A

Infection with Ascaris lumbricoides usually occurs after individuals have visited places like sub Saharan Africa or the far east. Unlike ancyclostoma duodenale infection there is usually evidence of both worms and eggs in the stool. The absence of pruritus makes enterobius less likely. The presence of coughing may be due to the migration of the larva through the lungs.

3802
Q

Hookworms that anchor in proximal small bowel

Most infections are asymptomatic although may cause iron deficiency anaemia

Larvae may be found in stools left at ambient temperature, otherwise infection is difficult to diagnose

Infection occurs as a result of cutaneous penetration, migrates to lungs, coughed up and then swallowed

Treatment is with mebendazole

A

Ancylostoma duodenale

3803
Q

Due to organism Enterobius vermicularis

Common cause of pruritus ani

Diagnosis usually made by placing scotch tape at the anus, this will trap eggs that can then be viewed microscopically

Treatment is with mebendazole

A

Enterobiasis

3804
Q

Due to infection with roundworm Ascaris lumbricoides

Infections begin in gut following ingestion, then penetrate duodenal wall to migrate to lungs, coughed up and swallowed, cycle begins again

Diagnosis is made by identification of worm or eggs within faeces

Treatment is with mebendazole

A

Ascariasis

3805
Q

Due to infection with Strongyloides stercoralis

Rare in west

Organism is a nematode living in duodenum of host

Initial infection is via skin penetration. They then migrate to lungs and are coughed up and swallowed. Then mature in small bowel are excreted and cycle begins again

An auto infective cycle is also recognised where larvae will penetrate colonic wall

Individuals may be asymptomatic, although they may also have respiratory disease and skin lesions

Diagnosis is usually made by stool microscopy

In the UK mebendazole is used for treatment

A

Strongyloidiasis

3806
Q

Protozoal infection

Organisms produce cysts which are excreted and thereby cause new infections

Symptoms consist of diarrhoea and cramping abdominal pains. Symptoms are worse in immunosuppressed people

Cysts may be identified in stools

Treatment is with metronidazole

A

Cryptosporidium

3807
Q

Diarrhoeal infection caused by protozoa

Infections occur as a result of ingestion of cysts

Symptoms are usually gastrointestinal with abdominal pain, bloating and passage of soft or loose stools

Diagnosis is by serology or stool microscopy

First line treatment is with metronidazole

A

Giardiasis

3808
Q

Classification of causes of urinary tract obstruction

A

Luminal

Mural

Extramural

3809
Q

Luminal causes of urinary tract obstruction

A

Stones

Blood clots

Sloughed papilla

3810
Q

Mural causes of urinary tract obstruction

A

Congenital/acquired stricture

Tumour: renal, ureteric, bladder

Neuromuscular dysfunction

3811
Q

Extramural causes of urinary tract obstruction

A

Prostatic enlargement

Abdo/pelvic mass/tumour

Retroperitoneal fibrosis

3812
Q

Presentation of acute upper urinary tract obstruction

A

Loin pain-> groin

3813
Q

Presentation of acute lower urinary tract obstruction

A

Bladder outflow obstruction precedes severe suprapubic pain with distended palpable bladder

3814
Q

Presentation of chronic upper urinary tract obstruction

A

Flank pain

Renal pain (may be polyuric)

3815
Q

Presentation of chronic lower urinary tract obstruction

A

Frequency, hesitancy, poor stream, terminal dribbling, overflow incontinence

Distended, palpable bladder +/- large prostate PR

3816
Q

Ix of urinary tract obstruction

A

Bloods: FBC, U+E

Urine: dip, MC+S

Imaging: USS- hydronephrosis/hydrouretur

Anterograde/retrograde ureterograms (allow therapeutic drainage)

Radionucleotide imaging: renal function

CT/MRI

3817
Q

Mx of upper urinary tract obstruction

A

Nephrostomy

Ureteric stent

3818
Q

Mx of lower urinary tract obstruction

A

Urethral or suprapubic catheter- may be a large post-obstructive diuresis

3819
Q

Cx of ureteric stents

A

Common:

Infection

Haematuria

Trigonal irritation

Encrustation

Rare:

Obstruction

Ureteric rupture

Stent migration

3820
Q

Causes of urethral stricture

A

Trauma: instrumentation, pelvic fractures

Infection e.g. gonorrhoea

CTx

Balanatitis xerotica obliterans

3821
Q

Presentation of urethral stricture

A

Voiding difficulty

Hesitancy

Strangury

Poor stream

Terminal dribbling

Pis en deux

3822
Q

Pis en deux

A

Urinary urgency shortly after voiding

3823
Q

Examination in urethral stricture

A

PR: exclude prostatic cause

Palpate urethra through penis

Examine meatus

3824
Q

Ix of urethral stricture

A

Urodynamcs: reduced peak flow, increased micturition time

Urethroscopy and cystoscopy

Retrograde urethrogram

3825
Q

Mx of urethral stricture

A

Internal urethrotomy

Dilatation

Stent

3826
Q

Pathogenesis of obstructive uropathy

A

Acute retention on a chronic background may go unnoticed for days due to lack of pain

Se Cr may be up to 1500uM

Renal function should return to normal over days although some background impairment may remain

3827
Q

Cx of obstructive uropathy

A

Hyperkalaemia

Metabolic acidosis

Post-obstructive diuresis

Na and HCO3 losing nephropathy

Infection

3828
Q

Post-obstructive diuresis

A

Kidneys produce a lot of urine in the acute phase after relief of obstruction

Must keep up with losses to avoid dehydration

3829
Q

Na and HCO3 losing nephropathy post obstruction

A

Diuresis-> loss of Na and HCO3

May require replacement with 1.26% NaHCO3

3830
Q

Classification of causes of urinary retention

A

Obstructive

Neurological

Myogenic

3831
Q

Mechanical causes of urinary retention

A

BPH

Urethral stricture

Clots, stones

Constipation

3832
Q

Dynamic causes of obstructive urinary retention

A

Increased smooth muscle tone (alpha adrenergic)

Post-op pain

Drugs

3833
Q

Neurological causes of urinary retention

A

Interruption of sensory or motor innervation

pelvic sx, ms, dm, spinal injury/compression

3834
Q

Myogenic causes of urinary retention

A

Over-distension of the bladder

Post-anaesthesia

High EtOH intake

3835
Q

Suprapubic tenderness

Palpable bladder: dull to percussion, can’t get beneath it

Large prostate on PR (check anal tone and sacral sensation)

<1l drained on catheterisation

A

Acute urinary retention

3836
Q

Ix in acute urinary retention

A

Blood: FBC, U+E, PSA (prior to PR)

Urine: dip, MC+S

Imaging: US- bladder volume, hydropnephrosis

Pelvic XR

3837
Q

Mx of acute urinary retention

A

Conservative:

analgesia, privacy, walking, running water or hot bath

Catheterise:

Use correct catheter (e.g. 3 way if clots) +/- STAT gent cover

Hourly UO + replace: post-obstruction diuresis

Tamsulosin: redcued risk of recatheterisation after retention

TWOC after 24-72h: can be d/c and have OPD f/U. More likely to be successful if predisposing factor and lower residual volume

3838
Q

TURP in acute urinary retention

A

Failed TWOC

Impaired renal function

Elective

3839
Q

How can chronic urinary retention be classified?

A

High pressure

Low pressure

3840
Q

Features of high pressure chronic urinary retention

A

High detrusor pressure @ end of micturition

Typically bladder outflow obstruction-> bilateral hydronpehrosis and impaired renal function

3841
Q

Features of low pressure chronic urinary retention

A

Low detrusor pressure @ end of micturition

Large volume retention with very compliant bladder

Kidney able to excrete urine

No hydronephrosis therefore normal renal function

3842
Q

Insidiuous presentation as bladder capacity increases

Typically painless

Overflow incontinence/ nocturnal enuresis

Acute on chronic picture possible

Lower abdo mass

UTI

Renal failure

A

?Chronic urinary retention

3843
Q

Mx of high pressure urinary retention

A

Catheterise if: renal impairment, pain, infection

Hourly UO and replace post-obstruction diuresis

Consider TURP before TWOC

3844
Q

Mx of low pressure chronic urinary retention

A

Avoid catheterisation if possible due to risk of introducing infection

Early TURP: often do poorly due to poor detrusor function.

Need CISC or permanent catheter

3845
Q

Advantages of suprapubic catheterisation

A

Reduceed UTIs

Reduced stricture formation

TWOC w/o catheter removal

Pt preference: increased comfort

Maintain sexual function

3846
Q

Disadvantages of suprapubic catheterisation

A

More complex: need skills

Serious cx can occur

3847
Q

CI to suprapubic catheterisation

A

Known or suspected bladder carcinoma

Undiagnosed haematuria

Previous lower abdominal surgery: adhesions of small bowel to abdominal wall

3848
Q

Features of CISC

A

Clean intermittent self-catheterisation

Alternative to indwelling catheter in AUR and CUR

Also useful in pts who fail to void after TURP

3849
Q

Causes of false haematuria

A

Beetroot

Rifampicin

Porphyria

PV bleed

3850
Q

How can the causes of true haematuria be classified

A

General

Renal

Uretur

Bladder

Prostate

Urethra

3851
Q

General causes of true haematuria

A

HSP

Bleeding diathesis

3852
Q

Renal causes of true haematuria

A

Infarct

Trauma inc. stones

Infection

Neoplasm

GN

Polycystic kidneys

3853
Q

Uretur causes of true haematuria

A

Stone

Tumour

3854
Q

Bladder causes of true haematuria

A

Infection

Stones

Tumour

Exercise

3855
Q

Prostate causes of true haematuria

A

BPH

Prostatitis

Tumour

3856
Q

Urethral causes of true haematuria

A

Infection

Stones

Trauma

Tumour

3857
Q

Haematuria at beginning of stream

A

Urethral

3858
Q

Haematuria throughout stream

A

Renal/systemic, bladder

3859
Q

Haematuria at end of stream

A

Bladder stone

Schistosomiasis

3860
Q

Clinical features to determine in haematuria

A

Timing

Painful or painless

Obstructive symptoms

Systemic symptoms

3861
Q

Ix in haematuria

A

Bloods: FBC, U+E. clotting

Urine: dip, MC+S, cytology

Imaging:

Renal US

IVU

Flexible cystoscopy and biopsy

CT/MRI

Renal angiography

3862
Q

Aetiology of peri-aorititis

A

Idiopathic retroperitoneal fibrosis

Inflammatory AAAs

Perianeurysmal RPF

RPF 2o to malignancy e.g. lymphoma

3863
Q

Idiopathic retroperitoneal fibrosis pathophysiology

A

Autoimmune vasculitis

Fibrinoid necrosis of vasa vasorum

Affects aorta and small/medium sized retroperitoneal vessels

Ureters are embedded in dense, fibrous tissue-> bilateral obstruction

3864
Q

Associations of idiopathic retroperitoneal fibrosis

A

Drugs: beta blockers, bromocriptine, methysergide, methyldopa

AI disease: thyroiditis, SLE, ANCA+ vasculitis

Smoking

Asbestos

3865
Q

Middle aged male

Vague loin, back or abdo pain

Raised BP

Chronic urinary tract obstruction

A

?Para-aortitis

3866
Q

Ix in idiopathic retroperitoneal fibrosis

A

Blood: raised U and Cr, raised ESR/CRP, reduced Hb

US: bialteral hydronephrosis + medial ureteric deviation

CT/MRI: peri-aortic mass

Biopsy to exclude cancer

3867
Q

Rx in peri-aortitis

A

Relieve obstruction: retrograde stent placement

Ureterolysis: dissection of uretur from retroperitoneal tissue

+/- immunosuppression

3868
Q

Epidemiology of urolithiasis

A

Lifetime incidence: 15%

Young men: peak age 20-40y. Sex M>F 3:1

3869
Q

Pathophysiology of urolithiasis

A

Increased concentration of urinary solute

Reduced urine volume

Urinary stasis

3870
Q

What are the common anatomical sites of urolithiasis

A

Pelviureteric junction

Crossing the iliac vessels at the pelvic brim

Under the vas or uterine artery

Vesicoureteric junction

3871
Q

What is the most common type of renal stone?

A

Calcium oxalate

3872
Q

Struvite stones associated with

A

Proteus infection (staghorn calculi)

3873
Q

Types of renal stone

A

Calcium oxalate: 75%

Struvite: 15%

Urate: 5% (radiolucent)

Cystine: 1% (faint)

3874
Q

Factors associated with urolithiasis

A

Dehydration

Hypercalcaemia: 1o HPT, immobilisation

Increased oxalate excretion: tea, strawberries

UTIs

Hyperuricaemia: gout

Urinary tract abnromalities e.g. bladder diverticulae

Drugs: frusemide, thiazides

3875
Q

Ureteric colic=

A

Severe loin pain radiating to the groin

Associated with n/v

Patient cannot lie still

3876
Q

Severe loin pain radiating to goin

Bladder irritability: frequency, dysuria, haematuria

Strangury: painful urinary tenesmus

Suprapubic pain radiating to tip of penis or in labia

Pain and haematuria worse at the end of micturition

Other possible features:

UTI

Haematuria

Sterile pyruira

Anuria

Examination:

usually no loin tenderness

Haematuria

A

?Urolithiasis

3877
Q

Ix in urolithiasis

A

Urine: dip- haematuria. MC+S

Blood: FBC, U+E, Ca, PO4, urate

Imaging:

KUB XR

USS- hydronephrosis

CT KUB: gold standard

IVU

Functional scans

3878
Q

CT KUB stone visualisation

A

99% of stones visualised

3879
Q

Featuers of IVU

A

600x radiation dose of KUB

IV contrast injected and control, immediate and serial films taken until taken at level of obstruction

Abnormal findings:

Failure of flow to the bladder

Standing column of contrast

Clubbing of the calcyces

Delayed, dense nephrogran- no flow from kidney

3880
Q

CI to IVU

A

Contrast allergy

Severe asthma

Meformin

Pregnancy

3881
Q

Different types of functional kidney scan

A

DMSA: dimeracptosuriccinic acid

DTPA: diethylenetriamene penta-acetic acid

MAG-3

3882
Q

Prevention of renal stones

A

Drink plenty

Treat UTI rapidly

Reduce oxalate intake: chocolate, tea, strawberries

3883
Q

Initial Rx in urolithiasis

A

Analgesia:

diclofenac 75mg PO/IM or 100mg PR

Opioids if NSAIDs CI e.g. pethidine

Fluids: IV if unable to tolerate PO

Abx if infection e.g. cefuroxime 1.5mg IV TDS

3884
Q

Conservative Mx of renal stones

A

<5mm in lower 1/3 of uretur

90-95% pass spontaneously

Can discharge patient with analgesia

Sieve urine to collect stone for OPD analysis

3885
Q

Indications for medical expulsive therapy in urolithiasis

A

Stone 5-10mm

Stone expected to pass

3886
Q

Drugs used in medical expulsive therapy in urolithiasis

A

CCB or alpha antag: nifedipine or tamsulosin +/- pred

Most pass within 48h, 80% within 30d

3887
Q

Indications for active stone removal in urolithiasis

A

Low likelihood of spontaneous passage e.g. >10mm

Pesistent obstuciton

Renal insufficeincy

Infection

3888
Q

What are the options for active stone removal in urolithiasis

A

Extracorporeal shockwave lithotripsy

Ureterorenoscopy + Dormier basket removal

Percutaneous nephrolithotomy

Lap or Open surgery

3889
Q

What determines options for stone removal

A

Location and size of stone

3890
Q

Stone in renal pelvis

>20mm

Option

A

PNL or URS

2nd line: SWL

3891
Q

Stone in renal pelvis

<20mm

A

SWL

2nd line: URS, PNL

3892
Q

Stone in proximal uretur

>10mm

A

URS or SWL

3893
Q

Stone in proximal uretur

<10mm

A

SWL

3894
Q

Stone in distal uretur

>10mm

A

URS

2nd line: SWL

3895
Q

Stone in distal uretur

<10mm

A

URS or SWL

3896
Q

Indications for extracorporeal shockwave lithotripsy

A

Stones <20mm in kidney or proximal uretur

3897
Q

SE of SWL

A

Renal injury may-> raised BP

3898
Q

CI to SWL

A

Pregnancy

AAA

Bleeding diathesis

3899
Q

indications for ureterorenoscopy and dormier basket removal

A

Stone >10mm in distal uretur or if SWL failed

Stone >20mm in renal pelvis

3900
Q

Indications for percutaneous nephrolithotomy

A

Stone >20mm in renal pelvis

e.g. staghorn calculus: do DMSA first

3901
Q

Febrile with renal obstruction

A

Surgical emergency

Percutaneous nephrostomy or ureteric stent

IVAbx: cefurime, 1.5g IV TDS

3902
Q

Epidemiology of RCC

A

90% of renal cancer

55y/o

Sex: M>F 2:1

3903
Q

RFs for RCC

A

Obesity

Smoking

HTN

Dialysis (15% of patients develop RCC)

4% heritable e.g. VHL syndrome

3904
Q

Pathology of renal cell carcinoma

A

Adenocarcinoma from proximal renal tubular epithleium

Subtypes:

Clear cell (glycogen): 70-80%

Papillary: 15%

Chromophobe: 5%

Collecting duct 1%

3905
Q

Most common presentation of RCC

A

50% incidental finding

3906
Q

Triad in renal cell carcinoma

A

Haematuria, loin pain, loin mass

3907
Q

50% incidental finding

Triad: haematuria, loin pain, loin mass

Systemic: anorexia, malaise, weight loss, PUO

Clot retention
invasion of L renal vein-> varicocele

Cannonball mets-> SOB

A

RCC

3908
Q

What are the paraneoplastic features of RCC

A

EPO-> polycythaemia

PTHrP-> hypercalcaemia

Renin-> HTN

ACTH-> Cushing’s syndrome

Amyloidosis

Deranged LFTs

3909
Q

Spread of RCC

A

Direct: renal vein

Lymphatic

Haematogenous: bone, liver and lung

3910
Q

Ix in RCC

A

Blood: polycythaemia, ESR, U+E, ALP, Ca

Urine: dip, cytology

Imaging:

CXR- cannonball mets

US: mass

IVU: filling defect

CT/MRI

3911
Q

Robson staging of RCC

A

Confined to kidney

Involves perinephric fat but not Garota’s fascia

Spread into renal vein

Spread to adjacent/distant organs

3912
Q

Mx of RCC

A

Medical:

reserved for patients with poor px

Temsirolimus (mTOR inhibitor)

Surgical:

Radical nephrectomy

Consider partial if small tumour or 1 kidney

3913
Q

Epidemiology of TCC

A

2nd commonest renal cancer

50-80y

M>F: 4:1

3914
Q

RFs for TCC Renal

A

Smoking

Amine exposure (rubber industry)

Cyclophosphamide

3915
Q

Pathology of TCC

A

Highly malignant

Locations: bladder (50%), uretur, renal pelvis

3916
Q

Presentation of TCC

A

Painless haematuria

Frequency, urgency, dysuria

Urinary tract obstruction

3917
Q

Ix in TCC renal

A

Urine cytology

CT/MRI

IVU: pelviceal filling defect

3918
Q

Mx of renal TCC

A

Nephouretectomy

Regular f/up: 50% develop bladder tumours

3919
Q

Childhood tumour of primitive renal tubules and mesenchymal cells

May be associated with Chr 11 mutation

May be associated with WAGR syndrome

A

Nephroblastoma

3920
Q

WAGR syndrome

A

Wilms

Aniridia

GU abnormaltities

Retardation

3921
Q

2-5y/o

5-10% bilateral

Abdominal mass that doesn’t cross the midline

haematuria

Abdo pain

HTN

A

?Wilm’s

3922
Q

Benign neoplasms of kidney

A

Cysts: very common

Renal papillary adenomas

Oncocytoma: eosinophilic cells with numerous mitochondria

Angiomyolipoma: seen in tubeorus sclersosis

3923
Q

Additional malignant renal neoplasms

A

SCC associated with chronic infected staghorn calculi

3924
Q

NB re: renal benign tumours

A

Commonly require nephrectomy to exclude malignancy

3925
Q

Epidemiology of bladder tumours

A

1:5000/y

M>F 4:1

3926
Q

Pathology of bladder tumours

A

TCC account for 90%

SCCs: associated with schistosomiasis

Adenocarcinoma

3927
Q

Natural hx of bladder tumours

A

Low-grade:

80%

Non-invasive, generally not life-threatening

High recurrence rate

High grade:

20%

Invasive and life-threatening

High recurrence rates

3928
Q

RFs for bladder tumours

A

Smoking

Amine exposure (rubber industry)

Previous renal TCC

Chronic cystitis

Schistosomiasis (SCC)

Urechral remnants (adenocarcinomas): embryological remnants of communication between umbilicus and bladder

Pelvic irradiation

3929
Q

Painless haematuria

Voiding irritability: dysuria, frequency, urgency

Recurrent UTIs

Retention and obstructive renal failure

Anaemia

Palpable bladder mass

Palpable liver

A

?bladder tumour

3930
Q

TNM staging of Bladder tumours

A

80% confined to mucosa

20% penetrate muscle (increased moratlity)

Tis: carcinoma in situ, not fetlt at EUA

Ta: confined to epithelium, not felt at EUA

T1: tumour in lamina propria: not felt

T2: superficial muscle involved: rubbery thickness

T3: deep muscle involved: mobile mass

T4: invasion of prostate, uterus, vag: fixed mass

3931
Q

Spread of bladder tumours

A

Local-> pelvic structures

Lymph-> iliac and para-aortic nodes

Haem-> bones, liver and lungs

3932
Q

Histological classification of bladder tumours

A

Grade 1: well differentiated

Grade 2: intermediate

Grade 3: poorly differentiated

3933
Q

Ix in bladder tumours

A

Urine: dip (sterile pyuria), cytology

IVU: filling defects

Cystoscopy with biopsy- diagnostic

Bimanual EUA helps to assess spread

CT/MRI helps stage

3934
Q

Mx of bladder tumours

A

Depends on histological grade and the presence of dissemination

3935
Q

Mx of bladder cancer:

Tis, Ta, T1

A

80% of all patients

diathermy via transurethral cystoscopy/TURBT

Intravesicular chemo: mitomycin C

Intravesicular immunotherapy: BCG

3936
Q

Mx of bladder cancer:

T2, T3 (invasive)

A

Radical cystectomy with ileal conduit is gold standard

RTx: worse 5ys but preserves bladder: salvage cystectomy can be performed

Adjuvant chemo: M-VAC

Neoadjuvant chemo may have a role

3937
Q

Mx of bladder cancer:

T4

A

Palliate CT/RTx

Long-term catheterisation

Urinary diversions

3938
Q

Cx of bladder cancer treatment

A

Massive haemorrhage

Cystectomy-> sexual and urinary malfunction

3939
Q

F/U of bladder TCC

A

Up to 70% of bladder tumours recur, therefore intensive F/U is required

Hx, ex and regular cystoscopy

High risk tumours: every 3m for 2y then ever 6m

Low risk: @9m then yearly

3940
Q

Px in bladder cancer

A

Depends on age and stage

Non invasive: 95% 5ys

T2: 40-50%

T3: 25%

T4 <1y

3941
Q

Epidemiology of BPH

A

70% @60y

90% @80y

3942
Q

Pathophysiology of BPH

A

Benign nodular or diffuse hyperplasia of stromal and epithelial cells

Affects inner (transitional) layer of prostate (cf. Ca)-> urethral compression

DHT produced from testosteroe in stromal cells by 5-a reductase enzyme

DHT-induced GFs-> raised stromal cells and reduced epithelial cell death

3943
Q

Presentation of BPH

categorisation

A

Storage symptoms:

nocturia, frequency, urgency, overflow incontinence

Voiding symptoms:

hesitancy, straining, poor stream/flow + terminal dribbling, strangury, incomplete emptying

Bladder stones: 2o to stasis

UTI: 2o to satasis

3944
Q

Examination in BPH

A

PR: smoothly enlarged prostate

Definable median sulcus

Bladder not usually palpable unless acute-on-chronic obstruction

3945
Q

Ix in BPH

A

Bloods: U+E, PSA
Urine: dip, MC+S

Imaging: transrectal US +/- biopsy

Urodynamics: pressure/flow cystometry

Voiding diary

3946
Q

Conservative Mx of BPH

A

Reduce caffeine,

EtOH

Double voiding

Bladder training: hold on-> increase time between boiding

3947
Q

Medical mx of BPH

A

Useful in mild disease while awaiting TURp

1st line: alpha blockers: tamsulosin, doxazosin

2nd line: 5alpha reductase inhibitors: finasterid

3948
Q

Use of alpha blockers in BPH

A

Tamsulosin, doxazosin

Relax prostate smooth muscle

SE: drowsiness, reduced BP, depression, ED, weight gain, extra-pyramidal signs

3949
Q

Use of 5 alpha reductase inhibitors in BPH

A

Finasteride

Inhibit conversion of testosterone to DHT

Preferred if significantly enlarged prostate

SE: secreted in semen, ED

3950
Q

Indications for surgical mx of BPH

A

`Symptoms affect QoL

Cx of BPH

3951
Q

What are the surgical options for BPH

A

TURP

TUIP

Transurethral electrovaporisation of prostate

Laser prostatectomy

Open retropubic prostatectomy

3952
Q

Features of TURP

A

Cystoscopic resection of lateral and middle lobes

<14% become impotent

3953
Q

TUIP

A

Transurethral incision of prostate (TUIP)

less destrction than TURP so less risk to sexual function

Similar benefits if small prostate (<30g)

3954
Q

Features of laser prostatectomy

A

Reduced ED and retrograde ejaculation

Similar efficacy as TURP

3955
Q

Use of open retropubic prostatectomy

A

Used for very large prostates >100g

3956
Q

What are the immediate cx of TURP

A

TUR syndrome: absorption of large quantity of fluids-> hyponatraemia

Haemorrhage

3957
Q

What are the early cx of TURP

A

Haemorrhage

Infection

Clot retention: requires bladder irrigation

3958
Q

What are the late cx of TURP

A

Retrograde ejaculation: common

ED: 10%

Incontinence: 10%

Urethral stricture

Recurrence

3959
Q

What is the epidemiology of prostate cancer

A

Commonest male cancer

3rd commonest cause of male cancer death

80% of men >80y

Increased in blacks

3960
Q

Pathology of prostate cancer

A

Adenocarcinoma

Peripheral zone of prostate

3961
Q

Usually asymptomatic

Urinary: nocturia, frequency, hesitancy, poor stream, terminal dribbling, obstruction

Systemic: weight loss, fatigue

Mets: bone pain

A

Prostate Ca

3962
Q

Examination findings in prostate ca

A

Hard irregular prostate on PR

Loss of midline sulcus

3963
Q

Spread of prostate cancer

A

Local: seminal vesicles, bladder, rectum

lymph: para-aortic nodes

Haemo: sclerotic bony lesions

3964
Q

Ix in prostate cancer

A

Bloods: PSA, U+E, acid and ALP, Ca

Imaging: XR chest and spine

transrectal US + biopsy

Bone scan

Staging MRI: contrast enhancing magnetic nanoparticles increases detection of affected nodes

3965
Q

Features of PSA

A

Proteloytic enzyme used in liquefaction of ejaculate

Not specific for prostate Ca: increases with age, PR, TURP and prostatitis

>4ngml: 40-90% sensitivie, 60-90% specific.

Only 1 in 3 will have cancer

Normal in 30% of small cancers

3966
Q

What can be used to grade prostate cancer

A

Gleason Grade

Score 2 worst affected areas

Sum is inversely proportional to px

3967
Q

TNM staging of prostate Ca

A

Tis: carcinoma in situ

T1: incidental finding on TURP or raised PSA

T2: intracapsular tumour with deformation of prostate

T3: extraprostatic extension

T4: fixed to pelvis + invading neighbouring structures

N1-4: 1 or more LNs involved

M1: distant mets e.g. spine

3968
Q

Prognostic factors in prostate cancer

A

Help determine whether to pursue radical Rx

Age

Pre-Rx PSA

tumour stage

Tumour grade

3969
Q

Issues with mx of prostate cancer

A

Difficult to know which tumours are indolent and will not lead to mortality before something else

Radical therapy associated with significant morbidity

3970
Q

Conservative mx of prostate cancer

A

Close monitoring with DRE and PSA

3971
Q

Radical mx of prostate ca

A

Radical prostatectomy (+ gosrelin if node +ve)

Performed laparoscopically with robot

Only improves survival vs active monitoring if <75y

Brachytherapy: implantation of palladium seeds

3972
Q

SEs of radical therapy for prostate cancer

A

ED, urinary incontinence, death

3973
Q

Use of radical prostatectomy in prostate cancer

A

Performed laparoscopically

Only improves survival vs active monitoring if <75y

3974
Q

Medical mx of prostate cancer

A

Used for metastatic or node +ve diseae

LHRH analgoues e.g. gosrelin: inhibits pituitay gonadotrophins-> reduce testosterone

Anti-andrognes: e.g. cyporterone actetate, flutamide

3975
Q

Symptomatic mx of prostate cancer

A

TURP for obstruction

Analgesia

RTx for bone mets/cord compression

3976
Q

PSA and screening

A

Population based screening not recommended in UK

PSA not an accurate tumour marker

ERSPC trial showed small mortality benefit, PLCO trial showed no benefit

Must balance mortality benefit with harm caused by over diagnosis and over treatment of indolent cancers

3977
Q

Aetiology of prostatitis

A

S. faecalis

E. coli

Chlamydia

3978
Q

Usually >35y

UTI/dysuria

Pain: low backache, pain on ejaculation

Haematospermia

Fever and rigors

Retention

Malaise

Pyrexial

Swollen/boggy/tender prostate

Examine testes to exclude epididymo-orchitits

A

Prostatitis

3979
Q

Ix in prostatitis

A

Blood: FBC, U+E, CRP

Urine dip: MC+S

3980
Q

Rx in prostatitis

A

Fluoroquinolone for 4 w

Analgesia

e.g. levofloxacin 500mg/d for 28d

3981
Q

Features of male urinary incontinence

A

Usually caused by prostatic enlargement

Urge incontinence or dribbling may result from partial retention

Retention may lead to overflow (palpable bladder after voiding)

TURP and pelvic surgery may weaken external urethral sphincter

3982
Q

Urinary incontinence in women

A

Stress:

Leakage from incompetent sphincter when raiesd IAP

Loss of small amounts of urine when coughing

Pelvic floor weakness

Urge/OAB

can’t hold urine

May have precipitatnt

Dx: urodynamic studies

3983
Q

Mx of urinary incontinence

A

Check

PR: faecal impaction

Palpable bladder after voiding: retention with overflow

DM

CNS: MS, parkinson’s, stroke, spinal trauma

Diuretics

Stress:

Pelvic floor exercises

Ring pessary

Duloxetine

Surgery: tension free vaginal tape

Urge:

Bladder training

Weight loss

Anti-AChM: tolterodine, imipramine

3984
Q

Epidemiology of undescended testes

A

3% at birth

1% at 1y

Unilateral 4x commoner cf. bilateral: should have genetic testing if bilateral

Commoner in prems: incidence up to 30%

3985
Q

Normal descent of testes

A

Testes remain in abdomen (retroperitoneal) until 7m

Gubernaculum connects inferior pole of testis to scrotum

Testes descend through inguinal canal to scrotum with an out-pouching of peritoneum: processus vaginalis

3986
Q

Def: cryptorchidism

A

Complete absence of testis from scrotum

Anorchism= absence of both testes

3987
Q

Def: retractile testis

A

Normal development but excessive cremasteric reflex

Testicle often found at external inguinal ring

Will descend, no Rx required

3988
Q

Def: maldescended testis

A

Found anywhere along normal path of descent

Testis and scrotum are usually under-developed

Often associated with patent processus vaginalis

3989
Q

Def: ectopic testis

A

Found outside line of descent

Usually in superior inguinal pouch (anterior to external oblique aponeurosis)

Abdominal, perineal, penile, femoral triangle

3990
Q

Cx of undescended testes

A

Infertility

10x risk of malignancy (remains after surgery)

Increased risk of trauma

Increased risk of torsion

Associated with hernias (90%) or UT abnormalities

3991
Q

Reasons for intervening in undescended testes

A

Restores potential for spermatogensis

Make ca surveillance easier

3992
Q

Surgical mx of undescended testes

A

Orchidopexy by Dartos Pouch Procedure

Perform before 2y

Mobilisation of testis and cord

Removal of patent processus

testicle brought through a hole made in the dartos muscle to lie in a subcutaneous pouch

Dartos prevents retraction

3993
Q

Hormonal mx of undescended testis

A

Beta-HCG may be tried if testis is in inguinal canal

3994
Q

Aetiology of testicular torsion

A

Usually 2o to some exertion or minor trauma

Occurs because testicles don’t have a large bare area to attach to scrotal wall:

Tunica vaginalis invests whole of testicle

Free hanging clapper-bell testicle can twist on its mesentry

3995
Q

Usually 10-25y/o

Sudden onset severe pain in one testis

May have lower abdo pain (testis supplied by T10)

Associated with n+v

May be hx of previous testicular pain

A

?Testicular torsion

3996
Q

Inflammation of one testis: hot, swollen, extremely tender

Testis rides high and lie transversely

A

Testicular torsion

3997
Q

Ddx in ?testicular torsion

A

Epididymo-orchtiis: older patient, UTI symptoms, more gardual onset

Torted Hydatid of Morgagni: remnant of mullerian duct, younger patient, less pain. Tiny ble dot visible on scrotum

tumour

Trauma

Strangulated hernia

Appendicitis

3998
Q

Ix in torsion

A

Doppler US may demonstrate absence of flow

must not delay surgical exploration

3999
Q

Mx of ?torsion

A

Surgical emergency: 4-6h window from onset of pain to salvage testis

Inform senior

NBM

IV access: analgesia, bloods: FBC, U+E, G+S, clotting

Sx: consent for possible orchidectomy, bilateral orchidopexy: suture testes to scrotum

if no torsion found and epidydiom-orchtitis dx- take fluid for bacteriology and rx with abx

4000
Q

Groin/scrotal lump:

Can’t get above

A

Inguinoscrotal hernia

4001
Q

Groin/scrotal lump:

Separate, cystic

A

Epididymal cyst

4002
Q

Groin/scrotal lump:

separate, solid

A

Varicocele

Sperm granuloma

Epidydimitis

4003
Q

Groin/scrotal lump:

Testicular, cystic

A

hydrocele

4004
Q

Groin/scrotal lump:

testicular, solid

A

Tumour

Orchitis

haematocele

4005
Q

Develop in adulthood

Contiain clear or milky fluid

Lie above and behind testis

Remove if symptomatic

A

Epididymal cyst

4006
Q

Feel like bag of worms in scrotum

may be visible dilated veins

reduced size on lying down

Patient may complain of dull ache

May lead to oligospermia-> reduced fertility

A

Varicocele (dilated veins of pampiniform plexus)

4007
Q

Pathology of testicular varicocele

A

1o: left side commoner, drain into LRV
2o: left renal tumour has tracked down renal vein-> testicular vein obstruction

4008
Q

Mx of testicular varicocele

A

Conservative: scrotal support

Surgical: clipping the testicular vein (open or lap)

4009
Q

Painful lump of extravasated sperm after vasectomy

A

Sperm granuloma

4010
Q

Collection of serous fluid within tunica vaginalis

Can be primary: associated with patent processus vaginalis, commoner, larger, tense, younger men

Can be secondary: tumour, trauma, infection, smaller, less tense

A

Hydrocele

4011
Q

Ix in testicular hydrocele

A

US testicle to exclude tumour

4012
Q

Mx of testicular hydrocele

A

May resolve spontaneously:

surgery- Lord’s repair: plication of the sac

Jaboulay’s repair: eversion of the sac

Aspiration: usually recur so not 1st line

Send fluid for cytology and MC+S

4013
Q

Blood in tunica vaginalis

Hx of trauma

May need drainage or incision

A

haematocele

4014
Q

Aetiology of epididymo-orchitis

A

STI: chlamydia, gonorrhoea

Ascending UTI: E. Coli

Mumps

4015
Q

Sudden onset tender testicular swelling

Dysuria

Sweats, fever

Tender, red, warm swollen testis and epidydimitis

Prehn’s sign positive

Secondary hydrocele

urethral discharge

A

Epididymo-orchitis

4016
Q

Prehn’s sign

A

Positive Prehn’s sign indicates there is pain relief with lifting the affected testicle, which points towards epididymitis

4017
Q

Ix in epididymo-orchitis

A

Blood: FBC, CRP

Urine: dip, MC+S (first catch)

Urethral swab and STI screen

US: may be needed to exclude abscess

4018
Q

Cx of epididymo orchitis

A

May-> infertility

4019
Q

Mx of epididymo-orchitis

A

Bed rest

Analgesia

Scrotal support

Abx: doxy or cipro

Drain abscess if present

4020
Q

Epidemiology of testicular tumours

A

Commonest male malignancies from 15-44y

Whites > blacks 5:1

4021
Q

Painless testicular lump- often noticed after trauma

Haematospermia

2o hydrocele

Mets: SOB from lung mets

Abdo mass: para-aortic lymphadenopathy

Hormones: gynaecomastia, virilisation
Contralateral tumour in 5%

A

Testicular tumours

4022
Q

RFs for testicular tumours

A

Undescended testis: 10% occur in this group

Infant hernia

Infertility

4023
Q

How can the different types of testicular tumour be classified?

A

Germ Cell

Sex-cord Stromal

lymphoma/leukaemia

4024
Q

What classification of testicular tumour is the most common?

A

Germ cell (95%)

4025
Q

What are the subtypes of germ cell testicular tumours

A

Pure Seminomas (40%)

Non-seminomas (inc. mixed): (60%)

Mixed

Teratoma

Yolk sac

Choriocarcinoma

4026
Q

Commonest single subtype of testicular tumour

30-40

Raised beta HCG in 15%

Raised ALP in some

Very radiosensitive

A

Seminoma

4027
Q

Commonest NSGCT

A

Mixed

4028
Q

Testicular tumour

Arise from all 3 germ layers

Common and benign in children

Rare and malignant in adults: 15-30y

Secrete beta HCG and or AFP

Chemosensitive

A

Teratoma

4029
Q

Teratoma: secretes Two

A

Beta HCG

and or AFP

4030
Q

Commonest testicular tumour in children

A

Yolk sac

4031
Q

Testicular tumour

Raised +++ BetaHCG

A

Choriocarcinoma

4032
Q

What are the sex cord stromal subclassifications of testicular tumours

A

Leydig cell

Sertoli cell

Both mostly benign

4033
Q

Leydig cell tumours may secrete

A

Androgens or oestrogens

4034
Q

Sertoli cell tumours may secrete

A

Oestrogens

4035
Q

What is the commonest malignant testicular mass >60y

A

NHL

4036
Q

What is the commonest malignant testicular mass <5

A

ALL

4037
Q

What can be used to stage testicular tumours?

A

Royal Marsden Staging

4038
Q

Royal Marsden Staging of Testicular tumours

A
  1. Disease only in testis
  2. Para-aortic nodes involved (below diaphragm)
  3. Supra and infra-diaphragmatic LNs involved
  4. Extra-lymphatic spread: lungs, liver
4039
Q

Ix in testicular tumours

A

Tuour markers: useful for monitoring

Raised AFP and hCG in 90% of teratomes

Raised hCG in 15% of seminomas

Normal AFP in pure seminomas

Scrotum US

Staging: CXR, CT

NB: percutaneous needle biopsy should not be performed due to risk of seeding along the needle tract

4040
Q

Mx of testicular tumours: both testes involved

A

Semen can be cryopreserved

4041
Q

Mx of stage 1-2 seminoma

A

Inguinal orchidectomy + RTx

Groin incision allows cord clamping to prevent seeding

4042
Q

Mx of stage 3-4 seminoma

A

Inguinal orchidectomy + RTx + chemo (BEP)

Bleomycin, etoposide, cisPlatin

4043
Q

Mx of stage 1 testicular non-seminoma

A

Inguinal orchidecctomy and surveillance

4044
Q

Mx of stage 2 non-seminoma

A

Orchidectomy + chemo + para-aortic LN dissection

4045
Q

Mx of stage 3 non-seminomatous testicular tumour

A

Orchidectomy + chemo

4046
Q

F/U in testicular tumours

A

Typically within 18-24m

Repeat CT scanning and tumour markers

4047
Q

Px in testicular tumours

A

Stage 1: 98% 5ys

Stage 2: 85%

Stage 4: 60%

4048
Q

Def: balanatitis

A

Acute inflammation of foreskin and glans

4049
Q

Cause of balantitis

A

Strep, staph, candida

4050
Q

RFs of balantitis

A

DM

Young children with tight foreskin

4051
Q

Rx of balantitis

A

Hygiene advice

Abx

Circumcision

4052
Q

Phimosis

A

Foreskin occludes the meatus

4053
Q

Child

Recurrent balantitis and ballooning

A

Phimosis

4054
Q

mx of childhood phimosis

A

Gentle retraction

Steroid creams

Circumcision

4055
Q

Adult presentation of phimosis

A

Dyspareunia, infection

4056
Q

Mx of adult phimosis

A

Circumcision

4057
Q

Balantitis xerotica obliterans

A

Thickening of foreskin and glans-> phimosis + meatal narrowing

4058
Q

Pathology of paraphimosis

A

Tight foreskin is retracted and becomes irreplaceable

Reduced venous return-> oedema and swelling of the glans

Can rarely lead to ischaemia

4059
Q

Causes of paraphimosis

A

Catheterisation, masturbation, intercourse

4060
Q

Mx of paraphimosis

A

Manual reduction: use ice and lignocaine jelly

May require glans aspiration or dorsal slit

4061
Q

Hypospadia

A

Developmental abnormality of the position of the urethral opening

Opens on ventral surface of penis

4062
Q

Epispadia

A

Developmental abnormality of the position of the urethral opening

Opens on the dorsal surface

4063
Q

Epidemiology of penile cancer

A

1:100,000/y in UK

commoner in far east and africa

4064
Q

Aetiology of penile cancer

A

V. rare if circumcised

RFs: HPV, chronic irritation

4065
Q

Pathology of penile cancer

A

Erythroplasia of Querat-> Penile CIS

SCC

4066
Q

Chronic fungating penile ulcer

Bloody/purulent discharge

Inguinal nodes in 50% at presentation

A

Penile cancer

4067
Q

Mx of penile cancer

A

Medical: early growths with no urethral invovlement: DXT and iridium wires

Surgical:

Amputation required if urethral involvement

LN dissection

4068
Q

A 24-year-old man presents with localised spinal pain over 2 months which is worsened on movement. He is known to be an IVDU. He has no history suggestive of tuberculosis. The pain is now excruciating at rest and not improving with analgesia. He has a temperature of 39 oC.

A

In an IVDU with back pain and pyrexia have a high suspicion for osteomylelitis. The most likely organism is staph aureus and the cervical spine is the most common region affected. TB tends to affect the thoracic spine and in other causes of osteomyelitis the lumbar spine is affected.

4069
Q

A 56-year-old lady has undergone a Hartman’s procedure for diverticulitis. 6 months post operatively she complains of painless passage of blood stained mucous per rectum.

A

Rectal diversion may result in proctitis.

4070
Q

Bright red rectal bleeding

Painful bleeding that occurs post defecation in small volumes. Usually antecedent features of constipation

Muco-epithelial defect usually in the midline posteriorly (anterior fissures more likely to be due to underlying disease)

A

Fissure in ano

4071
Q

Bright red or mixed blood

Bleeding that is accompanied by other symptoms such as altered bowel habit, malaise, history of fissures (especially anterior) and abscesses.

Perineal inspection may show fissures or fistulae. Proctoscopy may demonstrate indurated mucosa and possibly strictures. Skip lesions may be noted at colonoscopy.

A

Crohns disease

4072
Q

Bright red bleeding often mixed with stool

Diarrhoea, weight loss, nocturnal incontinence, passage of mucous PR.

Proctitis is the most marked finding. Peri anal disease is usually absent. Colonoscopy will show continuous mucosal lesion.

A

Ulcerative colitis

4073
Q

Bright red blood mixed volumes

Alteration of bowel habit. Tenesmus may be present. Symptoms of metastatic disease.

Usually obvious mucosal abnormality. Lesion may be fixed or mobile depending upon disease extent. Surrounding mucosa often normal, although polyps may be present.

A

Rectal cancer

4074
Q

A 35-year-old woman has undergone a wide local excision. The histology shows an invasive lobular carcinoma present at 3 of the resection margins. Cavity shavings taken at the original operation are also involved. Sentinel node biopsy was negative.

A

This patient has an extensive disease process and lobular cancers are notorious for being multifocal. In this case a mastectomy is the safest next step.

4075
Q

A 28-year-old female presents with a painless lump in the upper outer quadrant of her left breast. Imaging using ultrasound is indeterminate (U3). Two core biopsies have now been performed and both show normal breast tissue (B1

A

The imaging and biopsy results are not concordant. At this stage an excision biopsy is the safest option

4076
Q

A cyclist loses control and falls off the side of a road landing on the bicycle handlebars. CT scanning shows a large amount of retroperitoneal air.

A

Retroperitoneal air is more likely with a duodenal injury. As it is largely retroperitoneal. A handlebar type injury is the commonest cause and the pancreas should be carefully inspected as it too may be injured. It would be unusual for the ileum to be injured in this type of scenario as it is mobile.

4077
Q

Retroperitoneal air is more likely with a duodenal injury. As it is largely retroperitoneal. A handlebar type injury is the commonest cause and the pancreas should be carefully inspected as it too may be injured. It would be unusual for the ileum to be injured in this type of scenario as it is mobile.

A
  1. Small bowel injury is the most common type of injury in this scenario. The enteric contents will tend to result in a large amount of intra abdominal fluid.
4078
Q

Unusual injuries. In blunt trauma most injuries occur within 4cm of the carina. Features suggesting this injury include haemoptysis and surgical emphysema. These injuries have a very large air leak and may have tension pneumothorax.

A

Tracheobronchial tree injury

4079
Q

Post-trauma

Usually cardiac arrhythmias, often overlying sternal fracture. Perform echocardiography to exclude pericardial effusions and tamponade. Risk of arrhythmias falls after 24 hours

A

Cardiac contusion

4080
Q

Typically, these patients have a viral prodrome followed by anaemia, often with haemoglobin concentrations falling below 5.0 g/dL and reticulocytosis.

A

Transient aplastic crisis in SCD following parvovirus B19 infection

4081
Q

A 44-year-old man attends for counselling with regards to a vasectomy. Which one of the following statements is true regarding vasectomy?

Vasectomy is effective immediately

Female sterilisation is more effective

Two negative semen samples should be obtained at 2 and 4 weeks before other contraceptive methods are stopped

Chronic testicular pain is seen in more than 5% of patients

Sexual intercourse should be avoided for one month to reduce the chance of a sperm granuloma

A

Chronic testicular pain is seen in more than 5% of patients

4082
Q

Features of male sterilisation

A

Failure rate: 1/2000

Simple operation, can be done under LA, go home after a couple of hours

Doesn’t work immediately.

Requires semen analysis 2 weeks following procedure before a man can hvae unprotected sex

4083
Q

Cx of vasectomy

A

Bruising

Haematoma

Infection

Sperm ganuloma

Chronic testicular pain (affects 5-30% of men)

4084
Q

Female sterilisation features

A

1/200 fail

Usually done laparoscopically under GA

Day case

Many different techniques

4085
Q

Cx of female sterilisation

A

Increased risk of ectopic if sterilisation fails

General Cx of GA/lap

4086
Q

Wallace’s rule of 9s for burns

A

head + neck = 9%

Each arm= 9%

Each anterior part of leg= 9%

Each posterior part of leg= 9%

Ant chest= 9%

Post chest= 9%

Ant abdo and post abdo= 9% each

4087
Q

What is the most accurate method for assessing the extent of a burn

A

Lund and Browder chart

4088
Q

How to calculate fluid replacement in burns?

A

Parkland formula:

volume= total body surface area of the burn x weight (kg) x 4

Half of the fluid is administered in the first 8h

4089
Q

A 53-year-old woman presents with a bloody nipple discharge. On mammography there is calcification behind the nipple areolar complex. A core biopsy shows background benign change, but cells that show comedo necrosis which have not breached the basement membrane.

A

Comedo necrosis is a feature of high nuclear grade ductal carcinoma in situ. It is has a high risk of being associated with foci of invasion.

4090
Q

A 74-year-old woman presents with a breast lump. On examination is has a soft consistency. The lump is removed and sliced apart. Macroscopically there is a grey, gelatinous surface

A

Mucinous carcinomas comprise 2-3% of all breast cancers. They are one of the special type of carcinomas. These have a better prognosis that tumours of Non Special Type (NST) and axillary nodal disease is rare in this group

4091
Q

A 74-year-old woman presents with an erythematous rash originating in the nipple. It is spreading to the surrounding areolar area and the associated normal tissue.

A

Paget’s is associated with DCIS or invasive carcinoma.Unlike eczema of the nipple which predominantly affects the areolar region, Pagets will usually affect the nipple first and then spread to the areolar area. Diagnosis is made by punch biopsy.

4092
Q

A 28-year-old Indian woman, who is 18 weeks pregnant, presents with increasing shortness of breath, chest pain and coughing clear sputum. She is apyrexial, blood pressure is 140/80 mmHg, heart rate 130 bpm and saturations 94% on 15L oxygen. On examination there is a mid diastolic murmur, there are bibasal crepitations and mild pedal oedema. She suddenly deteriorates and has a respiratory arrest. Her chest x-ray shows a whiteout of both of her lungs.

A

Mitral stenosis is the commonest cause of cardiac abnormality occurring in pregnant women. Mitral stenosis is becoming less common in the UK population, however should be considered in women from countries were there is a higher incidence of rheumatic heart disease. Mitral stenosis causes a mid diastolic murmur which may be difficult to auscultate unless the patient is placed into the left lateral position. These patients are at risk of atrial fibrillation (up to 40%), which can also contribute to rapid decompensation. Physiological changes in pregnancy may cause an otherwise asymptomatic patient to suddenly deteriorate. Balloon valvuloplasty is the treatment of choice.

4093
Q

A 28-year-old woman, who is 30 weeks pregnant, presents with sudden onset chest pain associated with loss of consciousness. Her blood pressure is 170/90 mmHg, saturations on 15L oxygen 93%, heart rate 120 bpm and she is apyrexial. On examination there is an early diastolic murmur, occasional bibasal creptitations and mild peal oedema. An ECG shows ST elevation in leads II, III and aVF.

A

Aortic dissection is associated with the 3rd trimester of pregnancy, connective tissue disorders (Marfan’s, Ehlers- Danlos) and bicuspid valve. Patients may complain of a tearing chest pain or syncope. Clinically they may be hypertensive. The right coronary artery may become involved in the dissection, causing myocardial infarct in up to 2% cases (hence ST elevation in the inferior leads). An aortic regurgitant murmur may be auscultated.

4094
Q

A 73-year-old lifelong heavy smoker presents to the vascular clinic with symptoms of foot ulceration and rest pain. On examination her foot has areas of gangrene and pulses are impalpable.

A

0.3

This is critical limb ischaemia. Values of 0.3 are typical in this setting and urgent further imaging is needed. Debridement of necrosis prior to improving arterial inflow carries a high risk of limb loss.

4095
Q

A 63-year-old man presents with a claudication distance of 15 yards. He is a lifelong heavy smoker. On examination his foot is hyperaemic and there is a small ulcer at the tip of his great toe.

A

0.5

Hyperaemia may occur in association with severe vascular disease and is referred to surgically as a “sunset foot”. ABPI is usually higher than 0.3, but seldom greater than 0.5. Especially when associated with hyperaemic changes and ulceration. Urgent further imaging and risk factor modification is needed.

4096
Q

A 77-year-old morbidly obese man with type 2 diabetes presents with leg pain at rest. His symptoms are worst at night and sometimes improve during the day. He has no areas of ulceration.

A

>1.2

Type 2 diabetes may have vessel calcification. This will result in abnormally high ABPI readings. Pain of this nature in diabetics is usually neuropathic and if a duplex scan is normal then treatment with an agent such as duloxetine is sometimes helpful.

4097
Q

A 38-year-old lady presents with symptoms of obstructed defecation that date back to the birth of her second child by use of ventouse. She passes mucous and suffers from pelvic pain. Digital rectal examination and barium enema are normal.

A

Rectal intussceception

Rectal intussceception (internal rectal prolapse) typically presents with symptoms of obstructed defecation. The pathology is best demonstrated by a defecating procotogram rather than barium enema.

4098
Q

Triad of features in TUR syndrome

A

Hyponatraemia: dilutional

Fluid overload

Glycine toxicity

Mx: fluid restriction and the treatment of cx associated with the hyponatraemia

4099
Q

A 4-year-old presents with sudden onset of dysphagia. He undergoes an upper GI endoscopy and a large bolus of food is identified in the mid oesophagus. He has no significant history, other than a tracheo-oesophageal fistula repair soon after birth.

A

The correct answer is Benign oesophageal stricture

Children with tracheo-oesophageal fistulas will commonly develop oesophageal strictures following repair. These may require regular dilations throughout childhood.

4100
Q

CVP reading that excludes pulmoanry oedema?

A

CVP 18mmHg

4101
Q

Stages of ARDS

A

Early: exudative phase of injury with associated oedema

Late stage: repair and consists of fibroproliferative changes

Subsequent scarring may rseult in poor lung function

4102
Q

Features of lung recruitment manoeuvres

A

Transient increases in transpulmonary pressure designed to open up collapsed airless alveoli

Primarily used in ARDS

Multiple methods have been described

40cmH20 for 40-60 seconds

3 consecutive sighs/min with a plateau pressure of 45cmH2O

2 minutes of peak pressure of 50cmH2O and PEEP above upper inflection point (obese/trauma patients may require >60-70cmH2O)

long slow increase in inspiratory pressure up to 40 cmH2O (RAMP)

stepped increase in pressure (e.g. Staircase Recruitment Manoeuvre)

Proning can also be considered a recruitment manoeuvre, and other recruitment manoeuvres are more more effective in the prone position

4103
Q

A 34-year-old man is suffering from septic shock and receives and infusion of Dextran 70. Which of the following complications may potentially ensue?

Anaphylaxis

Vomiting

Acute hepatic failure

Digital necrosis

Deep vein thrombosis

A

Dextran 40 and 70 have higher incidence of anaphylaxis than either gelatins or starches.

Dextrans are branched polysaccharide molecules. Dextran 40 and 70 are available. The higher molecular weight dextran 70 may persist for up to 8 hours. They inhibit platelet aggregation and leucocyte plugging in the microcirculation. Thereby improving flow through the microcirculation, primarily of use in sepsis.
Unlike many other intravenous fluids Dextrans are a recognised cause of anaphylaxis.

4104
Q

A 63-year-old lady undergoes an axillary clearance for breast cancer. She makes steady progress. However, 8 weeks post operatively she still suffers from severe shoulder pain. On examination she has reduced active movements in all planes and loss of passive external rotation.

A

Adhesive capsulitis

Frozen shoulder passes through an initial painful stage followed by a period of joint stiffness. With physiotherapy the problem will usually resolve although it may take up to 2 years to do so.

4105
Q

A 78-year-old man complains of a long history of shoulder pain and more recently weakness. On examination active attempts at abduction are impaired. Passive movements are normal.

A

Rotator cuff tear

Rotator cuff tears are common in elderly people and may occur following minor trauma or as a result of long standing impingement. Tears greater than 2cm should generally be repaired surgically.

4106
Q

A 28-year-old man complains of pain and weakness in the shoulder. He has recently been unwell with glandular fever from which he is fully recovered. On examination there is some evidence of muscle wasting and a degree of winging of the scapula. Power during active movements is impaired.

A

Parsonage - Turner syndrome

This is a peripheral neuropathy that may complicate viral illnesses and usually resolves spontaneously.

4107
Q

Please select the source of innervation for the region described. Each option may be used once, more than once or not at all.

  1. The skin on the palmar aspect of the thumb
  2. The nail bed of the index finger
  3. The skin overlying the medial aspect of the palm
A

Median

median

Ulnar

4108
Q

A 38-year-old man is playing football when he slips over during a tackle. His knee is painful immediately following the fall. Several hours later he notices that the knee has become swollen. Following a course of non steroidal anti inflammatory drugs and rest the situation improves. However, complains of recurrent pain. On assessment in clinic you notice that it is impossible to fully extend the knee, although the patient is able to do so when asked.

A

he correct answer is Torn meniscus

Theme from September 2012 Exam
Twisting sporting injuries followed by delayed onset of knee swelling and locking are strongly suggestive of a menisceal tear. Arthroscopic menisectomy is the usual treatment.

4109
Q

A 28-year-old professional footballer is admitted to the emergency department. During a tackle he is twisted with his knee flexed. He hears a loud crack and his knee rapidly becomes swollen.

A

The correct answer is Anterior cruciate ligament rupture

This is common in footballers as the football boot studs stick to the ground and high twisting force is applied to a flexed knee. Rapid joint swelling also supports the diagnosis.

4110
Q

A 65-year-old diabetic female presents with a painless ulcer at the medial malleolus, it has been present for the past 16 years. On examination she has evidence of truncal varicosities and a brownish discolouration of the skin overlying the affected area.

A

Venous insufficiency

Venous ulcers are usually associated with features of venous insufficiency. These include haemosiderin deposition and varicose veins. Neuropathic ulcers will tend to present at sites of pressure, which is not typically at the medial malleolus.

4111
Q

A 71-year-old man presents with a painful lower calf ulcer, mild pitting oedema and an ABPI of 0.3.

A

Chronic obliterative arterial disease

Painful ulcers associated with a low ABPI are usually arterial in nature. The question does not indicate that features of chronic venous insufficiency are present. Patients may have mild pitting oedema as many vascular patients will also have ischaemic heart disease and elevated right heart pressures. The absence of more compelling signs of venous insufficiency makes a mixed ulcer less likely.

4112
Q

A 28-year-old female presents with a small nodule located on the back of her neck. It is excised for cosmetic reasons and the histology report states that the lesion consists of a sebum filled lesion surrounded by the outer root sheath of a hair follicle.

A

Pilar cyst

Pilar cysts may contain foul smelling cheesy material and are surrounded by the outer part of a hair follicle. Because of their histological appearances they are more correctly termed pilar cysts than sebaceous cysts.

4113
Q

A 40-year-old lady trips and falls through a glass door and sustains a severe laceration to her left arm. Amongst her injuries it is noticed that she has lost the ability to adduct the fingers of her left hand.

A

Ulnar nerve

The interossei are supplied by the ulnar nerve.

4114
Q

What is used to give Px in breast Ca?

A

Nottingham Prognostic Index

4115
Q

How is NPI calulated

A

Tumour size x 0.2 + LN score + grade score

4116
Q

A 19-year-old sportswoman presents with knee pain which is worse on walking down the stairs and when sitting still. On examination there is wasting of the quadriceps and pseudolocking of the knee.

A

Chondromalacia patellae

A teenage girl with knee pain on walking down the stairs is characteristic for chondromalacia patellae (anterior knee pain). Most cases are managed with physiotherapy.

4117
Q

A tall 18-year-old male athlete is admitted to the emergency room after being hit in the knee by a hockey stick. On examination his knee is tense and swollen. X-ray shows no fractures.

A

Dislocated patella

A patella dislocation is a common cause of haemarthrosis and many will spontaneously reduce when the leg is straightened. In the chronic setting physiotherapy is used to strengthen the quadriceps muscles.

4118
Q

An athletic 15-year-old boy presents with knee pain of 3 weeks duration. It is worst during activity and settles with rest. On examination there is tenderness overlying the tibial tuberosity and an associated swelling at this site.

A

Osgood Schlatters disease

Athletic boys and girls may develop this condition in their teenage years. It is caused by multiple micro fractures at the point of insertion of the tendon into the tibial tuberosity. Most cases settle with physiotherapy and rest.

4119
Q

Hx and Ex in carpal tunnel syndrome

A

Pain/pins and needles in thumb, index, middle finger e.g. at night

Patient flicks hand to obtain relief

Weakness of thumb abduction

Wasting of thenar eminence

Tinel’s sign: tapping causes paraesthesia)

Phalen’s sign

4120
Q

Causes of carpal tunnel syndrome

MEDIAN TRAP

A

Myxoedema

Edema premnestrually

DM

Idiopathic

Acromegaly

Neoplasm

Trauma

Rheumatoid arthritis

Amyloidoisis

Pregnancy

4121
Q

A 72-year-old man is diagnosed with prostate cancer and goserelin (Zoladex) is prescribed. Which one of the following is it most important to co-prescribe for the first three weeks of treatment?

Tamoxifen

Lansoprazole

Allopurinol

Cyproterone acetate

Tamsulosin

A

Anti-androgen treatment such as cyproterone acetate should be co-prescribed when starting gonadorelin analogues due to the risk of tumour flare. This phenomenon is secondary to initial stimulation of luteinising hormone release by the pituitary gland resulting in increased testosterone levels.

The BNF advises starting cyproterone acetate 3 days before the gonadorelin analogue.

4122
Q

Compartments of the lower limb

A

Anterior

Peroneal compartment

Superficial posterior compartment

Deep posterior compartment

4123
Q

Muscles in the anterior compartment of the lower limb

A

Tibialis anterior

EDL

Peroneus tertius

EHL

4124
Q

Innervation of the anterior compartment of the lower limb

A

Deep pernoeal nerve

4125
Q

Innervation of the peroneal compartment of the lower limb

A

Superficial peroneal nerve

4126
Q

Innervation of the psoterior compartment of the lower limb

A

Tibial nerve

4127
Q

Muscles in the peroneal compartment of the lower limb

A

Peroneus longus

Peroneus brevis

4128
Q

Muscles in the superficial posterior compartment of the lower limb

A

Gastrocnemius

Soleus

4129
Q

Muscles in the deep posterior compartment of the lower limb

A

FDL

FHL

Tibialis posterior

4130
Q

Action of tibialis anterior

A

Dorsiflexis ankle

Inverts foot

4131
Q

Action of EDL

A

Extends lateral 4 toes, dorsiflexes ankle joint

4132
Q

Action of peroneus tertius

A

Dorsiflexes ankle, everts foot

4133
Q

Action of EHL

A

Dorsiflexes ankle joint, extends big toe

4134
Q

Action of peronues longus

A

Everts foot, assits in plantar flexion

4135
Q

Action of peroneus brevis

A

Plantar flexes the ankle joint

4136
Q

Action of gastrocnemius

A

Plantar flexes the foot, may also flex the knee

4137
Q

Action of soleus

A

Plantar flexor

4138
Q

Action of FDL

A

Flexes lateral four toes

4139
Q

Action of FHL

A

Flexes the great toe

4140
Q

Action of tibialis posterior

A

Plantar flexor, inverts the foot

4141
Q

A 73-year-old male presents with a collapse and is brought to the emergency department. On examination he has a cold, painful left hand and forearm.

A

Sudden arterial embolus will affect the axillary artery in up to 30% cases. Because of the acute nature of the condition there is no time for the development of a collateral circulation so the limb is usually pale and painful. Emboli occur usually occur as a result of atrial fibrillation. Fast atrial fibrillation can cause syncope and an acute embolus.

4142
Q

A 52-year-old lady presents with an episode of nipple discharge. It is usually clear in nature. On examination the discharge is seen to originate from a single duct and although it appears clear, when the discharge is tested with a labstix it is shown to contain blood. Imaging and examination shows no obvious mass lesion.

A

The correct answer is Intraductal papilloma

Intraductal papilloma usually cause single duct discharge. The fluid is often clear, although it may be blood stained. If the fluid is tested with a labstix (little point in routine practice) then it will usually contain small amounts of blood. A microdocechtomy may be performed.

4143
Q

RFs for TCC bladder

A

Smoking

Exposure to aniline dyes int he printing and textile industry

Rubber manufacture

Cyclophosphamide

4144
Q

RFs for SCC bladder

A

Schistosomiasis

BCG treatment

Smoking

4145
Q

A 20-year-old complains of severe pain and swelling of the scrotum after a cystoscopy. He had mumps as a child. The testis is tender. The urine dipstick is positive for leucocytes.

A

Epididymo-orchitis: acute pain and swelling after urological intervention. To differentiate from testicular torsion there is usually pyrexia and positive urine dipstick.

4146
Q

An 8-year-old presents with scrotal swelling. He has just recovered from an acute viral illness with swelling of the parotid glands. On examination both testes are tender and slightly swollen.

A

Orchitis

Orchitis may be associated with mumps viral infections.

4147
Q

There is decreased secretion of which one of the following hormones in response to major surgery:

Insulin

Cortisol

Renin

Anti diuretic hormone

Prolactin

A

Endocrine parameters reduced in stress response:

Insulin

Testosterone

Oestrogen

4148
Q

Hormones increased by stress response

A

GH

Cortisol
Renin

ACTH

Aldosterone

Prolactin

ADH

Glucagon

4149
Q

An 18-year-old female presents with 3 nodules in the right lobe of the thyroid. Clinically she is euthyroid and there is associated cervical lymphadenopathy. She has no family history of thyroid disease.

A

Papillary carcinoma

Papillary thyroid cancer is the most common type of thyroid cancer and are the more common in females (M:F=1:3). Papillary tumours are more likely to develop lymphatic spread than follicular tumours.

4150
Q

A 10-year-old boy undergoes a delayed open reduction and fixation of a significantly displaced supracondylar fracture. On the ward he complains of significant forearm pain and paraesthesia of the hand. Radial pulse is normal.

A

Fasciotomy

The delay is the significant factor here. These injuries often have neurovascular compromise and inactivity now places him at risk of developing complications. In compartment syndrome the loss of arterial pulsation occurs late.

4151
Q

A 28-year-old man falls onto an outstretched hand. On examination there is tenderness of the anatomical snuffbox. However, forearm and hand x-rays are normal.

A

Discharge home with futura splint and fracture clinic appointment

This could well be a scaphoid fracture and should be temporarily immobilised pending further review. A futura splint will immobilise better than an arm sling for this problem.

4152
Q

What can be used to classify open #s?

A

Gustilo and Anderson Classification System

4153
Q

Components of Gustilo and Anderson

A

Grade 1:

Low energy wound <1cm

Grade 2:

>1cm wound with moderate soft tissue damage

Grade 3:

High energy wound >1cm with extensive soft tissue damage

A: adeqate soft tissue coverage

B: inadequate soft tissue coverage

C: associated arterial injury

4154
Q

A 43-year-old man has been troubled with dysphagia for many years. He is known to have achalasia and has had numerous dilatations. Over the past 6 weeks his dysphagia has worsened. At endoscopy a friable mass is noted in the oesophagus.

A

Squamous cell carcinoma

The risk of squamous cell carcinoma of the oesophagus is increased in people with achalasia. The condition often presents late and has a poor prognosis.

4155
Q

A Schatzki ring or Schatzki–Gary ring

A

is a narrowing of the lower esophagus that can cause difficulty swallowing (dysphagia). The narrowing is caused by a ring of mucosal tissue (which lines the esophagus) or muscular tissue.[1] A Schatzki ring is a specific type of “esophageal ring”, and Schatzki rings are further subdivided into those above the esophagus/stomach junction (A rings),[2] and those found at the squamocolumnar junction in the lower esophagus (B rings).[3]

Patients with Schatzki rings can develop intermittent difficulty swallowing or, more seriously, a completely blocked esophagus. The ring is named after the German-American physician Richard Schatzki.

4156
Q

What are the critera for brain stem death testing

A

Deep coma of known aetiology.

Reversible causes excluded

No sedation

Normal electrolytes

4157
Q

Testing for brain death

A

Fixed pupils which do not respond to sharp changes in intensity of incident light

No corneal reflex

Absent VOR (no eye movements following the slow injection of at least 50ml of ice-cold water in each ear in turn- caloric test)

No response to supraorbital pressure

No cough reflex to bronchial stimulat or gagging response to pharyngeal stimulation

No observed repiratory effort in response to disconnection of the ventilator for ~5 minutes,

4158
Q

A 56-year-old man undergoes a difficult colonoscopy for assessment of a caecal cancer. 48 hours after the procedure he is admitted with septicaemia. His abdomen is soft and non tender. Blood cultures grow gram positive cocci.

A

Streptococcus bovis

Streptococcus bovis septicaemia is associated with carcinoma of the colon. It also can also cause endocarditis.

4159
Q

An 8-year-old child presents with enlarged tonsils that meet in the midline and are covered with a white film that bleeds when you attempt to remove it. He is pyrexial but otherwise well.

A

Acute bacterial tonsillitis

Theme from April 2012 Exam
In acute tonsillitis the tonsils will often meet in the midline and may be covered with a membrane. Individuals who are systemically well are unlikely to have diptheria.

4160
Q

A 10-year-old child presents with enlarged tonsils that meet in the midline. Oropharyngeal examination confirms this finding and you also notice peticheal haemorrhages affecting the oropharynx. On systemic examination he is noted to have splenomegaly.

A

A combination of pharyngitis and tonsillitis is often seen in glandular fever. Antibiotics containing penicillin may produce a rash when given in this situation, leading to a mistaken label of allergy.

4161
Q

A 62-year-old man is admitted with dull lower back pain and abdominal discomfort. On examination he is hypertensive and a lower abdominal fullness is elicited on examination. An abdominal ultrasound demonstrates hydronephrosis and intravenous urography demonstrated medially displaced ureters. A CT scan shows a periaortic mass

A

Retroperitoneal fibrosis

Retroperitoneal fibrosis is an uncommon condition and its aetiology is poorly understood. In a significant proportion the ureters are displaced medially. In most retroperitoneal malignancies they are displaced laterally. Hypertension is another common finding. A CT scan will often show a para-aortic mass

4162
Q

A 32-year-old man is admitted with a distended tense abdomen. He previously underwent a difficult appendicectomy 1 year previously and was discharged. At laparotomy the abdomen is filled with a gelatinous substance.

A

Pseudomyxoma peritonei

Pseudomyxoma is classically associated with mucin production and the appendix is the commonest source.

Pseudomyxoma peritoneii- Curative treatment is peritonectomy (Sugarbaker procedure) and heated intra peritoneal chemotherapy.

4163
Q

A 73-year-old man develops disseminated intravascular coagulation following an abdominal aortic aneurysm repair. He receives an infusion of cryoprecipitate. What is the major constituent of this infusion?

Factor VIII

Factor IX

Protein C

Protein S

Factor V

A

Cryoprecipitate

Blood product made from plasma

Usually transfused as 6 unit pool

Indications include massive haemorrhage and uncontrolled bleeding due to haemophilia

Composition

AgentQuantity

Factor VIII100IU

Fibrinogen250mg

von Willebrand factorVariable

Factor XIIIVariable

4164
Q

A 42-year-old lady who has systemic lupus erythematosus presents to the clinic with a 5 day history of a painful purple lesion on her index finger. On examination she has a tender red lesion on the index finger.

A

Oslers nodes

Osler nodes are normally described as tender, purple/red raised lesions with a pale centre. These lesions occur as a result of immune complex deposition. These occur most often in association with endocarditis. However, other causes include SLE, gonorrhoea, typhoid and haemolytic anaemia.

4165
Q

An 85-year-old man presents with a cough and haemoptysis. He has a modest smoking history of 15 pack years. He is found to have a tumour located in the right main bronchus, with no evidence of metastatic disease. He decides no undergo any treatment and he remains well for a further 12 months before developing symptomatic metastasis.

A

Squamous cell carcinoma

Squamous cell carcinomas are reported to be more slow growing and are typically centrally located. Small cell carcinomas are usually centrally located. However, small cell carcinomas would seldom be associated with a survival of a year without treatment.

4166
Q

A female infant is born by cesarean section at 38 weeks gestation for foetal distress. The attending paediatricians notice that she has a single palmar crease and an anti mongoloid slant to her eyes. Soon after the birth the mother tries to feed the child who has a projectile vomit about 10 minutes after feeding. On examination she has a soft, non distended abdomen.

A

Duodenal atresia

Proximally sited atresia will produce high volume vomits which may or may not be bile stained. Abdominal distension is characteristically absent. Whilst under resuscitated children may be a little dehydrated they are seldom severely ill. The presence of Trisomy 21 (palmar and eye signs) increases the likelihood of duodenal atresia.

4167
Q

A 34-year-old women trips over and falls into a bonfire whilst intoxicated at a party. She suffers burns to her arms, torso and face. These are calculated to be 25% body surface area. She is otherwise stable. The burns to the torso are superficial, her left forearm has a full thickness burn and the burns to her face are superficial. There is no airway compromise. She has received 1000ml of intravenous Hartman’s solution, with a further 1000ml prescribed to run over 4 hours.

A

The correct answer is Transfer to regional burn centre once stabilised

This women has been resuscitated and requires transfer for specialist management

4168
Q

A 20-year-old man is trapped in a warehouse fire. He has sustained 60% burns to his torso and limbs. The limb burns are partial thickness but the torso burns are full thickness. He was intubated by paramedics at the scene and is receiving intravenous fluids. His ventilation pressure requirements are rising.

A

Escharotomy

He requires an escharotomy as this will be contributing to impaired ventilation

4169
Q

A 68-year-old man is on the colorectal ward following resection of an extremely large adenocarcinoma in his descending colon. During the operation he had a left hemicolectomy. As the tumour was invading surrounding structures, two loops of small bowel had to be excised along with a partial cystectomy.

Now five days after the procedure he is managing well, with pain well controlled. His catheter has drained 2000ml in the last 24 hours. The abdominal surgical drain is still producing 200-300 ml of clear yellow fluid per day and you are concerned that his bladder wall repair following the partial cystectomy may be leaking urine. Which investigation should you order to offer the most definitive result to assess whether the bladder suture line has healed?

Cystoscopy

CT

Ultrasound of the kidneys, ureters and bladder

X-ray of the kidneys, ureters and bladder

Cystogram

A

A cystogram involves passing radiopaque dye into the bladder, then performing radiographs to assess the course of the bladder contents. This provides evidence of whether there is any radiopaque fluid that has escaped the bladder and is free in the abdominal cavity.

4170
Q

A 56-year-old man is involved in a road traffic accident. He is found to have a pelvic fracture. He reports that he has some lower abdominal pain. He has peritonism in the lower abdomen. The nursing staff report that he has not passed any urine. A CT scan shows evidence of free fluid.

A

Bladder rupture

A pelvic fracture and lower abdominal peritonism should raise suspicions of bladder rupture (especially as this man cannot pass urine).

4171
Q

A 52-year-old man falls off his bike. He is found to have a pelvic fracture. On examination he is found to have perineal oedema and on PR the prostate is not palpable. A urine dipstick shows blood.

A

Membranous urethral rupture

A pelvic fracture and highly displaced prostate should indicate a diagnosis of membranous urethral rupture.

4172
Q

How to differentiate between bulbar and membranous urethral ruputre

A

Bulbar rupture:

Most common

Straddle type injury e.g. bicycles

Triad signs: urinary retention, perineal haematoma, blood at the meatus

Membarnous rupture:

Can be extra or intraperitoneal

Commonly due to pelvic fracture

Penile or perineal oedema/haematoma

PR: prostate displaced upwards (beware coexisiting retroperitoneal haematomas as they make examination difficult)

4173
Q
A
4175
Q

With which of the following blood products is iatrogenic septicaemia with a gram positive organism most likely?

Cryoprecipitate

Platelets

Packed red cells

Factor VIII concentrate

Factor IX concentrate

A

Platelets are stored at room temperature and must be used soon after collection. This places them at increased risk of culturing gram positive organisms. Iatrogenic infection with gram negative organisms is more likely with packed red cells as these are stored at 4 degrees.
Infections with blood products of this nature are both rare.

4176
Q

A 23-year-old rugby player falls directly onto his shoulder. There is pain and swelling of the shoulder joint. The clavicle is prominent and there appears to be a step deformity.

A

Acromioclavicular dislocation

Acromioclavicular joint (ACJ) dislocation normally occurs secondary to direct injury to the superior aspect of the acromion. Loss of shoulder contour and prominent clavicle are key features. Note; rotator cuff tears rarely occur in the second decade.

4177
Q

A 22-year-old man falls over and presents to casualty. A shoulder x-ray is performed, the radiologist comments that a Hill-Sachs lesion is present.

A

Glenohumeral dislocation

A Hill-Sachs lesion is when the cartilage surface of the humerus is in contact with the rim of the glenoid. About 50% of anterior glenohumeral dislocations are associated with this lesion.

4178
Q

Mondor’s diease of the breast

A

Mondor’s disease (also known as “Mondor’s syndrome of superficial thrombophlebitis”[1]) is a rare condition which involves thrombophlebitis of the superficial veins of the breast and anterior chest wall. It sometimes occurs in the arm or penis.[2] In axilla, this condition is known as axillary web syndrome.[3]

Patients with this disease often have abrupt onset of superficial pain, with possible swelling and redness of a limited area of their anterior chest wall or breast. There is usually a lump present, which may be somewhat linear and tender. Because of the possibility of the lump being from another cause, patients are often referred for mammogram and/or breast ultrasound.[4]

4179
Q

Features of Pb poisoning

A

abdominal pain

peripheral neuropathy (mainly motor)

fatigue

constipation

blue lines on gum margin (only 20% of adult patients, very rare in children)

4180
Q

A 28-year-old man undergoes a ileocaecal resection and end ileostomy for Crohn’s disease. One year later he presents with a deep painful ulcer at his stoma site.

A

Pyoderma gangrenosum

Associated with inflammatory bowel disease/RA

Can occur at stoma sites

Erythematous nodules or pustules which ulcerate

Submit answer

4181
Q

Triad of symptoms in fat embolism

A

Respiratory

Neurological

Petechial rash (tends to occur after the first 2 symptoms)

4182
Q

A 35-year-old male presents with haematuria. He is found to have bilateral masses in the flanks. He has a history of epilepsy and learning disability.

A

Angiomyolipoma

This patient has tuberous sclerosis. This is associated with angiomyolipoma, which is present in 60-80% patients. It is a benign lesion.

4183
Q

A 73-year-old lady is undergoing chemotherapy for treatment of acute leukaemia. She develops symptoms of renal colic. Her urine tests positive for blood. A KUB x-ray shows no evidence of stones.

A

Uric acid

Chemotherapy and cell death can increase uric acid levels. In this acute setting the uric acid stones are unlikely to be coated with calcium and will therefore be radiolucent.

4184
Q

A 16-year-old boy presents with renal colic. His parents both have a similar history of the condition. His urine tests positive for blood. A KUB style x-ray shows a relatively radiodense stone in the region of the mid ureter.

A

Cystine

Cystine stones are associated with an inherited metabolic disorder.

4185
Q

Associated with chronic straining and constipation. Histology shows mucosal thickening, lamina propria replaced with collagen and smooth muscle (fibromuscular obliteration)

Submit answer

A

Solitary rectal ulcer

4186
Q

A 32-year-old man is involved in a house fire and sustains extensive partial thickness burns to his torso and thigh. Two weeks post operatively he develops oedema of both lower legs. The most likely cause of this is:

Iliofemoral deep vein thrombosis

Venous obstruction due to scarring

Hypoalbuminaemia

Excessive administration of intravenous fluids

None of the above

A

Loss of plasma proteins is the most common cause of oedema developing in this time frame.

4187
Q

Features of pulmoanry artery occlusion pressure monitoring

A

Indirect measure of left atrial pressure and thus filling pressure of the left heart.

When combined with measurements of SVR and CO it is possible to accurately classify the potential cause of shock

4188
Q

How to calculate SVR

A

Derived from aortic pressure, RAP and CO

SVR= 80(mean aortic pressure-mean right atrial pressure)/CO

4189
Q

Normal PAOP

A

8-12mmHg

4190
Q

Low PAOP

A

<5: hypovolaemia

4191
Q

Low PAOP with pulmonary oedema

A

<5= ARDS

4192
Q

High PAOP

A

>18: suggestive of overload e.g. cardiogenic pulmonary oedema

4193
Q

A 2 month old baby presents with jaundice. He has an elevated conjugated bilirubin level. Diagnosis is confirmed by cholangiography during surgery.

A

Roux-en-Y portojejunostomy

This child has biliary atresia. The aim is to avoid liver transplantation (however, most will come to transplant in time).

4194
Q

Which one of the following is not a risk factor for the development of calcium oxalate and calcium phosphate renal stones?

Bendroflumethiazide

Aminophylline

Acetazolamide

Furosemide

Prednisolone

A

Bendroflumethiazide may help prevent the formation of calcium based renal stones. It may however theoretically increase the risk of urate based stones

4195
Q

What is the Mackler Triad for Boerhaave syndrome

A

Vomiting

Thoracic pain

Subcutaneous emphysema

Commonly presents in middle aged men with a background of alcohol abuse

4196
Q

A 52-year-old male presents with tearing central chest pain. On examination he has an aortic regurgitation murmur. An ECG shows ST elevation in leads II, III and aVF.

A

Proximal aortic dissection

An inferior myocardial infarction and AR murmur should raise suspicions of an ascending aorta dissection rather than an inferior myocardial infarction alone. Also the history is more suggestive of a dissection. Other features may include pericardial effusion, carotid dissection and absent subclavian pulse.

4197
Q

A 60-year-old male is admitted to A&E with a fall. He lives with his wife and still works as a restaurant manager. He has a past history of benign prostatic hypertrophy and is currently taking tamsulosin. He is otherwise fit and healthy. On examination there is right hip tenderness on movement in all directions. A hip x-ray confirms an intertrochanteric fracture.

A

ynamic hip screw

The blood supply to the femoral head may be intact and the fracture should heal with compression type devices such as gamma nails or dynamic hip screws. The latter device being the most commonly performed therapeutic intervention.

4198
Q

An 86-year-old retired pharmacist is admitted to A&E following a fall. She complains of right hip pain. She is known to have hypertension and is currently on bendrofluazide. She lives alone and mobilises with a Zimmer frame. Her right leg is shortened and externally rotated. A hip x-ray confirms a displaced intracapsular fracture.

A

Hemiarthroplasty

Hemiarthroplasty is offered to older, less mobile individuals compared to fracture reduction and fixation in younger patients.

4199
Q

A 74-year-old male is admitted to A&E with a fall. He is known to have rheumatoid arthritis and is on methotrexate and paracetamol. He lives alone in a bungalow and enjoys playing golf. He is independent with his ADLs. He complains of left groin pain, therefore has a hip x-ray which confirms a displaced intracapsular fracture.

A

Total hip replacement

This patient has pre-existing joint disease, good level of activity and a relatively high life expectancy, therefore THR is preferable to hemiarthroplasty.

4200
Q

Base excess

A

Calculated figure wich provides an estimate of the metabolic componenet of acid-base balance

Defined as the amount of H+ ions that would be required to return the pH of the blood to 7.35 if the pCO2 were adjusted to normal

BE >+3: metabolic alkalsois

BE < -3= metabolic acidosis

4201
Q

A 2 week old baby is referred to the surgical team by the paediatricians. They are concerned because the child has a painful area of macerated tissue at the site of the umbilicus. On examination a clear- yellowish fluid is seen to be draining from the umbilicus when the baby cries.

A

Persistent urachus

A patent urachus will present with umbilical urinary discharge. The skin may become macerated if not properly cared for. The discharge is most likely to be present when intra-abdominal pressure is raised. It is associated with posterior urethral valves.

4202
Q

A premature neonate is born by emergency cesarean section at 29 weeks gestation. He initially seems to be progressing well. However, the team are concerned because he becomes systemically septic and on examination has a swollen and erythematous umbilicus.

A

Omphalitis

Infection from omphalitis may spread rapidly and cause severe sepsis especially in immunologically compromised, premature neonates.

4203
Q

A baby boy is born by elective cesarean section at 39 weeks gestation. He initially seems to progress well and is discharged from hospital the following day. The parents bring the child to the clinic at 10 days of age and are concerned at the presence of a profuse and foul smelling discharge at the site of the umbilicus. On examination the umbilicus has some prominent granulation tissue. When the baby cries a small trickle of brownish fluid is seen to pass from the umbilicus.

A

Persistent vitello-intestinal duct

A persistent vitello-intestinal duct may allow the persistent and ongoing discharge of small bowel content from the umbilicus. This fluid may be very irritant to the surrounding skin.

4204
Q

A 63-year-old man presents with a locally unresectable gastrointestinal stromal tumour. Biopsies confirm that it is KIT positive.

A

Imatinib

Imatinib is licensed for treatment of GIST in the United Kingdom for this situation. The guidance from the National Institute of Clinical evidence is that patients be reviewed at 12 weeks after initiating therapy.

4205
Q

A 28-year-old man undergoes an ileocaecal resection to treat terminal ileal Crohns disease. Post operatively he attends the clinic and complains of diarrhoea. His CRP is within normal limits and small bowel enteroclysis shows no focal changes. Which of the following interventions is most likely to be beneficial?

5 ASA drugs

Azathioprine

Pulsed methylprednisolone

Infliximab

Oral cholestyramine

A

Malabsorption of bile salts is a common cause of diarrhoea following ileal resection. A normal small bowel study and CRP effectively excludes active Crohns disease and therefore immunomodulator drugs are not appropriate.

4206
Q

A 78-year-old man is referred to the clinic by his general practitioner. For many years he noticed a smooth swelling approximately 5cm anterior to the tragus of his right ear. Apart from being a heavy smoker he has no co-morbidities. What is the most likely diagnosis?

Pleomorphic adenoma

Liposarcoma

Warthins tumour

Adenocarcinoma

None of the above

A

Warthins tumours are most common in elderly smokers. They have a relatively benign and indolent course. They are usually well circumscribed as illustrated below:

4207
Q

An 18 month old boy is brought to the emergency room by his parents. He was found in bed with a nappy filled with dark red blood. He is haemodynamically unstable and requires a blood transfusion. Prior to this episode he was well with no prior medical history. What is the most likely cause?

Necrotising enterocolitis

Anal fissure

Oesophageal varices

Meckels diverticulum

Crohns disease

A

Meckels diverticulum is the number one cause of painless massive GI bleeding requiring a transfusion in children between the ages of 1 and 2 years.

4208
Q

Margins of excision related to Breslow thickness

0-1mm thick

A

1cm

4209
Q

Margins of excision related to Breslow thickness

1-2mm thick

A

1-2cm depending on site and pathological features

4210
Q

Margins of excision related to Breslow thickness

2-4mm thick

A

2-3cm depending on site and pathological features

4211
Q

Margins of excision related to Breslow thickness

>4mm thick

A

3cm

4212
Q

A 23-year-old man suffers a road traffic accident and is due for emergency surgery. Which anaesthetic agent carries the greatest risk of adrenal suppression in the peri-operative period?

Ketamine

Etomidate

Sevoflurane

Desflurane

Propofol

A

Although etomidate is widely used in emergency settings due to its favourable haemodynamic profile and cerebroprotective effects, it is associated with depressed corticosteroid synthesis by inhibition of 11-beta-hydroxylase. Adrenal suppression can increase mortality, especially in the post-operative recovery period.

4213
Q

intravenous anaesthetic agent

barbiturate: enhance effects of GABA (less specific than benzodiazepines)

risk of severe vasospasm → tissue necrosis if accidentally injected into vein

A

Thiopentone

4214
Q

intravenous anaesthetic agent

mode of action unknown

rapidly metabolised therefore fast recovery

adverse effects: pain on injection site, cardiovascular and respiratory depressant

can be used as an infusion

A

Proprofol

4215
Q

intravenous anaesthetic agent

similar to thiopentone but with less cardiovascular and respiratory depression

adverse effects: adrenocortical suppression, excitatory effects during induction and recovery

Submit answer

A

Etomidate

4216
Q

A 56-year-old man presents with an adenocarcinoma of the mid oesophagus. Staging investigations show no metastatic disease and he is otherwise fit.

A

McKeown oesophagectomy

He requires a total (3 stage oesophagectomy). This is also called a McKeown oesophagectomy. Ivor Lewis type resections are concluded in the mid third of the oesophagus and would not adequately treat this disease

4217
Q

A 43-year-old man with an adenocarcinoma of the distal oesophagus. His staging investigations are negative for metastatic disease.

A

Ivor Lewis oesophagectomy

This requires an Ivor Lewis oesophagectomy with resection of the distal oesophagus and an intrathoracic anastomosis

4218
Q

A 43-year-old lady presents with severe chest pain. Investigations demonstrate a dissecting aneurysm of the ascending aorta which originates at the aortic valve. What is the optimal long term treatment?

Endovascular stent

Medical therapy with beta blockers

Medical therapy with ACE inhibitors

Sutured aortic repair

Aortic root replacement

A

Proximal aortic dissections are generally managed with surgical aortic root replacement. The proximal origin of the dissection together with chest pain (which may occur in all types of aortic dissection) raises concerns about the possibility of coronary ostial involvement (which precludes stenting). There is no role for attempted suture repair in this situation.

4219
Q

A 48-year-old lady presents with discomfort in the right breast. On examination she has a discrete soft fluctuant area in the upper outer quadrant of her right breast. A mammogram is performed and a “halo sign” is seen by the radiologist.

A

Breast cyst

Lesions such as breast cysts compress the underlying fat and produce a radiolucent area (halo sign). If symptomatic these cysts should be aspirated.

4220
Q

A 55-year-old lady presents with discomfort in the right breast. On clinical examination a small lesion is identified and clinical appearances suggest fibroadenoma. Imaging confirms the presence of a fibroadenoma alone. A core biopsy is taken, this confirms the presence of the fibroadenoma. However, the pathologist notices that a small area of lobular carcinoma in situ is also present in the biopsy. What is the best management?

Whole breast irradiation

Simple mastectomy

Mastectomy and sentinal lymph node biopsy

Wide local excision and sentinel lymph node biopsy

Breast MRI scan

A

Lobular carcinoma in situ has a low association with invasive malignancy. It is seldom associated with microcalcification and therefore MRI is the best tool for determining disease extent. Resection of in situ disease is not generally recommended and most surgeons would simply pursue a policy of close clinical and radiological follow up.

4221
Q

An 80-year-old woman has a hip fracture. Her calcium is normal. She has never been given a diagnosis of osteoporosis.

A

Risedronate and calcium supplements

The osteoporosis guidelines state if a postmenopausal woman has a fracture she should be put on bisphosphonates (there is no need for a DEXA scan).

4222
Q

A 60-year-old man presents with recurrent renal stones. He is found to have a calcium of 2.72 (elevated) and a PTH of 12 (elevated).

A

Exploration and parathyroidectomy

This patient has primary hyperparathyroidism and nephrolithiasis, which is an indication for parathyroidectomy.

4223
Q

An 82-year-old woman from a nursing home is admitted to the orthopaedic ward with a hip fracture. She is acutely confused and agitated. Her Calcium is 2.95 (elevated).

A

Intravenous fluid (0.9% N.Saline)

This patient needs rehydration due to hypercalcaemia. An intravenous bisphosphonate is indicated if the Ca is above 3.

4224
Q

A 49-year-old woman presents with a tender lump around the areola associated with a green nipple discharge.

A

Mammary duct ectasia

4225
Q

A 16-year-old boy is hit by a car and sustains a blow to the right side of his head. He is initially conscious but on arrival in the emergency department is comatose. On examination his right pupil is fixed and dilated. The neurosurgeons plan immediate surgery. What type of initial approach should be made?

Left parieto-temporal craniotomy

Right parieto-temporal craniotomy

Posterior fossa craniotomy

Left parieto-temporal burr holes

None of the above

A

A unilateral dilated pupil is a classic sign of transtentorial herniation. The medial aspect of the temporal lobe (uncus) herniates across the tentorium and causes pressure on the ipsilateral oculomotor nerve, interrupting parasympathetic input to the eye and resulting in a dilated pupil. In addition the brainstem is compressed. As the ipsilateral oculomotor nerve is being compressed, craniotomy (rather than Burr Holes) should be made on the ipsilateral side.

4226
Q

Risk factors for developing TURP syndrome?

A

Surgical time >1hr

Height of bag >70cm

Resected >60h

Large blood loss

Perforation

Large amount of fluid used

Poorly controlled CHF

4227
Q

A 3-month old infant presents with recurrent abdominal pain, colicky in nature and intestinal obstruction. Imaging results reveal a right sided midline defect where there is herniation of the transverse colon into the thoracic cavity. What is the most likely diagnosis?

Bochdalek hernia

Umbilical hernia

Para umbilical hernia

Femoral hernia

Morgagni hernia

A

The right sided midline defect is referring to herniation through the foramen of Morgagni, 60% of Morgagni hernias contain transverse colon whereas Bochdalek hernias tend to be left sided, containing stomach.

4228
Q

A 26-year-old man presents to the emergency department with a swelling over his left elbow after a fall on an outstretched hand. On examination, he has tenderness over the proximal part of his forearm, and has severely restricted supination and pronation movements.

A

Fracture of the radial head is common in young adults. It is usually caused by a fall on the outstretched hand. On examination, there is marked local tenderness over the head of the radius, impaired movements at the elbow, and a sharp pain at the lateral side of the elbow at the extremes of rotation (pronation and supination).

4229
Q

A 56-year-old lady presents with a painful swelling over the lower end of the forearm following a fall. Imaging reveals a distal radial fracture with disruption of the distal radio-ulnar joint.

A

Galeazzi fractures occur after a fall on the hand with a rotational force superimposed on it. On examination, there is bruising, swelling and tenderness over the lower end of the forearm. X- Rays reveal a displaced fracture of the radius and a prominent ulnar head due to dislocation of the inferior radio-ulnar joint.

4230
Q

A 3 day old neonate is developing increasing problems with feeding. On examination she has a pan systolic murmur and her forearms have not developed properly.

A

Oesophageal atresia

This child has VACTERL, which is a combination of Vertebral, Ano-rectal, Cardiac, Tracheo-oesophageal, Renal and Radial limb anomalies. Half of babies with oesophageal atresia will have VACTERL.

4231
Q

A 63-year-old man is admitted with severe right sided loin pain to the Emergency Department. A urine dipstick shows blood +++, leucocytes +, protein +. An abdominal radiograph is therefore ordered which shows a stag-horn calculus in the right renal pelvis. What are stag-horn calculi normally composed of?

Xanthine

Magnesium ammonium phosphate

Calcium oxalate

Uric acid

Magnesium calcium phosphate

A

Magnesium ammonium phosphate

4232
Q

Relations of the cavernous sinus

A

Medial: pituitary fossa, Sphenoid sinus

Lateral: temporal lobe

4233
Q

Contents of the cavernous sinus

A

Lateral wall from top to bottom

Oculomotor nerve

Trochlear nerve

V1

V2

Contents of the sinus: from medial to lateral

Internal carotid artery and sympathetic plexus

Abducens nerve

4234
Q

pain, opthalmoplegia, proptosis, trigeminal nerve lesion (opthalmic branch) and Horner’s syndrome.

A

Cavernous sinus syndrome

4235
Q

A 38-year-old women undergoes a gastric bypass procedure. Post operatively she attends the clinic and complains that following a meal she develops vertigo and develops crampy abdominal pain. What is the most likely underlying explanation?

Insulin resistance

Irritable bowel syndrome

Biliary colic

Dumping syndrome

Enterogastric reflux

A

Dumping syndrome, which can be divided into early and late, may occur following gastric surgery. It occurs as a result of a hyperosmolar load rapidly entering the proximal jejunum. Osmosis drags water into the lumen, this results in lumen distension (pain) and then diarrhoea. Excessive insulin release also occurs and results in hypoglycaemic symptoms.

4236
Q

A 15 year-old boy presents to the out-patient clinic with tiredness, recurrent throat and chest infections, and gradual loss of vision. Multiple x-rays show brittle bones with no differentiation between the cortex and the medulla.

A

Osteopetrosis is an autosomal recessive condition. It is commonest in young adults. They may present with symptoms of anaemia or thrombocytopaenia due to decreased marrow space. Radiology reveals a lack of differentiation between the cortex and the medulla described as marble bone. These bones are very dense and brittle.

4237
Q

A 5-year-old boy presents with a painful limp. The symptoms have been present for 8 weeks. Two hip x-rays have been performed and appear normal.

A

Perthes’ disease should be suspected in boys over 4 years old presenting with a limp. Early disease can be missed on x-ray. MRI is increasingly replacing bone scans as the second line investigation of choice

4238
Q

A 70-year-old coal miner presents with 3 weeks of haematuria and bruising. He is normally fit and well. He is on no medications. His results reveal:
Hb 9.0
WCC 11
Pl 255
PT 16 (normal)
APTT 58 (increased)
Thrombin time 20 (normal).

A

Acquired haemophilia

Factor 8 acquired disorder. The elderly, pregnancy, malignancy and autoimmune conditions are associated with acquired haemophilia. Prolonged APTT is key to the diagnosis. Management involves steroids.

4239
Q

Epiphysitis of the vertebral joints is the main pathological process

Predominantly affects adolescents

Symptoms include back pain and stiffness

X-ray changes include epiphyseal plate disturbance and anterior wedging

Clinical features include progressive kyphosis (at least 3 vertebrae must be involved)

Minor cases may be managed with physiotherapy and analgesia, more severe cases may require bracing or surgical stabilisation

A

Scheuermann’s disease

4240
Q

Consists of curvature of the spine in the coronal plane

Divisible into structural and non structural, the latter being commonest in adolescent females who develop minor postural changes only. Postural scoliosis will typically disappear on manoeuvres such as bending forwards

Structural scoliosis affects > 1 vertebral body and is divisible into idiopathic, congential and neuromuscular in origin. It is not correctable by alterations in posture

Within structural scoliosis, idiopathic is the most common type

Severe, or progressive structural disease is often managed surgically with bilateral rod stabilisation of the spine

A

Scoliosis

4241
Q

Congenital or acquired deficiency of the pars interarticularis of the neural arch of a particular vertebral body, usually affects L4/ L5

May be asymptomatic and affects up to 5% of the population

Spondylolysis is the commonest cause of spondylolisthesis in children

Asymptomatic cases do not require treatment

A

Spondylolysis

4242
Q

This occurs when one vertebra is displaced relative to its immediate inferior vertebral body

May occur as a result of stress fracture or spondylolysis

Traumatic cases may show the classic “Scotty Dog” appearance on plain films

Treatment depends upon the extent of deformity and associated neurological symptoms, minor cases may be actively monitored. Individuals with radicular symptoms or signs will usually require spinal decompression and stabilisation

A

Spondylolisthesis

4243
Q

An 18-year-old student is involved in a car crash, with another car crashing into the side of the car. A chest x-ray shows an indistinct left hemidiaphragm.

A

Diaphragmatic rupture

A lateral blunt injury during a road traffic accident is a common cause of diaphragmatic rupture. Diagnosis is usually evident on chest x-ray. CXR changes include non visible diaphragm, bowel loops in the hemithorax and displacement of the mediastinum. In most cases direct surgical repair is the best option.

4244
Q

A 19-year-old student falls from a 2nd floor window. He is persistently hypotensive despite fluid resuscitation. A chest x-ray shows depression of the left main bronchus and deviation of the trachea to the right. Lung markers can be seen in the peripheries of both thoraces.

A

Aorta rupture

He has a deceleration injury, with persistent hypotension (contained haematoma). This should indicate aorta rupture. Widened mediastinum may not always be present on a chest x-ray. A CT angiogram will provide clearer evidence of the extent of injury. The presence of persistent hypotension, from a early stage is more consistent with haematoma than a tension pneumothorax in which it occurs as a final periarrest phenomena.

4245
Q

Intra-articular fracture of the first carpometacarpal joint

Impact on flexed metacarpal, caused by fist fights

X-ray: triangular fragment at ulnar base of metacarpal

A

Bennett’s fracture

4246
Q

A patient is recovering from a Sistrunk’s procedure, what lesion was treated with this operation?

A

Thyroglossal cyst

This is the procedure for excision of the cyst and its associated track. Excision must be complete and thus a small segment of the hyoid bone is removed to gain access to the upper part of the cyst tract.

4247
Q

A patient presents with a facial nerve palsy. This occurred following repeat excision of a facial lump. The histology report remarks on the biphasic appearance of the lesion and mucinous connective tissue.

A

Pleomorphic adenoma of the parotid

The histological features are as described with a classic biphasic (mixed stromal and epithelial elements), although benign local recurrence can complicate incomplete excision. As this is a benign lesion direct extension into the facial nerve is unlikely to occur. Facial nerve injury can happen during repeat parotid surgery.

4248
Q

A 56-year-old man is admitted with passage of a large volume of blood per rectum. On examination he is tachycardic, his abdomen is soft, although he has marked dilated veins on his abdominal wall. Proctoscopy reveals large dilated veins with stigmata of recent haemorrhage.

A

IV terlipressin

Rectal varices are a recognised complication of portal hypertension. In the first instance they can be managed with medical therapy to lower pressure in the portal venous system. TIPSS may be considered. Whilst band ligation is an option, attempting to inject these in same way as haemorroids would carry a high risk of precipitating further haemorrhage.

4249
Q

A 73-year-old lady is admitted with dark red PR bleeding. She undergoes an OGD which is normal. Digital rectal examination shows blood but no masses. She becomes tachycardic and BP is 95/40.

A

Angiography of mesenteric artery

This women is actively bleeding and mesenteric angiography may localise the bleeding. Colonoscopy in this situation is seldom helpful or successful.

4250
Q

A 68-year-old man with ulcerative colitis is admitted with an exacerbation. You are called to see him because he is having brisk PR bleeding. He has been on Intravenous hydrocortisone for 5 days. The gastroenterologists have done an OGD to exclude a duodenal ulcer, this was normal.

A

A 68-year-old man with ulcerative colitis is admitted with an exacerbation. You are called to see him because he is having brisk PR bleeding. He has been on Intravenous hydrocortisone for 5 days. The gastroenterologists have done an OGD to exclude a duodenal ulcer, this was normal.

4251
Q

Mutation of PTEN gene on chromosome 10q22, dominant

Macrocephaly
Multiple intestinal hamartomas
Multiple trichilemmomas
89% risk of cancer at any site
16% risk of colorectal cancer

A

Cowden disease

4252
Q

Biallelic mutation of mut Y human homologue (MYH) on chromosome 1p, recessive

Multiple colonic polyps
Later onset right sided cancers more common than in FAP
100% cancer risk by age 60

A

MYH associated polyposis

4253
Q

A 30-year-old woman presents with pain and swelling of the left shoulder. There is a large radiolucent lesion in the head of the humerus extending to the subchondral plate.

A

Giant cell tumour

Giant cell tumours on x-ray have a ‘soap bubble’ appearance. They present as pain or pathological fractures. They commonly metastasize to the lungs.

4254
Q

A 72-year-old woman has a lumbar vertebral crush fracture. She has hypocalcaemia and a low urinary calcium.

A

Osteomalacia

Hypocalcemia and low urinary calcium are biochemical features of osteomalacia.

4255
Q

A 16-year-old boy presents with severe groin pain after kicking a football. Imaging confirms a pelvic fracture. A previous pelvic x-ray performed 2 weeks ago shows a lytic lesion with ‘onion type’ periosteal reaction.

A

A Ewings sarcoma is most common in males between 10-20 years. It can occur in girls. A lytic lesion with a lamellated or onion type periosteal reaction is a classical finding on x-rays. Most patients present with metastatic disease with a 5 year prognosis between 5-10%.

4256
Q

A 22-year-old man suffers 20% partial and full thickness burns in a house fire. There is an associated inhalational injury. It is decided to administer intravenous fluids to replace fluid losses. Which of the intravenous fluids listed below should be used for initial resuscitation?

Dextran 40

5% Dextrose

Fresh frozen plasma

Hartmans solution

Blood

A

In most units a crystalloid such as Hartmans (Ringers lactate) is administered initially. Controversy does remain and some units do prefer colloid. Should this leak in the interstial tissues this may increase the risk of oedema.

4257
Q

Indications for fluid resus in burns

A

: >15% total body area burns in adults (>10% children)

The main aim of resuscitation is to prevent the burn deepening

Most fluid is lost 24h after injury

First 8-12h fluid shifts from intravascular to interstitial fluid compartments

Therefore circulatory volume can be compromised. However fluid resuscitation causes more fluid into the interstitial compartment especially colloid (therefore avoided in first 8-24h)

Protein loss occurs

4258
Q

Fluid resuscitation formula in burns

A

Parkland formula (crystalloid only)

Total fluid requirement in 24h= 4ml x total burn surface area x bodyweight (kg)

1/2 in first 8 hours.

Aim for urine output of 0.5-1ml/kg/hr in adults (increase rate of fluid to achieve this)

Starting point of resus is time of injury

4259
Q

Fluid management in burns after 24h

A

Colloid infusion is begun at a rate of 0.5ml x total burn surface area x bodyweight

Maintenance crystalloid is continued at rate of 1.5ml x total burn SA x weight

Colloids used include albumin and FFP

Antioxidants may be used to minimise inflammatory cascade

High tension electrical injuries and inhalational injuries require more fluid.

4260
Q

A 17-year-old male is admitted with lower abdominal discomfort. He has been suffering from intermittent right iliac fossa pain for the past few months. His past medical history includes a negative colonoscopy and gastroscopy for iron deficiency anaemia. The pain is worse after meals. Inflammatory markers are normal.

A

Meckels diverticulum

This scenario should raise suspicion for Meckels as these may contain ectopic gastric mucosa which may secrete acid with subsequent bleeding and ulceration

4261
Q

Undisplaced intracapsular fracture

Nil comorbidities

A

Internal fixation, especially if young

4262
Q

Undisplaced intracapsular fracture

Major illness or advanced organ specific disease

A

Hemiarthroplasty

4263
Q

Displaced intracapsular fracture

No comorbidities

<70y/o

A

Internal fixation if possible

Hip aryhoplasty if not

4264
Q

Displaced intracapsular fracture

No comorbidities

>70 years old

A

THR

4265
Q

Displaced intracapasular fracture

Major comorbidities/immobile

A

Hemiarthroplasty

4266
Q

Extracapsular fracture (non special type)

A

Dynamic hip screw

4267
Q

Extracapsular fracture

(reverse oblique, transverse or subtronchateric)

A

Intramedulllary device

4268
Q

A 72-year-old retired teacher is admitted to A&E with a fall and hip pain. He is normally fit and well. He lives with his son in a detached, 2 storey house. A hip x-ray confirms an extracapsular fracture.

A

Dynamic hip screw

Extracapsular fractures should be treated surgically. Since the blood supply to the femoral head is not compromised joint replacement is not usually warranted

4269
Q

A 72-year-old retired teacher is admitted to A&E with a fall and hip pain. He is normally fit and well. He lives with his son in a detached, 2 storey house. A hip x-ray confirms an subtrochanteric fracture.

A

Intramedullary device

Intramedullary device is normally recommended for reverse oblique, transverse or subtrochanteric fractures.

4270
Q

An 86-year-old retired pharmacist is admitted to A&E following a fall. She complains of right hip pain. She is known to have hypertension and is currently on bendrofluazide. She lives alone and does not mobilise. Her right leg is shortened and externally rotated. A hip x-ray confirms a displaced intracapsular fracture.

A

Hemiarthroplasty non cemented prosthesis

This patient warrants a hemiarthroplasty due to reduced mobility and older age. The anterolateral approach is recommended in the SIGN guidelines. In this case most surgeons would not use a cemented prosthesis.

4271
Q

Theme: Muscle relaxants

A.Atracurium

B.Suxamethonium

C.Pancuronium

D.Vecuronium

E.Curare

An agent that is degraded by hydrolysis and may produce histamine release.

An agent which should be avoided in a 23-year-old man with burns and bilateral tibial fractures after being trapped in a car accident for 2 hours

An agent with a half life of less than 10 minutes.

A

Atracurium

Atracurium is degraded by a process of ester hydrolysis. This uses non specific plasma esterases.

Suxamethonium

Suxamethonium may induce hyperkalaemia as it induces generalised muscular contractions. In patients with likely extensive tissue necrosis this may be sufficient to produce cardiac arrest.

Suxamethonium

Suxamethonium is extremely rapidly metabolised, acetylcholinesterases degrade the drug within minutes. In patients who lack this enzyme the drug may last far longer.

4272
Q

Laryngocoele

A

Air filled sac that communicates with the larynx

Can be congenital or acquired (e.g. due to high pressure- COPD)

If it is limited to the paraglottic space it presents with stridor and hoarseness.

If it protrudes through the thyrohyoid membrane it presents as a reducible lump in the anterior triangle of the neck which recurs with coughing, sneezing, etc.

4273
Q

What is a fistula? What is the exception

A

Abnormal connection between two epithelial surfaces.

AV fistula which is a connection between two endothelial surfaces

4274
Q

Hernia through the triangle bounded by external oblique, lat dorsi and iliac crest?

A

Petit hernia (lumbar hernia)

4275
Q

Hernia passing through a traingle bounded by 12th rib, sacrospinalis muscle medially and interanl oblique laterally

A

Grynfelt hernia

4276
Q

Radiographic triad in splenic injury

A

Elevated left hemidiaphragm

Left lower lobe atelectasis

Left pleural effusion

4277
Q

Describe the levles of foot amputaiton

A
4278
Q

Primary lesions giving rise to painful arc?

A

Incomplete tear of supraspinatus

Chronic supraspinatus tendonitis

Subacromial bursitis

Crack fracture of the greater tuberosity

4279
Q

Differentiating between supraspinatus tendonitis and impingement syndrome

A

Full can test against resistance- weakness +/- supraspinatus lesion or tear dependent on degree of weakness

Empty can test force, assess for weakness or pain

impingement= painful arc

4280
Q

Components of Whipple’s Procedure

4 things removed

3 anastomoses

A

Pancreatic head, common bile duct and gallbladder, distal stomach and some of the duodenum.

3 anastomoses: hepato-jejnostomy, pancreatic jejunostomy and a gastrojejunostomy

4281
Q

Schwart test

A

Saphena varix will demonstrate a fluid thrill when the greater saphenous vein is tapped distally

4282
Q

Anatomy of Kocher’s incision

A

3-5cm below the costal margin

Anterior rectus sheath exposed and divided along the line of excision exposing the rectus muscles which may then be divdided, taking care to enure haemostasis when branches of the superior epigastric vessels are sacrificed.

Incision is extended through the lateral abdominal wall muscles which are split to allow better access.

8th and 9th intercostal nerves lie between the IO and transversus muscles. 8th nerve often sacrificed.

Incision is deepened through the posterior rectus sheath and peritoneum.